Download as pdf or txt
Download as pdf or txt
You are on page 1of 668

Downloaded From : www.EasyEngineering.

net

ww
w.E
a syE
ngi
nee
rin
g.n
et

**Note : Other Websites/Blogs Owners we requested you, Please do not Copy


(or) Republish this Material.
This copy is NOT FOR SALE.
**Disclimers : EasyEngineering does not own this book/materials, neither created nor
scanned. we provide the links which is already available on the internet. For any quarries,
Disclaimer are requested to kindly contact us. We assured you we will do our best. We DO
NOT SUPPORT PIRACY, this copy was provided for students who are financially troubled but deserving
to learn. From : www.EasyEngineering.net
Downloaded Thank You and God Bless!
Downloaded From : www.EasyEngineering.net

ww
w.E
asy
En
gin
eer
ing
.ne
t

Downloaded From : www.EasyEngineering.net


Downloaded From : www.EasyEngineering.net

ENGINEERING
MATHEMATICS -I
SECOND EDITION

ww
w.E P.B. Bhaskar Rao
M.Sc., Ph.D.
Retd. Professor, Former Chairman, Board of Studies,

a Department of Mathematics

syE
Osmania University
Hyderabad

ngi
S.K.V.S.Sriramachary nee M. Bhujanga Rao
M.A., M.Phil., B.Ed.
Professor & Head (Retd.) rin M.Sc., Ph.D.
Professor, Dept. of Mathematics
Department of Mathematics
University College of Engineering g.n
University College of Engineering
(Autonomous)
(Autonomous)
Osmania University
Hyderabad
et
Director of Centre for Distance Education
Osmania University
Hyderabad

BSP BS Publications
4-4-309, Giriraj Lane, Sultan Bazar,
Hyderabad - 500 095 A.P.
Phone: 040 - 23445688
e-mail: contactus@bspublications.net

Downloaded From : www.EasyEngineering.net


Downloaded From : www.EasyEngineering.net

Copyright © 2008, by Publisher

All rights reserved.

No part of this book or parts thereof may be reproduced, stored in a i


retrieval syste'm or transmitted in any language or by any means,
electronic, mechanical, photocopying, recording or otherwise without
the prior written permission of the publishers,

ww
w.E
Published by :
a
BSP BS Publications syE
ngi
=:;;;;= 4-4-309, Giriraj Lane, Syltan Bazar,
Hyderabad - 500 095 - A. P.
Phone: 040-23445688
e-mail: contactus@bspublications.net nee
www.bspublications.net
rin
g.n
Printed at
et
Adithya Art Printers
Hyderabad.

ISBN: 978-81-7800-151-7

Downloaded From : www.EasyEngineering.net


Downloaded From : www.EasyEngineering.net

Contents

ww
CHAPTER -1

w.E
Ordinary Differential Equations of
First Order and First Degree ..................................................... 1

CHAPTER -2
a syE
Linear Differential Equations with

ngi
Constant Coefficients and Laplace Transforms ...................... 69

CHAPTER-3
Mean Value Theorems and nee
rin
Functions of Several Variables .............................................. 111

CHAPTER-4
g.n
Curvature and Curve Tracing ................................................ 213

CHAPTER-5
et
Application of Integration to
Areas, Lengths, Volumes and Surface areas ........................ 313

CHAPTER-6
Sequences of Series .............................................................. 385 _

CHAPTER-7
Vector Differentiation ............................................................. 475

CHAPTER-8
Laplace Transforms ............................................................... 623

Downloaded From : www.EasyEngineering.net


Downloaded From : www.EasyEngineering.net

ww
w.E
a syE
ngi
"This page is Intentionally Left Blank"
nee
rin
g.n
et

Downloaded From : www.EasyEngineering.net


Downloaded From : www.EasyEngineering.net

1w
ww
.Ea
Ordinary Differential Equations of
First Order and First Degree syE
ngi
1.1 Introduction
nee
Differential euqtions play an important role in many applications in the field of

rin
science and engineering, such as (i) problems relating to motion of particles
(ii) problems involving bending of beams (iii) stability of electric system, etc. For

g.n
example, Newton's law of cooling states that the rate of change of temperature of

e
a body varies as the excess temperature of the body to that of its surroundings. If

t
8(t) is the temperature of the body at time 't' and 8 0 is the temperature of the room
de
in which the body is kept, then dt gives the rate of change of temperature with

time.
de
dt = K(8 - 8 0) ; K is constant

Similarly Newton's second law of motion for a particle of mass m moving in a


straight line can be written as
d 2x
m dt 2 =F
Where m is the mass, x is the distance of the particle at time 't' measured from
a fixed origin and F the external impressed force.

Downloaded From : www.EasyEngineering.net


Downloaded From : www.EasyEngineering.net

2 Engineering Mathematics - I

A differential equation is an equation involving an unknown function and its


derivatives. Ifthere is only one independent variable and one dependent variable the
equation is called (Ill ordinary differential equation.
If there are more than one independent variable the equation is called a partial
differential equation as this involves partial derivatives.
For example:

d3y dy
4 +3x _ y=e( .... (a)
3
dx dx

ww d3y J4 ( d2YJ8 (dY) 12 6 _ 8


w .Ea
(-dx;3- + -dx-2 + -dx + Y -x .... (b)

syE .... (c)

ngi
nee .... (d)

rin
g.n
.... (e)

e t
.... (t)

.... (g)

The first four equations (a), (b), (c) and (d) are ordinary differential equations and
the remaining three are partial differential equations.
Order 0/ a differential equation: The order of a differential equation is the order
of the highest ordered derivative appearing in the equation.
Degree 0/ a differential equation: The degree of a differential equation is the
power to which the highest ordered derivative appears in the equation after clearing
the radicals if any.

Downloaded From : www.EasyEngineering.net


Downloaded From : www.EasyEngineering.net

Ordinary Differential Equations of First Order and First Degree 3

In the above examples:


Example: 1.1(a) is a differential equation of order 3 and degree I.
Example: 1.1(b) is of third order and fourth degree differential equation.
Example: 1.1(c) is a second order, first degree differential equation.
Example: 1.1(d) is a second order, second degree ditferential equation.

1.2 Example
Formation of an ordinary D.E :

ww The differential equations ar~ formed by eliminating all the arbitrary constants
th~t areinvolved in the functional relationship between the dependent and independent

w.E
variables.
For example:
y =

asy
cx2 + c 2 where c is an arbitrary constant.
To eliminate 'c': (only one constant)
.... (I)

From(l)
dv
En = c.2x+ 0

gin
_0

dx
I (~V
c= -
eer
2x dx
Substitution of c in (I) gives

y _I dy x2+ _1_(dy )2 ing


.ne
=
2x d\: 4x 2 d);

d
~
( dx
)2 + 2x 3 2d
dx
- 4x2y = 0 t
is the required D.E and y = cx2 + c 2 is called the solution of the D.E.
Note:
Depending on the number of constants in the given equation differentiate it as
many number oftimes successively. Then the elimination of the arbitrary constants
from the resulting equations and the given equation gives the required differential
equation whose order is equal to the number of constants.

1.3 Example
Eliminate the arbitrary constants a, b from xy + x 2 = aeX + be-X and form the
differential equation.

Downloaded From : www.EasyEngineering.net


Downloaded From : www.EasyEngineering.net

4 Engineering Mathematics - I

Solution:
The given equation is
xy + .x2 = ae-'" + be-x ..... ( I)
The number of arbitrary constants is two. Differentiating (I) w.r., to 'x' two
times successively.
dy _
x dx + Y + 2x = ae-' - be-x .....(2)

d2y ely dy
X ---2 + - + - + 2.1 = aex + be-x .... (3)

ww d-c dx dx
From (I), (2) and (3) el imination of a, b gives the D.E.

w.E
from (I) and (3) we get
d l )' 2dy . .

asy
x --?
dx-
+ - - + 2 = xy + x 2 IS the requIred D_E.
dx

1.4 Example
En
a.x2 + by =1 gin
Form the differential equation by eliminating the constants a and b from

Solution:
Differentiating
e eri
ax2 + by = I w.r.t 'x' ng. .... (I)
dy
2ax+ 2by-
d-c
Again differentiating wr.t., 'x'
= 0
net .... (2)

d 2y dy dy
2a+2by - ? +2b-.- =0 .... (3)
dx- dx dx
Elimination of a, b from (I), (2) and (3) gives
x-? i -I
x yYl 0 =0
(Y.h + yl2 0

Expanding the determinant we get


2 y
x d y +x(d )2 _ y(dY)=o
2
dx dx dx

Downloaded From : www.EasyEngineering.net


Downloaded From : www.EasyEngineering.net

Ordinary Differential Equations of First Order and First Degree 5

1.5 Example
Form the differential equation by eliminating the constants from
y = a secx + b tan x
Solution
Given equation is
y= asecx + btanx .... (I)
Differentiating w.r. to 'x'

ww dy
dx
= asecx tanx + bsec 2x .... (2)

w.E dy
= secx[a tan x + b sec xl .... (3)

asy
dx

Further differentiation gives

d 2y En = a sec x tan 2x + a sec3 x + h2sec 2x tan x


-,
2
(X
gin
I.e., d Y
--1
dx-
2
= asecx tan x
2 1
+ bsec-xtanx
eer
+ bsec 2xtanx + asec 3x

d 2y ing
I.e., dx 2
.ne
= secx t'lnx(atanx + bsecx) + sec 2x(btanx + asecx) ..... (4)

Substituting
asecx + btanx =y from (I)
t
atanx + bsecx = --
(~) from (2)
and
secx
in (3) we get

dY)
[-f)
d2
=
(
secx tanx ~ + sec x(y)
--
2
dx secx

d 2y dy
i.e., - -2 - tanx - - ysec2x = 0
dx dx

Downloaded From : www.EasyEngineering.net


Downloaded From : www.EasyEngineering.net

6 Engineering Mathematics - I

1.6 Example
Form the differential equation of all circles passing through the origin and having <

their centres on the x - axis.


Solution
Take any tangent to the circle as y-axis, the centre lies on x-axis. Let 'a' be the
radius of the circle.
Then centre is (a, 0)
:. Equation of the circle is (x - a)2 + .r a2 .... (1)

ww
=

w .Ea
x
,
syE x

y' ngi
Fig. 1.1 nee
Differentiating (I) w.r. to 'x'
rin
dy
2(x - a) + 2y - = 0 g.n
x- a =
dx
dy
-y-
dx
e t
dy
a=x+ y - .... (2)
dx
From (1) and (2)

[x-(x+ y:lJ' y' =[x+ y:J


+

.r (:r +.r = x2 +.r (:r + 2XY ( : )

2XY : + x2 - .r = 0 is the required D.E


Downloaded From : www.EasyEngineering.net
Downloaded From : www.EasyEngineering.net

Ordinary Differential Equations of First Order and First Degree 7

1.7 Example
Form the differential equation of all central conics whose axes coincide with the
axes of coordinates.
Solution
The equation of all central conics whose axes coincide with the axes is
ax2 + bl = 1 .... (1)
Differentiating (1) w.r.t., x

ww dy
2ax+ 2by -
dx
= 0 .... (2)

w Differentiating (2) w.r.t., 'x' again

.Ea 2
d y
a+by - 2 +b -
y )2 (d
=0 .... (3)

syE dx

Eliminating a, b from (1), (2)(3)


dx

x2
n y2 gin
dy
e
r ering
X y-
dx
o =0
2
y~+ ~
d 2 (d 0
dx dx
.ne
=>
dx·
y
dx dx t
Xyd2; +x(d )2 _y(dY)=O is the required D.E.

Exercise - 1(a)
1. Eliminate the arbitrary constants from the following and find the
corresponding differential equation :
(i) y = mx + c (m, c arbitrary constants)

(ii) y = a e2x + b e-2x (a, b arbitrary. constants)

d2
[ Ans : ~ - 4y - 0]
dx 2

Downloaded From : www.EasyEngineering.net


Downloaded From : www.EasyEngineering.net

8 Engineering Mathematics - I

(iii) y = ax + bx 2 (a, b arb. constants)

d2 dy
[ADS: x 2 ----?
dx
- 2x -d + 2y
x
= 0]

(iv) (x - h)2 + (y - kf = a2, (h, k a, b arb. constants)

ww (v) y = (a + bx)e-X , (a, b arb. constants)

d 2y 2dy

w.E a sin x + b cosx (a, b arb. constants)


[ Ans: - 2 + -
dx dx
+Y = OJ

asy
(vi) y =

d 2y

En [ADS: - l
2
(X
+ Y = 0]

gin
2. Find the differential equation of all circles with centre on the line y = x and having
radius' I '.

e eri
ng.
3. Form the differential equation of all the circles with centre on the line y
passing through the origin. net = -x and

[ADS. : (xl + y) (: -1) = 2(y - x) (x + y:)]


4. Find the differentiall equation of all the parabolas with vertex at the origin and foci
on the x - axis.
dy
[ADS: y -2xy dx = 0]

5. Find the differential equation of all parabolas with the origin as focus and axis along
x-axis.
?
dy dy -
[ADS: 2x dx + Y ( dx) -Y = 0]

Downloaded From : www.EasyEngineering.net


Downloaded From : www.EasyEngineering.net

Ordinary Differential Equations of First Order and First Degree 9

Methods to Solve

1.8 The differential equations of the first order and of the First
Degree:
1.8.1 Separation of Variables
Sometimes the differential equation
/

dy
dx = q(x, y)

ww can be written as

w.E f(x) dx + g(y)dy = 0


if the variables can be separated.
.... (I)

asy
Integration of (1) gives the solution of the equation.

i.e., En
Jf(x)dx+ Jg(y)dy=c

gin
where c is an arbitrary constant.

1.8.2 Example eer


Solve et"tany dx + (1-~sec2ydy = 0
ing
Solution
.ne
The given equation
e-'"tany dx + (1-e-'")sec 2ydy =0 t
can be rearranged as

e( sec 2 y
- - d x + - - dy=O
I-eX tany

2
eX Jsec y
Integrating --dx+ - - dy=c
J I_eX tany

-Iog( e-'" -I ) + log t~ll1Y = c

tany
i.e., -X-I = c or tan y = c(e X -I)
e -

Downloaded From : www.EasyEngineering.net


Downloaded From : www.EasyEngineering.net

10 Engineering Mathematics - I

1.8.3 Example
dy
Solve - = 1 + xl +
dx
Y + xly
Solution
The given differential equation can be written as

dy = (1 +xl) (1 + y)
dx

ww dy x3
-1- 2 = (1+xl)dx => tan-I y = x + - + C
+y 3

w.E
1.8.4 Example
dy
Solve dx - 2xy = x, where YCO) = 1

Solution asy
dy En
gin
- =x(1+2y)
dx
dy
I+2y =xdxeer
On integration ing
I
"2 Iog(1+2y) = 2 +c
x2
.ne .... (1)

Giveny= 1 when x = 0
t
Substituting in (1)
1
"2 Iog(3) = 0 + C

C= ~ log3 = log(v'J)
Hence the solution is
2
"21 Iog(I +2y)= 2x + Iogv'J (i.e.,) log (1+2Y)
- 3 - =xl

Downloaded From : www.EasyEngineering.net


Downloaded From : www.EasyEngineering.net

Ordinary Differential Equations of First Order and First Degree 11

1.8.5 Example
dy
Solve - =(4x+y+ If
dx
Solution
dy
-= (4x + y + 1)2 .... (I)
dx
Substituting 4x + y + I = t in (I)
dy dt
4+-

ww
=-
dx dx

w.E i.e.,
dy
dx
= dt -4
dx

asy
dt
- -4 = t 2
dx
Integrating
En
gin
i.e.,
I t
-tan-I-=x+c eer
2 2
ing
tan-I (4X+;+ I) = 2(x + c)
.ne
The solution can also be written as
4x + y + I = 2tan(2x + c)
where C is an arbitrary constant.
t
Exercise -1(b)

1.9 Solve the Following Differential Equations

1. : =~Y [ADS: ~ + e-Y = c]

2. (2 - x)dy - (3 + y) dx = 0 [ADS (3 + y) (2 -x) = c]

Downloaded From : www.EasyEngineering.net


Downloaded From : www.EasyEngineering.net

12 Engineering Mathematics - I

[ ADs: yc = (a + x) (I - ay) ]

dy
6. (x-y)2 dx = a2 [ADs: (x - y) + log ( x- y-a) = x + c]
x-y+a

dy
[ADs: 2(3x+4y+ 1)= .J3tan- 1 2.J3x+c]
ww
7. - =(3x+4y+ 1)2
dx

8.
w.Edy =(2x+y+
dx
If [ ADs: 1 tan-I (2X +
.J2 .J2Y + I ) = x +c]

dy asy
9. dx = tan (x + y)
En [ADs: log[sin(x + y) + cos(x + y)] = x - y + c]

10 dy = 2 gin [ADs: (x + 2y - 3) - 410g (x + 2y + I) =x + c ]


. dx (x+2y-3)
eer
1.9.1 Homogeneous Equations
The differential equation of the form ing
dy f(x,y) .ne
dx g(x,y)
where f, g are homogeneous functions of same degree in x, y is called a
t
homogenous differential equation.
Such a differential equation can be written as

.... (I)

Substitutingy = vx
dy dv
- =v+x-
dx dx

Downloaded From : www.EasyEngineering.net


Downloaded From : www.EasyEngineering.net

Ordinary Differential Equations of First Order and First Degree 13

The D. E (I) becomes

dv \11 (v)
v + x dx = ~(v)

dx ~(v)dv
-; + v~(v)-\lf(v) = 0

Integration yields the solution

J-
dx
+
J"'()~(v)dv( ) = c where v = -y
ww \
X v'" v - \11 v x

w
1.9.2 Example

Solve
.Ea
(xl +
dy
I) -dx =xy

Solution
syE
ngi
· . dy dv
S U bstltuttngy = vx, -d = v + x - nee
dv
x dx
xvx rin
v + x-
dx
= 2
x +v x
2 2

g.n
dv
v+ x -
dx
=--
v
1+ v 2
e t
dv v
x- = - -2- v
dx 1+ v

dv _v 3
x-=--
dx 1+ v 2

dx JI
~+ JI-dv=c
J-+
X v3 v

Downloaded From : www.EasyEngineering.net


Downloaded From : www.EasyEngineering.net

14 Engineering Mathematics - I

v-2
logx + - + logv = e
-2
1
logxv- - = e
2V2

logx - ~(y)
x 2y
X2
= e ~ 2ylogy - xl = 2ey

i.e., 2ylogy = 2ey + x 2

ww
1.9.3 Example
dy y2
w.E
Solve x-+-=y
dx x

Solution
asy
dy

En
xl-+y=xy
dx

i.e., gin
Substitutingy = vx, e eri
dy dv
-=v+x- ng.
dx dx
net
dv
v+x - =v-vl
dx
dv
x - =-vl
dx
dx dv
-+- =0
X v2
Integration yields
V-2+1 1
logx+ - - =e ~logx-- =c
-2+1 v

Downloaded From : www.EasyEngineering.net


Downloaded From : www.EasyEngineering.net

Ordinary Differential Equations of First Order and First Degree 15

x
logx - - = c
y

ylogx -x = cy or ylogx = x + cy

1.9.4 Example

Solve [x+ YSin(Yx)]dx = xSin(Yx)dy

ww Solution
The given differential equation can be written as

w.E dy x+ ysin(X)

dx asxSin(j~)
= __

yE
--,---.0.-,.-----.:-

Substitutingy = vx ~
n
dy dy
- = v + x. -
dx dx gin
dv
v+x -
x+ vxsinv e eri
ng.
= ----
dx xsinsin(v)

dv l+vsin
v+x- = - - -
dx sinv net
dv l+vsinv 1
x-= -v=-.-
dx sin v SI11 v

dx
sin v dv = -
x

Integrating
-cosv = logx + c

logx + cos(Yx) = c

Downloaded From : www.EasyEngineering.net


Downloaded From : www.EasyEngineering.net

16 Engineering Mathematics - I

Exercise - 1 (c)

1.10 Solve the Following Equations

dy y2
l. dx - xy-x2 [ADS : y = ceix ]

2. (2 - 2xy)dx = (x 2 - 2xy)dy [ ADS : xy(y - x) = c ]

3. 2xy + ~ - r) dy = 0 r + y = cy ]
ww dx
[ADS:

w.E [ADS: logx = 2tan- i (Yx) + c]

asy
5. xdy - ydx = ~ x 2 + y2 dx

En y

gin [ ADS: cos - = logcx ]


x

7. xcos(Yx) (ydx + xdy) = ysin (Yx) (xdy - ydx) eer [ADS: sec (Yx) = yxc]
ing
8. (ry - x 3 )dy - ~ + xy) dx = 0 [ ADS:
2
.ne
y~ x + y = cx.e
2 tan
-I(YI)
Ix
]

9. (r + y) dx = 2xydy ( ADS: (r - y) = xc ] t
dy
10. X dx = y[logy - logx + 1] ( ADS : y = xecx ]

1.10.1 Non-homogeneous Differential Equations


The D.E of the form
dy = ax+by+c
.... (J)
dx Ax+By+C

where a, b, c A, B, C are constants, is called a non-homogeneous dif.ferential


equation.

Downloaded From : www.EasyEngineering.net


Downloaded From : www.EasyEngineering.net

Ordinary Differential Equations of First Order and First Degree 17

Case (i)
a b
If --::;:.-
A B
Substituting x = X + h, Y = y + K
where h, k are (constants) to be chosen so as to satisfy.
ah + bk + c = 0, Ah + Bk + C = 0
Solving these equations, values of h, k are obtained.
The given D.E then reduces to a homogeneous D.E.

ww dY aX +bY

w.E dX AX +BY
which is then solved taking Y = VX and then
substitute X
Case (ii)
=
asy
x - hand Y = y - k in the solution.

If
a b
En
A B
gin
Then the differential equation will be of the fonn

dy _ (ax + by)+c e eri


dx - m(ax+by)+C
ng. .... (2)

since Ax + By will be constant m times ax + by.


Now substitute
ax + by = t,
net
Differentiation gives

a+bdy=dt
dx dx
dt
--a
i.e., dy dx
-=--
dx b
D.E (2) then reduces to
dt
--a t+c
~=---
b mt+C

Downloaded From : www.EasyEngineering.net


Downloaded From : www.EasyEngineering.net

18 Engineering Mathematics - I

Then the solution is obtained by using the method of separation of variables.

1.10.2 Example
dy x+2y-3
Solve
dx 2x+ y-3

Solution
dy x+2y-3
= ... - (I)
dx 2x+ y-3

ww Substituting

w.E x = X + h, y = Y + k
where h, k are chosen to satisfY h + 2k - 3 = 0 and 2h + k - 3 = 0
solving we get
h=I,k=I
asy
En
.... (2)
i.e., we take x = X + I, Y= Y + I
The D.E (I) reduces to gin
dY
dX
X-2Y
=---
2X+Y e eri
dY
Substituting Y = VX, dX = V + X dX
dV
ng.
dV X+2VX
V+X-=---
net
dX 2X+VX

~
2
V+XdV =1+2V XdV =1-V
dX 2+V dX 2+V

2+v dv=dX
I-v X

dX +(1 +_3_) dv = 0
X v-I

10gX + v + 3 log(v-I) = loge

Downloaded From : www.EasyEngineering.net


Downloaded From : www.EasyEngineering.net

Ordinary Differential Equations of First Order and First Degree 19

v + log( v-I )3 + logX = loge


v-log(v-I)3 X = loge

~-X
3
Y V-x
X +10 )
X=loge

I.e., 10g(Y - X)3 + XY = Xloge


log(y - \ - x + \)3 + (x - \) (y - I) = (x -I) loge
log(y - x)3 = (x - 1) (loge - y + \)

ww
1.10.3 Example

w.E Solve (2x + 3y + \) dx + (2y- 3x + 5)dy = 0


Solution
dy
asy
2x+3y+ 1
dx
En
2y-3x+5

gin
Substituting x = X + h, y = Y = k, : = :

eer
Choosing h, k so that 2h + 3k + 1 = 0, 3h - 3k - 5 = 0
we get h = 1, k = -1
ing
The given differential equation reduces to
dV 2X+3Y .ne
Substituting
dX 3X-2Y
t
Y=YX
dV dV
- =Y+X-
dX dX
dV 2X+3YX
Y + X dX = 3X -2YX

Y+X dV =2+3Y
dX 3-2Y

X dV = 2(1 + y2 )
dX 3-2Y

Downloaded From : www.EasyEngineering.net


Downloaded From : www.EasyEngineering.net

20 Engineering Mathematics - I

3-2V)dV= 2dX
( 1+ V2 X
Integrating

3 -dv
f 2v
- - --dv=2 f-+c fdX
1+ v 2 1+ v 2 X
3tan- 1(V) - log( I + v2) = 210gX + c

3tan 1 (~ )-IO~ X';/') ~ 210g)( + c


ww B~ X=x-h=x-I
w.E Y=y-k=y+1

asy )-IOg[ -Il~: ~ I)'] ~


3tan-1 (~:: (x + 210g(x- I) + c
En
1.10.4, Example
gin
Solve
dy =x-y+l
dx 2x-2y eer
Solution
i ng.
-dy = --'------'-'--
dx
(x- y)+1
2(x- y) n et
Substituting x - y = V

dY=I_ dV
dx dx

l_dV=V+I
dx 2V

dV_ V+l_V-1
-- I------
dx 2V 2V

2V
- dV=dx
V-I

Downloaded From : www.EasyEngineering.net


Downloaded From : www.EasyEngineering.net

Ordinary Differential Equations of First Order and First Degree 21

Integrating

2 JV-I+I
dV=x+c
V-I

2[V + 10g(V-l)] = x+c


2[x - y + log(x - y -1)1 = x+c
210g(x - y - I) = 2y - x + c
or (x - y -1)2 = e2j~x C

ww
1.10.5 Example

w.E Solve
dy
dx
4x+6y+1
2x+3y -5

Solution
asy
(~V
-=-
d'(En 2(2x+3y)+ I
(2x+3y)-5

Substituting gin
2x + 3y = V
eer
dy (N
2+3-=- ing
dx dx

-2]
.ne
dy
dx
=~[dV
3 £Ix t
The differential equation reduces to

~(dV -2)=- -2V + I


3 dx V-5

dV
-
6V+3
=2+ - -
IO-2V+6V+3
= ------
13+4V
dx 5-V 5-v 5-V

5-V
4V+13 dV=dx

Downloaded From : www.EasyEngineering.net


Downloaded From : www.EasyEngineering.net

22 Engineering Mathematics - I

3
Integrating fl 1-
J~
3
4V + 13
) dv = _ 4 Idx + e

33
v- -log(4V+ 13)=-4x+e
4

33
(2x + 3y) - - log[4(2x + 3y) + 13] = - 4xm + C
4

ww i.e., 4(6x+3y-e)=33Iog[4(2x+3y)+ 13]

Exercise -1(d)

1.11
w.E
Solve the Following Equations

dy y-x+5 asy
\. - + -=-------
dx y+x+3
En
gin y+4 1 y+4
1
[Ans:tan- J (
eer
x-I ) +"2 log[ ( x-I ) 2 +1 =log(x-I)+loge]

dy x+2y-3 ing
2. dx - 2x+3y-5
.ne
[Ans: (2+Ji) IOg[y-1 -
x-I ,,3
~] - (2-Ji) log [y-I +
x-I ,,3
~]+ ,,3~ logx= loge] t
3 _dy = _2y'------_x_-_4
. dx y-3x+3

[Ans: (x - 2)2 - 5(x - 2) (y - 3) + (y - 3)2 = e {2(y-3)-[5+h1{X-2)]}]


2{y -3)-(5 -h1(x-2»

4. (2x+5y+ l)dx-(5x+2y- l)dy=O

[ Ans : (x + y) 7 = e ( x - y - ~ r]
Downloaded From : www.EasyEngineering.net
Downloaded From : www.EasyEngineering.net

Ordinary Differential Equations of First Order and First Degree 23

dy y-x+ 1
5.
dx y+x+5

y-2) I
2
[ADS : tan~l ( x _ 3 + log [(y - 2)2 + (x - 3)2] = loge]

dy _ 3y+2x+4
6. dx 4x+6y+5

[ADS: .7'j(2x+3y)- :9 (14x+2ly+22)=x+c]

ww
7. (4x-6y-l)dx+(3y-2x-2)dy=O

w.E 3
[ADS: x - y + 41og(8x - 12y - 5) = c ]

8.
dy
dx = 2x - a
x-y+3
2y + 5
syE
ngi [ ADS: (x - y) + log[2 + x - y ]
x
= -
2
+ C]

1.12 Linear Differential Equations nee


A differential equation of the from : rin
+ py = q where p, q are functions of 'x',

g.n
alone or constants is said to be a linear differential equation offirst order. Multiplying
both sides of the equation by el pta [called the integrating factor (I.F)] we get,

el ptlr dy + el ptlr py = q .e Iptlr


et
.... (i)
dx

The left hand side is the differential coefficient of y. e l pcb:


(i) can be written as
1ptlr
d(y.e ) = qe 1ptlr

Integrating

which gives the desired solution.

Downloaded From : www.EasyEngineering.net


Downloaded From : www.EasyEngineering.net

24 Engineering Mathematics - I

Note:
In some cases a differential equation can be reduced to the linear form by taking 'y'
as independent variable and x as the dependent variable.
The D.E is written as

p" q I are functions of y or constants

ww Now the I.F = eJ


Pldy

w Solution is

.Ea x.e
JPldy
=
fql·e Jpl"Y •
+c

1.12.1 Example syE


Solve
n (I + x 2 )
dy
-
dx
gin
+ 2yx - 6x2 = 0

Solution
e
Rearranging the given differential equation to the form
eri
dy
dx + py = q ng.
We have

dy 2x
-+--y=--
6x 2
n et
dx I +X2 1+X2

2x 6x 2
Here p= I+X2 ,q= I+X2

~dx
I.F= e Jpdx =e J I+x2
log(I+X2 )
I.F = e = (1 + x 2 )
Solution is given by

y(1. F) = fp(I.F)dx + c

Downloaded From : www.EasyEngineering.net


Downloaded From : www.EasyEngineering.net

Ordinary Differential Equations of First Order and First Degree 25

1.12.2 Example

ww Solve xlogx
dy
- +y=2Iogx
dx

w.E
Solution
dy y 2

asy
-+--=-
dx xlogx x

En
I 2
p= - - andq=-
Here
xlogx x
gin
e eri
Solution is

J~ logxdx + c
ng.
ylogx =

(IogX)2
net
ylogx=2 +c
2

1.12.3 Example
dy Y (xsmx+cosx
xcosx -+ . ) =I
Solve
dx
Solution
dy xsin+ cosx
-+y.----
dx xcosx xcosx

xsin x + cosx I
p= ,q=--
xcosx xcosx

Downloaded From : www.EasyEngineering.net


Downloaded From : www.EasyEngineering.net

26 Engineering Mathematics - I

J ~Slll X~.COS~ dx
I.F = e XCDSX

= e(log(xsecx)

I. F = e1og(x sec x)

= xsecx

ww Solution is

w.E y(xsec x) = f_l-


xcosx
x x sec x dx + c

xysec x
asy =
2
Jsec xdx E c

xysecx
En = tanx+c

1.12.5 Example
gin
Solve eer
dy + 2ytanx = sinx given that y = 0 where x = ~,

Solution
~

ing 3

dy + 2ytanx = sinx .ne


dx
p = 2tanx
t
q = sinx

ef
2 tan xd!:
IF =

~ = e210gsecx

ylF= JqxIF.dx+c

2
ysec 2x = Jsecx.sec xdx + c

Downloaded From : www.EasyEngineering.net


Downloaded From : www.EasyEngineering.net

Ordinary Differential Equations of First Order and First Degree 27

ysec 2x = Jsec x tan xlir + c


')
ysec-x = secx + (' .... (I)

Given that y = 0 when x = ;7j


1C
o= sec - + c => C = -2
3

Substituting c = -2 in (I)

ww ysec 2x = secx - 2

w.E
1.12.6
is the required solution

Example

asy dy
Solve (x + 2.v) -, = y

Solution En (X

+2.v gin yd
y

e
X =

er
dx

dx
--- =
dy
2.v x
y ing
is in the form of .ne
t
-1
P = - q = 2,1
I ' 1 Y
Y

Downloaded From : www.EasyEngineering.net


Downloaded From : www.EasyEngineering.net

28 Engineering Mathematics - I

x-1
y
= f2y 2 1
x-dy+c
y

x
- =y+C
Y

1.13.7 Example

ww Solve (x + y+ I) -
dy
dx
= 1

Solution
w.E dy
dx- X =y+l
asy
PI=-lq\,=y+1
En
f-1dy
IF =
=
e

e- Y gin
Solution is given by

x(IF) = fql(/F)dy+c
e eri
xe-Y f(y + I)e-Y dy + c ng.
net
=

i.e., or x +y + 2 = ce Y

Exercise 1(e)

1.13 Solve the Following Differential Equations


I. (I + y)dx = (t~n-Iy - x)dx
[ADS: xetan- I y = tan-Iyetan-Iy --etan- I y + c]
dy
2. cos2x - + Y = tanx
dx
[ADS: Y = ce-tanx - tanx -1]

Downloaded From : www.EasyEngineering.net


Downloaded From : www.EasyEngineering.net

Ordinary Differential Equations of First Order and First Degree 29

dy
3. x - +2y-x2 1og=0
dx

c I .r2
(ADS: y = -? +- x210gx --I
x- 4 16

dy n/
4. dx + ycot x = 4x cosec.x, if y = 0, when x = ~2

n2

ww
5. yeYdx = (y + 2x&)dy
(ADS: ysinx = 2x2 - -
2

w
6. (x + 31) dy .Ea
= y
( ADS : xy~2 =c- e~Y I

dx
syE 3
21 + cy ]
ngi ( ADS : x =

7. (xy -I )
dx+ y
dy
3 + y3 = 0
nee
rin I
. I
I ADS: x
g.n
= ce Y + - + II
Y

8.
dy
-
dx
= x 3 - 2xy if y = 2 when x = I e t
[ADS: 2y - x 2 + 1 = 4 el~~ ]

dy x+ ycosx
10. dx =
I +sinx

x2
[ADS: y(l+sinx) = c - -]
2

Downloaded From : www.EasyEngineering.net


Downloaded From : www.EasyEngineering.net

30 Engineering Mathematics - I

1.13.1 Non-linear Differential Equation of First Order


Ber noulli j. equation:
The differential equation of the form
ely
dx + PY = llyn .... (I)

where p, q are functions ofx alone is said to be a Bernoulli's differential equation.


Dividing (I) throughout by yn

ww dy
y-n_ + py-n+l
dx
=q .... (2)

w.E
Substitutingynt-l(coefficient ofp) "" v
1 dy dv

(2) reduces to
asy
(1-n) - - = -
y" dx dx

En
I dv
- - - +pv=q
(I-n)dl'
gin
dv
- + (1-n)pv = (I-n)q
dx e eri
which is linear in v.
ng.
net
The avove D.E can be solved by using the method given in 1.12.1 example.

1.13.2 Example

Solve dy - ytanx = ysecx


dx
Solution
dy
dx - ytanx = ysecx .... (I)

dy 1
y-2 -d - - tan x = sec x
x Y

Substituting

--
y =v,

Downloaded From : www.EasyEngineering.net


Downloaded From : www.EasyEngineering.net

Ordinary Differential Equations of First Order and First Degree 31

1 dy dv
+--=-
/ dx dx
(1) reduces to
dv
dx + vtan x = sec x

is linear in v.
Here p = tanx, q = secx

ww IF = e
fpd<

w.E = e
flanXd<

= asy
= elogsec x

secx

En
SolutIon of th [) E (I) IS therefore

v(lF) = Jq(IF}dx + c gin


v. secx = Jsecx.secxdx + C e eri
vsecx.= J sec xdx + c
2 ng.
Substituting
net
1
v=--
Y

1
- - sec x = tan x + c
y

i.e., y(tan x + c) + sec x = 0

1.13.3 Example
Solve (3xy + .0) dx - 3i2dy = 0
Solution
dy
3x2 - - 3xy
dx
= .0

Downloaded From : www.EasyEngineering.net


Downloaded From : www.EasyEngineering.net

32 Engineering Mathematics - I

is in Bernoulli's fonn
1 dy _
"7 dx - xy - 3x 2

Substituting
-1
=v
y

D.E reduces to

ww dv I
---v=-
dx xy
1
3x 2
.... (1)

w.E which is linear in v


1 1
Here
asy
p=~, q= 3x 2

En
IF = e
Jpdx

=
gine
J!d\
x

=x
eer
Solution of the D.E (1) is

v(lF) = fq(lF)dx+c ing


.ne
-..!..x= f_l_. xdx + c
y

x 1
3x 2
t
--=-Iogx+c
Y 3

i.e., y(110g x + c) + x = 0
1.13.4 Example
dy
Solve dx + (2xtan- 1y -.x3) (1 + y) = 0

Solution
dy dv
1+ y2 dx = dx

Downloaded From : www.EasyEngineering.net


Downloaded From : www.EasyEngineering.net

Ordinary Differential Equations of First Order and First Degree 33

The given differential equation becomes


dv
- + 2xv=x3 .... (1)
d-c
which is linear in v
Here p = 2x, q = x 3

IF = e J
2xdx
=e
x2

Solution of the D.E (1) is

ww v(lF) = Jq(IF}dx+ c

w Writing

.Ea
x2 = t

xdx = 2dt
1

2
v.e x = {e 2+c
syE r I dt

ngi
nee
r ing
1.14.5 Example
dy .ne
Solve tany dx + tanx

Solution
= cosy cos3x
t
dy
tany d'C + tanx = cosy cos 3x

Dividing by cosy throughout


dy
secy tany dx + secy tanx = cos 3x

Substituting secy = v, we get


dy dv
secy tany dx d'C

Downloaded From : www.EasyEngineering.net


Downloaded From : www.EasyEngineering.net

34 Engineering Mathematics - I

dv
dx + v . tal1.X = cos3x .... (I)

is linear in v
Here p = tal1.X, q = cos3x

IF = f pdt = flallXdy = secx


e e
Solution of the D.E (I) is

ww v(IF) = fq(IF)dx + c

w
3
v. secx = fcos xxsecxdx+c

.Ea secy secx =


2
fcos xdx + c

syEI
secx secy =
I +cos2x
2 +c

x sin2x ngi
secx secy = -+--+c
2 4
nee
1.13.6 Example
rin
dy
Solve -d
.
= (SII1.X -
. cosx
smy) - - g.n
Solution
x cosy
e t
dy. ."
cosy dx + smycosx = SII1.XCOSX

Substituting siny = v

dy dv
cosy-=-
dx dx

The gIven equation reduces to

dv .
dx + vcosx = SII1.XCOSX .. .. (I)

Downloaded From : www.EasyEngineering.net


Downloaded From : www.EasyEngineering.net

Ordinary Differential Equations of First Order and First Degree 35

is linear in v
Here p = COSX, q = SIl1 X cos X

IF
~
= e
fpdr = e feo') '(tl\ =e
~1Il X

Solution of the D.E (I) is

v.e SlIlt =
J. xcosx.e m"'. I + c
SII1 (X

write sinx = t=> cosx dx = £II in the RHS

ww
w.E siny e;lIlt = [te t - el] + c
sinyeSlI1\"
asy = eSItlX [sinx-l] + c

En Exercise - 4(f)

1.14 Solve the Following Differential Equations


gin
I. (ylogx -I )ydx = x(~y

e eri 1
(Ans: -
y
ng.
= 1 + logx + ex )

2.
dy .
- cosx + YSlnx =
dx
r.::::::::
...; ysecx
net
(Ans :2y y 12 -Jsecx = tanx + 2c )

( Ans : ~ = - sin 2x -sinx -~ + ce 2SIIl\" )


y- 2

dy tany
4. ---- = (I+x) ~secy
£Ix l+x
[Ans: siny = (1 + x) (eX + c) )

Downloaded From : www.EasyEngineering.net


Downloaded From : www.EasyEngineering.net

36 Engineering Mathematics - I

dy
5. x - + ylogy = xyeX
dx
I ADs : xlogy = (x - I)e" +c J

dy 2 x3
6. 3-+--y=-)
dx x+ I y

x6 2x 5 X4
I ADs: (x + Iii = -
6
+-
5
+- +C
4

ww 7.
dy
dx
+ ytanx = isecx

w.E dy
I ADs: cos2x = y(c + 2sin x) J

8.
dx X
2 1
Y +xy
asy
En
gin
1.14.1 Exact Differential Equations
eer
ing
Let us consider the differential equation Mdx + Ndy = 0 where M, N are functions
ofx,y.

.ne
If this equation is to be exact, then it must have been derived by directly
differentiating some function ofx,y.
Hence Mdx + Ndy = du, say t
.... (I)
But from differential calculus
au
du = -dr:+-dy
au
.... (2)
ax ay
From (I) and (2) we get

au N= au
M= ax' ay

aM
-=--
a2u
Now and -ax- - -ax-ay-
ay ayax

Downloaded From : www.EasyEngineering.net


Downloaded From : www.EasyEngineering.net

Ordinary Differential Equations of First Order and First Degree 37

aM
-
oN.IS tIle con d··
= - ItlOn ..lor exactness.
oy AX
:. The differential equation Mdx + N~v = 0

.IS exact 'f aM aN


I -=-
oy ax
Then the solution is expressed in the form

+
ww (treatingyas (integrate w.r.t y those

wconstant integrate w.r.t x)


Note: .Ea
terms that are independent of x)

syE
IfN has no term independent ofx then the solution is

1.14.2 Example
fMdx
n
= e

gin
Solve (x + 2y - 3) dy - (2x - Y + \ )dx = 0
e eri
Solution
(x+2y - 3)dy - (2x - Y + I) dx = 0 ng.
M = -(2x - Y+ \)
N = (x + 2y - 3)
aM
n et
-=\
oy
aN
-=\
ax
The given differential equation is exact
The solution is

- f(2x - y + \ )dx + fe x + 2y - 3) = (ry = e


_2X2 2i
- -2- + yx - x + 2 - 3y = e

=> I - x 2 + xy - x - 3y = e

Downloaded From : www.EasyEngineering.net


Downloaded From : www.EasyEngineering.net

38 Engineering Mathematics - I

1.14.3 Example
Solve (x 2+ y.)dr: + 2xy dy = 0
Solution
(x 2 + .V) dx + 2xy dy = 0
M=x2+y. N = 2xy
aM
-=2y
aM
- =2y
av ax
ww The given differential equation is exact
2
}h + f2xydy = c
w.E Solution is f(x + y2

asy
1.14.4 Example

-,:~, /Y
E n1gi
Solve (I + e )dx +
nee
[I.'... ; dy = 0

Solution
r
(I + e'x Y ) dx + e x'Y (
J -;;, ) dy = 0
ing
.ne
M = J + e: Y , N = e,i
t x'y [
J - Yx)
t
aM
-=-
-x ,y'
e'Y
~ i
oM oN
oy ox
The given differential equation is exact

Downloaded From : www.EasyEngineering.net


Downloaded From : www.EasyEngineering.net

Ordinary Differential Equations of First Order and First Degree 39

Solution is

x
X + e Y (y) = c

Exercise - 4(g)

1.15 Solve the Following Differential Equations


ww1. (el' + 1)cosxdx + eJ'sinx~v = 0

w .Ea
2. (vcosx + siny + y)tb' + (sinx + xcosy + x)dy = 0
lADs: (oY + 1)sinx = c I

3. (x 2 - ay)dx = (ax - ;l)(~vsyE I ADs: ysinx + (siny + y)x = c I

ngi ADS: x 3 + ),3 - 3ll.\y = C I


4. (ax + hy + g)d'C + (hx + hy + j)dy"= 0
nee 2
hi':'
I ADs·.
ax
rin
- 2 + (liy + g)x + .!.
+ fy = c I

5. (x 2 + 1- a2 )xdx + (x 2 -.v - P)ydy = 0


g.n
1.15.1 Integrating factors
I ADS : x4 + 2x21
e
- 2a2x2 - l - 2b 2;l = 4c

t
I

If the differential equation Mdx + Ndy = 0 is not exact, it can be made exact by
multiplying it with some function of x, y. Such a function is called an integrating
jactor.
Rule!t' for fillt/illg the illtegrtltillg factors :
1. Integrating factors found by inspection:

Example
Solve x dy- ydx = 0

Solution
xdy- ydx = 0

Downloaded From : www.EasyEngineering.net


Downloaded From : www.EasyEngineering.net

40 Engineering Mathematics - I

Dividing by x2
xdy- ydx
---'-,=-=--
2
= 0
x

On integration

Yx =c

ww First method /0 find an integrating jac/or :


If the differential equation Mdx + Ndy is not exact, but is homogenous and

w.E
Mx + Ny 7:- 0, then the integrating factor is 1
Mx+Ny
. Multiply the differential

asy
equation by IF. The DE becomes exact.
1.15.2 Example
En
Solve
Solution
gin
(x 2y - 2xy2)dx - (x 3 - 3x2y)dy =0

eer
(x 2y - 2xy)dx - (x 3 - 3x2y)dy =0 .... (I)

The differential equation (I) is homogeneous


ing
M
aM
= x 2y- 2xy N
aN
= - (x 3 - 3x2y)
.ne
-
oy
=x2-4xy -
ax =-3x2 + 6xy
t
The DE is not exact and
Mx + Ny = x 2y 7:- 0
) 1
IF = Mx+ Ny - x 2 y2

Multiplying the DE by the integrating factor ~


• x y

(
X2Y -2X
y2
2 2
)d _(X2 -3X3Y )dY
X 2 2
= 0 .... (2)
X Y X Y

Downloaded From : www.EasyEngineering.net


Downloaded From : www.EasyEngineering.net

Ordinary Differential Equations of First Order and First Degree 41

1 2 -x 3
write M = --- N =-+-
I Y X I y2 Y

aMI aNI
--=-=--
then ry ax y2

DE (2) is exact
Solution is

ww
w .Ea
x
- - 210gx + 3logy + c
y

i.e.,
syE
or n x

gin
1.15.3 Second Method to Find the Integrating Factor
e eri
If the differential equation Mdx + Ndy = 0 is not exact and is of the form
j(xy)ydx + g(xy)xdy = 0 ng.
and Mx- Ny *- 0
1
n et
then Mx _ Ny is an integrating factor

1.15.4 Example
Solve (x 2y2 + xy + 1)ydx + (x2y2 - xy + 1)xdy =0
Solution
(x2y2 + xy + 1)ydx + (x2y2 - xy + 1)xdy = 0 .... (1)
M = x2j3 + xy2 + y, N = x2y2 - x2y + x
Mx - Ny = 2x2y2 *- 0
1 1
Hence the IF- =--
- Mx- Ny 2x 2y2

Downloaded From : www.EasyEngineering.net


Downloaded From : www.EasyEngineering.net

42 Engineering Mathematics - I

Multiplying (I) by IF
2
(X y 2 +xy+ 1)+ (X 2
y 2 -xy+ I) = 0
2x 2y2 2x 2y2

2
I{ I 2} I{ II}
- y+-+-
x x 2y
dx+- x - - + -1
2 y xy-
dy=O .... (2)

ww
w.E DE (2) is exact

aJl sy r Jl
Solution of the DE (I) is

~ 'y+~+-!-tx+~ 'x_J.-+_I_ )dY = c


2

1[
E x

I]nIg
x- y 2 y xy
1

in
- xy + log x - - - -logy = c
2 xy 2
eer
1.15.5 Example
Solve (xysin xy + cos xy)ydx + (xysin xy - cos xy)xdy = 0 ing
Solutiou
.ne
(xysin xy + cos xy)dx + (xysinxy - cosxy)xdy = 0
M = (xysin xy + cos x,v)y, N = (xysin xy - cos xy)x t
.... (1)

Mx - Ny = 2xycosxy =F- 0

IF=---
Mx- NY 2xycosxy

Multiplyingthe DE by 2xycosxy

The DE reduces to

~(ytanXY+~)dX+~( xtanxy- ~) dy = c .... (2)

Downloaded From : www.EasyEngineering.net


Downloaded From : www.EasyEngineering.net

Ordinary Differential Equations of First Order and First Degree 43

which is exact (verify)


.. Solution is

1 1 1
-Iogsecxy + -Iogx - - log y = c
2 2 2

ww I.e.,
~
logxsecxy = 2c + logy. Taking 2c as log A
xsecxy = Ay

w .Ea Exercise - 4(h)

I. (x 3;
syE
1.16 Solve the Following Differential Equations
+ x 2y2 + xy + 1)ydx + (x 3; xli - xy + 1)xdy

ngi
- = 0
1
r ADS: xy - - - 210gy = c I

2. (xli + xy + I )ydx + (xli - xy + 1)xdy = 0


nee xy

r
r ADS: ing
xy + logx - logy - -
1
= c]

.ne
xy

[ ADS: logx2 - logy - _I = c ]


xy
t
4. (x4y4 + x 2 i + xy)y dx + (x 4y4 - x 2i + xy)xdy = 0

I ADS : ~ x 2i - _I - logx - logy = c ]


2 xy

1.16.1 Third Method to Find the Integrating Factor


aM aN

If Mdx + Ndy = 0 is not exact and _a-=-~_ _ax_ is a function of x alone say fix),
N

then the IF =e
J
j (x)dr

Downloaded From : www.EasyEngineering.net


Downloaded From : www.EasyEngineering.net

44 Engineering Mathematics - I

1.16.2 Example
Solve (x 2 + y + 6x )dx + yxdy = 0
Solution
(x 2 + Y + 6x)dx + yxdy = 0 ..... (1 )

M = x2 + Y + 6x N = yx
aM aN
ay= 3y, fu = y2

ww aM
ay-ili
aN
3y2 - y2 2

w.E N
is a function of x alone
2
y x X

asy
IF = e f~
2tJx
= x2

En
Multiplying the given DE by x2
x2(x2 + y + 6x)dx + yi3dy = 0 gin ..... (2)

-
aMI
ay =3x2y e eri
ng.
DE (2) is exact
net
Solutionis
4 2 2 2
f(x + x / + 6x 3 }tx + fy x dY = c

1.16.3 Example
Solve (x2 + y) dx - 2xydy = 0
Solution
(x2 + y)dx - 2xydy = 0 ..... (1 )

Downloaded From : www.EasyEngineering.net


Downloaded From : www.EasyEngineering.net

Ordinary Differential Equations of First Order and First Degree 45

M = .xl + .0, N = - 2xy


aM aN
ay = 2y, ox = -2y

aM aN
ax ax 2y+2y -2
N -2xy x

is a function of x alone

f -~dx
ww IF = e x
1
=-
x2

w.E
MuItiplyingthe DE by
1
IF= -
asy
x2

[ l) E
The given DE reduces to

n
1+-
2
x
2y
dx,.--dy=O
x gin ..... (2)

aM, _ 2y
-a --2'
oN,
-=-
2y e eri
y X

(2) is exact ng.


Solution of (1) is
y2 2y
net
Jl+-dx+
x
J--dy=c
2
x
y2
x- - =c
x

·1.16.4 Fourth Method to Find the Integrating Factor

If the differential equation Mdx + Ndy = 0 is not exact and


aN aM
ax M By
1 is a
[

function of y alone, say fly)

Downloaded From : www.EasyEngineering.net


Downloaded From : www.EasyEngineering.net

46 Engineering Mathematics - I

Then IF =
e
f t(y)<ly

1.16.5 Example
Solve (xy + y) dx + 2(.x2.0 + x + y4)dy = 0
Solution
(xy + y)dx + 2(.x2.0 + x + y4)dy = 0
M = .xy3 + y N = 2(.x2.0 + x + y4)

ww oM
--- =
8y
3xy + 1
oN
-
ox =4xy2+2

w.E -a;oN -8;OM] (4xy2 + 2)-(3xy2 + I)


(as
=

yE M xi+y

n gin
is a function ofy alone
e y

eri
f~d)' ng.
F=e Y =y

Multiplying the differential equation by I.F =Y


net
(xy4 + .0) dx + 2(.x2Y + xy + yS)dy = 0 ..... ( I)

oMI .,3 oNI .,3


--=4xy +2y - =4xy +2y
8y ox
The differential equation (I) is exact

Solution is f{xl + i }:Lx + 2 f(x 2 i + xy + yS) dy = 0

Downloaded From : www.EasyEngineering.net


Downloaded From : www.EasyEngineering.net

Ordinary Differential Equations of First Order and First Degree 47

1.16.5 Example
Solve (3x 2y4 + 2xy)dx + (2x 3_Y' - x2)(~V = 0

Solution

aN _ aM _ (6x /
2 -2x)-(12x 2y 3 +2xL 6x 2/ -4x -_3.
ww ax ay - l
3x 2 oJ- 2x - y{3x 2/ + 2x ) - Y

w.E - is a function of y alone

J~ dy I

asy IF = e Y =-?
Y

E
Multiplying the DE by
ngi
~, the DE reduces to
y

(3x 2y 2+-y
2X) dx+ (3 nee
x2) dy= 0
2x y-7
r ing
aNI =6xy- 2x
ax i .ne
:. DE (I) is exact

Solution of the DE is
t

2
x3 2 2 x
3-y+--=c
3 y 2

Downloaded From : www.EasyEngineering.net


Downloaded From : www.EasyEngineering.net

48 Engineering Mathematics - I

Exercise - 4(i)

1.17 Solve the Following Differential Equations


I. (x + y3 + I )dx + ydy = 0
2 3x
X e
2 x 3x 2 e 3x 3 e 3x
ADs: (ADs: - - - - e +--+(y + l)-=e I
3 9 933

2. (x 2 + Y + 2x)dx + 2ydy = 0

ww 3. 2xydy - (x 2 + Y + I)dr = 0

w .Ea
I ADs: y - x 2 + I = ex I

syE
5. (y4 + 2y)dx + (xy3 + 2y4 - 4x)dy ngi
= 0

nee 2x
I ADs: xy + -) + Y.1 = e ]

r ing
y

.ne
1.17.1 Fifth Method for Finding Integrating Factor t
If the differential equation
Mdx + Ntry= 0 .... (l)
is not exact and is of the form
xliyb(mydx + nxdy) + Xf y(pydx + qxdy) = 0
when a, b, r, s, m, n, p, q are constants
Then the IF = xlII
where h k are constants such that the equation (I) after multiplying with IF
becomes exact.

Downloaded From : www.EasyEngineering.net


Downloaded From : www.EasyEngineering.net

Ordinary Differential Equations of First Order and First Degree 49

1.17.2 Example
Solve xy\ydx + 2xdy) + 3ydx + 5xdy) = 0

Solution
xy3(ydx + 2xdy) + (3ydx + 5xdy) = 0 ..... (1 )

The IF = x"1
Multiplying (I) by IF = xhl it must become exact
(xh+1 1+1 + 3x" 1+1)dx + (2x ilj .? + 1+3 + 5xil f-/ I){~Y = 0 ..... (2)

wwis exact if

w.E aM
- ay = (4 + k)xH1 .1+3 + 3x\k + 1)1
.
asy
aN
-
ax =
En
2(h + 2) xh+I /+ 3+5(h + 1)x"1

For DE (2) to be exact gin


aM aN eer
ax ax ing
.ne
Comparing the coefficients on both sides
2h - k = 0 and Sh - 3k = -2
t
Solving h = 2, k= 4

Substituting h = 2, k = 4 in (I) DE (I) reduces to

(x 3y 8 + 3x2yS) dx + (2x4y1 + 5x3y 4)dy = 0 which is exact

.. Solution is

Downloaded From : www.EasyEngineering.net


Downloaded From : www.EasyEngineering.net

50 Engineering Mathematics - I

1.17.3 Example

Solve (3x + 2y)ydx + 2x(2x + 3y)dy = 0

Solution
(3x + 2y)ydx + 2x(2x + 3y)dy = 0 .... (1)

IF = 0/,
MUltiplying (1) by IF

ww (0/,+2 + 30+2 /,+I)dx + (2x h+3/, - 0+ 1/,+I)dy = 0 .... (2)

w.E oM
-
ox = 3(k + 2)0/,+1 + 2(k + 1) 0+2yK

oN
-
ox asy = 2(h + 3) x h+2/, ---(h + 1)0/,+ 1

En
The DE(1) is to be exact

oM ON gin
ox ox e e ri
2(k + 1) = 2(h + 3) => h = - % -/i
k=
ng.
Comparing the coefficients
k + 2 = - (h + J)
net
and 2(k+ 1) = 2(h + 3) => h = -,% k= -/i
Substituting h, k values in (2)

(
-5/
X /2 y/2
3/
+ 2x -1/
I II) ( II -II
12 yl2 dx + 2X 72 Y 12
-3/ II) dy = 0
_ X /2 y72 .... (3)

(3) is an exact DE
2 -2/ 3/ II II
Solutionis --x 13 yl2 + 4X 72 y72 = c
3

Downloaded From : www.EasyEngineering.net


Downloaded From : www.EasyEngineering.net

Ordinary Differential Equations of First Order and First Degree 51

Exercise - 4(j)
I Solve the following differential equations

I. x(3ydx + 2xdy) + 8y4(vdr + 3xdy = 0

4 2 10 +1
3x 3+l y 3 X 1 7
I Ans: ---y 3 =c]

ww -
4
3
+1
10
- - +I
3

3.
w.E(y2 + 2x2y)dx + (2x 3 - xy)dy = 0
I I 2 3 3

asy ( Ans : 4x 2 y 2 __ X
3
2Y 2 = ex ]

4. (2.~y - 3y4)dx + (3x 3


En
+ 2xy3)dy = 0

gin
- 36, 24 _10 ' 15
(Ans: 5x 13 y 13_12x I]y I] =c]

cooling: e
1.17.4 Applications to geometry, law of natural growth and Newton's law of
eri
When some action is applied on a quantity the changes and the action affects all
the parts equally. The rate of change depends on the original quantity. ng.
For example the total population 'p' of a country increases with respect to time

't', say, then its rate of change with time is ~ . Under ideal condrtions the rate of
net
change of the population will be proportional to the total population at any given time
and is called the law of natural growth.
The growth of the population satisfies the differential equation
dp
- =kp
dt
where k > 0 is a constant
Solving the differential equation we have
p = cekl
dx
The rate at which a quantity x is decreasing is given by dt and this is negative.

Downloaded From : www.EasyEngineering.net


Downloaded From : www.EasyEngineering.net

52 Engineering Mathematics - I

This rate of decrease or decay is found to be proportional to x itself


dx
- =-kx (where k> 0 is a constant)
dt
is the law of decomposition
Solving the DE x = ce-kl
Newton s law of cooling:
Newton's law of cooling states that the rate of decrease of the temperature of
a body is proportional to the difference of the temperature 9fthe body and that of

ww its surrounding medium.


Let 0 be the temperature of the body at time 't' and 00 the temperature of its

w.E
surrounding medium.
:. Difference between the temperatures = 0 - 8 0 (0 > 00) and the rate of decrease

asy dO
of the temperature of the body is - dt' since the body is cooling oc.

En de

gin
Using Newton's law ofcooling - -
dt
oc(O - 0 )
0

or
-dO
dt = k(O -
eer
00 ) where k> 0 is a constant of proportionality

-
dO
= -k (0 - 0 ) ing
dt

f~=-k
0

.ne
0-0
fdt+c
0

10g(0 - 00) = - kt + c or 0=0 o + Ce-kl


t
1.17.4 Example
The mass of crystalline deposit increases at a rate which"js proportional to its
mass at that time. The deposit has started around a crystal seed of 5 grams. Find an
expression of its mass at time 't'. Ifin 30 minutes the mass of the deposit increases
by'l' gram, what will be the mass of the deposit after 10 hours.
Solution
Let m be the mass of crystalline at time 't' then by law of growth

dm=k' dm
-=dt
dt ' km

Downloaded From : www.EasyEngineering.net


Downloaded From : www.EasyEngineering.net

Ordinary Differential Equations of First Order and First Degree 53

on integration
log m = kt + c .... (I)

initially when t = 0, m = 5
(1) => log5 = 0 - c
c=-log5
SubstitutilllJ; c value in (1)
log m = kt-log5

ww I.e., kt= 10g( ~) .... (2)

w .Ea
When t = 30 minutes mass deposit increases by , I ' gram
m = 5 + 1 = 6 grams
Substituting syE
t=
1
"2 n
(hours), m =6
gin
e eri
ng.
Substituting in (2)
n et
log(1Y t= 109; .... (3)

Now to find the mass after 10 hours (i.e t = 10)


from (3) we get

IOg(H IO~ log 7~ log (7) ~ IOg(%)"


x

6)20
m = 5"5 ( grams

Downloaded From : www.EasyEngineering.net


Downloaded From : www.EasyEngineering.net

54 Engineering Mathematics - I

1.17.5 Example
The rate at whi~h a certain substance decomposes in a certain solution at any
instant is proportional to the amount of it present in the solution at that instant.
Initially, there are 27 grams and three hours later, it is found that 8 grams are lett.
How much substance will be left after one more hour.
Solution
If m grams is the amount of the substance left in the solution at time 't', then the rate

at which it decomposes is dm , which is proportional to m.

ww dl
By law of decay

w.E dm
dt = - km (k> 0)

asf~y =-k fdt+c

En
gin
logm = -kt+ c
Initially when t = 0, m = 27
.... (I)

From (1) we have


eer
=>
log27 = - k.O +c
ing
c= log27
Substitution of'c' in (I) gives .ne
log m = - kt + log27

10g(;; )= - kt
t
.... (2)

It is given that m = 8 when t = 3


.. From (2)
8
log (2 ) = - k, 3
7

8
-k= log ( 27 )X
2
-k=log-
3

Downloaded From : www.EasyEngineering.net


Downloaded From : www.EasyEngineering.net

Ordinary Differential Equations of First Order and First Degree 55

Then (2) becomes

10g~ =IOg(%} ... (3)

when t = 4

4
III 2

ww log 27 = log ( "3 )

w.E m=27 x (%r grams

asy
16
3
m=
En grams.

1.17.6 Example
gin
The number x of bacteria in a culture grow at a rate proportional to x. The value

e eri
ofx was initially 50 and increased to 150 in one hour what will be the value ofx after

1
12" hour.
ng.
Solution
dx
-=/0:
dt
net
dx
- =kdt
x

logx = kt + c .... (I)

c is the constant of integration


when t= O,x= 50
.. log 50 = k.O + c
or c= log50
(1) => logx = kt + log50

Downloaded From : www.EasyEngineering.net


Downloaded From : www.EasyEngineering.net

56 Engineering Mathematics - I

x
log- =kl
50
x = 150, when 1= 1
150
.. log 50 = k.I

or k= log3
(2) then gives

ww log (
x
5O) = flog3
w.E we want to find x when I = "2
3

asy
En
gin
X
50 = (3)2
3

eer3

1.17.7 Example
X = 50 (3)2 grams
ing
.ne
The rate of cooling of a body is proportional to the difference between the temperature
of the body and the surrounding air. If the air temperature is 20°C and the body
cools for 20 minutes from 140°C to 80°C, find when the temperature will be 35°C.
Solution
If 8 is the temperature of the body at time '1' then from Newton's law of cooling
t
-d8 de
-a(8-20) ~ - =-k(8-20)
dl dl

f~
8-20
= - k rldl + c
J'
log(8 -20) = - kt + c .... (I)

Initially when
t = 0, 8 = 140
log(8 - 20) = 0 + c,

Downloaded From : www.EasyEngineering.net


Downloaded From : www.EasyEngineering.net

Ordinary Differential Equations of First Order and First Degree 57

c = log( 120), (I) reduces to


log(140 - 20) = - kt + log 120
or +kt = log( 120) - 10g(S - 20) .... (2)
It is given that q = 80 when t = 20 minutes
k. 20= logI20-log(80-20)

I (120)
k= 20 log 60

ww k= -log2
20

w.E Substituting in (2)


1

asy
(2 0 IOg2)t= logI20-log(S-20)

En
It is required to find t when
0= 35°c

gin 0

t=
logI20-log(35 - 20)
I eer
20 10g( \25 )

-log2
20
log2
ing
10g(8) 2010g23 60 10g2 .
t= 20 - - = - - = - - =60mmutes .ne
log2 log2 log2

Exercise - 4(k)
t
I. In a certain reaction, the rate of conversion of a substance at time "t' is proportional
to the quantity of the substance still untransformed at that instant. At the end of one
hour 60 grams while at the end offour hours 21 grams remain. How many grams of
the first substance was there initially?
( ADs: 85 grams approximately I
2. The rate of growth of a bacteria is proportional to the number present. If initially
there were 100 bacteria and the amount doubles in '1' hour, how many bacteria will
1
be there after 2"2 hours.

(ADs: 564 I

Downloaded From : www.EasyEngineering.net


Downloaded From : www.EasyEngineering.net

58 Engineering Mathematics - I

3. Under certain conditions cane sugar in water is converted into dextrose at a rate
which is proportional to the amount unconverted at any time. 1f75 grams was there
at time t = 0.0 and 8 grams are converted during the first 30 minutes find the amount
I
converted in 12 hour.

[ADS: 21.5 gms]

4. The rate of cooling of a body is proportional to the difference between the temperature
of the body and the surrounding air. If the surrounding air is kept at 30°c and the

ww body cools from 80°c to 60°c in 20 minutes. Find the temperature ofthe body after
40 minutes.

w.E [ADS: 48°c J

5. If the air is maintained at 30°c and the temperature of the body cools from 80°c to

asy
60°c in 20 minutes. Find the temperature of the body after 40 minutes.

En [ ADS: 48°c]

gin
6. The rate at which a heated body cools in air is proportional to the difference between
the temperature of the body and that of the surrounding air. A body originally at 80°

e
cools down to 60°c in 20 minutes the temperature of the air being 40°c what will be

eri
the temperature of the body after 40 minutes from the original temperature.

ng. [ADS: 50°c J

net
7. The rate at which bacteria multiply is proportional to the instantaneous number
present. If the original number doubles in 2 hours in how many hours will it triple.

[ ADS: 210g3 J
log2

8. Water at temperature 100°C cools in 10 minutes to 80°C in a room of temperature


25°C. Find the temperature of water after 20 minutes.
[ ADS: 65.5°c]

9. A cup of coffee at temperature 100°C is placed in a room whose temperature is


15°C and it cools to 60°C in 5 minutes, find its temperature after a further interval of
5 minutes.
[ADS: 38.8°c ]

Downloaded From : www.EasyEngineering.net


Downloaded From : www.EasyEngineering.net

Ordinary Differential Equations of First Order and First Degree 59

Orthogonal Trajectories

1.18.0 Definition (1)


A trajectory of a family of curves is a curve cutting all th.e members of the system
according to some law. For example a curve cutting a family of curves at a constant
angle is a trajectory.
Definition (2)
If a curve cuts every member of a given family of curves at right angles, it is called
an orthogonal Trajectory. The orthogonal trajectories of a given family of curves _

ww themselves form a family of Curves. If the two families of curves are such that
each member of either family cuts each member of the other family at right angles

w then the members of one family are known as the orthogonal trajectories of the
other.

.Ea
In two dimensional problems in the flow of heat, the curves along which the heat

syE
flow takes place and the isothermal curves or loci of points at the same temperature
are orthogonal trajectories. In hydrodynamics, the flow of water from a lake into
narrow channel produces a family of streamlines which are orthogonal trajectories

ngi
to the curves of equal Velocity Potential. In the flow of electricity in thin
conducting sheets, the paths along which the current flows are the orthogonal

nee
trajectories of the equipotential curves and vice - versa.

1.18.1 Orthogonal Trajectories: Cartesian Coordinates


Let f(x,y,c)=O
r ing ..... (1)

.ne
be a family of curves, where c is a parameter. We can form a first order differential
equation by eliminating 'c' from (1)

i.e., F(X,y,:)=o
\
t
..... (2)

is the differential equation whose general solution is (1 )


If the two curves are orthogonal (curves intersecting at right angles) the product of
the slopes of the tangents at their point of intersection must be equal to -I.

Suppose (x, y) is the point of intersection of the curve (I) and its orthogonal
trajectory .

Downloaded From : www.EasyEngineering.net


Downloaded From : www.EasyEngineering.net

60 Engineering Mathematics - I

At this point slope of the tangent to the curve (I) is dy and -dx is the slope of
dx dy.
the tangent to the orthogonal trajectory. Therefore on replacing dy by - dx in (2)
dx dy
, the equation thus obtained is the differential equation of the family of orthogonal
trajectories of the family(l) .

. . ¢( xJI, - :)=0 ... (3)

is the differential equation of the system of orthogonal trajectories, and its solution

ww
is the fami Iy of orthogonal trajectories of (I) .

w
1.18.2 Orthogonal Trajectories - Polar Coordinates
Suppose
.Ea
f(r,O,c) = 0 ..... (1)

syEF(r,o, :~ )
is the family of curves where c is the arbitrary constant.

=0
We can form a differential equation
ngi ..... (2)

nee
of the family (I), after elimination of the constant 'c'.
Let ¢ be the angle between the radius vector and the tangent at any point (r, 0) .
on a member of the family of curves.
rin
Then
dO
tan¢=r-
dr g.n .... (3)

Let ¢I be the angle between the radius vector and the tangent at any point (lj, ~ )
on the trajectory.
et
Then tan;/,
dBI
= r,·1 _
Y'I dr, ..... (4)
I

At a point of intersection of the given curve and the orthogonal trajectory

lj=r, 01 =0 Hence tan¢1 = -cot¢


From (3) and (4),
dOl 1 dr 1 dr dOl
lj dr = --; dO i.e., -; dO = -lj dlj
l

Downloaded From : www.EasyEngineering.net


Downloaded From : www.EasyEngineering.net

Ordinary Differential Equations of First Order and First Degree 61

Hence the differential equation of the orthogonal trajectory is obtained by


·. dB fi 1 dr. J dB fi dr
Sll bstltutmg - r - or - - , I.e., -r- - or-
dr r dB dr dB

Therefore the differential equation F (r, B, _r2 ~~) = 0 ..... (5)

gives the differential equation of orthogonal trajectory of the family of cllrves(l)

Solution of (5) is the orthogonal trajectory of the family of curves.

ww
1.18.3 Example

w
Find the orthogonal trajectory of the family of curves a/
parameter.
.Ea
= x 3 , where a is variable

Solution:
syE
a/ = x
ngi
3
Given family of curves is ..... ( I)

Differentiating (I) w.r.t. 'x'; 2ay dy


dx
= 3x 2
nee ..... (2)

Eliminating 'a' from (1) and (2)

. (X3) dy _
r ing
.. 2 / .y dx -3x
2

.ne
2x
dy
dx
=3y ..... (3) t
is the differential equation of the family (I). Now replacing : by - : in(3),

gives the differential equation of the orthogonal trajectory to (I) as -2x dx = 3y


dy
23 2
Integrating both sides -2~ = L + c . Therefore
2 2
2X2 + 3/ = c is the equation of orthogonal trajectory of (I)

Downloaded From : www.EasyEngineering.net


Downloaded From : www.EasyEngineering.net

62 Engineering Mathematics - I

1.18.4 Example
Find the orthogonal trajectory of the family of parabolas y2 = 4ax where 'a' is the
parameter.
Solution:
y2 =4ax ..... (1)

differentiating (I) w.r.t. 'x'; y dy = 2a ..... (2)


dx

ww Eliminating 'a' from (I) and (2). y2 = 2X(Y:)

w.E .. y=2x
dy
dx
..... (3)

asy
is the differential equation of the family (1)

dy by _ dx in(3)
Replacing
dx dy
En
Y=2X( -: J gin ..... (4)

eer
is the differential equation of the orthogonal trajectory to (I). Integrating (4)

ing
fydy = -2 fxdx + c; 2X2 + y2 = c is the orthogonal trajectory of (I)
1.18.5 Example .ne
Find the orthogonal trajectories of ~2 + (
a
2

a +A
2
= 1 whereA is the parameter. t
Solution:
X2 y2
-+ =1 ..... (1)
a2 a2 +A
Differentiating (I) w.r.t. 'x'; 2x2 + 22y dy =0
a a +A dx

..... (2)

Downloaded From : www.EasyEngineering.net


Downloaded From : www.EasyEngineering.net

Ordinary Differential Equations of First Order and First Degree 63

)
x- xy
Eliminating A from (I) and (2) -2- - - = 1
a a 2 --dy
dx
2
:. ( x - a 2 ) : = xy ..... (3) ---

is the differential equation of the family (I)

··
S ub stltutmg -dx
- fOor -dy.111 (3) ; (2
X -a 2)( -d- ) = xy
Y .
I.e,
dy dx dx

ww dY~-( x' x a
2
y )dx
w.E
is the differential equation of the orthogonal trajectory to (1)
Y
2
2 X
2
Integrating we get

x +l
2
= 2a 2 asy-=a logx---
2 2+c

En
log x + C is the orthogonal trajectory of the family of curves( 1)

1.18.6 Example
gin
e
Find the orthogonal trajectories of the family of coaxial circles
x + l + 2gx + C = 0 where g is the parameter.
2

eri
Solution:
x 2 + y2 + 2gx + C = 0
ng.
Given

Differentiating (I) w.r.t. 'x' ;


dy -
2x+2y-+2g+0=O
dx
net ..... (1 )

..... (2)

2
Eliminating 'g' from (I) and (2) x + y2 - 2x ( x + y : ) + c = 0

y2 _ x 2 _ 2xy dy + c = 0 ..... (3)


dx
is the differential equation of the family of (1 )

Substituting· - dx for dy in (3) we get,


dy dx
dx
y2 _ x 2 + 2xy - + C = 0 ..... (4)
dy

Downloaded From : www.EasyEngineering.net


Downloaded From : www.EasyEngineering.net

64 Engineering Mathematics - I

Which is the differential equation of orthogonal traject{)ry. SimplifYing


dx 1 2 c+ /
2x---x = - - - ..... (5)
dy y y
is a linear equation of the Bernoulli's form (non - linear differential equation of
first order and first degree)

Substituting x 2 = t, 2x dx = dt in (5)
dy dy
dt t (c+ / )
---= ..... (6)

ww is linear in t.
dy y y

w.E I I·
--dy
:.I.F=e Y =-
1
Y
General solution of(6) is
af sy
En
C
t.! = - + y2 .!dy+k
y y y
t
-=-y+-+k.
C
gin
y y
x 2 + Y - ky - C =0 e eri
( since t = x 2 )
is the equation of the orthogonal trajectory of (I)
ng.
1.18.7 Example
Find the equation of the system of orthogonal trajectories of the parabolas
net
r = 2a , where 'a' is the parameter.
l+cosO
Solution:
2a = r (1 + cos 0) ..... (1)

~ a = rcos 0/2
2
2a = 2r cos 2 0/2
log a = logr + 21ogcosO/2
.Differentiating w.r .to '0 '

o=! dr + 2 (-sinO/2).(l/2)=O i.e,


r dO cosO/2

Downloaded From : www.EasyEngineering.net


Downloaded From : www.EasyEngineering.net

Ordinary Differential Equations of First Order and First Degree 65

1 dr
---tanB/2 = 0 ..... (2)
r dB
2 £10 elr.
Substituting -r - for - m (2) we get
dr dB

1(
- -r-?dB)
r
-
dr
-tanB/ 2 =0

dr

ww -
r
= -cotB/2dB ..... (3)

w.E
is the differential equation of the 0I1hogonai trajectory .
Integrating (3)
log r
a
log(~) syE
= -2 log sin B/2 + log c
2
= logsin B/2
ngi
c .
n 2
-=sm B 2=--'--------'-
r 2
/ ( 1- cos B)

eer
:. 2c = r (1 - cos D) is the equation of orthogonal trajectory of the fami Iy of ( I )
ing
1.18.8 Example
.ne
Find the orthogonal trajectory of
Solution:
rill = alii cosmB where 'a' is the parameter.
t
Given rill = a cosmB
ln
..... (1)
mlogr = mloga+ log cos mB

Differentiating w.r.to 'B' ;


m -dr=
- 1 ( -msmmB
. )
r dB cosmB

~ dr = -tan(mB) ..... (2)


r dB
is the differential equation of the family of curves (I)
dr 2 dB .
Replacing - by -r - m(2)
dB d,.

Downloaded From : www.EasyEngineering.net


Downloaded From : www.EasyEngineering.net

66 Engineering Mathematics - I

~.(_r2 da) = -tan (rna)


r dr
dr = cot (rna)da ..... (3)
r
is the differential equation of the orthogonal trajectory.
Integrating (3)

'log r = J.-Iogsin (rna) + loge


ww rn
log rln = log e sin (rna) ; rill
lll
= e sin (rna) is the orthogonal trajectory of the
lll

w
family (I)
.Ea
syE
Exercise 4 (s)

1.
variable parameter.
y=ax 2
n
Find the orthogonal trajectories of the following family of curves, where a

gin 2
[ADS: x +21 =e]
IS a

(i)

(ii) xy=a 2 e eri 2


[ADS: x - y-
?
=e ]
(iii) x 2 -xy+ l =a 2
ng.
[ADs: (x-y)=e(x+y)2]

(iv) 2X2 + y2 = kx [ ADs: x


2
n= - y2 log ( ;
et)]

(v) x 2 _ y2 = a2 [ADs: xy = e]

(vi) X2/3 + y2/3 = a2/3 [ADs: y4/3 _ X4/3 = e 4/3}

2. Show that the system of confocal and coaxial Parabolas l = 4a (x + a) is self


orthogonal.
3. 3
Find the orthogonal trajectories of the curves 3xy = x 3 - a , a being the parameter.

4. Prove that orthogonal trajectories of a system of circles x 2 + y2 - ay = 0 is


x 2 + y2 -bx =0

Downloaded From : www.EasyEngineering.net


Downloaded From : www.EasyEngineering.net

Ordinary Differential Equations of First Order and First Degree 67

5. Find the orthogonal trajectories of the following family of curves.


-
0'
--

(i) r=aB [ADS: r = ce 2 I


(ii) r =e aO [ ADS: (log)2 + B2 = c I
(iii) rn sinnB = a" [ADS: rll eosnB = e" I
2
(iv) reosB = asin B [ ADS: r = e (3 + eos 2B) ]

(v) r = a(l +eosB) [ADS: r = c(l-eosB) I

ww (vi) r = aeosB
2
(vii) r2 = a eos2B
[ADS: r = esinO I
= C- Sin 20 )
w [ADS: r ? ?

.Ea
syE
ngi
nee
rin
g.n
e t

Downloaded From : www.EasyEngineering.net


Downloaded From : www.EasyEngineering.net

ww
w.E
a s yE
"This page is Intentionally Left Blank"
ngi
nee
rin
g.n
e t

Downloaded From : www.EasyEngineering.net


Downloaded From : www.EasyEngineering.net

2w
ww
.Ea
Linear Differential Equations with
syE
Constant Coefficients and Laplace Transforms

2.1.1 Let
n gin
e
(i) the differential operator" ~" be donoted by 'D'
dx eri
ng.
(ii) PI' P2' P3' ............. Pn be either functions of x or constants

(L.D.E) of order n is given by


D nY + PI Dn- I Y + P2 Dn- 2Y + ................. PrJ! -- R
n
(iii) R be a function of x then the general form of a linear differential equation

et
.... (I)
or simply (Dn + ppn-I + ............ +Pn)Y = 0, If PI' P2' ..................., Pn are constants
then (I) is called a L D E with constant coefficients.
Denoting the differential operator (Dn + PI Dn - I + .............. + Pn) by f (D),
(I ) can be written as
f(D)y = R .... (2)

2.1.2 Consl·d er the D.E, dy


dx + Py = 0

Separating the variables dy = - P(x) dx


y

Downloaded From : www.EasyEngineering.net


Downloaded From : www.EasyEngineering.net

70 Engineering Mathematics - I

Integration yields logy= fp(x)dx => y =Ce -fp(x)dx + logc ..... (3)

Solution isy= Ce-


fpdx
Suppose p is a constant = -m, say, then solution is y = Cernx
(i.e.) the solution of (D - m)y = 0 is y = Ce rnx .... (4)
2.1.3 In this chapter the attention is mostly confined to L.D.E.S with constant coefficients.
If R = 0 then equation (2) becomes

ww f(O)y =

We shall take, here afterwards, f(m)


0
= 0 as the auxiliary equation, (A.E).

w.E
2.1.4 Consider a second order L.O.E
(D2 + a\ D + ~)y =0 ..... (6)
A.E. is m 2 + aim + a2 asy= o.

En
Let m\, m 2 be the roots of this equation.
Now four cases arise.
(i) m\, m2 are real and distinct gin
(ii) m\, m 2 are real and equal e eri
(iii) m\, m2 are complex and distinct
(iv) m\, m2 are complex and equal ng.
2.1.5 Case i : (6) can be written as
[0 2 - (m\ + m 2 ) D + m\m 2 ] y = 0 or (D - m2 ) (0 - m\) y = 0
net
.... (7) -
Call (0 - m l ) y = y
(7) now becomes (0 - m 2) Y = 0

From (4) it follows that Y = Ce D12X

From (7) (D - m\)y = Ce D12X


I.F. = e-rn\X

Solution is y . e-m\x = C fe(m,-m')x dx + C\ .... (8)

C C
y.e-rn\x= e(rnrrn\)x dx + CI' Call as C 2 .
m 2 -m! m 2 -m!

Downloaded From : www.EasyEngineering.net


Downloaded From : www.EasyEngineering.net

Linear Differential Equations with Constant Coefficients ... 71

(i.e.,) y = C2 e-1Il 2x + C, elll,x where C, and C 2 are arbitary constants


Similarly ifm" m2, .......... I11n are real and dinstinct roots off(l11) = 0 then
y = C, e'n,~ -I- C 2 el11 2x -I- ....•....•..•..•.. + C n elllnx .... (I)

(I) is the solution of f(O)y = 0 where C" C 2, ........... , C n are arbitrary constants.

2.1.6 (Case ii) : m, = m2,


From (8) it follows that the solution of(6) is given by

ww :. y = (Cx + C,) elll,x

w.ESimilarly if m, is repeated say 'r' times then solution of(D - m,Y y


y = (C, + C 2x + C 3x2 + .............................. + c;-r-') elll,x
= 0 is
..... (II)

asy
(II) is the solution corresponding to a root m, repeated r times

En
2.1.7 Case (iii) : Ill" m2 are complex, say (a ± if3)
Then solution of(6) is given by
y = A eta ± I~)X + B e(a- gin
IfJJX where A and B are arbitrary constants
(i.e.,) y =
eer
Ae ax . e Vlx + Beax.e-1jJx = eax [A(cosjJx + isinjJx) + B (cosjJx' - isinjJx)]
= eax [C, cos/lr + C 2 sin,lk]
ing ..... (III)
where A + B = C, and i(A - B) = C 2
(III) gives the solution corresponding to two complex conjugate roots (a± ifJ) .ne
2.1.8 Case (iv) : If (a ± ifJ) are repeated say's' times then the corresponding solution
(follows from case (ii) and (iii) is given by
t
y = ~[(C, + C 2x + C 3x2 + ........................ + CsXS- ' ) ] cosf3x
+ (d, + d 2x + d 3x2 + ....................... + dsxS- ' ) + sinf3x] .... (IV)

Example 2.1.9

d2 y dy
Solve - 2 +5-+6y=O
dx dx
Sol. A.E is m2 + 5m + 6 => (m + 2) (m + 3) = 0
:.m=-2,m=-3.
Solution is y = Ae-2x -I- B e-3x

Downloaded From : www.EasyEngineering.net


Downloaded From : www.EasyEngineering.net

72 Engineering Mathematics - I

Example 2.1.10
d2y dy
Solve dx 2 -11 dx +30y = 0

Sol. A.E is m2 - 11 m + 30 => (m - 5) (m - 6) = 0


Solution is y = Ae 5x + B e6x
Example 2.1.11
d2y dy
Solve2 dx 2 +5 dx -12y=0

ww
Sol. A.E is 2m 2 + 5m - 12 => (2m - 3)(m + 4) = 0

w.E
m =3/2, m=-4

asy
3
:. Solution is y = C 1 e2x + c 2 e--4x
Example 2.1.12
d 3y d2y En
Solve -+4--6y=0
dx3 dx 2
gin
Sol. m3 + 4m2 + m - 6 = 0
(m - I) (m + 2) (m + 3) = 0 eer
:. Solution is y = cl~ + c 2 e-2x + C 3 e-3x ing
Exercise - 2(a)
.ne
Solve the following differences:

1.
d 2y dy
-+2--3y=0
2
Ans : y = A~ + Be-3x
t
dx dx
2
d y _ 3y = 0 Ans: y = A~ + Be-X
2.
dx 2
d 3y d2y dy
3. -+2--5--6y=0
3 2
Ans : y = Ae-X + Be2x + Ce-3x
dx dx dx
d2 y dy
4. 9 -+18--16y=0
2
dx dx

d2y dy
5. -+3-+2y=O
2
Ans : y = Ae-X + Be-2x
dx dx

Downloaded From : www.EasyEngineering.net


Downloaded From : www.EasyEngineering.net

Linear Differential Equations with Constant Coefficients ... 73

Solved Examples
Example 2.1.13

dZy dy
Solve -z +6-+9y=0
dx dX'

Sol. A.E is mZ + 6m + 9 = °
i.e. (m + 3)z = 0, m = -3,-3
:. Solution is y = e-3x (A + Bx)

ww
Example 2.1.14

w.E 3 2
d3
Solve~+~=O
dx
dZ
dx
Sol. A.E is m + mZ =
3
°a Z
°
:. m=O,O,-1 syE
=> m (m + I) =

ngi
Solution is y = (A + Bx)e oX + C e-x = (A + Bx) + Ce-X
Example 2.1.15

Solve
d4y
- 4 + 18-
1
dZy
+ 81y=0
n eer
dx dx~

Sol. A.E. is m4 -18m 2 + 81 = ° ing


:. (mz - 9) (m z - 9) = ° .ne
:. m = 3, 3, -3,-3
Solution is y = (A + Bx) e-3x + (C + xD)e 3x = (C(x + Cz)e-3x + (C 3x -t- C 4 )e 3x
t
Exercise 2 (b)
d2y dy
I. Solve -Z +10- +25y=0 Ans : y = e-5x (Ax + B)
dx dx

2. Ans : y = (Ax + B)e2x + cex

3. Ans : y = (Ax + B) + (Cx + D)~

4. Ans : y = (M + Bx + C) e-t

Downloaded From : www.EasyEngineering.net


Downloaded From : www.EasyEngineering.net

74 Engineering Mathematics - I

Solved Examples
Example 2.1.16

d2 y dy
Solve -2 +4- +9y=0
dx dx

Sol. A.E. is m2 + 4m + 9 =0
m=-2±i/5
a = -2 and f3 = /5
ww
:. Solution is y = e-2x (Acos x /5 + Bsin /5 x)

w.E
Example 2.1.17

d 3y
Solve dx 3 + y= 0
asy
Sol. A.E. is m3 + 1 = 0
En
:. (m + 1)( m2 - m + I) = 0
gin
1+J3i
:.m=-I,m=--
2 eer
ing
.ne
Example 2.1.18
d4 y
Solve -+2-+3-+2-+y=0
d3 y d2 y dy
t
dx 4 dx' dx 2 dx

Sol: A.E. is m4 + 2m 3 + 3m 2 + 2m + 1 = 0
(m 2 + m + 1)2 = 0

-1±J3i -1±J3i
:. m=
2 2

Downloaded From : www.EasyEngineering.net


Downloaded From : www.EasyEngineering.net

Linear Differential Equations with Constant Coefficients ... 75

Exercise 2 (c)

I. Ans: y = Acos J7x + Bsin J7x


d 2y dy x x
2. Solve2- 2 +4- +3y=O Ans : y = e-x (Acos.fi + Bsin .fi )
dx dx

3.

ww
w 4.
.Ea
d4 y
Solve dx4 -64y = 0

Ans : y =
syE
C\e2x + C 2e-2x + e-x (C 3cosfix+c 4 sin fix)

+ ~(C3cos.J3x+C6sinfix)
ngi
2.2.1 Consider the O.E.f{O) y = R
nee .... (I)

Operating both sides with f(IO) (called inverse operator)


rin
1
we have f(O) [f(O)]y =
I
f(O) R g.n
I
or y = f(O) R
e t
This solution is called the particular Integral of(l)
while the solution off(O) y = 0
is called the complementary function (C.F.) ..... (2)
Suppose y\ is C.F and Y2 is P.I for (I)
Thenf{O) y\ = 0 from (2)
andf{O) Y2 = R from (1)
Hencef(O) [y\ + Y2] = f(0) y\ +f(O) Y2
=O+R=R

Downloaded From : www.EasyEngineering.net


Downloaded From : www.EasyEngineering.net

76 Engineering Mathematics - I

Which shows thatYI + Y2 is also a solution of(l)'Yl + Y2 (i.e.) C.F. + P.I is called the
most general solution of(l).
I
2.2.2 Calculation of R
D-I11,

D _ nl, R = y, say, Operating both sides with (0-m 1), we get (0-m 1) Y = R

dy
(i.e.) dx - m1y= R

ww I.F.lse

:. Solution is given by
=e-ml x
f-mjtU

w y.e-mf = fRe-lIl/ dx+c


.Ea
(i.e) y = Ce-ml X + ell/IX fRe-1Il IX dx = C.r. + P.I

P.I.=e mI X fH£lIllx dx syE


If m =O~
'0
R= fRdx ngi
nee
I

I
Thus the operator 0 stands for integration

Example 2.2.3
I
Find the value of - D x
r ing
m} e'+ xe;" - ~;'l.net
-5

Sol. D 5 ~ Fe" fe".xdx ~ <' 0" [ - +('

x I
-----
5 25

Example 2.2.4

1 I
Sol. eX
02 _50+6 = (D-2){D-3) .(eX)

I I
= (0-2) . (0-3)
(eX) I 3x -3x x dx
= (0-2) e e.e
f =
I
(0-2) . e
3x

2
I -- r -2x .exdx = -e
I - (eX) = - -I e 2x Je I 2 x e-xdx=-ef
I 2 x.e- t
2 (0-2) 2 2 2 • 2

Downloaded From : www.EasyEngineering.net


Downloaded From : www.EasyEngineering.net

Linear Differential Equations with Constant Coefficients ... 77

Example 2.2.5
Solve (02+a 2) y = tan ax

Sol. A.E is m2 + a2 = 0 => m = ±ai

C.F. is y = C I cos ax + C 2sin ax

P.I =
I
tan ax = -
I
- I- - -I - J tanax
(0+ ai)(O- ai) 2ai [ O-ai D+ ai

ww 2ai [e lax J e -a/\ tan axdx - e -IllX J eat" tan axdxJ

w.E r alX
Je - tanaxdx =
J. . sinax
(cos ax -Ismax) - - dx = JSin ax- i
(l-cos ax)
dx
2

asy cos ax cos ax

= - -- -
cosax
a En i isin ax
-\og(sec at' + tan ax) + - -
a a

dx gin
cosax i isinax

eer
an
lilly
Je tanax = - - - + - \og(sec ax + tan ax) - - -
a a a

II
= (Cosax+aisinax) [-cos ax + isin ax -ilog (sec ax + tan ax] ing
.ne
a
[-\- icos ax log(sec ax + tan ax) + sin ax \og(sec ax + tan ax)]
t
\
:. P.l. = - 2
? [-2icos ax log(sec ax + tan ax)]
a-,

cos ax
- - - - log(sec ax + tan ax)
a

:. Most general solution is

. cos ax
y = C 1 cos ax + C 2 SIl1 ax - - - 2- log(sec ax + tan ax)
a

Downloaded From : www.EasyEngineering.net


Downloaded From : www.EasyEngineering.net

78 Engineering Mathematics - I

Example 2.2.6
Solve (02-50+6)y = ~
Sol. A.E. is m2-5m+6 = 0
C.F. is y = C)e 2x + C2 e3x
I eX
P.1. isy= 0 2 -50+6 (eX) = 2 (from 5.2.4)

:. Complete solution (C.S.) is y = C.F. + P.I.

ww
w.E Exercise 2 (d)
I
I. 0 cosx
asy I
3. 0 (x 2 )
En
I
4. 0-2 sinx gin
x3
3 eer (2sinx+ cos x)
Ans: (I)+sillx (2) e-x (3) (4)
5
ing
Solve the following:
.ne
6.
d2y
dx 2 -
dy
4 dx + 3y = e2x t
d2 y
7. -2 +y=5x+3
dx

d2 y dy
8. dx 2 - 3 dx + 2y = e-2x

9.

10.

Downloaded From : www.EasyEngineering.net


Downloaded From : www.EasyEngineering.net

Linear Differential Equations with Constant Coefficients ... 79

xsin ax
II. Ans : y = C I cos ax + C 2sin ax + C 2sin ax + - - -
a

cos ax
+ -2- log cos ax
a
12. (D 2 + 9) Y = tan 3x Ans : y = C1cos 3x + C 2sin 3x

cos3x
- - - log(sec 3x + tan 3x)
9

ww
Methods of finding P.1.
We shall now consider the methods for finding P.1. where R is of some special form

w.E
2.3.1 Particular integral

a
Where R is of the form eax
Case (i) iff(a) =F 0
syE
Since De ax
j{D) ear
= aeax ;
=
D2e ar
ngi
= a2 ear ...... Dn(eax) = an.ear
(Dn + K1D n- 1 + K 2 Dn- 2 + ...... K,)e ax

nee
= (an + K1a n- 1 + .......... + Kn)e an = j{a) ear

Operating with _I- on both sides


rin
feD)
g.n
- I
feD)
[f(D)e a,] =
I
- (-) [f(a)elU]
f D et
(i.e) eax = 1 f(a) eax = f(a). [ feD)
feD) I e ax]

1 a, 1 at
(or) --e = --e
feD) f(a)

1 1 1
Note: If K is a constant then feD) (K) = feD) K.eax = f(m)·K

3x I 3 I 3
for example -D-:02-_-2-D-+-l e 9 _ 6 + I = e x = "4 e x

Downloaded From : www.EasyEngineering.net


Downloaded From : www.EasyEngineering.net

80 Engineering Mathematics - I

2.3.2 Case ii) If./{a) = 0 then it is possible to write./{O) as <1>(0) (O-ay where <I>(a) i:- 0
Suppose r = 1 then
1 I 1
P.1. = - - eax = - - - - e(lX
f(O) O-a <1>(0)

1 1 1
- - eax = - - - - eax
O-a $(a) $(a) O-a

Je~a\ .eaxdx = <I>~a) ea\dx = <I>~a) ell<.x


ww = <l>(la) eax .... (I)

w.E 1 ax 1 I ax 1
ifr=2then f(O) e = (0-a)2 <1>(0) e = <I>(a) (0-a)2 ea~

a syE
1 1
- - [xeax]
$(a) O-a

P.1. =
1
f(O) e
1
<I>( a) ngi
ax- = - - e ax- -X
. L2
2

In general, if (O-a) is repeated 'r' times then nee


1 I x2
P.1. = f ( D) eax = $( a) ea~. --;:! rin
g.n
Example 2.3.3
Solved Examples
et
d 2y dy
Solve - + 4 - + 5y = 13e3x
d~2 dx

Sol: (0 2 + 40 + 5) = 13e3x
A.E. is m2 + 4m + 5 = 0, giving m = -2 ± i
C.F. is y = e-2x (Acosx + Bsinx)
I
PI (13e3x)
.. = 0 2 +40+5

replace '0' by '3'


13e3x 13e3x eh
9+ 12+5 26 2

Downloaded From : www.EasyEngineering.net


Downloaded From : www.EasyEngineering.net

Linear Differential Equations with Constant Coefficients ... 81

h
e
:. Complete solution is y = C.F. + P.1. = e-2X(Acosx + Bsinx) + 2
Example 2.3.4
d2 y dy J. dy
Solve dx 2 + 4 dx + 5y = 2e-- x , gIven that x = 0, y = I and dx =-2

Sol. Given equation isJ(O)y = 2e-2x were f(O) = 0 2 + 40 + 5


:. A.E. is m2 + 4m + 5 = 0, giving m = -2±i

ww :. C.F. is y = e-2x (Acosx+Bsinx)


I
e 2x
2e- 2 ,
2x

w.E
P.l. = 0 2 +40+5 2- = 4-8+5 = 2e-

:. Complete solution is y = C.F. + P.1. = e-2x(Acosx + Bsinx) + 2e-2x .... (4)


Now yeO) = I, (given)
asy
Substituting in (4)
En
yeO) = I(A) + 2; A =-\.
gin
yl(x) = e-2x(-Asinx + Bcosx) -2e-2x (Acosx + B sinx) -4e-2x

yl(O) = B-2A -4, yl(O) = -2 (given)


e eri
:. -2 = B-2A-4, => B = 0
:. y(x) = e-2x (-cosx) + 2e-2x ng.
Example 2.3.5
= 2e-2x - e-2x cosx
net
d3 y d2 y dy
Solve - 3 + 2 -2 + - =e 2x
dx dx dx

Sol. A.E. is m3 + 2m2 + m = 0 ; m(m+ 1)2 = 0


m = 0, -I, -I.
C.F. = C1eOx + (C 2 +C 3x)e-X

I 2x
e 2x
P.l. = 03 + 202 + 0 e = 18

Downloaded From : www.EasyEngineering.net


Downloaded From : www.EasyEngineering.net

82 Engineering Mathematics - I

Example 2.3.6_
d2 y dy
Solve - , -3 - +2y=e-~
dx- dx

Sol. f(O) = 0 2-30+2


A.E. is m2 _ 3m + 2 = 0 => (m - 2) (m - I) = 0; m = 1,2
here f(l) = 0

I I I I I
.. (0-1) (0-2) eX= (0-1) (1-2) e = (0-1) eX=-eX
X
~ (using 5.3.2.(3)

ww:. Most general solution is


:. y
w.E
= CteX + C2 e2x - xe n
2.4.1 R is of the form sin ax or cos a\"
0 2 (sin ax) =:' (~a2i~ sin ax
asy
(02f (sin ax) = (_a2)2 sin ax
En
(0 2)3 (sin ax) = (_a2)3 sin ax
gin
e eri
(02)n (sin ax) = (_a2)n sin ax ng.
Hence j(02) sin ax = j(-a2) sin ax, provided f( -a2)

1 2' _ 1 2 .
:;i: 0 net
j(O ) SIl1 ax - (,)j(-a )SIl1 ax
1 (0 ) 2
1 0-

2
Qr sin ax = 1(02 ) j(-a ) sin ax

I. sinax
1(02 ) SIl1 ax = f(-a 2 )

Similarly it can be shown that I 2 cos ax ax = I, cos ax


f(-O ) 1(-a-)

Downloaded From : www.EasyEngineering.net


Downloaded From : www.EasyEngineering.net

Linear Differential Equations with Constant Coefficients ... 83

Ifj{O) vanishes when 0 2 is replaced by _a2 then we write

I 1 .
1ar
j(O) j(O)
ww [sin ax] = [Imaginary part of e ]

w.E 1 .
= I.P of - - e 1ar
j(D)
1 asy 1 .
Similarly j(O)
En
[cosar] = R.P. of j(O) e 'ar

gin
Solved Examples
Example 2.4.2
eer
Solve
d2 y
- -3 -
dy
+ 2y = sm3x
.
ing
.ne
2
dx dx

A.E. is m2 -3m+2 = 0; m = 1,2


Sol.
t
I. 1 . -1. (3 D - 7) .
P.I., (sm3x)= 9 30 2 (sm3x)= (3 7 (sm3x)= 9D 2 49 (sm3x)
0- - 3D + 2 - - + D + .) +

(3D - 7 ) . 9cos3x + 7 sin 3x


= -81-49 (sm3x)=--0-2=-_-3D-+-2-

Hence complete solution is

Downloaded From : www.EasyEngineering.net


Downloaded From : www.EasyEngineering.net

84 Engineering Mathematics - I

Example 2.4.3
Solve (D3 -D) Y = sillX
Sol. A.E. is m 3 -m =0
m=O, m =±1
C.F. is C1e°.x + c2eX + C3e-x = C 1 + C2e x + C3e-x
I.Sit1X I I
P.1. = D3 _ D = D2 _ 1 . D (sitlX)

ww I
= - - - . cosx=
D2_1
-co·sx
-1-1
x
= --SillX
2

w.E
The complete solution is

I 2
asy
y = C + C eX + C e-x -~SillX
3 2

Example 2.4.4
En
Solve (D2 - D + 1) y = cos2x
gin
1±J3i
Sol. A.E. is m2 - m + 1 = 0 ~ m = 2
e eri
:. C.F. = e
~~
2
fi
(Acos-x + B sin- x)
2 2
fi
ng.
I
P.I. = D2 _ D + 1 cos2x =
I
- 4- D+1
cos2x =
1
-3 - D
cos2x = -
net
(3-D)
9 - D2
cos2x

(3 - D )cos2x I
13 =- 13 (2sin2x + 3cos2x)

C.F. + P.1. = e 2
x
fi BSIl1-x
( Acos-x+
2
. fi
2
1 _-1 (2sin2x + 3cos2x)
13

Example 2.4.5
Solve D2(D2+9) = sin2x + 5
Sol. A.E. is m2(m 2+9) = 0 ~ m = 0, 0, + 3i
C.F. = (C I + C2x) + C3 cos3x + C4sin3x

Downloaded From : www.EasyEngineering.net


Downloaded From : www.EasyEngineering.net

Linear Differential Equations with Constant Coefficients ... 85

P.1. = D2 (D2 + 9») sin2x + (D2 + 9) D2

+ ~._I_(I)
2
sin 2x = _ sin 2x + 5x
-4(-4+9z) 9 D2 20 18

Example 2.4.6
(D 4 -2D 3 + 2D2 - 2D + 1 ) y
ww
Sol.
Solve
A.E. is f(D) = D4 - 2D 3 + 2D2 - 2D + 1
= eX + sin2(x/2)

w.E m4 -2m 3 + 2m2 - 2m + 1 = 0


= (m-l )2 (m2+1) = 0
:. m= I, I,m=i,-i
asy
En
C.F. is (C t +C 2x)eX + (C 3cosx + C 4sinx)
1 1
P.1. = - - eX + - - sin2(x/2)
f(D) f(D) gin
eer
ing
_1_
f(D)
.
Sin
2 _
(x/2) -
1 .
(? ) Sin (x/2) -
0
(D-It D-+l
2 _
?
(D-It D +1
1
(2
.ne
)

~~ [(0_1),1(0' + I) (I-COSX)] ~ ~ [-I -20~


(0' 1)(0' + X
t
I) COS 1
1 1 1
= - - --.-- (cosx)
2 - 4D D2 + 1
1 lID
= "2 + "4 . D2 + 1 .
(D2) (cosx)

= ~+~._l_.~(-sinx)
2 4 D2 + 1 (-I)
1 1 1
= "2 + "4 . D2 + 1 . sinx

Downloaded From : www.EasyEngineering.net


Downloaded From : www.EasyEngineering.net

86 Engineering Mathematics - I

1 1 1
[)2 + 1 sinx = I.P of 02 + 1 ex = I.P of (D +-i)---:(-D----:-i) ex

I x
= I.P of 2i XCiX = "2 I.P. of -i[cosx + isinx] = -~COSX

1 1
:. P.1. = "2 - "8 x cos.\"

:. Ilence the complete solution is

ww Y =(C +C x)~+C
1 x r 1 x 2
cosx+C sinx+ -.-e +---cosx

w.E I 2 3' 4 2 2!

Exercise 2(e)
2 8

Solve the following


asy
(1) En
(2) (D2+D+ l)y = sin2\" gin
Ans:
2
J3
e"r ( C,cos-x+ Co'
-
S IJj
/ 1 - X -1
2 13
e
- ('J
eri SIll '....
.... cos 2x+ 3' J X )]

3 x ng.
(3) (D6 + 1) y = sin"2 x.sin "2

jj-, x . x .fh x
net
. x
Ans. (C1COSX + C 2sinx) + e- 2 (C3cos"2 +C 4 SIIl"2)+ e2- (Cscos"2 +C 6SIIl"2)

x 1
+ -sinx+ -cos2x
12 126
(4) (D3 + 4Di y = Sin2x
xsinx cos2x
Ans. y = C 1 + C2 cos2x + C]sin2x --8- -16
I
2.5.1 P.1. is of form feD) xm

I
- - xm = [f(D)rl.x m
feD)

Downloaded From : www.EasyEngineering.net


Downloaded From : www.EasyEngineering.net

Linear Differential Equations with Constant Coefficients ... 87

Now expand [f(0)r 1 in ascending powers of 0 and retain as far as 0 111 and then
operate on xl11
Solved Examples
Example 2.5.2
Solve (0 2 + 50 + 6) y = x
Sol. A.E. is m2 + 5m + 6 = 0 => m = -2,-3
C.F. is C 1e- 2x + C 2e- 3x
2
1 1 (I 0 + 50)_1
ww = P.1. 0 2 + 5D + 6 x ="6 + 8 x

w.E = -
I
6
50
6 6
I 5
[1--] x= - [x--]
6
= -
x
6
---
5
36

asy
Solution is therefore)' = C e- 2x + C e-3x + - - -
I 2
x
6
5
36

Example 2.5.3
En
Solve
d3 v dy
_ . - 3 - -2y=x2 gin
dx'
Sol. A.E is m3 - 3m -2
dx
= 0 => (m-2) (m+ 1)2 = 0 eer
C.F. is C 1e 2x + (C 2 + C3x)e-X
ing
P.1. = ,
x
2

0- -30-2 2
2
3
(1_(D -3D»1,.2
., .ne
=--
1
[1+
222
0 3 -3D
+(
02 -30 , .,
t+ ....... ]x- t
_ -I 3D 9 2 2 _ 1 2 6x 9 _ I 2
- - [ I - - + - D ] x ---[x --+-] - - [2x -6x+9]
224 2 224

Complete Solution is y = C 1e 2x + (C 2 + C 3x)e-X - ~ (2x 2 - 6x + 9)


4
Exercise 2(f)
I. Solve (03 - 302 +2)y = x

Downloaded From : www.EasyEngineering.net


Downloaded From : www.EasyEngineering.net

88 Engineering Mathematics - I

2. Solve (D2 + 2D + 3)y = x +.~


14-6x
Ans. y = e-x (C I cos fix + C2 sin J;:) +
27
3. Solve (D2 + 2D + I) Y = cos2x + x 2

Ans. y = (y + C e-> -~(3COS2X -


2 2sin 2x) + (x 2 - 2x)

4. (D2 - 4D + 4) y = x 2 + sin2x + e3t"


, 2X2 + X + 3 cos 2x 3

2.6.1
ww Ans. y = (C I + C2x)c~X + - - 8- -8- + e x

w.E I
To find - - (e<txy) where Y is a function of x
feD)

D(elU"Y I) asy
c<l\D(Y I) + aelU"Y I
= = ea\"(D+a)YI

En
Similarly D2(ea:.V I) = e aY (D+a)2y I

gin
D3(elU"Y ) = e<tt"(D+a)3YI
1

Dn(elU"Y I) = ea\"(D+a)nYI eer


feD) (ea\v I) = e<tt"f(D+a)Y I
ing
Calling f(D+a)Y I = Y, we get Y I = f(D+a) Y .ne
feD) rea,
f(D+a)
I
Y] = elU"Y or elU"
I
f(D+a)
I
Y - - - elU"Y
- feD)
t
_I_ elU" Y = elU" I Y
feD) f(D+a)
2.6.2 Note

1. While finding _1- elU" when f(a) = 0 we can employ the above method for
feD)

Downloaded From : www.EasyEngineering.net


Downloaded From : www.EasyEngineering.net

Linear Differential Equations with Constant Coefficients 000 89

2. Further _1_,_ [cos ax or sin ax] when f( -a2) = 0 we can write


/(0-)

_1_2- [cos ax or sin at]


f(O)

1 \
= f(02) [R.P. or I.P of eaT] = R.P. or I.P. of e1a\". f(O + ia)2 .\

Solved Examples

ww dy 2
2.6.3 Solve: -2 - 2 -
dx
dy
dx
- 4y = tfcosx

Sol.
w A.E. is m2 - 2m + 4 = 0
m= \
.Ea± fJi
C.F. is eX(C I cosfJx+CzSinfJx)

P.1.
1
tf cosx
syE = eX?
1
cosx

ngi
=
0 2 -20+4 (0+\)"-2(0+\)+4

=
\
tf 02 + 20 + 1- 20 _ 2 + 4 (cosx)
nee
=
I
0 +3
~X
t;
) x
(cosx = e .
- -
2
cosx
_12 + 3
eo' cosx
=---
2 r ing
:. Complete solution is y = CoF. + P.1.
e'cosx .ne
Example 2.6.4
y = eX (C I cosfJx + C 2sin fJx) + 2
t
Solve (02-2)y = tfcosx

Sol. A.E. is m2 - 2 =0 m=± J2


C .F, is C Ie.fix + C 2 e~.fix

1 1
P.1. = (0 2 _ 2) tf cosx = tf. (0 + 1/ _ 2 cosx = tf. 0 2 + 20-1 . cosx

= tf. . cosx
20-2
(20+2) eX(-2sinx+2cosx) eX .
= tf . cosx = = -(Sin X - cosx)
40 2 -4 -4-4 4

Downloaded From : www.EasyEngineering.net


Downloaded From : www.EasyEngineering.net

90 Engineering Mathematics - I

eX
Y = CeJ2x +C e-- J2 • + -(sinx-cosx)
I 2 4

Example 2.6.5

3
d d
Solve ---.E.
3
_7~ - 6y = e 2x + xe 2x
dx dx
Sol. A.E.ism 3 -7m-6=O:. m=-1,3,-2

ww :.C.F.

P.1. =
= Cle-x + C2e3x + C3e-2 t"
(1+x) 2x

w.E o -70-6
3 e

1
=e2x
asy
(0+2)3 -7(0+2)-6
(I+x)

= e2x En
~------

gin
3 2
0 -60 +50-12

e eri
_e2x 50 _e 2x 5 ng.
12

_e 2x
= -
(1+-)(1 +x)= -
12

17
[x+-]
12
[I+x+-]
12
net
12 12

x 3x 2x
e 2x 17
Y = C I e- + C2e + C3 e- - -
12
[x + - ]
12

Example 2.6.6

d3y d2y dy
Solve - -3 3 - +
2
3 - - y = eX(1 +x)
dx dx dx
Sol. A.E. is m3 - 3m 2 + 3m - 1 = 0 or (m - 1)3 = 0 or m = I, I, I.
C.F. - (C I xl + C2 x + C3 )eX

Downloaded From : www.EasyEngineering.net


Downloaded From : www.EasyEngineering.net

Linear Differential Equations with Constant Coefficients ... 91

P.1. = (0 _ 1)3 . (x + I) ~

e'.(x+l) 1 .1 x2
= (0+1-1)3 =~. D3 (x+ 1)=tf. 02 (T+ X
)

x3
X4
Complete solution is y = (C 1 x + C~ + C3)2
~ + ~ (24 + (;)

ww
Example 2.6.7

w.E Solve
d~y dy
- , - 4- +y = e2x sin2x

Sol.
de dx
asy J3
A.E. is m2 - 4m + I = 0, m = 2 ±
c.F.
En
= C,e(2+J3)( + C e(2
z
,)1)(

PI -
I
gin
e2x sin2x =
eh
sin2x
.. - OZ-40+1

e 2x
eer
(D+2)2-4(0+2)+1

e 2x sin2x
= -
0 -3 2
- sin2x =
-4-3
= --
1"
7
eX sin2x
ing
Complete solution is
.ne
Exercise 2(g)
t
I.

3x

Ans : y = (C 1 + C 2x) ~ + 8e (al - 4x + 3)

2. Solve (03 - 70 - 6) Y + e2x + x 2e3x


1 2 +-x+-)
Ans:y= C 1e-x +C 2e- 2x+C 3e 3x_e 2X_(x 5 169
12 6 72
3. Solve (0 2 - 20 + 4)y = eXcos 2x

Ans: y = ~(C, cos3Fx + C 2 sin3Fx)+le


x
-~sin 2x

Downloaded From : www.EasyEngineering.net


Downloaded From : www.EasyEngineering.net

92 Engineering Mathematics - I

4.

Ans. y = Clcosx + C2 sinx -~ ~(2cosx - sinx)


5

5.

13 C3sm-x
Ans.y= C le- x +e7i (C 2COS-X+ ' 13) - - 1 e 2x.(11cosx-3slnx)
.

ww 2 2

I~O) [xv] where v is a function ofx


130

2.7.1 To find
w.E
O(xv l) = xOv l + vI
asy
02(xv l) = O(xDvl + vI) = x02v I + 20v I = x0 2v I + (0 2) vI

En
Similarly 03(xv l ) = x03v I + (03) vI
in general on((xv l ) = xOnv l + (on) vI
gin
(here 1 indicates differentiation w.r.t '0')
Thus, f(O)(xv l ) =o.x(O)vl + fl(O)vl e eri ..... (1)
now let f(O)vl = v
ng.
1
vI = I(O)'v net
1 1
From(1) j(0) [x'/(D) v]=xv + fl(O)[/(O) v]

Operating both sides with I/O)

_1_ _ _1_ __1_ I _I_


x /(0) v - 1(0) (xv) + I(O)·f (0) [/(0) .v]

Downloaded From : www.EasyEngineering.net


Downloaded From : www.EasyEngineering.net

Linear Differential Equations with Constant Coefficients""" 93

2.7.2. Note: For finding [xlll.(sinax or cosax)] when m > 1 the above method becomes very
lengthy. Hence we can take it as [Xlll. (R.P or I.P of e/a,"] and apply the method
(2.6.1 ).
Solved Examples
Example 2.7.3
Solve (D 2 + 9) y = xsinx
Sol. A.E. is m2 + 9 = 0 =>m = ± 3;
1 .) 1 I .

ww P. I. -2-(XSIllX = {x--,-.2D}.-,-.sll1x
="
D +9
[using the result of2.7.1. v = sitlX]
D-+9 D-+9

w.E 1
D" + 9
I. x.
8
1 2
= {x--,-2D} . - Stox= -Stox- - - - cosx
8 D2 + 9 8

=
x.
8"
2
Stox - 8"
asy 1 x.
. 8" . cosx = 8"
1
SltlX - 32 cosx

En
Complete solution is y
gin
= C1cos3x + C 2sin3x + ~ sinx -
8
_I cosx
32
Example 2.7.4
e eri
d2 y
Find the solution of - , + 9y = xcos2x
dx- ng.
Sol. A.E. is m2 + 9 = 0 => m = ± ; Jj net
1
and P.1. = D2 + 9 (xcos2x)

1
= Real part of D2 +9 x(cos2x + isin2x)

= R.P. of -1- xe 2"IX = R.P. ofe2"IX 1 2"


x = R.P. ofe IX
1 x
D2 +9 (D+2i)2 +9 D2 +4iD+5

1+ D2 + 4iD )~t x= R.P. -e


21x 21x
= R.Pof -e ( (
1- D2 + 4iD) x
5 5 5 5

Downloaded From : www.EasyEngineering.net


Downloaded From : www.EasyEngineering.net

94 Engineering Mathematics - I

2lX
e 4iO 0 2
= R.P. of -5- [I ----+----]x
. 5 5
e 2lX 4i
= R.P. of -5- [x- 5]

I 4
= 5 [xcos2x + 5 sin2xJ
Most general solution is

c 2sIn. 3x + -1 [xcos2x + -4 Sin


. 2X ]

ww
Example 2.7.5
Y = C ,cos3x +
5 5

w.E
Solve (0 2 - 40 + 4)y
A.E is m 2 - 4111 + 4
= 8e2x x2sin2x
~ m
Sol.
asy
C.F. is (C,x + C 2)e2x
=0 = 2, 2

I -- (0 -8 2)2 e2x . x-
P.. - En
·')s,·n2x - 8e2x 1
(0 + 2 _ 2i x2sin2x = 8e2x _1_ ') sin2x
0 2 x-

~2 x 2 e 2ix ] = 8e2x[I.P. of e2ix .


gin
= 8e2x [I.P of
e (0-2i)
1 x2 ]

eri
- 8 2x[I P f 2ix
- e . 0 e
1
-4+4iO+02
x2 ]
ng.
· -1
= 8e2x [I.P of e 2IX (4) [1 -
4iO+02
4

. 4iO+ 0 2
r I,)
x- net
= -2e2x[I.P. of e 21x 1- 4 r l
x2

2 2
= -2e2x [J.P. of e 2ix [1 + 4iD + 0 + (4iO + 0 )2]x2
4 4
2
=_2e2x [J.P ofe2 ix [x 2 + 8ix+2 + 16i 02 (x 2 )]
4 4
2.<
= - ~ [J.P. of (cos2x + isin2x) (4x 2 + 8ix - 30)
2
-e 2.<
= -2- .J.P. of [4x2cos2x + 8ixcos2x - 30cos2x + i4x2sin2x - 8xsin2x - 30isin2x]
.
= -e2x[4xcos2x + 2x2sin2x - ISsin2x]

Downloaded From : www.EasyEngineering.net


Downloaded From : www.EasyEngineering.net

Linear Differential Equations with Constant Coefficients ... 95

General solution is y = C.F. + P.1.


Y = (C IX + C 2)e 2\" - e2~[2x2sin2x + 4xeos2x - 15sin2x]

Exercise 2(h)
1. Solve (0 2 -1) Y = x 2eos3x

1 26 6
Ans. y = Cle-' + C2e- x - 10 (x 2eos3x - 50 eos3x - "5 xsin3x)

2. Solve (0 2 +4)y = xsin 2x

ww X
"4 ["2
I

w.E3.
Ans. y = Cleos2x + C 2sin2x +

Solve (04 + 202 + 1)y = x 2eosx


+ sin2x]

asy I I 3
12
Ans. y =

En
(Clx + C 2 )eosx + (C 3x + C 4 ) sinx - 48 (x 4-9x2) cosx + x sinx

4.
gin
Solve (0 2 - 20 + 1)y = xe-'"sinx

Ans. Clcos3x + C 2sin3x + eer I 4


"5 (xcos2x + "5 sin2x)J
5. Solve (0 2 - 60 + 13)y = 8e3xsin2x ing
Ans. y = e3x[Clcos2x + C 2sin2x] - 2e3x xcos2x
.ne
6. Solve (04 + 0 2 + 1)y = e 2
-:::: xfj
cos (--)
2 t
-x
1 e2 1 1 1
+ C4 sin(-xfj)] + - [xcos(-xfj) + r::; xsin(-x.J3)]
2 4 2 -..13 2

d2 y
7. Solve - , + Y = e-'"sinx
dx-

I
Ans. Clcosx + C 2sinx - "5 e-'" (2cosx - sinx)

Downloaded From : www.EasyEngineering.net


Downloaded From : www.EasyEngineering.net

96 Engineering Mathematics - I

2.8.1 Cauchy's Homogenious Linear equation


Most general form of the homogeneous equation is
(xnon + K1x ll- 1 on-l + ..... + Kn- 1 xO + =X .... (I)

Where kl' k2' ........ kn are constants and 'X' is either a constant or a function of x
To solve such equations it is convenient to transform them into linear equations with
constant coefficients with the substitution x = eZ i.e. z = logx.

dy = dy dz = ~ dy .... (2)
d\: dz' dr x dz

ww
w.E
I 'dy 1 d 2 y dy
= -l----t----]
x x dz X dz2 asy ( .: dz = x)

__, d2y dy
- 2(d 2 - d ) En ..... (3 )
x z z
gin
e eri ..... (4)

d
By taking '0' to stand for dz' ng.
dy
(2), (3), (4) ~ x dx =
dy
dz = Oy
net

..... (5)

dn
Xll -Z = 0(0-1) (D-2) ......... (O-n+l)y
dx"

By substituting (5) in the given equation (') it transforms into a linear differential
equation with constant coefficients.

Downloaded From : www.EasyEngineering.net


Downloaded From : www.EasyEngineering.net

Linear Differential Equations with Constant Coefficients ... 97

Solved Examples
2
d y dy
2.8.2 Solve x 2 m"2 -x d~" + 4y = 0
Sol. Writing x = el. (i) reduces to
d
[D(D-I) -D + 4tv = 0, where D = -
dz
(D2_2D+4)y=00
A.E. is m2 - 2m + 4 = 0 :::) m = 1 ± fi i

ww :. C.F. is given by y
substituting z = logt,
= (C 1cos,,'3z+C 2 sinfiz)e
I

Y w (J3logx) (J310gx)
= Clcos + C~sin e logx

.Eax)y = [C ICOS (fi log + C 2 sin (fi logx )lx

2.8.3 Solve syE


d 2y
x2_~ - 3x-
dv
" - 4y = 2x2 ..... (i)
dx-

Sol. Writing x = e Z ngi dx


(i) reduces to
[D(D-I) + 3D -
(D 2 - D
ne + 3D -
4lY =
4)y = 2e 21.
2e 2/ ,

eri
A.E. ism 2 +2m-4=0 ~m=-1 ±J5
ng.
C.F. is y = (C1e

2
(I.JS)L

P.1. = -D-2-+-2-D - 4 (e--) = 4 + 4 _ 4 e


+ C 2e

")_
(h'5)L)

2 2_
- =
2e 2z e 2z
-4- = 2
n et
Most general solution is

y = ( C e-(Il /"S)Z
1
+C
2
e (I JS)7 + e~z 1

Downloaded From : www.EasyEngineering.net


Downloaded From : www.EasyEngineering.net

98 Engineering Mathematics - I

2
2.8.4 Solve.xl --?-
d
d\'
2y = x 2 +-
X
1

Sol. Taking x = eZ equation (i) reduces to


[D(0-1)-21v = e 2x + e-Z
A.E. is m2-m-2 = 0 ~m = 2,-1
:. C.F. is y = C l e2x + C2e-z

1 I 1 1

wwP.1. =
(D-2)(D+l)
e2z +
(D-2)(D+l)
e--Z =
(D-2)(2+1)
e2z +
(-1-2)(D+I)
e- Z

w 1 ~_
---e-- -
3(D-2)
.Ea 3(D+I)
I __
e-

=e2z I (I) e-3 (I) syE


n
3(D+2-2) - 3(D-I+l)

= .!. e2z . .!. (I) - .!. e-z .!. (1 ) gin


3
1 _
D
I
3 D
e eri
= -
3
ze2x - - ze-z
3
ng.
Complete solution C.F + P.1.

I.e. Y = C e2z + C e-= + - z( e2x


I 2 3
1
-- e-Z )
n et
I
Y = C1 + C e-Iogx + - (e2logx
e210gx 2 - e-1ogX)logx
3
1 1 1
Y = C x 2 + C - + - (x 2 - - )Iogx
I 2 x 3 x

Sol. Taking x = ez the equation reduces to


[D(O-I) (0-2) + 2D(0-1) +21v = 10ez + IOe-z
(D 3-D 2+2)y = IO(ez+e-Z )

Downloaded From : www.EasyEngineering.net


Downloaded From : www.EasyEngineering.net

Linear Differential Equations with Constant Coefficients ... 99

A.E. is m3-m 2+2 = 0


=> (m+ I) (m 2 -2m+2) = 0
(i.e.) m =-1, m = I± i
:. C.F. is y = C1e-z +e-'(C 2cosz + C 2sinz)
10 IO z
P.1. = J , e= + D3 _ D2 + 2 e-
D' - D- +2
10e: 10e- z
+ - - - - :2 : - - - -
1-1+2 (D + 1)(D - 2D + 2)

ww = 5e-'z + 10 e -L = 5e -L + 2e -L . - -I - - I

w.E 1
5( D + I)

5e-z + 2e-z D (I) = 5e-z + 2e-z . z


(D - I) + 1

asy
=

Complete solution = C.F. + P.1.


En
C1e-z + e=(C 2cosz + C sinz) + 5e-z + 2e-z . z logx
gin
=
3
x

= -
C1
x
+ x(C 2cos(logx) + C3sm(logx)) + 5x + 2

e eri
2.8.6 Solve
d y
x 2 dx 2
2
dy
+ x dx + Y =
.
logx sm(logx) ng.
Sol. Writing z = logx the equation reduces to
[D(D-I) + D+ IlY = zsinz
net
(D 2 + l)y = zsinz
A.E. is m2 + 1 = 0 => m = ±i
C.F. is y = C1cosz + C2 sinz
1 •
P.I. = -2- zsmz
o +1
1 1
= I.P. of -2- ze 'Z = J.P. of e'z z
o +1 (0+i)2 +1

1
= I.P. of e'z ---:-,----z
0- -1+20i+l

Downloaded From : www.EasyEngineering.net


Downloaded From : www.EasyEngineering.net

100 Engineering Mathematics - I

= I.P. of e= I Z
D2 +2Di

= I P of e1z -
1 (
1+ -
0)'-1 Z
. 2Di 2i

I.P. of e1z _I_(z_~)


ww _e .i(z2 J
=
2Di 2i

w.E =IPof-- - - -Iz


.
IZ

2 2 2i

asy [2 . = Imaginary part of


-I . .
(cosz + ISInZ) [22 + 2.J]
Z U

En Z2
= - - cosz
I.
-- ZSInZ =
z.
- - (SInZgin+ 2zcosz)

e
2 4 4
Hence complete solution is
eri
ng.
= C,cos(logx) + C2sin(logx) - lo;X net
[sin(logx) +2(logx) cos(logx)]

Example 2.8.7
Solve(x2 02-xO+4)y = cos(logx) + xsin(logx)
Sol. Taking x = eZ or Z = logx the equation reduces to
[0(0-1) -0 + 4lY = cosz + eZ sinz.

A.E. is m2 - 2m + 4 = 0 => m =1±J3i


C.F. =ez (C,cosJ]z+C 2sinJ]z)
I I .
P.I. = 2 COSZ + 2
Z
e smz
D -2D+4 D -2D+4

Downloaded From : www.EasyEngineering.net


Downloaded From : www.EasyEngineering.net

Linear Differential Equations with Constant Coefficients ... 101

L
1 e •
= --.cosz+ 1 smz
3-20 (D+I)--2(0+1)+4

(3+20) eZsinz 3cosz-2sinz eZsinz


---cosz+ - - = +--
9+4 3 13 3
The complete solution is y = C.F. + P.1.

ww =
.
x[C1cos J3 (Iogx) + C2Slll v(logx) ] +
~ 1 .
13 [3cos(logx) -2sll1(1ogx)] +
xsin{logx)
3

w
Example 2.8.8

.Ea
Legendre's Linear equation:
An equation of the form
d ll syE dll-1y
(ax+b)11 dx~ + K1(ax+b)n-l dx n - I + ...... + koY
n gin
Where K's are constants and X is either a constant or a function of x is called
= X

with constant coeficients by the substitution. e


Legendre's linear equation. To solve such an equation we transform it into linear equation

eri
ax + b = eZ or z = log (ax +b)
dy = dy dz = _a_ dy ng.
Now
dx

(ax + b)2 -
dz'dx

dy
= a-
ax+b'dz

dy
= aOy where 0 = dz
d
n et
dx dz

3
d
I

Similarly (ax + b)3 ---f


dx
= a3 D(O-l) (0-2)y

by substituting these values the given equation reduces to linear equation with constant
coefficients.

Downloaded From : www.EasyEngineering.net


Downloaded From : www.EasyEngineering.net

102 Engineering Mathematics - I

Solved Examples
Example 2.8.9
d 2y dy .
Solve (J+x)2 dx 2 + (I +x) dx + Y = sm2[log(l +x)] .... (I)

Sol. Write I +x = e => log( I +x) = Z


dy d2y d
(I+x) dx = Dy, (I+x)2 -? = D(O-l)y where D = -
dz
dx-

ww
Substituting these values (1) reduces to
(D2_D+D+ I)y = sin2z; (i.e) (D2+1)y = sin2z

w.E
A.E. is m2 + I
C.F.
= 0 => m =
= C ICOSZ + C2sinz
±i

1
P.1. = -2 - sin2z = asy sin 2z
--
~4+1
= --
sin 2z
~3
0 +1

En
· ·IS = C .F + PIC
C omp Iete so IutlOn
gin
. = lCOSZ + C' sin32z
2Sll1Z - - -

.
= C1cos[log(l+x)] + C2slI1[log(l+x)]-
eer
sin 2(1og(l + x))
"
.J

Example 2.8.10
ing
d 2y dy
Solve (2x+3)2 dx 2 +6(2x+3) dx + 6y= log(2x+3)
.ne
Sol. Writing 2x+3 = eZ or z= log(2x+3)
the given equation reduces to [2 2.D(0-1) + 6.2D + 6]y = z
t
d
i.e. (4D 2+8D+6)y = z where D = -
dz

A.E. is 4m2 + 8m + 6 = 0 => 2m2 + 4m + 3 = 0


~2±J2i 1
=>m= 2 =-l+J2i

C.F. is y = e-Z [Acos ~ z + Bsin ~ z]

Downloaded From : www.EasyEngineering.net


Downloaded From : www.EasyEngineering.net

Linear Differential Equations with Constant Coefficients 000 103

I 1 1 I 40 :m 2
Pol. = 1 z = - 40 2D2'Z = -6 [1+- +-3-rl z
40- +80+6 6 3
1+-- + ----
3 3

1 z
= -
6
r1--40
3
]z=
1 4
- [z- - ] = - --
6 3 6
2
9

Complete solution is y = C.F. + P.1.

=e-Z[ Acos r;::z+


1 B' z --
1 ] + -
SIl1-Z 2
..;2 ,fi 6 9

ww = -2
I
e
1
3 [Acos( r;:: log(2x+3»+ Bsin(
1 1 ')
log(2x+3))] + -6 log (2x+3)- ::..

w x+..;2..;2

.Ea Exercise 2(h)


9

I. (x 202 - 3xO + 4) Y = 2x2 syE


Solve the following differential equations

Ans. (C I + C2logx~ + x 2 (Iogx)2


ngi
2. (x 2D2 - 3xD + 5)y = sin(log(x»
Ans. x 2(C Icoslogx + C2sinlogx) + (coslogx + sinlogx) nee
3. (x 2D2 - xD +2)y = xlogx
rin
Ans. x(Clcoslogx + C 2sinlogx) + xlogx
g.n
4. (x 2D2-3xD+5)y = x 2 sinlogx

Ans. x2(C)coslogx + C2sinlog~) - ~ x2 1ogxcoslogr


e t
2 2 (_,-s_in_l--,og~x-.:.)_+_1
5. (x D -3xD+l)y
= -
x

Ans. x2[C Icosh (filogx) + C.,sinh( J3log~) + _I + _1- [5sin(logx) - 6cos(\ogx)]


- 6x 61x

Ans.

Downloaded From : www.EasyEngineering.net


Downloaded From : www.EasyEngineering.net

104 Engineering Mathematics - I

8. (x20 2+xO+I)y= (I0gx)2 + sin(logx)


Ans. C I coslogx + C2sinlogx + (Iogx? -x[2coslogx - sinlogx]
d 2y dy
9. (2x-I)2 -2 + (2x-l) - +2y = 0
dx dx
C
Ans. y= C 1(2x-l)+ ~
2x+ I

d2 dy
10. (5+2x)2 dx; - 6 (5+2x) dx + 8y = 6x

ww
Ans. y = C (5+2x)2+.fi + C (5+2xi t fi _ 3x _ 45

w.E 2
I 2 2 8

asy
11. (2x+I)2 d y _ 6 (2x+l) dy + 16y = 8 (l+2x)2
dx 2 dx
Ans. y

d2y En
= C 1 + C2 log(2x+ 1) + [log(2x+ 1)2](2x+ 1)2

12.
dy
(2x+3) - 2 -(2x+3) --12y=6x
dx dx gin
Ans. C 1(2x+3)2 + Ci2X+3f~eer - 2x + %
2.9.1 Linear Differential equations of second order ing
d 2y dy .ne
The general form is dx 2 + P dx + Qy = R

where P, Q, R are functions of x.


t
Method of variation of Parameters
Let the C.F. of the equation
2
d Y + P dy + Qy = R
dx 2 dx
be y=Au+ Bv
where A and B are constants and u, v are two independent solutions-,of
d 2y dy
dx2 + P dx +Qy = R .... (I)

Downloaded From : www.EasyEngineering.net


Downloaded From : www.EasyEngineering.net

Linear Differential Equations with Constant Coefficients ... 105

be y = Au + Bv .... (2)
where A and B are constants and u, v are two independent solutions of

d 2y dy
- 2 +P-+QllJ= 0 .... (3)
dx dx .J'

such that 1I2 + PU I + Qu = 0 and v2 + PV I + Qv = 0


d 2u du
where tl2 = dx 2 ' ul = dx etc.

ww In order to obtain the solution of the equation (I) the arbitrary constants A and Bare
treated as arbitrary functions of x and are chosen in such a way that

w.E
y = A(x)u + B(x)v satisfies (1)
Differentiation of(2) gives
.... (4)

a
= All,

syE
+ lIAI + BVI + vB, (suffixes indicating the order of the derivative)
Now we chose A and B such that Alu + BI V = 0 .... (5)

So, ngi
Again differentiating
nee
d2y
dx 2 = (Au 2 + lIIAI) + (Bv2 + vIB,)
rin .... (6)

substituting (4), (5) and (6) in (I) we get


g.n
(or)
[AlI 2 + BV2 + Alu, + Blv,] + P[Au, + Bvd + Q(Au + Bv] = R

A(u2 + Pu, + Qlt) + B(v2 + PV I + Qv) + Allt, + BI vI = R


et
Alu, + Biv i = R (u and v are solutions of(3) .... (7)
Solving (5) and (7) we get
dA -vR dB uR
- =A = and - = B = ---
dx I uv, --u, v dx I 1/V, -II, v

integrating we obtain

A(x)= J -vRdx +CI;B(x)=


(uv, -1I,v)
J uR
lIV, -u,v
+C
2
.... (8)

where C I and C 2 are arbitrary constants


Substituting (8) in (4) we get the complete general solution of the equation (I)

Downloaded From : www.EasyEngineering.net


Downloaded From : www.EasyEngineering.net

106 Engineering Mathematics - I

Working Rule: First find two independent solutions 1I and v of(3). Then C.F. is given by
y = Au+Bv where A and B are arbitrary constants. Treating A and B as functions of x, we
have the solution of (1) as y = AI(x) 1I + BI(x) u where A(x) and B(x) are given by (8)

Solved Examples
Example 2.9.2

d2 2
Solve by the method of variation of parameters ~- y = --x
dx 2 1+c

ww
Sol. A.E. is m2 - I = 0 => m = ± I
C.F. is y = A~ + Bc-x

w
Assuming A and B as functions of x such that the given equation is satisfied by
y =
.Ea
Ac-'" + Bc-X we have

dy
dx
= AcX - Be-x + ~-+e-x-
dA
dx-
syE dB
dx
= A~ - Be--x .... (I)

dA ngidB
Choosing A and B so that ~~ + e--x dx = 0
nee .... (2)

rin .... (3)

Substituting (I) to (3) in the given equation, we get g.n


~-
dA
dx
+e-x -
dB
dx
=--
2
l+c x
e t
.... (4)

dA -x
Solving (2) and (4) we get -dx = _c_ .... (5)
eX + I
X
dB C
and - =--- .... (6)
dx eX + I

= log (~+ I) -e-x-x


Integrating (5) and (6) A = _e- x + log( 1+e-') - x + C" B = -Iog( 1+~) + C 2

Downloaded From : www.EasyEngineering.net


Downloaded From : www.EasyEngineering.net

Linear Differential Equations with Constant Coefficients ... 107

Hence the solution is


y=[-e-x+log(ex + 1)-x+C,1~+[C2-log(1 +~)]e-X

(or) y=C,ex +C 2 e-x -1 +elllog(e'+I)-x]-e-'\'log(el+I)J

Remarks: If(i) I +P+Q=Otheny=~(ii) I-P+Q=Otheny=e-'x


and (iii) P + Qx = 0 then y = x are solutions of D2y + POy + Qv = 0
Example 2.9.3
Solve by the method of variat ion of parameters

ww d 3y
- \ + (I -
(ix"
d
cOlt") ~ -
dx
cotx = sin 2x .... (I)

w.Edx -
Sol:
3
d y
dx 3 + (I - cotx)
dv
cotx = 0 .... (2)

asy
C.F.P = I - cott Q = -cou comparing with original eqn.

En
:. I - P + Q = I - I + colx - cotx = 0

gin
showing that v = e-X is a solution or (2)
To find another independent solution of(2)
let 11
eer
= eO-x . w

By this substitution, the equation (2) reduces to


ing
d 2 w dw 2 -x
-+-(l-cotx--e
dx 2 dr c- x
)=0
.ne
or
d2w dw
- 2 = (l+cotx)- ~
i(~)
dw = I + cou
t
dx dx
dX

dw dw
log - = x + logsinx or - =eTsinx
dx dx

:. w= JeXsinxdx=- e; (cosx-sinx)

eX . I .
u = e-x [--(cosx-smx)] = - - (cosx - Slnx)
2 2
The second independent solution can be taken as cosx - sinx

Downloaded From : www.EasyEngineering.net


Downloaded From : www.EasyEngineering.net

108 Engineering Mathematics - I

Thus solution of(2) is


y = A (COS X - silu) + Be-x
For finding the solution of (I) we treat A and B as functions of x
i.e. y = A(x) (cosx - sinx) + B(x).e-x is the solution of (I) where A(x) and B(x) are
obtained by solving
(cosx - sinx) A I + e-x . BI = 0 .... (4)
and .... (5)
dA I

ww
solving (4) and (5) ~ = -2 sinx

and
w -dB = ~--
dx
sin2x

integrating we get
-ex

.Ef' a
4
(I-cos2x)
--'-----'-
4

A = - -I
2 syE
cosx
slflxdx+C 1 =--+C 1
2
.... (6)

and ngi .... (7)

Hence the complete solution is nee


y = C 1(cosx - sinx) + C2e-x -~ (sin2x - 2cos2x) rin
Example 2.9.4
10

g.n
Solve by the method of variation of parameters
2
e t
(I - x) d y + x dy _ Y = (I _ x)2
dx 2 dx
The given equation can be written as
d 2y x dy I
dx 2 + I-x dx - l-x Y = I-x .... (I)

x I
p= _ . Q= - andX= I-x
I-x' I-x
Clearly P + Qx = 0
. . d2y X dy I
Hence y = x IS a-solutIon of - 2 + -I- dx - --y = 0 .... (2)
dx -x I-x

Downloaded From : www.EasyEngineering.net


Downloaded From : www.EasyEngineering.net

Linear Differential Equations with Constant Coefficients ... 109

To obtain the second independent solution of(2)


take y = vx

d 2v dv x 2
Then (2) reduces to -'} + dx (-+-.1) = 0
dx- I-x x

d . dv dv x 2
- [-]+- [-+-] = 0
dx dx dx I-x x

d dv
~~ x 2 I-x-I 2 1 2
- - =- - - - =+( ) -- =+1----

ww d,,-
dx
1- x x I- x x I- x x

w.E lo.g(:) =x+ log(x-I)-logx2

dv
-
x-I
= -.e-t= e-t [-+-, ]asy 1 (-I)
dx 2 x
En X x-

:. v
1
= e"'" .-
x gin
The second independent solution is xv = e"'"
e eri
solution of equation (2) is Y = Ae"'" + Bx
To find the solution of (I) treat A and B as functions of x such that ng. .... (3)

dA
dt"
dB
e"'" - + x -
dx
= 0 net
.... (4)

dA dB
and, e"'" - + x - = I -x .... (5)
dx dx

dA dB
Solving (4) and (5) we get d; = -xe-X and dx = I

Hence the complete solution is y = A e"'" + B.x


= [C1+e-x(I+x)]e"'" + (x+C 2 ) x = C1e"'" + Cr + x 2 + (I+x)

Downloaded From : www.EasyEngineering.net


Downloaded From : www.EasyEngineering.net

110 Engineering Mathematics - I

Exercise (i)
Solve the following by the method of variation of parameters
d 2y
I. - + a2y = sec ax
dx 2

· I
Ans. y = C,cos a:r + C'2sm x .
ax + -SIl1 ax + -cos ax Iog(cos a,·)
a a

d2 y
2. dx 2 + Y = tanx

ww Ans. y = -[Iog(secx + tanx)] cosx + C,cosx + C2sinx

3.
d2 y
--T
dx-
w.E
+ 4y = cosec 2x

asy
Ans. y = C,cos 2x + C2sin2x + xcos 2x + sin 2~ log(sin 2x)

4.
d2
x2~-2x(l+x)
dy
-+2(1 +x)y=x3 En
2
dx dx
gin
Ans. y = C, xe 2x + C
x 2

r - 4-"4
x
e eri
ng.
net

Downloaded From : www.EasyEngineering.net


Downloaded From : www.EasyEngineering.net

3
ww
w.ETheorems and
Mean Value
asy Variables
Functions of Several
En
gin
3.1.0 This chapter deals with
(i)
(ii)
(iii)
Rolle's theorem
ee rin
Lagrange's mean value theorem also called as first mean value theorem.
Cauchy's Mean Value theorem
(iv)
(v)
Higher Mean Value theorems.
Curvature (vi) Centre of curvature. g.n
3.1.1
(vii) Evolutes (viii) Envelopes

Rolle's Theorem
e t
Ifj{x) is (i) continuous in [a, b], (ii) differentiahle in (a, b)
and (iii)j{a) = j{b),
then there exists a'c'E(a,b) 3f'(C)=0
Proof: Suppose
(i) f (x) is a constant function throughout the interval [ a, b ],
then f' (x) = 0 VX E ( (I, h) Hence theorem is proved ...
(ii) f (x) is not a constant function in [ (I ,h ]. As f(x) is continuous in [a,b],
there exists a maximum value say, at 'c' (a ~ C ~ b) and a minimum value,
sayat'{f (a~d~h) for f(x) in(a,b).

Downloaded From : www.EasyEngineering.net


Downloaded From : www.EasyEngineering.net

112 Engineering Mathematics - I

f (c) 7:- f (d) and at least one of them is different from f ((l) == f (b)
Suppose f( c) 7:- f( a) and 'c + h' be a point in the neighborhood of 'c' ,.
/(c+II)- /(c)
then $; 0 when II> O. .... ( 1)
h

/(c+I1)- /(c)
and ;::: 0 when h < O. .... (2)
Iz

ww Further f(x)

... / ,(c) == h~O


is differentiable in (a, h).

Lt /(c + h) - /(c)
w.E h As h -t 0 we get from (I) and (2) that -

LI
h-~O h asy
/(c+h)- /(c)
$; 0 and Lt
h~O
/(c+h)- /(c)
h
.
;::: 0 respectively.

i.e., 1'( c) $; 0 Enand 1'( c) ~ 0 simultaneously => 1'( c) == 0


Similarly the theorem can be proved when gin f (d) 7:- f (a)
3.1.2 Geometrical interpretation of Rolle's theorem e eri
Let P and Q be two points on the curve y=f(x). AP==BQ ordinates f(a)=f(b» and
the curve is continuous from P to Q. It can be shown that there is at least one ng.
point on the curve y = f(x) between x = a and x = b at which the tangent to the curve
is parallel to x-axis. . net
y
x=a y=a

P Q

f(a) .i(b)

x' 0 A B x

y'

fix) is a constant function

Downloaded From : www.EasyEngineering.net


Downloaded From : www.EasyEngineering.net

Mean Value Theorems and Functions of Several Variables 113

x=c
y

Q
f(a) f(b)
f(d)

o A B x
y'

ww j(c) andj(d) are both different fromj(a) = j(b)

w .Ea
y x=a
x=c
x=b

syE
f(a) ngi
f(c) f(d)

o A
nee B x
y'
rin
j(c) -:F j(a) andj(d) = j(a) = j(b)
g.n
x=a x=b
e t
y
x=d

f(a) f(b)
f(d)

x' 0 A B x
y'

j(d) * j(a) and j(c) = j(a) = j(b)

Downloaded From : www.EasyEngineering.net


Downloaded From : www.EasyEngineering.net

114 Eng~neering Mathematics - I

3.1.3 Verify Rolle's theorem for / (x) = log { :;a:a:) }in ( a,b )
Solution:

Consider the function J (x) = 10g{ x(2 + ah)} in the interval ( a, b ).


x a+b

2
J(x+h)- J(x) I[ (x+h)2 +llh I x +abj
Lt = Lt
- log - og---
h~O± + h~O± h ( a + b) x
ww h ( a + b) ( x + h)

~rlOg (x +_~h) + 2xll+h2 -log x+ hj


2
=
w.Ehl
+h~O±
Lt
(x 2 +ah) x

asy 1
= Lt -1 [ log {'1+ 2xh + h
r"~O± h
En 7
x- + ab
} -log {h}]
1+-
x
= Lt [2X
+h~O± x 2 + ab
1
--+O(h)
x
]

where O(h) indicates terms of order h and higher powers of h.


gin
I I( X) = 2
2x
- ~
x +ab x e eri
which indicates that I (x)
ng.
is differentiable in ( a,b ) and hence continuous also.

net
2
(a +ab)
Further I (a ) = log ( ) = log 1 = 0
a a+b
2
(b +ab)
I (b) = log ( ) = 0. Thus I (a ) = 0 = I (b)
b a+b
All the conditions of Rolle's theorem are satisfied. Hence 3c( a < c < b) such
that I I( C) =0
. 2c 1 2
II ()
C =0 gIves 2 --=O~c =ab or c=±ab
c +ab c
Clearly c = +ab is the G.M of a and b and so E (a, 1)
The theorem is thus verified.

Downloaded From : www.EasyEngineering.net


Downloaded From : www.EasyEngineering.net

Mean Value Theorems and Functions of Several Variables 115

3 .1.4 Ven'fy Rolle's t I1eorem '"lor the function


. j"() S1l1X.10
x = --- (0 ,Tl )
eX

Solution:

f(x) =~lIl<x in (O,Tl)


e

Lt
f(x+h)- f(x)
= Lt -1 [Sin(x+h) -Sinx]
-
h-... O± h h--.O± h e( ail) e'

e< [Sin ( x+ h) - e" sin x]


ww = Lt-
iI--.O± h x
ex+1I .e

w = LI -
1 .Ea
Sin(x+h)-Sinx{l+ h + h + .. ..1
I! 2!
-----------X+-il~x--------~
2

f
}HO± h
syE e .f!

~ n~ gi
=h--.O±
Lt -
h
----~--~--~------------
ext-II nee
1 2 cos ( x + ). sin ( ) - h {sin x + 0 ( h )}

rin
S1l1. (h) g.n
Lt !h 2COS(X+!!').
=II--.O± 2
( h ) -{sinx+O(h)}
2
e t
f' ( x ) = cos x ~ S1l1 x
e
Thus f (x) is differentiable in (0, Tl) and hence continuous there.

Further f (0) = 0 = f (Tl) .AIl the conditions of Rolle's theorem are satisfied.
:. 3e( 0 < e < Tl) f'( e) = 0
such that

(i.e.,)
cose-sine
------ = 0 => e = -Tl (pnnclpal
.. value)
eC 4
and clearly e = Tl E (0, Tl) . Hence the theorem is verified.
4

Downloaded From : www.EasyEngineering.net


Downloaded From : www.EasyEngineering.net

116 Engineering Mathematics - I

3.1.5 Example
Verify Rolle's theorem for the function.f{x) = Ixl in (-1, I).
Solution
Here .f{x) = - x for -I < x < 0

= 0 for x = 0
=x for 0 <x < I
}(-I) = 1,.f{I)= I;

ww Hence .f{-I)=.f{I)

w.E /'{x} =-1 for -1.:sx.:sO

asy
/'{x} = 1 for

En
:. f'{x} does not exist at x = 0 and hence j(x) is not differentiable in (- 1, I)

gin
.. Rolle's theorem is not applicable to the functionj(x) = Ixl in (-1, I).

y e eri
y = f(x) = Ixl
ng.
------------------~~------------------x
net
y'

Fig. 3.1 (The curve is not smooth at x = 0).

Downloaded From : www.EasyEngineering.net


Downloaded From : www.EasyEngineering.net

Mean Value Theorems and Functions of Several Variables 117

Exercise - 3(A)

I. Verify Rolle's theorem for the following functions :


1. lex) = x (x + 3)e-xI2 m (-3.0) [Ans:c=-2]
2. j(x) = sin x m (O.n) [Ans : c = n/2]

3. j(x) = e" (sin x - cos x m (:.?~) [Ans : c = n]

ww (.'s;-
+6: J6]
w.E
4. j(x) = log m (2,3) [Ans : c =

5. fix) asy = (x- a)m (x- b)n in (a, b), m > 0, 11 > 0 [Ans : c =
{mb+ na}
{m+ 11}
]

6. fix) En
= xl - 5x + 7 in (2, 3) [Ans : c = 5/2]

3.2.1 Lagrange's Mean Value Theorem (First mean Value Theorem): gin
e eri
Ifj(x) is (i) continuous in [a, b] and (ii) differentiable in (a, b) then ::3 at least one

"alue 'c' in (a, b) 3 f'{c) = f{bi- f{a)


-a ng.
Proof: Define a new function «j>x
where A is a constant 3
=

«j>(a) = «j>(b)
fix) + A.\
n et
..... ( I)

i.e., fia) + A (a) = fib) + Ab

i.e., A = _ [f{b)- }'(a)] ..... (2)


b-a

j(x) and Ax are continuous in [a, b]


Hence «j>(x) is continuous in [a, b] ..... (3)
j(x) and A x are differential in (a, b),
Hence «j>(x) is differential in (a, b) ..... (4)
andj(a) = «j>(b) ..... (5)

Downloaded From : www.EasyEngineering.net


Downloaded From : www.EasyEngineering.net

118 Engineering Mathematics - I

(3), (4), (5) show that ~(x) satisfies all the conditions of Rolle's theorem.

3 atleast one value c in (a, b) ;) ~' = 0


~' (x) = f' (x) + A
~' (c) = f' (c) + A = 0
A = - f'(c) ..... (6)

From (2) and (6) it follows that

ww f'{c) = j'(b)- f{a)


b-a

w.E
3.2.2 Geometrical Interpretation of Langrange's Mean Value Theorem
P and Q are two points on the continuous curve y = j(x) corresponding to x = a
and x = b respectively.
asy
.. P[a, j(a)],
En Q [b,j(b)] are two points on the curve.

· ... h . PdQ·
SI ope on t he Ime JOll1lng t e pomts an IS
gin f{b)-
b_a
f{a) ' R·IS a pomt
. h
on t e

e eri
curve between P and Q corresponding to x = c, so that f'{c) is the slope of the
tangent line at R [c,j(c)].

f'{c) =
f{b)- f{a)
b_a ng.
means that the tangent at R is parallel to the chord P Q.

y
net
Q

p
f(c) f(b)
f(a)

x' x=a x=c


0 x=b x

y'

Fig. 3.2

Hence this theorem tells that there is at least one point R on the curve PQ
where the tangent to the curve is parallel to the chord PQ.

Downloaded From : www.EasyEngineering.net


Downloaded From : www.EasyEngineering.net

Mean Value Theorems and Functions of Several Variables 119

3.2.3 Example
Verity Lagrange's theorem for the functionf(x) = (x - I) (x- 2) (x- 3) in (0, 4).
Solution
j(x) = (x - I)(x - 2)(x - 3) and a = 0, b = 4
j(x) = x 3 - 6 Xl + II x - 6 is an algebric polynomial and (0, 4) is a finite interval.
Hencej(x) is differentiable in (0, 4) and is continuous in [0,4] showing that the
conditions of Lagrange's Mean Value theorem are satisfied.
:3 atleast one value 'c' in (0,4), such that

ww f'{c) f{b)- f{a)


=
b-a
..... ( I)

w.E j(0) = - 6, f(4) = 6


f'{x) = 3 x2 - 12 x + II

asy
f'{c) = 3c2 - 12 c + 11

From (I)
E° n
3c2 _ 12 c + II = 6 - (- 6)
4-0

gin
3c2 - 12 c + 8 =
6±2fj
c=
3
e° )er
The point c = 6 ± 2fj
3
W h·ICh CIear Iy I·Ie .111 ( ,4 .
ing
3.2.4 Example .ne
Verify Langrange's Mean Value theorem for the functionj(x)
Solution
= ~ in (0, I) :
t
j(x) = ~ is differentiable in (0, I) and continuous in [0, I]
.. :3 atleast one value 'c' in (0, 1) such that
3 l'{c) = f{b)- f{a) ..... ( I)
b-a

'{ ) = f(l) - f(O)


3
f c 1-0 ..... ( 1)

j(0) = eO = I, f(l) = e

f'{x) = ~ gives f'{c) = eC

Downloaded From : www.EasyEngineering.net


Downloaded From : www.EasyEngineering.net

120 Engineering Mathematics - I

e-I '
From (I) eC = -
1-0

c=log(e-I), thisc EO, I)

3.2.5 Example
Verify Langrange's Mean Value theorem for the functionj(x) = 5x2 + 7x + 6
in (3,4).
Solution
j(x) is an algebraic polynomial and the interval (3,4) is finite, j(x) is differentiable

ww in (3,4) and continuous in [3,4].


. . 3 atleast one value c r.(3, 4) such that

w .Ea
j'(e) = f(ll) - f(3)
4-3
..... (1 )

3 f'{c}
syE
= f{h}- f{a}
b-a

ngi
j(3) = 72,j(4) = 114, f'{x} = lOx + 7

IOc+7=
114-72
4-3 nee
c = 3, 5 E, (3, 4)
r ing
Exercise - 3(8)
.ne
I. Verify Langrange's Mean Value theorem for the following functions :
I. j{x) = x(x - I) (x - 2) in (0, X) t
2. j(x)=logxin(1,e)

3. j(x) = x 2 _ 3x _ I in ( -~ 1, I;)
4. j(x) = a2 - 7x + lOin (2, 5)
3.3.1 Cauchy's Mean Value Theorem
If two functionsj(x) and g(x) are (i) continuous in [a, b] (ii) differential in (a, b)
and (iii) g'(x) =F- 0 in (a, b) then 3 atleast one value 'c' in (a, b) 3
f'{c} f{b}- f{a}
g'{c} = g{b}-g{a}

Downloaded From : www.EasyEngineering.net


Downloaded From : www.EasyEngineering.net

Mean Value Theorems and Functions of Several Variables 121

Proof:
Define a new functionj(x) = j(x) + A g (x) .....( I)

Where A is a constant such that 4>(a) + 4>(b) ..... (2)

j(a) + A g (a) = j(h) + A g (h)

[j{b}- j{a}]
..... (3)
A=- [g{b}-g{a}]

ww j{x), A g (x) are continuous in [a, b)

w Hence 4>(x) is continuous in [a, b)

.Ea
j(x), A g (x) are differentiable in (a, h) ..... (4)

and 4>(a) = 4>(b) syE


Hence 4>(x) is differentiable in (a, b) ..... (5)

..... (6)

n gin
(4), (5), (6) show that 4>(x) satisfies all the conditions of Rolle's theorem

But 4>'{x} = f'(x} + Ag' (x)


e
3 atleast one value c in (a, b) ;) 4>' (c) = 0

eri
4>'{c} = f'{c} + Ag' (c) = 0 ng.
=> A= -
/'{c}
g'{c}
n et
..... (7)

/'{c} /{b}- /(o)


From (3), (7) we get g'(c} = g(h}- /(o)

3.3.2 Example
1 1
Verify Cauchy's Mean Value theorem for j(x) = -? and g(x) = - in (0, h)
x- x

Solution

j(x), g(x) are differentiable in (a, b) and continuous in [a, b)

Downloaded From : www.EasyEngineering.net


Downloaded From : www.EasyEngineering.net

122 Engineering Mathematics - I

::3 atleast one value 'c' in (a, b) :)


f'{c) f{b)- f{a)
g'{c) = g{b)- f{a)

Here f'{x) = ~
X3

-I
g'{x) =7

ww -2

w.E C 3
=--IT
//c 2
= 1

asy b a

2
c En a+b
ab

c
2ab gin
= --b which is the Harmonic Mean of , a' and 'b'

c E(a, b)
a+
e eri
3.3.3 Example ng.
Verify Cauchy's
Solution
Mea~ Valve Theorem for fix) = e, g(x) =
net
e-X in (3, 7)

fix), g(x) are differentiable in (3, 7) and continuous in [3, 7]

::3 atleast one value 'c' in (a, b) :)


f'{c) f{b)- f{a)
g'{c) = g{b)-g{a)
Here f'{x) = e
g'{x) = e-x

c = 5 E (3, 7)

Downloaded From : www.EasyEngineering.net


Downloaded From : www.EasyEngineering.net

Mean Value Theorems and Functions of Several Variables 123

Exercise - 3(C)

I. Considering the functionsj{x):::: x 2 , g (x):::: (x) in Cauchy's Mean value theorem for
(a, b) prove that 'c' is the arithmetic mean between a and b.
2. Verify Cauchy's Mean Value theorem for j{x):::: sin x, g(x):::: cos x in (a, b)
I
3. Verify Cauchy's Mean Value theorem for j{x):::: fx, &>(x):::: fx in (a, b)

3.4.1 Higher Mean Value Theorem with Lagrange's form of remainder


ww (Taylor's theorem with Lagrange's form of remainder):

w.E
If a functionj{x) is such that

(i) j(x); f'{x}, r{x} ..... fn - I (x) are continuous in [a, a + h]

(ii) asy
fW{x} exists in (a, a + h), then :3 at least one number '8' between '0' and' I'

En h hn - ' h"
1.
gin
3j{a + h):::: j{a) + ,f'{a} + ..... + - (
n -I,.
\"f(n-I)(a) + -F (ll + 9h)
n!

Proof: Define a new function.


e eri
j{x):::: j{x) + (a+h-x) f'{x} + (a+h-xY r(x) + ...
J! 2! ng.
+
(a + h - X ),,-1 /"-1
(-I)
n . (x) +
(a + h - x)"
n., .A
net ..... (1 )

where A is a constant 3 cp (a) :::: cp (a + h)


2
h j" (a) + 2!
cp(a) :::: j{a) + 1! h fW (a)+

hn - I hn
..... + - (
I",,r-I(a) + - A ..... (2)
n- J! n!
and cp (a + h) :::: j{a + h) ..... (3)

but cp (a) :::: cp (a + h) ..... (4)

Thus j(a + h) :::: cp (a + h) :::: cp (a)

Downloaded From : www.EasyEngineering.net


Downloaded From : www.EasyEngineering.net

124 Engineering Mathematics - I

Hence using (2) we get

h h2
j(a + h) = lea) + - j(a) + - I" (a) + ..... .
J! 2!

..... (5)

j(x), I'(x) , I" (x ), ... .fn-I(x) and (a + h - x) (a + h - x)2 etc continllolls in


[a, a + 17] and differentiable in (a, a + h)

ww Hence ~(x) is continuous in ra, a + 17] and differentiable in (a, a + 17) ..... (6)

w.E
(4) and (6) show that ~(x) satisfies all the conditions of Rolle's theorem.

:. :3 atleast one value 'c' (a < c < 0 + 11) ;) ~' (c) = 0

a
Write c = a + e 17 where 0 < e < I
syE
:. :3 e E (0, I) ;) ~' (0 + e 17) = 0
ngi ..... (7)

Differentiating (I) with respect to 'x'


nee
rin
I" (x)- (n-IXa+h-xt-
2 g.nJ
.,.
+ [(a+h-x)"J
(n-I) (n-I)
In-J/(x)j_ n(a+h-xt-
F;'n!
et A ..... (8)

From (7) we get

(h - eh),,-J
~' (0 + e 17) = [rea + eh] =0
(n-I)
..... (9)
From (5) and (9) it follows that
2
j(a + h) =
hi' (a) + -,
j(a) + -1' h I "
(a) + ...
. 2.

(0 < e < 1)

Downloaded From : www.EasyEngineering.net


Downloaded From : www.EasyEngineering.net

Mean Value Theorems and Functions of Several Variables 125

The last term ~ .!' (a + 0 h) is called Lagrange'sform of remainder.


n!

3.5.1 Higher Mean Value Theorem with Cauchy's form of remainder


(Taylor's theorem with Cauchy's form of remainder):

If a functionf{x) is such that (i)f(x), f'{x) , r{x) ..... fll(X) arc continuous in
[a, a + 11] and (ii),f'1(x) exists in (a, a + h) then 3 atleast one number '0' between
'0' and' 1' such that

ww f(a + h) = I!
.f{a) + h f' () h '" «(I) + ...
a + j!f
2

w .Ea
Proof: Define a new fUllction syE
~(x) = .f{x) +
I!
n
(a+h-x) ()
f' X +
(a+h-x)2
2! '
()
f" x + ....
gin
+
(a + h - x)"
{n-l}!
I
e
fll-l(x)+(a+h-x)A
eri ..... (1)

where A is constant 3 ~ (a + h) = ~ (a) ng. ..... (2)


From (1) ~ (a + h)

h
= f(a + 11)
h2 " h"-' ,
n et
~(a) = f(a) + -1'.1
'I

.
{a} + -
2!
f {a} + ... + -(-1)'.1 11-1 (a) + II
n- .
A ..... (4)

From (2), (3), (4)


,J
h " n- f"
f{a + h) = .f{a) + -" j (0) + -, (a) + .......
. 2.

.. ... (5)

f(x), f'{x) , r(x) ..... jil- I (X) and (a + h - x), (a + h - x)2 ...... are continuous
in [a, a + h] and differentiable in (a, a + 1/)

Downloaded From : www.EasyEngineering.net


Downloaded From : www.EasyEngineering.net

126 Engineering Mathematics - I

Hence <I>(x) is continuolls in [a, a + h] ..... (6)


<I>(x) is differentiable in (a, a + h) ..... (7)
(2), (6) (7) show that <I>(x) satisfies all the conditions of Rolle's theorem.

:. 3 atleast one number '0' in between '0' and' I' 3 <1>' (a + 0 h) = 0 ..... (8)

Differentiating (I) W.f. to 'x'


<I>'{x) = I'{x) + [(a+h-x) IW{x) - f'{x)

ww [
(a+h-x f I"'{x)- 2(a+h-x) f"{x)]
2! 2 + ... + ... +

w.E + [(a+h-x
llu1
) F(x)_{n_I){a+h-x)"2 f"'{X)] -A)

asy (11-1). (n -I).

I.e., <I>'{x) = En
(a+h-x)"-'
{n-I}. f"(x)-A ..... (9)

gin
From (8) and (9) we get

{h-Oh)"-I eer
<1>' (a + 0 h) = {n -I}. fll (a + 0 h) - A=0
ing
.. A =
h,,-I (I - 0),,-1
(n-I)' fll(a+Oh) .ne ..... (10)

From (9) and (10) t


2
hi' h j.w
flo + h) = j(a) + -" (a) + -, (a) + ....
. 2.

h"{I-OY--1
The last term ( ) fll (a + 0 h) is called Cauchy's form of remainder.
n-I!

Downloaded From : www.EasyEngineering.net


Downloaded From : www.EasyEngineering.net

Mean Value Theorems and Functions of Several Variables 127

3.5.2 Alternate form of Lagrange's Mean Value theorem


If f(x) is i) continuous in the closed interval [a. a + h] and ii) derivable in
the open interval ( a . a + h ) then there exists at least one number B, 0 < B<1,
such that f(a+h)=f(a)+f'(a+Bh).
Proof: In ( 2.1 ) put b =a + h ,then from the proof of L . M . V theorem there
exists 'c' ,

such that
f'(c) = f(a+h)- f(a) ....... ( I)
a+h-a
ww writing c = a + Bh we have 0 < B < 1 and (I) becomes

w f'(a + Bh) = f(a+ h)- f( a) => f( a + h) = f(a)+ hf'(a+Bh)

3.5.3 Example: .Ea a+h-a

syE
If f(x+h)=f(x)+hf'(x+Bh), O<B<l, find the value of B when f(x)
is any quadratic expression.
Solution:
ngi
Let f (x) = ax 2 + bx + c , then
f(x+h)=a(x+h)2 +h(x+h)+c
nee
= f(x)+h(2ax+b+ah) .......(1)
It is given that = f (x + h) = f (x) + hf' (x + Bh) rin
....... (2)
g.n
From ( I ) and (2) we get,
f'(x+ Bh) = 2ax +b +ah (i.e.,) 2a(x + Bh)+ h = 2ax +b +ah
.
e t
(sincef'(x) = 2ax+b) => 2B = 1 or B= 1;2

3.5.4 Alternate form of Cauchy's Mean value theorem


If f (x) and g (x) are (i) continuQ,Us in the closed interval [ a, a+h] (ii) derivable

in the open interval ( a . a+ h ) and (iii) g' ( x) *- 0 at any point in the open interval
( a, a+ h ) , then there exists at least one number B , 0 < B < 1, such that
f'(a+Bh) f(a+h)- f(a)
g'(a+Bh) - g(a+h)-g(a)
Note: Lagrange's Mean Value theorem can be deduced from C . M . V theorem by
replacing g (x) with x in the interval ( a, b )

Downloaded From : www.EasyEngineering.net


Downloaded From : www.EasyEngineering.net

128 Engineering Mathematics - I

= b , g ( a ) = a and g X ) = 1
We get g ( b) I(

f'(C) f(b)-l(a)
-- = . which is L. M. V. theorem.
1 b-a
Example: Using Cauchy's Mean Value theorem, show that

sin b - sin a < b - a given that 0 < a < h < Jr


2
Solutio,,:
Taking f (x) = sin x and g (x) = x and applying C.M. V theorem we get that,

ww 3e such that
sinb-sina
b-a
= cos e where 0 < a < c < b < -
Jr
2

w .
Furt her cose < ] 111
.Ea
(0 Jr)2
;- :. sin b - sin a <
b-a
l' .(I.e.,) Sin b '
-SIna < (b
-a)

3.5.5 Example:
syE
I f f '( x) is continuous in [ a, b] and fll (x) exists in ( a, b ) then show that

ngi
I

f(b)-f(a)=(b-a)f '(a)+(b-a)2 fll(a) where a<e<b.


Solution:
2!
nee
Choose a function g( x)
g( x) = h( x) _(b - X)2 h( a) where
such that
b-a rin
h(x) = f(b)- f(a) -(b -X)f'(X) ........(1) g.n
Now g(a)=h(a)-h(a)=O and g(b)=h(b)-O=O from (I) and L.M.V
theorem.
e t
g(a)=O=g(b) .
It is given that f'ex) is continuous in [ a, b] and differentiable inC £I, b) .
Hence, so areh(x) and g (x) . :. g (x) satisfies all the conditions of
Rolle'stheorem.Hence 3e (a<e<b) 3g '(C)=0 ........(2)
g I ( X) = h' ( X ) + 2 (b - C! h ( a) from (I )
(b-a)
=-(b-X)f'1 (x)+ 2(b-X![f(b)_ f(a)-(b-a)/(a)]
(b-a)

Downloaded From : www.EasyEngineering.net


Downloaded From : www.EasyEngineering.net

Mean Value Theorems and Functions of Several Variables 129

Using (2) we get,

-C!
(b-c )f"( c) = 2( b [f(b) - f(a) -(b - a)f'( a)]
(b-a)
(b - a)2
or f(b)=f(a)+(b-a)f'(a)+ 2! f"(c) ........(3)
Taking (' = a + Biz where h = b - a and 0 <B<1

(3) gives f(a + h) = f( a)+ Jif'(a) + ~2! f'(a +Oh) where 0 < (1 < I

ww
3.5.6 Example:

w.E
f (x), g (x) and h(x) are three functions derivable in ( a , b ), show that there
exists c in ( 'I, b ) such that

a
f'(c) g'(c) h'(c)
f(a) g(a) h(a) =0 syE
f(b) g(b) h(b) ngi
deduce L. M . V and C . M . V theorems.
nee
Solution:
rin
f'(x) g'(c) h'(c)
¢ ( x) = f (a) g ( a) h(a) g.n
Consider the function
f(b) g(b) h(b)
=0 ........ ( I )
et
Clearly ¢(a) = 0 = ¢(b)
Also f( x),g( x) and h( x) are all derivable in (a, b),

:. All the conditions of Rolle's theorem are satisfied. Hence 3c E (a,b) such
that ¢' ( c ) = 0
f'(c) g'(c) h'(c)
(i.e.,) f (a) g ( a) h(a) = 0 ........ (2)

f(b) g(b) h(b)

Downloaded From : www.EasyEngineering.net


Downloaded From : www.EasyEngineering.net

130 Engineering Mathematics - I

Taking g(x) =x and h(x) = I we have from (2)

f'(e) I 0
f(a) a 1 =O=>f'(c)(a-h)-[f(a)-f(h)J=O
f(h) b 1

f(h)- f(a)
(i.e.,) f'(e) = where e E (a,b) ........ (3)
h-a
(3) shows that L . M . V theorem can be deduced from (I) Taking only h (x) =I
ww f'(e) g'(c) 0

w.E
we have, f (a ) g ( a )
j(b) g(b)
1 = 0 where c E ( a, b)

asy
=> f'(e)[g(a)-g(b)J-g'(c)[f(a)- f(b)J=O
f'(c) En
f(b)- f(a) . .
=> -(-) = ()
g' e
()
g h - g a
gin
where c E (a,b) which IS C. M . V theorem.

3.5.7 Example:
e
Apply Maclaurin's theorem with Lagrange's from of remainder for the function
eri
ng.
2 2
f (x) = eX and show that 1 + x + ~ ~ e ~ 1 + x + ~ eX for every x ~ 0 .
t

2 2
Solution:
Maclaurin's theorem with L form of remainder is
2 n-I n
net
f(X) = f(o)+~r'(o)+~ f"(O)+ ... +1~_1 in-I) (0)+ ~~ f" (Ox)
~ ~ ~
2 3 n-I n
X + -X
eX = 1+ X + T=) + ...... + IX X Ox (SInCe
~ _ 1 +- e
. Dn ( eX) = eX)
~ lJ ~ ~
........ (1)
where 0 < (} < 1
Taking n = 2 in (1) we get
X2
eX = 1+ X + ~ eOx ........ (2)

For every X ~ 0 we have eOx ~ eX where 0 < (} < 1

Downloaded From : www.EasyEngineering.net


Downloaded From : www.EasyEngineering.net

Mean Value Theorems and Functions of Several Variables 131

1 1
x- Ox 1 x- x
1+x+-e· S +x+-e
II ~
........ (3)
Further e
Ox
> 1 whenever x ~ 0 and 0 < 0 < 1
2 2
X X fh
l+x+-sl+x+-e
II II
........ (4)
From (2), (3) and (4) it follows that

ww 1 X
2
x Ox x x,
l+x+-s +x+-e =e sl+x+-e
II II
2 2

II
w.E
3.5.8 Example
. Taylor's theorem to prove that loge ( 1 + x ) < 1- x + x 2 3
2 3

Apply
asy whenever x> 0 .

Solution:
According to Taylor's theorem
En
gin h h2
I( a+h) = l(a)+11 1'( a)+ ~ f"(a) + ... + In-l f"- (a)+ R"
h"- I I

h" (1-0),,-" eer


where Rn = ~
n- .p
I" (a+Oh) , 0<0<1
ing
RII with Lagrange's form (n = p)is Rn =
h"
~ f" (a+Oh)
.ne
Substituting a = I and h = x in (1)

1(1 + x) = 1(1)+ ~f'(I)+


2 n-I

~ 1"(1)+ ... + 1:-1 in-I) (1) + ~ I" (1 +Ox)


II
t
(_1)"-1 In-l
Taking I (x ) = log x, I" (x ) = n
X
3
x2 x
.. 10g(l+x)=O+x--+ 30<O<I(Takingonlyupto3terms)
II 3(l+Bx)
Foranyx>O, (1+0x»1

Downloaded From : www.EasyEngineering.net


Downloaded From : www.EasyEngineering.net

132 Engineering Mathematics - I

3.6 Functions of Several Variables


3.6.1
Students are quite familiar with functions of a single independent variable.
Functions which depend on more than one independent variable are called
Functions of Several Variables.
(i) Volume V = nr2h ofa cylinder, r = radius, h = height,
curved surface area A = 2prh.
I
"2 xy, x = base, y = altitude
ww (ii) Area of a triangle A =

(iii) Volume of a rectangular parallelopiped v = xyz, where x = length, y = bredth

w.E z = height are all examples offunctions with more than one variable. (i) and
(ii) are examples offunctions oftwo variables and (iii) is a function of three
variables.
asy
Let z =j(x, y) is a function of two independent variables. Here x, yare independent

En
variables and z is the dependent variable and let the function f(x, y) be defined in
a region R.

gin
Suppose (x, y) be a movingpoint and (a, b) a fixed point in the region R ofthe xy-

P2' P3 etc. e
plane. The point (x,y) may approach (a, b) along different paths (see figure) PI'

eri
ng.
net

3.6.2 Concept of a Limit


Let (i) f(x, y) be defined in a region R
(ii) (x, y) tend to (a, b) along any path
and (iii) E < 0 be given.

Downloaded From : www.EasyEngineering.net


Downloaded From : www.EasyEngineering.net

Mean Value Theorems and Functions of Several Variables 133

If ::3 0 > 0 such that Ij(x, y) -II < E, V Ix - al < 0, Iy - bl < 0, thenj(x, y) is said to
tend to I.

We write Lt j(x, y) = 'I


x~a

y~b

or as Lt I(x, y) = I
(x,y)~(a,b)

ww
3.6.3 Example

w Consider
.Ea Lt
x-~2
y->!
x2 + y2
---'--, it can be seen that if
4xy
E = 0.01 (say) then

x2 + y2 5
- -8 <
syE
whenever Ix - 21 < 0.02, lv - II < 0.02 thus 0

n
E = 0.02
4 xy

satisfies
gin
)
x- +,v-
4xy
)

-"8
5
< 0.0 I. Hence the desired limit is ee"8 .r 5

ing
Note: 1
Let Lt f(x, y) = I and Lt g("(, v) = m .ne
then
«,y)-->(a,b) .

(i) Lt
(x,y)->(a,b)
[f ± g]
(X,Y)-'(a,b)'

= I±m t
(ii) Lt [fg] = /m
(x,y)->(a,b)

Lt I 1
and (iii) (x,y)->(a,h) g III

Note: 2

(x,y) LI
~ (a, b) exists iff LI
(x-~a) y~b
[ LI 1
I(x,b) == Lt [
(y~b) x~a
Lt f(x,h) 1

Downloaded From : www.EasyEngineering.net


Downloaded From : www.EasyEngineering.net

134 Engineering Mathematics - I

3.6.4 Example
~

x- + V
f(x, y) = . 2 ' find Lt as (x, v) ~ (2, I)
2x+ y .
Solution

LI [Lt
y~ x~2
2
x +y LI 1 [4
+ 5
2X+;2 . = y~ 4+ y2 =5=1
y]
..... (i)
2
Lt [Lt x +y ] Lt [X2 + I] 5
ww x~2 y~l
The two limits (i) and (ii) are equal.
2x+ y2 = x~2 2x+1 =5=1 ..... (ii)

w Henee .Ea Lt
'J
x' + Y
---=-;;-=1

3.6.5 Example
syE
(x,y) ~ (2,1) 2x + y2

J
x- - Y'
)
n gin
f(x, v)
.
= ? J
x- + y-
find whether the limit exits as (x, y)
e eri
~ (0, 0)

Solution
ng.
n et
I,I L/ 2 2]
x - Y LI I[""' I,/ x] - Y 2]
Thus
X ~ 0 [Y ~ 0 J
x- + y-
') 7:-
y ~ 0 ly ~ 0 x- + y ? )

lIenee the limit does not exist.

3.6.6 Example

Lt ~
xy
')

Find (x,y) ~ (0,0) y- - x-

Downloaded From : www.EasyEngineering.net


Downloaded From : www.EasyEngineering.net

Mean Value Theorems and Functions of Several Variables 135

Solution
Puty = 11/X

2
Lt xy Lt mx
(X,y)~(O,O) / __ X2 X~O m 2 x 2 __ X2

It is clear that limits will be different for different values of m i.e., the limit depends
upon the slope of the path along which (x. y) approaches (0,0). Hence the limit does not
exist.

Exercise - 3(0)

wwI. Examine whether the following limits exist. Find them if they exist.

(i)
w.E
(x.y)
Lt
~ (2,1)
x 2 ~ y2 + 4
3xy2
(ii)
(x, J~
Lt
-~
x 2 + 4y2
(0,0) y2 ~ 2X2

(iii)
(x.y)
Lt
~(2,2) xy~2x
asy
yx~2y
(iv)
Lt Y4
X

(x,y) ~ (0,0 ) x-J ~ y-J


4
~

Lt En J J

gin
x-y-
(v)
(x,y) ~ (0,0) x 2 + y2

3.7.1 Concept of Continuity e eri


Suppose (i) Lt
(~,y) ~(a,b)
j(x. y) exists
ng.
and (ii) Lt
(x,y)~(a,b)
f(.~, y) = j(a, b) net
thenj(x. y) is said to be continuous at (a. b).
Note: 1
Ifj(x. y) is said to be continuous at every point of a region R, then it is said to
be continuous in R.
Note: 2

Letj(x. y) and g(x, y) be continuous at (a, b) then f ± g,fg, and f (g -:t 0)


g
are all continuous at (~ b).

Downloaded From : www.EasyEngineering.net


Downloaded From : www.EasyEngineering.net

136 Engineering Mathematics - I

3.7.2 Example
Consider the function/ex, y) = x 2 + Y - 2x when (x, y) 7= (0, 0) and.l( I, I) =0
Lt f(x,y) = Lt [Lt (x 2 + y-J -- 2x) ]
(x,y) ~(I,I) x~ 1 y ~ I

=/(I, I)
Hence the function is continuolls at (1, 1)

ww
3.7.3 Example

w.EConsider the function f(x, y)


. .
=
2

x + y-
2
~ y J when (x, y) 7= (0,0) and.l(O, 0) = 0
Let (x, y) ~
asy
(0, 0) along the path y = mx

(.Y,y) En
Lt
~ (0,0)
f(x,y) =
Lt
~y 2
(x, y) ~ (0,0) x- + y
2 2

gin Lt x 2 m 2 y2

eer (x, y) ~ (0,0) x 2 + 1112 x 2

Ltm2x 2
(.r, y) ~ (0,0) I + m
--2 = in° g
=.1(0 0)

lienee .l(x, y) is continuous at (0, 0)


,

.ne
3.7.4 Example t
Consider the function.l(x, y) =
xy
x2 _ y2 (x, y 7= 0, 0), .1(0, 0) = °
• 2
Lt xy Lt mx
(x, y) ~ (0,0) x 2 _ y2 (x, y) ~ (0,0) x 2 (I - m 2 )

along the path y = niX

= - - 7=
1-JIl
III
2
.1(0, 0) except when III = °
Hence the given function is discontinuous at (0, 0)

Downloaded From : www.EasyEngineering.net


Downloaded From : www.EasyEngineering.net

Mean Value Theorems and Functions of Several Variables 137

3.7.5 Example
2 2
X Y
Consider the functionj(x, y) = (x, y) cf. (0, 0)
~X2 _ y2

= ° at (x, y) = (0, 0)
In this case it is convenient to introduce (polar co-ordinates). Substitutions x = rcosB,
y = rsinO.

x 2 y2
ww l"(2
r4cos20sin2B
+ y2 - Jr2 (cos 2
0 + sin 20)

w.E r'
-(sin 2 20)

a
4

syE 4

= {x2 + y2 ngy2 i
(x-, +y-, )3 2
4
nee
Now ~-
E.
provided /x/ <
I
E· 3
rin
4

/x - 0/ <
2

i3
lY - 0/ <
I g.n
thus when E and E3

et
..... (i)

Hence
Lt
(x,y)~(0,0)~X2 + y2
x 2 y2
---;===== = °
frol11 (i)

=j(O, 0)
2 2
j(x, y) = Jxx Y+ y2
2- is continuous at (0, 0)

Downloaded From : www.EasyEngineering.net


Downloaded From : www.EasyEngineering.net

138 Engineering Mathematics - I

Exercise - 3(E)

I. Investigate the continuity of the following:


(i) j(x, y) = 2x + ),2 } (x, y) t= (2, 3)
=0 (x, y) = (2, 3) at (2, 3)
Ans: Not continllous
xy
(ii) j(x.y) = - - , (x, y) t= (0, 0) at (0,0)

ww ~Xl + y2

w.E
=0 (x, y) = (0, 0)
Ans : Not continuolls.

a x-y
(iii) j(x,y) = - - ,
2y+x
=0 syE
(x, y) t= (0, 0)

(x, y) = (0,0) Ans : Not continuolls

3.8.1 Partial Differentiation ngi


nee
Consider z = j(x, y) where x and yare independent variables and z is the
dependent variable.

rin
Keeping one of the two variables x and y as constant and allowing the other to

g.n
vary we get a partial derivative of 'z' with respect to the variable that is varied.
Keep 'y' constant and allow x to vary, then partial derivative of z w.r. to 'x' is

obtained and is denoted by az or f (x, y) or Dj{x, y) or 8f


ax x ax
et
az Lt j(x+Jx,y)- j(x,y)
So =
ax ax~o bx

Similarly az = Lt f(x,y+Jy)- j(.x:,y)


By ay~o by

Note: 1
z = j(x, y) represents a surface in the cartesian co-ordinates (x, y, z) system.
The section of the surface z = jex, y) with the plane x = k (parallel to yz plane) is a
curve. (Similarly the sections with planes parallel to xy - plane and zx - planes also
will be curves).

Downloaded From : www.EasyEngineering.net


Downloaded From : www.EasyEngineering.net

Mean Value Theorems and Functions of-Several Variables 139

Note: 2

az)
( Ox gives the slope of the tangent at (x, y, z) to that curve
x=k

[
~z 1 gives the slope of the tangent at (x. y. z) to the curve obtained as
~ y=k

the section of the surface z =f(x. y) with the plane y = k.

3.8.2 Example

ww az , z = x3 + x2y + I

w.E
(i) To find
ay
we keep 'y' as constant

partial derivative w.r. to x of x 3 = 3x2


a syE
partial derivative w.r. to x of x 2y
partial derivative w.r. to x of I
=

0
2xy

n
=

..
az
ax = 3x2 + 2xy
gin
(ii) To find az , we keep 'x' as constant
ax
e eri
partial derivative w.r., to y ofxJ = 0 ng.
partial derivative w.r., to y of x y
partial derivative w.r., to y ofy2 = 2y
2
= x 2
n et
az
- =x2 + 2y
ay
3.8.3 Example
f= x 2 + 1- 2xy
af
- =2x-2y
ax
af
- =2y-2x
ay
3.8.4 Example
f= (x - y) (x + 2y)

Downloaded From : www.EasyEngineering.net


Downloaded From : www.EasyEngineering.net

140 Engineering Mathematics - I

This is a product, and the usual product rule applies


at' = (x - y) (1 + 0) + (x + 2y)( I - 0)
-'
ax
= x - y + x + 2y = 2x + y
aj
- = (x - y) (0 + 2) + (x + 2y) (0 - 1)
ay
= 2x - 2y - x - 2y = x - 4y

ww
3.8.5 Example

w
Solution
x+y
y
Iff= x-2 , Find af and aj

.Ea ax ay

syE
Applying the Quotient rule
aj
ax n
(x+ y)(I-O)-(x-2y)(I+O)
(x + y)2
gin
3y
(x+ y)2

aj
ay
(x+ y)(-2)-(x-2y)(O+I)
(x + y)2 e eri-3x
= (x + y)2

3.8.6 Example ng.


Iff = sin(ax + by), sino aj and aj
ax ay
n et
aj
- = cos(ax + by) x -
a (ax + by) = acos(ax + by)
ax ax
a a
ay = cos(ax + by) x ay (ax + by) = bcos(ax + by)

3.8.7 Example
Consider f = ax2 + hxy - by

Then
aj
-
at
= 2ax + hy - ' = 2by + hx
ax ' ay

The expressionix = 2ax + hy is itselfa function ofx andy. We could therefore


find its partial derivative with respect to x or y.

Downloaded From : www.EasyEngineering.net


Downloaded From : www.EasyEngineering.net

Mean Value Theorems and Functions of Several Variables 141

(i) If we differentiate it partially w.r. to x, we get ~ [Of]


ax ax
and this is

a
ax-
2
a I]
[- , = - (2ax + hy) = 2a
ax

ww This is called the second partial derivative of 'f \V.r. to 'x'.

!L (a~)
w.E If we differentiate Of partially w.r. to 'y' we get
ax ay ax
and this is -

written as - -
ail
ayax asy
En
gin
Thus
al
-
ax
= 2ay + hy, -8f
ay
= e
hx - 2by eri
ng.
net
Similar steps can be carried out with the expression for 8fay

a~ ~
2

ay
= (8f)
~ ay
= -2b

aI
axay
2
= al
ax
(8f)
~
= h

Downloaded From : www.EasyEngineering.net


Downloaded From : www.EasyEngineering.net

142 Engineering Mathematics - I

3.8.8 Example
2 2 2 2
al 8f a 1 a f a 1 a 1
Iff= ax3 + hx2y + bl, find - ' - ' - 2 ' - 2 , - - , - -
ax ay ax ay ayox axay
Solution
af
- =3ax2+2hxy~ -
8j ')
=hx2+3by
ax ay

ww
w.E
asy a a 2
1
In this example also, we see that - - = - ' -
2
f

En ayax axay
3.8.9 Example
gin
Iff= log(x2 + y), prove that

a21 e a21
eri
axay ayax
ng.
Solution
8j = 2x
net
a2f -4xy a2f -4xy
r
ayax = (x + y2 axay = {x + y2
2 2 r
axay ayax
3.8.10 Example

) au au
Ifu(x + y) =x2 + y~ then prove that ( - - - ) 2 =4( 1 - - - -)
au au
ax ay ax ay

Downloaded From : www.EasyEngineering.net


Downloaded From : www.EasyEngineering.net

Mean Value Theorems and Functions of Several Variables 143

Solution

Since u=
x+y
2
au x 2 +2xy- y2 . _au = _X +2xy+ y2
ax (X+y)2 'ay (X+y)2

au _ au 2 =[7
x- +2xy- y 2 +x-7-2xy- y-7]2
( ax ay ) (x+ y)2

ww
w.E ..... (i)

a syE
ngi ..... (ii)

(i) and (ii) prove the result n eer


3.8.11 Example
i ng.
a2 z a2 z
If z =

Solution
2(ax + byi - (x 2 + y) and a2 + b2 = 1, then prove that
ax net
-2 +-2 = 0
ay

az
ax = 4(ax + by) . a - 2x

a2 z
- = 4a2 -')~
ax 2
az
ay = 4(ax + by) . b - 2y

Downloaded From : www.EasyEngineering.net


Downloaded From : www.EasyEngineering.net

144 Engineering Mathematics - I

3.8.12 Example

If z = f ( -Xl] aZ az
then prove that x - + 2y- = 0
y 8.:r Oy

Solution

ww
w.E
a axsyE
x az + 2y az =f'(Xl][2X2 _ 2X2]=0
ay y y y
..... (i)

a 2z az a 2z
ngi
Differentiating (i) partially w.r., to 'x' .

x-+-+2y--=0
ax 2 ax axay nee ..... (ii)

Differentiating (i) w.r., to 'y' partially


a 2z a 2z
r ing
ayax
az
x--+2-+2y-=0
ax ay2 .ne ..... (iii)

Multiplying (ii) by 'x' and (iii) by 'y' and adding t


2
2 a z az a2z ::.2
u Z
a'Z ::.)
2 u- Z
x -+x-+2xy--+xy--+2y-+2y -=0
ax 2 ax axOy Oyax Oy ~l

az az
But x-+2y-=0
ax Oy
) 2 ::.)
2 a-z a z 2 u-Z
x -+3xy--+2y -=0
ax 2 axay Oy2

3.8.13 Example
If u = log(x3 + T + z3 - 3xyz) then prove tllat

Downloaded From : www.EasyEngineering.net


Downloaded From : www.EasyEngineering.net

Mean Value Theorems and Functions of Several Variables 145

Solution

...... (i)

ww au
Now
w.E ax
-3- - - - - - ( 3 x 1 - 3 yz)
x + y3 + Z3_ 3xyz

a all

au
~y
syE
3 3
x + y +z -3xyz
3 (3y2-3xz)

az
1 ~

ngi
-3- - - - - - ( 3 z - -3xv)
x + y3 + Z3 --3xyz .

au
- +- +-
au au nee
3x 2 +3yz-3 y 2 -3xz+3z 2 -3xy
= -----'--,-----'-------,.-------'-
Xl +./ +Z3 -3xyz
ax ay OZ
rin
3
x+y+z
(.: (x + y + z) (Xl
g.n
+ y2 + Z2 - xy - yz - zx)

Hence from (i)


= x 3 + y' + Z3 - 3xyz)
et
LHS = (! + ~ + ! J( x + : + z 1
=3(- 1
{x + y + Z )2
_ 1
{x + Y + Z )2
- 1
{x + Y + z f
1
-9
=--------:-
{x+ y+ Z)2

Downloaded From : www.EasyEngineering.net


Downloaded From : www.EasyEngineering.net

146 Engineering Mathematics - I

3.8.14 Example

r'
If II = en e 40 then find 'n' so that -au = -
1-a (r? au
-)
ao ,.2 ar a,.
Solution

au -_ nen 1
e
-r'
-40
+
On -40--
--I' (
.e .
~?
2
1
ae 4e

ww ..... (i)

w also
.Ea
syE
n gin
?]
e eri
-1[4lt
== 2e r 20 + 3r-u
ng.
n et ..... (ii)

Equating (i) and (ii) we get

n -3
e 2e

-3
Hence n==-
2

Downloaded From : www.EasyEngineering.net


Downloaded From : www.EasyEngineering.net

Mean Value Theorems and Functions of Several Variables 147

Alternative method :

Taking logarithms on both sides


)
r-
log u = n log 0 - - ..... (I)
40
Diff. (I) partially w.r.t 8

I au 11 r2 au ( r2)1l

ww --=-+-
u ae 8 u0 2
- = 11+-
ae
-
40 8 ..... (2)

w Diff.( I) partially w.r.to 'r'

.Eaar u
I au - r
--=-=:)-=--
20
au - u. r
or 28
s
ar yE
r--=---
) au -11.,.3

a( au) _-I [ u. 3r
28
ngi.1( -1/1")]
-
ar
I .2 -
ar
--
28
nee
2 +r --
20

ri ng.
n et
..... (3)

From (2) and (3) we get


r2 _3 r2
11+-=-+-
48 2 40
-3
.... 11=-
2

3.8.15 Example

?
If x- yY zZ = k then prove that at x = y =
a2 z
z, - - = [x log exr I
axBy

Downloaded From : www.EasyEngineering.net


Downloaded From : www.EasyEngineering.net

J48 Engineering Mathematics - I

Solution
Applying logarithms to.r y zZ = k, we get xlog x + ylog)' + zlog z = log k.
Differentiating this implicit function partially w.r., to 'x'.

x
1
x-+logx+ z.-+Iogz )az
z
-=0
ax
(I
az
(I + logx) + (1 + logz). ax = 0 ..... (i)

az (I + logy)
ww ax l+logz

w.E
/Illy
az
ay
=
(I + logy)
1+ logz

asy
Differentiating (i) partially w.r., to 'y'

En
{1+logz )-a2Z
- +aZ[1
- - .az]
- =0
ayax ax.z oy
Substituting x = y = z, we get gin
a I =0
(I + logx)_z_+-
2

axay x e eri
I I ng.
-axay- - - x{1 + logx) - x{loge + logx)

or - (xlogex) I
net
xlogex

Exercise - 3(F)

x+4y OJ of -17y 17x


1. If f= 4x _ y' find -8 and 7) (ADS: (4x- y)2' (4x-- y)2 ]
x Y

az az
2. If z = ax2 + bxy + by/inti ax' ay (ADS: 2q.x + ~y, bx + 2by]

Downloaded From : www.EasyEngineering.net


Downloaded From : www.EasyEngineering.net

Mean Value Theorems and Functions of Several Variables 149

. OZ {)z
3. Ifz = (ax + by)(a\" - by), tind -ax '-0y

oz oz
4. Ifz=tan(ax+ by), find ox'ay [ADS: asec 2(a," + by), bsec 2(ax + by»)

sin(ax + by) oz OZ
5. Ifz=
xy
find ox' OU.J'

ww [ADS: [
a\"cos(a\" + by) - sin(a\" + by) bycos(a\" + by) - sin(ar + b
x2y , x2y
Y2 )
I

w.E
6. If 11 = log(siny + ysinx), then prove that

a a2 u
-----
ax~v
syE
a211
~vJx

ngi
)
x- - y-
)
nee
ayax X
2
+ y- J

rin
8. If u = tan
I XY
.
~X2 + y2 + I
then prove that g.n
et
9. If u = --I .. x'-- prove that
ct 2 e 4c 't

au ) a2u
-=c---
at ax 2
10. If 11 = .I(x + Ky) + g(x - Ky) then show that

a2u = K2 a2u
ay2 ax 2

Downloaded From : www.EasyEngineering.net


Downloaded From : www.EasyEngineering.net

150 Engineering Mathematics - I

V z
II. If 1I =~ + - prove that
z x

x au + y ay + z au =: 0
ax . uy oz
12. I f II = log(x3 t- y3 -f :;3 ~3xyz), prove that
all au
-+-t----=:~--
UII 3
ax ~)I az x+y t- :;

I,,) I

ww
13. I f 1I =.I(r) W Ilere r = .jx" + y + z- prove t HIt

w x .Ea y z
14. [1' 11 = ~-
y+z z+x
syE
+ _.- -+ - - then prove that
y+z

n gin
e eri
l' ng. lADs: -I)

16. If u = xyf (; prove that


n et
au all
x-- + y-- = 211
Dx . ~v

17. If x = rcosO, y = rsin0, then prove that


0 20 0 1 0
- , +-) =0
ox- ay
18. If 1-1 = logr, r = x + );J - x 2y - xy2, prove that
3

19. lf 11 = x 2y + J,2z + z2x , prove that


au+ -au+ -au= (x+ V+z-)~
-
ax ~ az .
Downloaded From : www.EasyEngineering.net
Downloaded From : www.EasyEngineering.net

Mean Value Theorems and Functions of Several Variables 151

20. If u = .x3 + i - 3ary then prove that


a2u a211

21. If u = e ax + by j(ax - by), prove that


au all
h-+a-=2abu
ax 0;
2
X y-) z-)
22. If - ?- + - 2- + - 2 - - = I , prove that

ww a- +u

(-au)2
b +u c +u

+ (au)2 + (au)2 -2( allax au au)


w.E ax
-
0;
-
az
- x-+ y-+z-
By az
3.9.1 Composite Functions
asy
If u is a function of two variables x and yare themselves functions of an
independent variable I.
En
au au ax au av
then -=-.-+-.-'
at ax at ay al gin ..... (i)

e eri
Suppose now that 'u' is a function of the variables x and y, and x and yare
themselves functions of two other variables rand s.

then
au au ax au ay
-=-.-+-.- ng. ..... (A)
ar ax ar ay ar
all au ax au 0;
-=-.-+-.-
net..... (8)
'as ax as 0; as
II Ily if u is a function of rand s, where rand s are themselves functions of x and
ythen
au au or ax as
-=-.-+-.- ..... (C)
ax ar ax as ax
au au or au as
-=-.-+-.-
0; ar ay as ay
The second and higher-order partial derivatives of'u' can be obtained by repeated
application of the above formulae. Also, the formulae can be extended to functions
of three and more variabl~s.

Downloaded From : www.EasyEngineering.net


Downloaded From : www.EasyEngineering.net

152 Engineering Mathematics - I

3.9.2 Example
If z = fix, y), x = ell + e- v, y = e-l/ eV then show that
az az az az
---=x--y-
au av ax ay
Solution
az GZaX azOy az u az
-=--+--=-e --e
u
au ax .au Oy .ax ax . av .

ww az az ax az Oy
-=--+--=--e
av fu·av Oy·av fu·
az v az
--e
Oy.
v

w.E
By substraction

a az - az)_(
(-
au av
syE
- - e v)aZ
11
- - ("
ax
+e e -e v)aZ
-
ay
az y--
=x---
ax ay
az
ngi
3.9.3 Example nee
If u = j(x, y), x = rcos(J, y = sinO prove that
rin
g.n
Solution
et
we know that

au = au. ax + all .ay = au cosO + au sin 0


ar ax ar Oy ar ax Oy
au au ax au Oy au . au
- = - . - + - . - = -(-rsll1 0) + -(rcosO)
as ax as ay ae ax ay

(-Or )2+2 ()2 =cos-O.()2


au lau-
au +2cosOsmO-.-+
-
. auau
1

rae ax axOy

Downloaded From : www.EasyEngineering.net


Downloaded From : www.EasyEngineering.net

Mean Value Theorems and Functions of Several Variables 153

. ~ au 2 .
SIIl-O(- ) +SIIl 2
au -~
0(-) . au au
-2cosOsIIlO-.-+ COS 2 0(-au )2
c3y ax ax ay c3y

3.9.4 Example

ww If u = j{x - y, y - z, z - x) prove that

au + au + al~ = 0
w Solution .Ea
ax c3y az

syE
Let X = x - y, y = y - z, Z = z -x then u
all all ax au ay au az au au
= f(x, Y, Z)

-=-.-+-.-+-.-=---
ngi
ax ax ax ay ax az ax ax az
lilly
au au all au au az
ay = ay - ax' az az ay nee
Adding all these gives rin
au + au + au =0
ax c3y az g.n
3.9.5 Example
e t
If z = j{u, v) Whereu x2 -y and v = 2xy
=< prove that
az y-=2(x·
x-- az - y)az?
)-
(a) ax ay au

(b)

Solution
u = x2 - y, v = 2xy
au
ax =2x
- 'c3y au =-2y, av
ax =2y, av
c3y =2x

Downloaded From : www.EasyEngineering.net


Downloaded From : www.EasyEngineering.net

154 Engineering Mathematics - I

..... (i)

..... (ii)

From these, we get


at at 2 2 aZ
x - - y - = 2(x + y ) -

ww ax ay

From (i) and (ii) we obtain


au

w .E(-J az -
7
az
+( - )2 = {[ az az - az az-
4 x-+y- J? + [-y-+x- J? }

asy
ax By au av all av

En
gin
3.9.6 Example
e
If x = reos Bandy = rsill B, prove the following:
eri
1.
ax
-=eose·
as
ax
' as
. e.
-=-rsll1
'
ay = sine· By = rsinE>
'as ng.
2.
ar
ax
x
~X2 + y2
,
ar
By
y
~X2 + y2
ar

,
as
-
ax
=- ?
y
x- + y-? '
as
-
By
x net
x2 + y2

3. (:r +(: r =1

Downloaded From : www.EasyEngineering.net


Downloaded From : www.EasyEngineering.net

Mean Value Theorems and Functions of Several Variables 155

Solution
I. We have x = reos O. Differentiating this relation partially w.r.t., rand w.r.t., '0,
we get
ox = cosO· -o=r .
-
or ' ao -rsl110 ..... (i)

Similarly, the relation y = rsinO yields

oy = sin {} oy = reosO .... (ii)


or 'ao

ww 2. From the relations x = reosO, y


x 2 +1=,2
= rsinO, we get
..... (iii)

w .Ea y = tall 0
x
.... (iv)

syE Dr or
Differentiating (iii) paliially w.r.t., 'x' and w.1:l. ), 'we get -0 ;-0
x y
This gives
or x ngi
or y y
x
ox r Jx 2 + y" ' nee ..... (v)

Differentiating again (iv) partially w.r.t., 'x' and 'y' we get


rin
y( __1 ) = sec" 0 ao ,
x2
~ = sec" 0 ao
~v
g.n
which gives
ox x
e t
-ao = - -,
y = cos " 0 = --,
y =, y ,
..... (vi)
ox x· r- x- + y

00 cos" 0 x x
-=--=- .... (vii)
oy x ,.2

3. From (v) we get


2
or)2
( ox +(or)"
oy r- r-
=X=< =<
+y2 =1
r2 ..... (viii)

(::l\(:r~1
Downloaded From : www.EasyEngineering.net
Downloaded From : www.EasyEngineering.net

156 Engineering Mathematics - I

4. From (v) we also get

..... (ix)

..... (x)

Adding (ix) and (x)


a2 ,. a2r I

ww -+
2 -=-
ax ay2 r ..... (xi)

w.E
5. From (ix) and (x), we get

asy ..... (xii)

Differentiating ar
ay En
=Y partially w.r.t., 'x' we get
r

gin
eer
ing
So that
.ne
..... (xiii)

From (xii) and (xiii) we get that


t
..... (xiv)

6. From (vi) we get

Similarly from (vii), we get

..... (xvi)

Downloaded From : www.EasyEngineering.net


Downloaded From : www.EasyEngineering.net

Mean Value Theorems and Functions of Several Variables 157

Adding (xv) and (xvi) we get


2 ~
a 0 a-A 2xy 2xy 0
----:? + - 2 = -4- - - . 1 =
ax ~v r r

Note:
I. The variables (I', 0) related to (x, y) through the relations x = rcosO, y = rsinA are the

ww polar co-ordinates of the point whose cal1esian co-ordinates are (x. y).
2. In obtaining the resllits in (i) rand 0 are regarded as independent variables and

w.E
x and yare regarded as dependent variables.

3. In obtaining the results (ii) - (vi), x andy are regarded as independent variable~ and

asy
rand 0 are taken as dependent variables.

En
4. After partial differentiation is carried out, the final expression for the partial derivatives
are to be expressed in terms of the independent variables.

. gin
or ax ax ao .
5. From the expressIOn for - , - ; - , - , It may be observed that -
ar
is not the
ax ar as ax
eer ax
.
reclproca I 0
fax ax.IS not the reclproca
or an d ae . I 0f ao
ox'
ing
I n Clact we note IJere

ar = ax and ax = 1'2 as . Similarly ar = 0' ay = 1'2 as


ao ax 0' ar ' as 0'
. .ne
6.
ax ar
The results (i - vi) arc useful in calculations involving polar co-ordinates. t
Exercise - 3(G)

I. If u = x + y, v = xy and f = feu, v), show that


aj Of Of Of
x-+ y-=u-+2v-
ax 0' au av
2. If z = 1(r, y) and x = e" + e- X and y = e- II -ev show that
az az az az
---=x--y-
au av ax 0'

Downloaded From : www.EasyEngineering.net


Downloaded From : www.EasyEngineering.net

158 Engineering Mathematics - I

3. If z = ttu, v) where II '--= x2 - y2 and v = 2xy prove that


OZ UZ 2 2 ()z
x - -- v- == 2(x + y ) -
ox . oy OU

4. If II = (!"siny. v = e'cosy and z =.f(1I, v) prove that

ww 5. If Z = /og(u 2 + v). U = eX' +/.

!!..~ _ 2.\"(21/ 2 + I)
V = x2 -f y, show that

w.E
2
OZ _ 4Y1l + 1
ox - /1
2+V • ~v -
1/ + v

6. If w =.f(u. v) and u = x 2
asy-I, r 1 v = 2.\:v prove that

En l--+--
o2 W U-W
--+--=
olr
)
J 4(.2
) .\ +Y
2)
~v-
J
o-W

OU
;-,J
u-W
2)
ov-

x V gin
7. Ifu = -;V=-'--
z z
eer
and w =zandf=}(u, v, ll~ show that

Of Of Of Of
x-+y-+z-=w-
ox i oy oz ow ng.
8. If z = }(x, y), x = 11 cosh v, y = 1I sinh v prove that net
9. If z = }(u, v), It = Ix + II~Y, l' =- ~v - I11X, show that

a-J +
- z 0 2z (/ 2
ox ~2
2
- = +111 2)(2
0 Z+0-
- 2Z
0,,2 ov 2
1
10. If z =f(u, v), U = x2 -I, v = 1- x 2 show lhat
OZ oz
x-+ y-=O
~v ox

Downloaded From : www.EasyEngineering.net


Downloaded From : www.EasyEngineering.net

Mean Value Theorems and Functions of Several Variables 159

II. If x = ell /e/1/V.)' C~ ell sec)' fino the vallie of

( x~l!-ax + .}' Dy(~)' 1x (x ax'


vv + }'-~l
~v
lADS: 0 I

(
. II == 2
Ilmt I Iog(y-? -. x-)
) an d v::= Sill
. I( yx II
12. If w = .f(x2 - y, )~ .. z2, 22 -- x 2) prove that
-.!. ow + _~_ ow + _~ all' == 0

ww
13. If w = j(x, y), 11
x ax

e\ v
)' ~y z Dz
~ eY show that

w
0=

D2 w 3 2 \1'
.Ea --:=
3x~v
---.(111')
ouvv
14. If x::= -!vw,y::= j;;,z = -J;;;, syE Prove that
o<p 0(1) o<jl o<jl o<jl
x-+ v---+z--=u-+v--+w-
ax . ~Y
o<jl
oZ ngiov all all'
where <jl is a function of x, y, z.
nee
3.10.1 Homogeneous Function
Definition r ing
A functionj(x, y, Z, II .. .)

g(Yx '!-,!~
is said to be homogeneous in x, y,
.ne
Z, 11 ..... if it can be

written in the form xl1


x X
..... ) and is said to be of degree 11.
t
Ex. I. z = sin-I (~ ) + tan 1(~) is homogeneolls and is of degree '0'. Since

Z=xO[sin-I(~J+tan I:]

2 + 2
2• Z = x y .
IS IlOmogeneous an d'IS 0 f'd egree Isll1ce
'
x-y

Downloaded From : www.EasyEngineering.net


Downloaded From : www.EasyEngineering.net

160 Engineering Mathematics - I .

3. ~ = x 3sin(~) + A)!ZIOg(;1+ XZ2 tan I(~ 1

~ x'ISin(~)<~lot H~r tan'(; lJ


llence ~ is of degree' 3' and homogeneous in x, y, z.

3.10.2 Euler's Theorem

ww The following theorem, known as the Euler's theorem gives a very lIseful formula
for a specific combination of partial derivatives ofhomogeneolls functions.

w.E
Theorem :

asy
ffz is a homogeneous function ofx andy with degree '11' then
oz oz
En
x-+ y-=nz
ox . ~y
..... (i)

Proof :
gin
xllf(~)
e
Since Z is a homogeneous function of degree '11' we can write 'z' in the form.

eri
z=
ng.
Now ~: = I1X
Il - ~) + x l /(:)( ~;)
1f
(
net
= nxlllf(~)_xll 2y/(~)

and

xoxoz + yoz = f( xy) -x" 'y/ (y)


~y
I1X"
x
+ x" 'y/ (:Ix)
x

= nxllf(:Ix)

=nz [.:Z=Xllf(;)]

Downloaded From : www.EasyEngineering.net


Downloaded From : www.EasyEngineering.net

Mean Value Theorems and Functions of Several Variables

uz
x-+ V-=IIZ
oz
Thus oX . ~V

·3.10.3
Ir z is a homogeneous functions of x and y with degree 'n' then
2 J )
]OZ o-z 20-Z ( )
X - , + 2xy - - + y -i = 11 II - 1z ..... (ii)
ax - uxoy uy
Pr(}(l:

ww Diflerentiating (i) pUl1iaily w.r., to x, we obtain


02 Z OZ
x--+-+
02 Z OZ
y--=ll--
w.E ox 2

02 Z
ox oxoy
02 z
ax
UZ
I.e.,
ox'
asy
x-) + yx--=(n-I)-
oxoy ax
.... (iii)

En
Differentiating (i) par1ially w.r., to 'y', we obtain
02 Z 02 Z
gin
oz
y-+x--=(n-1)-
oyJ oyox oy ..... (iv)

eer
Multiplying (iii) by 'x' and (iv) by 'y' and adding we get

02 Z
? 02 Z ) 02 Z (OZ Oz)
x--? +2xy--+ y-? =(n-I) x-+y- =n(n-l)z ing
ox' oxuy oy' ox ry
.ne
3.10.3 General Formula
The Euler's theorem may be generalized to homogeneous functions of more.
than two variables. For a homogeneous function j(x, y, z) of degree 'n' in three
t
variables, the theorem can be read as :

x oj + y OJ + Z oj = nj
ax ry oz ..... (v)

In general, for a homogeneous functionj(xl ....x,,) of degree 'n' in 'k' independent


variables x\ .... x k the theorem reads
OJ OJ oj
xl - +x2 - + ..... +xk -
aXl aX 2 ax k =l1j ..... (vi)

The proofs of (v) (vi) are left to the student

Downloaded From : www.EasyEngineering.net


Downloaded From : www.EasyEngineering.net

162 Engineering Mathematics - I

3.10.4 Example

.
1/ 1I :~. cosec
-[;-;+JY]
I if;: + if;; ,thcn prove that x au au __ ~
ax + y ay - 6 tanu

Solution
We note that from the given cxpression for' u' we can write

cosecll
;-;+JY
= "il
.
;1.: f (say)=: ..... (i)
'Y.\" +vY

ww
w.E
Then
asy
En
gin
eer
ing
.ne
t
:. fis a homogeneous function of degree ~, Hence by Euler's theorem

W WI. I
x-+ y-=- j =-cosecu ..... (ii)
ax Oy 6 6

a a I
x-(cosecu) + y-(cosecl~ =: -cosecli
I.e.,
ax ay 6

Downloaded From : www.EasyEngineering.net


Downloaded From : www.EasyEngineering.net

Mean Value Theorems and Functions of Several Variables 163

Carrying Ollt partial differentiation, we get

I
--cosecu 1
au a 6 II
x-+ y-=-=----=--tanu
ax ~v - cosec II. cot u 6

x-
au + yau- = - -1 tanlt
ax ry 6

3.10.5 Example

J prove that x-+y-+z-=O


ww Ifu= j o( -,-,-
x y z
yzx
au au au
ax ry az

w.E o(x z)
Solution

asy 11= y
j -,-,-
y z x

E ngi
nee
0'( -----+---+-
r inJ g
all au au
x-+y-+z-=j
ax ry az
x z x y y z
y x y z z x
.ne
3.10.6 Example
=0
t
au all
If u = sin 1; + tan -1 ~ prove that x ax + Y ry = 0

Solution

u = f(x. y) is of degree '0'


By Euler's theorem

Downloaded From : www.EasyEngineering.net


Downloaded From : www.EasyEngineering.net

164 Engineering Mathematics - I

au ali
x-+ y-=o.u=o
ax 0'
. ,x ,y
Sill - = v, tan - - =w
Alter:
y x
u=v+w
au avow -::::-+-
-=-+-
au av Ow
ax ax ax' ay 0' 0'

ww x
y
. 1 av
-:::: Sill V, - = cosv-
y ax
w.E y - y ? aw
- = tan w, -} = sec- w.-
x x- ax
asy
-x av I
--:::: cosv- - = sec"w.-
) aw
y2
En 0" x 0'

gin
=0 eer
3.10.7 Example
ing
If ZI =
X3 + 1'3)
tan' - __
( x+y 0 - prove that
.ne
(i)
au au .
x - + y - = s1112u
ax oy
t
2 a-u a-u? au. .
J 2 ?

(ii) X - 2 +2xv--+ y-} =sm4u-sm2u


ax . axay ay-
Solution
Proof:
For the given expression for 'u' we have
x3 + y3
tanu = ----'-- = f, say
x+y

Downloaded From : www.EasyEngineering.net


Downloaded From : www.EasyEngineering.net

Mean Value Theorems and Functions of Several Variables 165

Ilere ~f' is a homogeneous functions of degree'2'

By Euler's thcorem
at" of
.\"-'
Ox +-
Oy .1' 7f'
= 1l.J =~ --:

I.e., X~(lanl/) I~ y~~~(tanll)= 2tanll


ax ~v

Carrying oul partial derivatives, we get

ww ?
xsec-II-;-~
:"\
vII
+ ysecII
?
all
- 2tanll

w.E au x - + y~~~
vx
all 2 tan 11
= --,,-
~v

.
S1l1211

asy
ceo
..... (i)
aX (~y sec ~ u

E
Next dirterentialing bL)th sides of the resull (i) partially \Y.r., to 'x' and 'J" wc get
alII all
ngi
alII
x ~-, + ~- + v .~-- =-- 2 cos 211 -
Ou
..... (ii)

n
ax- D.\· . axD)' Dx

a"11 D) II UII all


x -~ + )' - , + -~ = 2 cos 211 ~~ eer ..... (i i i)
(~VaX ~v' (Iy ~v
ing
(all .all)
Multiplying the result (ii) by x, the result (iii), by 'y' and adding the results we get

a~1I + y'
x', ~-,-
(1.\'-
,a"II +
~-:,~
ay
'1
~xy-- =
a"II
OXG)'
(2cos211 ~-l ne
x - + y---
Dx ~v
t
= (2cos211 -I ) sin211, (from i)
= sin411 - sin211

3.10.8 Example

Solution

Let and

Downloaded From : www.EasyEngineering.net


Downloaded From : www.EasyEngineering.net

166 Engineering Mathematics - I

Then
v and ware hOl1logeneous./llllclioll.\' (!ldegree '2 '. Applying 3.10.3 to 'v' and
'w' we get

..... (i)

!Illy ..... (ii)

ww
(i) - (ii) gives

w
Note:

.Ea
Letj(z) be a homogeneous function ofx and y of degree Il ; then from Euler's
theorem
('JI"
x_:'l+ y_:'l
syE
a" =I/f
ax Dy
ngi
i.e.,
.' f)z , az
x..f . ax + xi ay =
.
nee
oz OZ
1?/ => X ax + y ay = 7
Ilf
..... (i)

DifTerentiating (i) w.r., to y'partially


r
ja in
a'z aZ }I--=Il
J

\'-+-+
a'z [j")
'
J
--JJ'fj'" Z
/2 -ax g.n ..... (ii)

e
. ax 2 Dx . oxoy

Differentiating 0) w.r., to 'y' partially t


a z
2
a z az
x--+y-+-=n
. axay ay2 ry
2
[/2/2-jt' 1-az
ay ..... (iii)

(ii) x + (iii) Y gives

= Il[j '2 ~ f( ][x.az + y_~~] ..... (iv)


j2 OX~}I

Downloaded From : www.EasyEngineering.net


Downloaded From : www.EasyEngineering.net

Mean Value Theorems and Functions of Several Variables 167

Using (i) in (iv)

..... (v)

llere

ww
3.10.9 Example
If z = log(,3 + .vI __ x 2y __xy2), prove that
w.E
a=
x-+ v- =3
oz
(i)

a
ax . ry
syE
Note: (i) and (iv) can be llsed as formula in such cases
Hence f = t!, f' = c=, f" =
nazgi
e= and 1/ =3

From (i) of3.1 0.8 note x-+ v--=3


Dx . ~v nee
u=

1 )
rin ,

Form (v) on. 10.8 note


lO-Z a·z 2a'z "
x - , +2xv--+y -,=3.2--9=-_,
ax' . axoy ~v' g.n
3.10.10 Example et
Verify Euler's theorem for the function 1/(x, y, z) = Jx',z+y;
Solution

1I is a homogeneolls function of degree -=J.


2

..... (i)

uu ,
2u- = -z(x- + v-) . . 2x
, _,
So that 0;'" .

Downloaded From : www.EasyEngineering.net


Downloaded From : www.EasyEngineering.net

168 Engineering Mathematics - I

au ) ).)
2u~=-z(x- + y) - .2y
oy

gives au y~+z-·=-
x--+ au ou
ax oy oz
-z[
211
) I )
x- + y-
1
2
I

ww
U
= - - - -:;: ----1/ by (i)
211 2
Hence the result

w
3.10.11 Example
.Ea J
+Y -
J 3
of Df of
syE
X Z
If f = cos· I I then show that x - + }'- + z-'- = -cotf
"X 1 -t )'4 - Z4 AX . ~v oz
Solution
n gin
e eri
..... (i)

So that
From (i) we see that
coif= 11

is a homogeneous function of degree I. ng.


n
11

By Euler's theorem
all
x~+ y~+Z--=-ll
all au et
...... (iii)
ax ~ az
Form (ii), we find that

au = -sin l. Of, ou = -sinf i?l, au = -sin fo.l


ax . ax oy ~v oz . oz
Using these and (ii) and (iii), we get

-Sill .f[ x~
ox
of + z-'-
Of + y-'-
ay
Of) = cos j'
oz
Of Of Of
x-- + y - + z~ = -cotf
ax ~v oz

Downloaded From : www.EasyEngineering.net


Downloaded From : www.EasyEngineering.net

Mean Value Th.eorems and Functions of Several Variables 169

3.10.12 Example
,
. 2
1all au2 2
,all ...
' ,]1
x' + y3
Fmd x- - , + 2xy-- + y- - , ,If u = sm-- I
ax' axoy ~~!. [Xl + y2
, ,

Solution
,
x3~ + y31 ]~
ww Taking z = ,
[
Xl + y2
, then z = sinu =.f(u) say z is a homogeneous function or

w.E I
degree - - = n
12
From Euler's theorem asy
au
En
all j(u)
x - + y - = 11-,-
I sin II
= - - - - = --tan II
1
ax
gin
ay
,
f (ll)
2 1
12 cosu 12
. 1 ) ,"

Again
a.'/ eer
a'u .... a 11
2

axay
2 a-lI

ay'
( I) f
x -+.ay--+y -,=1111- - , -
f
11- f -I
,
f 3

= (-~)(- ~ -I)tanu - _I_(-tan u).tan ing


2
1I
12 12 144
.ne
=
tanu
144
1
--(13 + tan-u)
{-:./ = COSll, (' = -sin 11)
t
3.10.13 Example
2
_ Y )a u a 2II 1
1 a-lI
If u = tan' ( -=--- ) ,find x' --+ 2xy--+ y--2
X ax axay ay

Solution

Let z = 2:. degree of z is 0 and z = tanu


x
az
x-+ y-=o
az az 1 au af.
- =sec'u-
,all
-=sec'u-
ax ay ax ax oy ~v

Downloaded From : www.EasyEngineering.net


Downloaded From : www.EasyEngineering.net

170 Engineering Mathematics - I

..... (i)

x 02~ + au + y 02U = 0
..... (ii)
. uxoy oy ~l

(i) x + (ii) y gives

ww ) 0\' OlU
x--+2xv--+ y-=- x-+ y - =0
, 0\. (au au)
w.E 0'(2 . ax~y oy' ax oy

asy Exercise - 3(E)

En
I. Verify Euler's theorem for the following functions:

(a) z=--
xy
x+y gin (b)

eer Y1 )

ing
• 1 X} -
(d) z = SIl1
( x-y

( e) _. --I
( X
3})
Y (I) Z = (x
2
+ y-?+ z-?) 1
2 .ne
t
Z-SIl1 3 3
X +Y

2. If z = xyf (~) then show that x OZ + Y OZ = 2z


y ax ~

3. Show the following by using Euler's theorem:


x3 + / OZ OZ 5
(a)lfz= . thenx-+y-=-z
~x+ y ax· ~Y 2

x 3 + y3 OZ OZ
(b) Ifz= log ,then x-+ y-=2
x+ y ax oy

x-y 3 3)
87 OZ
( x-y
(c) Ifz=sin- I - - - - then x--=' + y - == 2 tan z
ax ~y

Downloaded From : www.EasyEngineering.net


Downloaded From : www.EasyEngineering.net

Mean Value Theorems and Functions of Several Variables 171

4. If z = tan-I ( x + /
J

lx+y
l' then x uz + y OZ = sin 2u
8x '~y

5. Ifz = sec~l[x2 + yll, then x a::: + yUZ =cotz


x+ y ox ~

6. If z is a homogeneous function of x and y with degree' 11' and z = f(u), prove that

x-+
all au
y-=n-'--
(u)
ux . ~ /(11)

ww . (y)
If z = x 3 1 SIll·-1 z a 2
a2
fiz
,show that x 2 --T + 2xy-- + y2 -2- = 20z
11
7.
w.E
8. If z = tan-I ~ X4 + y4
-
x ax- axay ~y

az oz . asy
, show that

(i) x - + y - = sm2z
ax oy En
ii)
,a
x- -
2
)
z
+ 2xy
0 2z
- - + v-
, 2z . gin a .
-) = s1I14::: - Sill 2z
(
ax' ax~v' ~v'
eer
ing
az .ne
ax
8z I
(i) x-- + y - = -tanz
ay 2 t
1
10.lfz=tal1- 1 [;l+YSin- [.:l show that

\ \-
x2 + y2
II. If z = cosec-I then show that

Downloaded From : www.EasyEngineering.net


Downloaded From : www.EasyEngineering.net

172 Engineering Mathematics - 1

a2 z 0-" z 2 0-" Z
X
2
-2
ax
+ 2xy - - + y ---:;-
aX~Y ay
tan zll31 J
= - - -+-tan- z
]

12 12 12

12.lfz=tan- 1 x +y 2 2) ,then show that


( x-y

ww
13. If z = log rand ,2 = x 2 + y
"a z
2
a z
2
)a z
2
then show that x- - 2 + 2xy-- + y - " + 1 == ()
ax axay ay

w.Ej + y4I
14. If z = sin-I
r
X4I
!
x5 _ y5
!
asy
then show that

En
gin
') 2 2
2 a-za z )a z 1 2
X - - 2 +2xy--+ y - , =--tanz(tan z-19)
ax ax~ ay 400

~ x 2+ y2] eer ;\2;\ 2 ; \2UZ


2
15. If z = log ( x+y find the value of x
2uZ
GX2
uZ

ing
+ 2xy axay + y ay2

3.11.1 Jacobian .ne


Ifu and v are functions of two independent variables x and y then the determinant

au au
t
ax ~ i.e.,
ux uy
Ov Ov v, vy is called the Jacobian of u, v with respect to x and y
ax au
and is denoted by

a(u, v)
a(x,y) or .J(~)
x,y

If u, l~ ware functions of three independent variables x, y, z then the determinant

Downloaded From : www.EasyEngineering.net


Downloaded From : www.EasyEngineering.net

Mean Value Theorems and Functions of Several Variables 173

lI( UI' 1I,

V( vI' Vz

W( Wy lV,

is called the Jacobian of 1I. V, w with respect to x. )~ z and is denoted by

a{/I, v, w)
or )(U,\" w)
a{x,y,=) x,y,z

ww Similarly

w.E a{U p 1l2·····uJ _ aX I aX 2


aUI uU I (J/I~
ax"

asy
a{X I'X 2.....x,,) - all" OU"
oX I aX2
au"
ax"

En
gin
e eri
SOME PROPERTIES OF JACOBIAN

3.11.2 Property ng.


a(u,v) a{x'Y)_1
a(x,y)' a{u, v) -
net
where J = a{lI, vI and _ a{x,y)
.J - 0(11, v}
a{x,y)
Proof:
Let u = u(x. y) and v = lI(x. y) so that u and v are functions of x and y. Then
differentiating these partially each with respect to II and v, we get

I := au . ax + all . 0'
ax all ay au

Downloaded From : www.EasyEngineering.net


Downloaded From : www.EasyEngineering.net

174 Engineering Mathematics - I

av ax av ay
O= - . - + - . -
ax au ay au
1= av .ax + av .ry
ax av ay av

au all ax ax
Now
a(lI, v) a(x,y) ax oy x all av
a(x,y)' a(u, v) av av ry ay

ww ax ay au av

w.E au -au
-
ax ay
av av
x
ax
-
au
ax
ry
all
ry
asy ax ay av av

En
(Interchanging rows and columns in 2 nd determinant)

gin ali ax au oy
-.-+-.- --+--
ax au oy au
au ax au ay
ax' av ay' au
eer av ax av ry
-.-+-.--
av ax av ay
--+--'
ax' av ~v' av
au au ry au
ing
=I~ ~1=1 Proved
.ne
3.11.3 Property t
a{u,v) _ a{u,v) a{r,s)
a{x,y) - a{r,s)' a(x,y)
where u, v are functions of r, sand r, s are functions of x, y.
Proof
By differentiation offunction ofa functions, partially; we get
au au ar au as
-=-.-+-.-;
ax ar ax as ax

Downloaded From : www.EasyEngineering.net


Downloaded From : www.EasyEngineering.net

Mean Value Theorems and Functions of Several Vanables 175

ou
-=-
or oy OS
au -+--'---_.
~ or .~ as· oy , ..... (i)

oval'
01'
-=~- - + avos
-_.
ox or .ox os· ox '
Ov 01' or 01' as
-=- -+--_.
and ~y or .oy os· ~y ,
all au (lr (Is

ww =?----=
o(u, v) a{r,s)
(1(r,s)· a(x,y)
(lr
uv
os
elv x
(Ix
us
(Iy
us
ilyl
w .Ea
i"lr

Oll
us

or as
all
(!y.

syE or os x ax ax
ov -oV (II' as
or as oy oy
ngi
(Interchanging rows and columns of the second determinant)

allor ou os ou nee
or Oll as
-.-+-.- -.-+-.-
or ox os ox
Ov or OV os
or oy os oy
ov or ov as rin
-.-+-.--
or ox os ox
-.-+-.-
or oy as ~ g.n
ou -ou
ox ~ _ o(lI,v)
e t
ov Ov - o(x,y) using (i)
- -
ox ~
3.11.4 Property
If u, v are functions of two independent variables x and y thell u, v, are

independent if
oo((U,V))oFO.
x,y
Otherwise they are dependent. Similarly if II, v, W

. . . . o(u, v, w) 0
(functions of x, v, z) are II1dependent If and only If ( ) oF
a x,y,Z

Downloaded From : www.EasyEngineering.net


Downloaded From : www.EasyEngineering.net

176 Englneenng Mathematics - I

Theorem
If the functions 11 1, 1I::...... 1I11 of the variables xI' x 2 ....• xn be defined by the
relations Il, ==.ll'",), 112 =flx ,. x 2), 113 = .li x ,. x 2• x 3 ) ..... 1/11 .I;,<x,. x 2 ..... x). cc

Then
O(lI p 1/2 .....U,,} OU, UU] OU 3 OU II
o(x"x2.....x,,) - ox, . oX1 'a~~ OXII

We know that

Similarly lor

ww Oil, Oil 2

w Then
.Ea 0(U"l/2 .. ·..U,J _ aX 2
ox,
Oil,
ox,
0111

oX1

syE
o(x, 'X 2 ..... X II )

ngi
all,
OXII
~1I2
OXII
aU
OXII
Ii

nee
OU,

rin
Oll] OU"
ox , ox , ox ,
0
Olll.
oX 2
Ollil
oX1 OU, Olll OU II g.n
ox , . oX 1 OXII
et
o o

3.11.5 Example

_ o(x,y)
If x = 112 -vl . Y = 2uv ,. tllld --(--)
0 1l,V

Solution

ox
We have x = u2 - v2 ~ -ox =
2lI. --=-2v
all OV

Downloaded From : www.EasyEngineering.net


Downloaded From : www.EasyEngineering.net

Mean Value Theorems and Functions of Several Variables 177

cYv 0)1
v = 2uv => --- = 2 v -"-- = 211
- V I I ' 01'

ox AX
V{X,y) _ or ()I~ ~ 1211 - 21'1 := 4(u-»
+)1'- )
O(II.~) - VI' ~F 21' 211
au VV
3.11.6 Example

ww If x ~ reose. y ~ rsintJ find - (- ) lllld-(--)


V(x, y) a( t, °)
, V r,O a x,y
w Solution

.Ea x = reosq, y = rsinO

=>
(It
~ =
(1,.
syE cos()o

-ax = -rsiIlO,
ngi
ov = rc()sf)
of)

ax ax
---
0(1
nee
o(x,y) _ or ao _Icoso -rsinol rin
Then
0(1',0) - ~v
or
~v
ao
- sinO .
r Sill 0
=

g.n
r(cos 2 0 + sin 20) = r

Again 12 = x 2 + y2 ; 0 = tal1- 1 ~
x
e t
or
2r-- = 2x
ox
(l() -- y -y
ax r+y
0 0 0
r-

or +x
- = -
ax I'

Dr y (1() -y -y
ay ,. ilx x2 +)'2 r
)

Downloaded From : www.EasyEngineering.net


Downloaded From : www.EasyEngineering.net

178 Engineering Mathematics - I

x x

lar ar x
a(,., 0) ~ ax ~}! Y
aCy,y) ~ ao ao -- ~ y
lay vy
3.11.7 Example

ww If x = uv, y = -
11

v
then show that .1./ ::.: I

w
Solution
.Ea ax at
syE
We have -=V, -=11
all al'
~F ay 11

ngi
= ---
)
VII v aI' v·

nee
')
2u- =
VU
)I,
r DII
2u-=x ing
.ne
But lr = xy ax . ay

av =!...
and v·
, x
=-, 2v
y ax y'
21'-=
aI'
ay
x
--?
y. t
y x
II" lly 2u 211
J =
Vr v)' I ~x

2vy 21:J!

X X X

4uvy 411vy 2uv· Y

I? V
=---v-
211v 2u

Downloaded From : www.EasyEngineering.net


Downloaded From : www.EasyEngineering.net

Mean Value Theorems and Functions of Several Variables 179

- 3.11.8 Example

Prove that JJ = 1 for x = e V secu, y = e V tanu


Solution
ax = e secu tanu, -=
ax e V secli
all av
ww and
oy
au = eV sec 2 u;
0;
- = ('
av
v
tanu

w .Ea
x"
Yv
=

I
"
e secutanu
I'
e sec-
)
II
v
e secu
e" tan 11
=
2"
-e secu = -xe
I'

Now sec 2u - lan 2


x 2 e- 2v - sye-yE 1I =

2v =
1
1

e2v = x 2 - y, also ngi


eVtanu
---
Y

. y
e" secll x
nee
SlIlli = -
X
rin
U
·-1
= Sill -;
(Y) an d v '21 Iog(x = 2
- Y-)) g.n
Du
i1x X~X2
y
- /
,
all
(Jy X~X2 -y 2
e t
av x Dv y
1 ? ?
ax x- - y ? '
i1y x- - y-

Y
lIx lIy X~X2 _ y2 ~X2 _ y2
J
Vt VI' X Y
x2 _ y2 x-1 - 1
y-

Downloaded From : www.EasyEngineering.net


Downloaded From : www.EasyEngineering.net

180 Engineering Mathematics - I

, \' I
JJ =xe ,-=1\'
xe

3.11.9 Example

ww VIV Wlt lIV I I I O(X, y, z)


If x=-,y=--,z=-- t Jen SlOW tlat - ( - '-)=4

w.E
II v W 01l,V,\I'

Solution

asy ax
Oll
ox
ov all'
oX VI\'

u 2
lI'

It
v
1/

En
o(x,y,z) _ oy
0(1/, V,-H')
oy
ov
oy
ow
11' Wit
2
II

gin Oil
OZ oz
ow
oz
V
V
V
II
v
uv

eer 011 OV IV IV

- VlI'

VlI'
Wli

- Wli
ltV

ltV '-2 '-2 ' 2 ing


VlI' lIW --l/V
II v W
.ne
-I
-I
t
-I

=-1(1-1)-1(-1-1)+ 1(1+1)=4

Exercise - 3(1)

I. If y + x + Z = 1I, Z + Y = 1Il: Z = lIVW, find o(x,y,z) [ADS: (1I


2
v) J
o(u, v, w)

Downloaded From : www.EasyEngineering.net


Downloaded From : www.EasyEngineering.net

Mean Value Theorems and Functions of Several Variables lSI

o(u, v, w)
2. If II = x+ 2/ - Z2, V = x2yz, W = 2X2 -xy find at the point (1,-1,0)
o(x,y,z)
Ans:19
3. If x==rcosB,}J==rsinB, find ~(x,Y) and !JS!~~~ and show that
. 8(r,0) 8(x,y)
o(x,y) o(r,O)
--'--. = 1 (Hint :relcr 3.6.6)
o(r,O) o(x,y)
XOXI X X, XIX)
I ,y, = - - - ,show that ----'----'--== 4 o(YI,VnY,)
4. If YI = - --- ,Y) = ---
XI - x2 - x, 0(X I ,X2,X3)

5. ww If YI ==1-XI'Y2 =xl (l-x2 ),y] ==xlxl(l-x,),provcthat


O(YI' Y), y, )
- = (1)3
- XI 2 x"
6. w.E
If x=a(coshu)cosv,y=a(sinhll)sinv, then show that
0(X I ,X2 ,X,)

asy
2
o(x, v) a
--,-'- = - (cosh 211 - cosh 2v)
0(11, v) 2
7.
En
If II = /(x l ), v = ¢(x l , x 2 ), W = If/(x!, X 2' Xl) then show that
Oell, v, w)
0(X I ,X2 ,X3 )
=---
011 Ov
ox
' oX • ox}
uw
gin
eer
i 2

8. If X
. 1')
= r S1l1 r.
do
cos 'I', • 0 . do
Y = r Slll 0 I oC X, y, z) ). 0
S1l1 or, Z = r cos - S lOW that - - - = ,.- S1I1

ox ox uy ing
o(r,a,¢)

or 00 o¢ sinOcos¢ reosOcos¢ .ne


- r sin Osin ¢
Solution:
o(x,y, z)
o(r,O,¢)
=
ay
or
oy
--
UO tJ¢
uy
= sin Osin ¢
cosO
r cos Osin ¢
-rsine
rsinO eos¢
o
t
OZ OZ
--
oz
or oe o¢
= sin ecos¢[ 0 +,.2 sin 2 Oeos¢ ] + r coseeos¢.r sineeosecos¢ +
+( -r sin esin ¢)[ -r sin 2 Osin¢ - r cos 2 esin ¢ ]
== r2 sin 3 Oeos 2 ¢ + r2 sin Ocos] Oeos 2 ¢ +,.2 sin 3 Bsin 2 ¢ + r2 cos 2 Osinesin 2 ¢
== r2 sin 3 0 + r1 sin () cos 2 () = ,.2 sin O.

Downloaded From : www.EasyEngineering.net


Downloaded From : www.EasyEngineering.net

182 Engineering Mathematics - I

3.12.0 JACOBIAN OF COMPOSITE FUNCTIONS:


Let (u,v) be functions of x,y where x,y themselves are functions of r,e .Then 1I,V arc
. e 8(1l, v)
. fi ..
composite unctlOlIls of r, and = 8(u, v) . 8(x,y) .
(from 3.6.3)
8(r,B) 8(x,y) 8(r,B)
Note 1: Ifu,v are functions ofx,y we may regard x,y as functions ofu,v.
e
Taking r = ll, = v in the above result, we have
au
-
8u
8(1l, v) 8(x,y) 8(u,v) 8u 8v 1 0
--- = =1

ww 8(x,y) . 8(1l, v) 8(1I, v) 8v


811
av
8v
0 1

w.E ..
8(1l, v) 8(x,y)
8(x,y) . 8(1l, v)
= 1 => .1.1 1 = 1

where J = 8(1l,:i,J = 8(x,y)


I
asy
8(x,y)
En
8(1l, v)

Note 2:
8(u, v, w)
8(r,e,¢)
=
8(x,y,z) 8(r,e,¢) gin
8(u, v, w) 8(x,y,z)
. where u,V,W are functions of x,y,z and x,y,z

are functions of r, B, ¢ .
eer
Also J./' = 1; where J= 8(1l, v, w) , .II
8(x,y,z)
= 8(x,y,z)
8(u,v,w) ing
3.12.1 Example: If II = xyz, V = x 2 + y2 + Z2, W = x+ y+ z find .I = 8(x,y,z) . .ne
Solution: we first evaluate .I
I
=
8(1l, V, w)
8(x,y,z)
,
8(1l, v, w)

since u,v,w
t
are explicitly given as

yz zx xy
functions of x,y,z. .II = 2x 2y 2x = -2(x - y)(y - z)(z - x)
1 1 1
Hence from the relation that .1.1
1
= 1, we have
J = 8(x,y,z) = -]
8(u, v, w) 2(x- y)(y-z)(z-x)

Downloaded From : www.EasyEngineering.net


Downloaded From : www.EasyEngineering.net

Mean Value Theorems and Functions of Several Variables 183

Exercise - 3(G)
) 1 • C(ll, v) 3
I. Ifu=2xy,v=x--y, whcrex=rcosO,y=rsmO show that ---=-4,..
o(r,O)
2. If x = ;;;;,y = Jwu,z =-r;;;;
where II = r sin OcosrjJ, v =,. sin Osin rjJ, W = rcosO,
v(x,y,z) 1 2 • 0
show that = --,. Slll
a(r,O,rjJ) 4

3.
ww
·
If x = ltV )'
,
ll+V
= --
1I-V'
_
fll1d - - -
v(u,\')
o(x,y)
(1I-V)2)
(A liS : --'------'-
411v

4.
·

·
w
II II = -
yz
x '
x +Y
zx
V = -
y ,
.Ea
xy
HI = -
z
prove that
8(x,y,z)
=
8( ll, V, lV) 4
D( ll, 0)

syE
I
5. 1I1I=--,O=tan-l(x)-tan-- (y) show that =0.
1- .xy v( x, y)
v(u,V,lI')
ngi
• 1
6. If lI=x--2y,v=x+y+z,w=x-2y+3z show that ----=1Ox+4
8(x,y,z)

3.12.2 Jacobian of implicit functions nee


Let 1I1' 1I2' 1I3, be implicit functions of x,, x 2' x3 so that
rin
jJu" 1I2' 113' x" x2, x3 ) = 0,
oj; + a.t; .aU I + a.t; .aU 2 + a.t; .aU 3
r= 1,2,3
g.n
e
Then = 0 and so on

then
aX I aUI oX I oU 2 aX I aU 3 oX I
a(.t; ,j~ '/1) 0(11" u), uJ
-a(u"U 2 ,1I))' a(X I ,X2 ,X3 )
t
I aj~ . all,. I oj~ . OUr I a.f! . Ollr
all,. ax! au, oX2 all, oX3
= I af2 . OUr I of2 . all,. I a.f~ . all r
Ollr ax! OUr aX 2 au,. aX 3
I a.t:l }u r I 'fJj~ . OUr I af3 . au,.
au,. ax! au,. oX2 all, aX3

(by the rule of prodllct of determinants)

Downloaded From : www.EasyEngineering.net


Downloaded From : www.EasyEngineering.net

184 Engineering Mathematics - I

_ aJ; W, _ ~t;
ax, aX2 ax)
_ ~f2 _ aj~ - aj~
au, QU 1 au)
0f, aj; 0f,
ax, aX2 ax)

= (_ I)' a(J;,j2''/;)
ww a(x"X2 ,X3 )
aCt; ,j~,f,)
w.E
Hence
a(u" 112' uJ _(_1)3 a(;;~-x~~~,"J
a(x, x2 ,xJ - ~V;,fI,lj)
asy
, a(U"l/2,lIJ
(The result if obvious)
3.12.3 Example En
x y gin
z
v'1-r2 v'1-r2
eer
If u = r:--:;' v = r:--:;' W = r:--:; prove that
v'1-r2

a(u, v, w) _ 1
y; ing
a(x,y,z) - (l-r2
r+ .ne
where
Solution
-? = x2 + z2
t
we have II (1-/ 2 ) =x2
=> II(u, v, W, x, y, z) = u2 (I _x2 - r _z2) - x2 = 0
Similarly ./; == v2 (1 - x 2 ~ r - z2) - r = 0
13 == w2 (1 - x 2 ~ r- z2) - z2 = 0

.... (i)

(see 3.13.2)

Downloaded From : www.EasyEngineering.net


Downloaded From : www.EasyEngineering.net

Mean Value Theorems and Functions of Several Variables 185

J
- 2u 2 x- 2x - 211-Y - 2112 z
aCt; ,.I~ ,.I;) _ -2v 2 x -2v 2 y-2y - 2v 2 z
8(x,y,z)
-2w 2 x - 2w 2 z -2z
)
- 211'- y

?
u2 + 1
)
u- 11-

= (-2x) (-2y)(-2z) v-
) 2 2
v +1 V

w2 +1
) )
w- 11'-

ww 0 u-?

w
)
-8xyz -I v-

.Ea 0 -I w2 +1

syE by C 1 -C 2
C2 -C 3
= -8xyz(u2 + v2 + w2 + I)
ngi
= -8xyz
[
--J
X2
+--- 2 +--? + 1
I-r- I-r l-r-
y2 Z2 ]

nee
? ')
x- + y + z- + 1-
') ,.,
1'-
rin
=-8xyz
-8xyz
1-r-
J

g.n
=--2
I-r
;also.

2u(l-x2 - / _Z2) o
e o t
8U;,.I;, .I;) o 2u(l_x2 - 0
= y2 _Z2)
8(ui' v2 , w3 )
o o 2u( 1- Xl _ yl _ Z2 )

8l1vw(l- Xl _ y2 _ Z2 )3

8 x . Y . z (l_r2)3=8~yz(l_r2)112
~ JI-? .J1-r 1
_. 8(11, V, w) 1)3 -8xyz ~ I
Hence trom (I), we have - - - - = (- - - , 8 (I 2)1/2 = r:-?
8(x,y,z) I-r- xyz -r ',"1_1'2

X Y Z
Cor: Given U=- V=- W=-
k' k' k

where k = VII - X
2
-
' f iIn d •,(x,y,z)
Y, - Z-, in terms ofk. lAns: k
S
]
(u, v, w)

Downloaded From : www.EasyEngineering.net


Downloaded From : www.EasyEngineering.net

186 Engineering Mathematics - I

Exercise - 3(H)

8(x,y,z) 2
I. Ifx+y+z = u ,y+z = uv, z = uvw, then prove that ----'----=--~ =- II V
8(u, v, w)
~)

8(u,v) x--y-
2. Ifx + l + u _v = O,uv + xy = 0, prove that
2 2 2

8(x,y) u 2 + v 2
3. If u1 =x1 +X2 +x3 + x,pU 1U 2 =x2 +X3 +X4 ,1lIU2113 =x4 then prove that
8(X p X2 ,X3,X4 ) 3 2
=1I1 .il2 ·113
ww
8(u l , 112 , u3' 114)
4. If u 3 +V3 +W3 =X+ y+z,u 2 +V2 +W2 =X3 + / +Z3,

w.E
t Ilen SlOW
I t Ilat
8(1I,v,w) (y-z)(z-x)(x-y)
= ---"'----'---'..-----'-----'-----'----"--
8(x,y,z) (v- w)(w-u)(u - v)
asy
3.7.4 Use of Jacobians in determining functional dependence and independence of

En
functions.

Let u and v be two functions of x and y connected by the relation v = f(u), then

gin
we say that u and v are functionally dependent. We shall prove that the condition for
functional dependence is

o(u, v) = 0 eer
o(x,y)
ing
Consider w = v - feu) = 0
.ne
Now w is a function of u and v where u, v are functions of x and y.
w is a composite function of x and y t
ow ow au Ow Ov
-=-.-+-.- ..... (i)
ax ax ax ov ax
ow ow au ow Ov
-=-.-+-.- ..... (ii)
ax au ax ov ax
But w considered as a function of x, y is identically zero.

Ow
i.e., -=0
ax '

Downloaded From : www.EasyEngineering.net


Downloaded From : www.EasyEngineering.net

Mean Value Theorems and Functions of Several Variables 187

From (i) and (ii) we get


Ow all ow ov
-.-+---.--=0 ..... (iii)
au ax av ax
Dw
--.--+-.-=
all ow av 0
and ..... (iv)
au ~v D1' ay
Ow aw
Eliminating -a-·-o
11 I'
from (iii) and (iv) we have

ww all aI'
ax ax
w .Ea
all -av
Oy ~v
=0

syE au au
--
ax ay
ou au =0
ax Oy ngi
nee
=>
a(u,11=o
a(x,y) r ing
The concept of functional dependence can be extended to any number of
.ne
variables. Thus if u, 1\ 11' are functions of x. y, z the 1I, v, w will be functionally
dependent (i.e., there will exist a relation between ll. 1\ 11') if

Q~!_,V, w) = 0
t
a(x,y,z)

3.14.5 Example

Are x -I- y - z, x - y + z, x 2 + y2 + z2 - 2yz functionally dependent? I f so, find


a relation between thelll.
Solution
Let lI=x+y-z ..... (i)

1'=x-y-l-z ..... (ii)


11' = Xl -I- I -I- i2 - 2yz ..... (iii)

Downloaded From : www.EasyEngineering.net


Downloaded From : www.EasyEngineering.net

188 Engineering Mathematics - I

\
o(U, v, w) = \
-\ -\
Then o(x,y,z)
2x 2y-2z 2z-2y

= -\ -\ =0
2x 2y-2z 2y-2z

(since 2nd and 3 rd columns are identical)

ww :. u, v, ware functionally dependent


To find the relation between u, v, w we notice that, from (i) and (ii)

w.E 11 + v = 2x,
From (iii)
U - v = 2(y-z)

asy
En
gin
e
u2 + v2 = 2w, the required relation eri
3.14.6 Example ng.
11,
If 11 = x + Y + z, v = x 2 + y2 -I- z2, W = x 3 + i
+ z3 - 3xyz then prove that
l~ ware not independent and find the relation between them.
net
Solution
.. ti c: . Id d . o(u, v, w) 0
CondItIOn or lunctlOna epen ence IS ( ) =
o x,y,z

Now o(U, v, w) 2x 2y 2z
o(x,y,z) - ? ? 2
3x- -3yz 3y- -3zx 3z -3xy

=6 X
2
x - yz

Downloaded From : www.EasyEngineering.net


Downloaded From : www.EasyEngineering.net

Mean Value Theorems and Functions of Several Variables 189

0 0
=6 x-y y-z z
)
(x- y)x+ y+z y - z(x + y + z) z- -xy

o o
= 6(x - y) (y - z) I z
x+ y+z x+ y+z z -xy

ww =0

w.E 11, V, IV are functionally dependent. We shall now find the relation between them

(x + y + z) (.x2 + I + z2 - xy - yz - zx)
asy
we have w = ..... (A)
Now u2 - V = (x + y + z)2 - (x 2 + I + z2)

En
= 2(xy + yz + zx)
We write (A) as
gin
W=U
[ U-V] eer
v - --
2
ing
Thus
.ne
3.14.7 Example

x+ y
Test whether u = - - , v = (
xy
)2 are functionally dependent and if so, find
t
x-y x-y
the relation between them.

Solution
Treating'lI, vas functions of x, y the condition for functional dependence is

a(u,v) =0
a(x,y)

x+y xy
11=--, V=-~-c-

x-y (x- y)2

Downloaded From : www.EasyEngineering.net


Downloaded From : www.EasyEngineering.net

190 Engineering Mathematics - I

{x-y)'I-{x+y)'1 -2y
We have ux = (X_y)2 = {X_y)2

(X- y).(I).{X+ y){l) 2x


II == --'---~---'--'---'--~'--'--'--

y (X_y)2 {x-yf

V = {x- y)2 y-2{x- y).l.xy


x (x _ y)4

ww _ {(X- y)y-2xy)
- (x- y)

w.E xy - y2 - 2xy _ - y{y + x)


{x- y)3 - (X- YY
asy X(X+ y)
lilly
En
Vy={
x-y )3

gin
a{u, V) lIx ll"

eer
a(x,y) = Vx Vy

=UV - v v
x y x y
ing
'-2y 2 .x{x+ y} + y{X+ y!. 2x J =0
(X- y) (X- y) {X- y} (X- y)- .ne
•. U, v are functionally dependent
We shall now find the relation between them
t
x+y
We have u=--
x-y

x-y
x+ y u

By componendo and dividendo


x 1+u
-=- A
Y I-li

Downloaded From : www.EasyEngineering.net


Downloaded From : www.EasyEngineering.net

Mean Value Theorems and Functions of Several Variables 191

o X x 1+11
Y
Now
xy
v = (x - y)' = (;
V

1 r y l-lt
by (A)

'V=C~:~2 J
u2(4v+ 1) = I

ww The required relation of functional depedence between lI, v.

w.E
3.14.8 Example
Examine for functional dependence of

asy
u = sin-I x + sin-1y

En
v=x~l-y2 +y,JI-x 2

gin
and find the relation between them, if it exists.
Solution
eer
We have u =-===
, ~1_x2 ing
.ne
t

lilly

a{u, v) 1I ( It )'

a{x,y) Vx Vy

Downloaded From : www.EasyEngineering.net


Downloaded From : www.EasyEngineering.net

192 Engineering Mathematics - I

:. ll, V are functionally dependent we shall now determine the relation between u and v

Let A=sin-1x,B=sin-1y; =>x=sinA y=sinB

ww The given function can then be written as u = A +B

w.E
2 2
v = sin A)1-sin B +sin BJl-sin A = sin(A+B)= sinu

Hence the required relation of functional dependence between u and v is v = sinu

3.14.8(a) If u asy
= eX sin y, v = eX cos y, show that u,v are functionally independent

En au au

av gin
-
X • X
a(u, v) ax ay e smy e cosy
Solution: The Jacobian = _ex * o.
a(x,y)
eer
ax
av
ay
eX cos y _ex sin y

:. x, v are functionally independent. ing


. x-y x+z
3.14.9 VerifY whether u = - - , v = - - are functionally dependent, and if so find .ne
the relation.
x+z y+z
t
Solution: Treating u,v as functions of x,y regarding z as an absolute constant, the
condition for functional dependent in

a(u, v) =0
a(x,y) ,
au
Now - =
y + z au 1 av 1 av =
- = - - - and -=~-,
x +z
ax (x+z/' ay x+z ax y+z ay (Y+Z)2

Downloaded From : www.EasyEngineering.net


Downloaded From : www.EasyEngineering.net

Mean Value Theorems and Functions of Several Variables 193

y+z
---
B(u,v) (x + Z)2 X+Z
= =0
B(x,y) 1 x+z
y+z (y + Z)2
u, v are functionally dependent.To lind the relation between u and v, we eliminate 'x'
between the given relation viz:
x-y x+z
u = ------------- (A), v = ---------------(B)
X+Z y+z
liZ+Y
ww
from(A) ux+vy = x-yo =>(I-u)x=uz+y => x = - -

+Y
1-11

w
liZ
+z
(y+Z)

.Ea
Substituting for 'x' in 'B', v = 1-11
y+z
=
I-l/' y+z I-u

syE
Hence the required functional relation between 'u' and 'v' is v( I -u) = I.

ngi
Exercise - 3(1)

I.
nee
Verify whether the following are functionally dependent and if so, lind the relation
between them.
x- y x+ y
(a) II = - - , V = - -
x+y x
(Ans: u( I+v) = 2)
rin
x-y
( b) u = - - , v = tan -\ x + tan -\ y (Ans: v = tan
-\
II) g.n
x+y
I-xy
(c)u=x-y,v= xy
(X+y)2
(Ans: 4v + 1I
2
=1)
e t
2. Prove that the following functions are not independent.
(a) II = X + 2y + Z, v = x- 2y + 3z, W = 2xy - 2z + 4yz - 2Z2
(b) II = x+ y- z, v = x- y+ z, w= x + l + Z2 - 2yz
2

3
(c) u = x + y + z, v = xy + yz + zx, w = x + l + Z3 - 3xyz

3. If u = x + y "v = y + z w = y(x + y + z) , show that u,v are connected by a


Z x xz
functional relation.

Downloaded From : www.EasyEngineering.net


Downloaded From : www.EasyEngineering.net

194 Engineering Mathematics - I

x-y
4. Examine for functional dependence between u = - - , v = tan-I x - tan -I y.1f
l-xy
dependent, find the relation. ( Ans: u=tan v)

5. Prove that functions II =Y + z, v = x + 2z 2 , W = x - 4yz - 21 are functionally


dependent and find the relation between them. (Ans: V=W+2l?)

3.15.1 Maxima and Minima


The method of finding the maxima and minima of functions of two and three

ww independent variables is discussed in this topic.


The maximum or minimum values are also called Stationary or extreme or turning

w.Evalues. For finding these values we use Taylor's theorem for functions of two
variables.

asy
3.15.2 Taylor's theorem for a function of two variables:
We know that Taylor's theorem for a function f(x) of a single variable 'x' is

~~En
f(x+h) = f(x) + hj\x) +
gin /I(X)+ ...... .
....( I)

eer
Now, consider f(x,y), a function of two independent variables. Keeping 'y' constant
and following (I) we get
. a h a
j(x+h,y+k) = f(x,y+k)+h-f(x,y+k)+---J j (x,y+k)+ .........(2)
2 2
ing
.

ax
Keeping 'x' constant and by applying (I), we get
2! ax-
.ne
a. ea
f(x+h,y+k) = f(x,y)+k-j(x'Y)+---2 f(x,y)+ ...... .
ay 2! ay
2
t
.... (3)
Substituting (3) in(2) we get

f(x+h,y+k) = [
. a. ea
j(x,y)+k a;f(x'Y)+2T ay2 f(x,y)+ ...... .
2
]

2
+ h- a. e a f(x,y)+ ...... .]
a [ f(x,y)+k-j(x,y)+---)
ax ay 2! ay
2
112 a [ aj. a
2
e
+ 2T al f(x,y)+k iY (x,y)+2T al f(x,y)+ ....... +..... .
]

Downloaded From : www.EasyEngineering.net


Downloaded From : www.EasyEngineering.net

Mean Value Theorems and Functions of Several Variables 195

Substituting (iii) in (ii) we get


2 2
f(x + hy + k) a k a f(x,y)+ ....]
=
[f(x,y)+k-f(x,y)---2
ay 2! ay

2
a [ f(x,y)+k-f(x'Y)+---2
+h- a a f(x,y)+ .....] e
ax ay 2! ay
2 2 2
h a [ f(x,y)+k-f(X'Y)+---2
+---2 a a f(x,y)+ .....] e
<2 ax ay < 2 ay

ww + .....

w.E[ aj 2 2
k a f + .....] + h-+hk--+
= f(x,y)+k-+-- a f .....] [aj 2

asy 2 ay < 2 ay ax axay

a +
+[~ E . . .] 2

ngi <2 ax
{

Thus n f(x + h. y + k) = f(x, y) + (h! + k


ee~ r)f

1
+~hax+kay
(a ing
a)2 f+·····
.ne ..... (iv)

Expression (iv) if known as Taylor's expansion for functions of two variables.


Writing x = a, y = b expansion about (a, b) is given by t
a a)2
If(a+h,b+k)=f(a,b)+ ( hax +kay ira,b)

[a a
1 h-+k- ]2 i((jb)+ .....
+-
1! ax ay ,
Now if we call h = x - a, y - b = k

we get f(x,y) =f(a, b) + ((x-a)! +(y-b)~)i((j'b)

Downloaded From : www.EasyEngineering.net


Downloaded From : www.EasyEngineering.net

196 Engineering Mathematics - I

a a )2 fiab) + ..... .
1 ( (x-a)-+{y-b)-
+-
2! ax ay'
..... (v)
Writing a = 0 = b, we have

f(x, y) = j(0, 0) + (x ~+y; )./(0,0) + ...... .. ... (vi)

This is Maclaurin's series for two varialbes.

ww
3.15.3 Example

w.E Solution

asy
We have f(x, y) =- ~2 + y2 => f{I,I) = e 2

En
Ix (x, y) = 2xe x
2+ 2
=> Ix (I, I) = 2e
2

gin
Y

eer
ing
X2 + y2 .ne
t
2 2 7
fyy (x, y) = e + 4Y eX +Y- => fyy (I, I) = 6e2
Hence putting these values in second form of Taylor's theorem, we get
1
e,2+y2 =e2[l +2(x-I)+2(y-I)]+ 2! [6(x-I)2+8(x-I)(y---I)+6(y---If+ .... ]

3.15.4 Example
Expand ~ siny in powers of x and y as far as terms of third degree
Solution
f(x, y) = ~siny => j(0, 0) = 0
Ix(x, y) = ~siny ,=>Ix(O, 0) = 0
J;,(x, y) = ~co,';y =>J;,(O, 0) = 1

Downloaded From : www.EasyEngineering.net


Downloaded From : www.EasyEngineering.net

Mean Value Theorems and Functions of Several Variables 197

lxlx. y) = ~siny -=>lxx(O, 0) = 0


lx/x. y) = ~C()sy -=>fxy(O, 0) = I
/y/x. y) = ~siny -=>/y/O, 0) "= 0
lxxi'" y) = ~siny -=>j~x..l0' 0) = 0
lxx/x. y) = ~sillY -=>j~-X/O, 0) = 1
j~J'Y(x, y) = ~siny -=>fxy/O, 0) = 0
fy;./x. y) = ~C()sy -=>1;'JY(x, y) = -1

ww Then by Taylor's theorem we have


f(x. y) = j{0, 0) + [xlx(O, 0) + y1;. (0, 0)]

w.E I
+ 21 2 .2
[xfxx (0,0) + 2xylxy (0,0) + yjXy(O, 0) ] + ......

a syE =
J
x-
J
y-
J
x-
0 + x(O) + y( 1) + - (0) + xy( 1) + - (0) + - (0) + .....
226

x y
=y+xy+---+ ..... ngii2

3.15.5 Example
2 6
nee
j(x. y) =
rin
x 2y + 3y - 2 in powers of (x-I) and (y + 2) by Taylor's theorem
Solution
g.n
j(x, y) =

+
f(a, b) + [(x-- a)lx (a, b) + (y - b)fy (a, b)]

I
21 (x - a)2 fxx(a, b) + 2(x -a) (y - b)j~ (ab)
et
+ (y - b)2.fyy (ab) + ..... .... (i)
Here a = 1, b = -2
f(x. y) = x 2 + 3y - 2 -=> j{ 1, -2) = -10
fJx.
x y) 2xy -=> J,x (I, -2) = -4
/y(x. y) =x2 + 3 -=> 1;,(1, -2) = 1+ 3 = 4
lxix. y) = 2y -=> lxx(\' -2) = 2(-2) =-4
fxy(x. y) = 2y -=> fxy(\' -2) = 2( 1) =2
j~(x. y) = 0 -=> j~(1, -2) = 0

Downloaded From : www.EasyEngineering.net


Downloaded From : www.EasyEngineering.net

198 Engineering Mathematics - I

11)=0
Jrxx.\ (x'J/ --- Jrxxx (I ' -2)=0
---,I'

' /x Y')/
j xxy\" = 2 => j'xx)'(I , -2) = 2

f' /x y,)/ = 0
, AYY\"
=> j'XJ'}'(I , -2) = 0
' /x Y')/ = () => j'»)')'(I , -2) = 0
j .Y.ly\"
Substituting a = -1, b = 2 and above values in (i) we have
x 2y + 3y - 2 = - 10 -4(x -I) + 4(y + 2) - 2(x-l;2
+ 2(x- -1) (y + 2) + (x - 1)2 Cv + 2)

ww
3.15.6 Example

w.E
Evaluate log[(l.03) I] + (0.98) '4 -1] ,approximately.

Solution

a syE
ngih)~ k)~
Then f(x + h. y + k) =
nee
IOg( (x + + (y + - I)

Assuming x = 1, y = 1, II = 0.03, k
rin= - 0 .02

We get F(x+h, y + k) = IOg[(l.03)~ +(0.98)~ -IJ g.n


=
ox oy [
oF OF)
F(x, y) + h-+ k - approximately ..... (A) . et
I 2
3 1 --3/4
of -x of -y
3 4
1 I I
ox oy - -

x 3 + y4 _I x 3 + y4 -1

= F(x, y) + h (OF)
ox + k [OF) .
oy approximately

Downloaded From : www.EasyEngineering.net


Downloaded From : www.EasyEngineering.net

Mean Value Theorems and Functions of Several Variables 199

~ IOg[ (1.03)~ + (0.98)~ -I]

= F(I, I) + 0.03 (aF) + (_O.02)[aF)


ax (1,1) ay (1,1)

~O.03 ll+I-IJ
r j to.oJ ~ 1~O.005
ll+I-IJ
approximately

ww (-: F(l,I) = 0)
w.E Exercise - 3(J)
asy
I.

En
Obtain the expansionllsing Taylor's theorem of the following:

I. xy2 + (I'~)
cos(xy) at
gin
eer
ing
2. xY at (I, I) lIpto second term
.
+ - ne
1
I ADS: xY = 1 +(x -I ) +(x -I) x (y -I ) 2
(x_I)2
tI

3. sinx. siny in powers of (x -~) and (y -~)


( ADS. ~+~(x_~)+~(V_~)_~(X_~)2
·2242' 444

4. eax sinby at (0, 0)


( ADS: hy + xyah + '" )

Downloaded From : www.EasyEngineering.net


Downloaded From : www.EasyEngineering.net

200 Engineering Mathematics - I

3
. ( x+ y)
5. Prove that sm(x +y) =x +Y- + .....
<3
2
6. Show that eY 10g(1 + x) = x + xy - ~ approximately
2

7. Expandf(x, y) = sinxy in powers of (x -1) and (y - ~) upto second degree term.


2
[ ADs • I - -1t (x - 1)2 - -1t(x -1 \ y - -1t) - -1 ( y - -1t)2 + ..... I

ww • 8 2 2 2 2

w.E
3.16.7 Maxima and Minima
The function F(x, y) is maximum at (x, y) if for all small positive or negative

asy
values of hand k; we have
f(x + h, y + k) - F(x, y) < 0 .... (i)

En
Similarly, F(x, y) is minimum at (x, y) iffor all small positive or negative values

gin
of hand k; we have
F(x + h, y + k) - F(x, y) > 0

eer
Thus ifF(x, y) has a minimum or maximum value then [F(x + h, Y + k) -F(x, y)]
keeps the same sign for all small positive or negative values of hand k.
Now using Taylor's expansion, we get ing
F(x + h, y + k) - F(x, y) haF aF)
-+k - .ne
t
=
( ox
ay

2 2 2
-f -2-(h2 a F +2hk a F +k2 a F)
+ 21 ax 2 axay 0'2 + ..... ..... (iii)

Assuming h, kto be sufficiently small; the sign of the expression on the left can
aF aF
be made to depend on that of h ax + k 0'

Hence the necessary condition for f(x, y) to be a maximum or minimum is that

Downloaded From : www.EasyEngineering.net


Downloaded From : www.EasyEngineering.net

Mean Value Theorems and Functions of Several Variables 201

of of
- = 0 and - = 0
ox oy
of of
I.e., p = 0 and q = 0 where p = ox' q = oy

2 2 2
then (iii) ~ f(x + h y -I- k) - j(x y) = -
I[ ,0 F 0 F , 0 F]
11- --) + 2hk-- + K- --) + .....
. '2! or ox8y oy-
I
= - r rh 2 + 2shk + tk2 ]
ww 02F 2
~l
2!

w .Ea =r- [ h-,


s=--
ox8y'

s t ,]
+ 2hk-+-K-
z= 0
~y-

r
syE) (2 ,II
2!

s
r

-
r

- s ) k-
=-
[(
2! )
ngi
h+-k
r
+ -,-k- +1-)
r- r-

=~[(h+~k)2 +(rt-.~.2)k:lne
2! r r-
eri
It has the same sign for all h, k if and only if (rt - s2) is positive.
ng.
When (i)

(ii)
(rt - s2) positive,j(x, y) is maximum for negative 'r' and minimum
for positive 'r'.
(rt - s2) is negative, we have neither a maximum nor minimum and
n et
such stationary points are called saddle points.
Solvingp = 0, q = 0, we get the extreme points
Extreme value :
.I(a, b) is said to be an extreme value ofJ(x, y) if it is either maximum or minimum.
3.16.8 Example
Find the maximum and minimum values of x 3 + 3xl- 31 + 4.
Solution
We have J(x, y) = x 3 + 3xl- 3x2 - 31 + 4
fx = 3x2 + 31- 6x, fy = 6xy - 6y, f)y = 6x- 6, 1;)' = 6y

Downloaded From : www.EasyEngineering.net


Downloaded From : www.EasyEngineering.net

202 Engineering Mathematics - I

We now solve
Ix = o,f;, = 0 simultaneously
I.C., x2+y~2x=0 ..... (i)

and xy ~Y = 0 ..... (ii)


xy ~ y = 0 => y = 0 or x = I
From x 2 + y ~ 2x = 0 we get
when y = 0, .~ ~ 2x = 0

ww x = 2 or x = 0

x= l,y~1 0 i.e.,y= I or~1


w when

(a) For
.Ea
=

Thus the points are (0, 0), (2, 0), (I, I), (I, ~ I)
x = 0 , y = 0 , r =j'xx = ~6 ' s = 0 ') t = ~f)
rt ~ syE
s2 = 36 ~ 0 > 0

But r=/xx = n
j(x, y) is stationary at x = 0, y = 0
~6 <0 gin
e eri
fix, y) is maximum at x = O,y = 0 and the maximum valuej{O, 0)
=4
(b) Por x = 2 , .y = 0, r =/xx = 6 ' s =/.xy = 0 , t =j'n' = 6
ng.
Again rt ~s2 > 0

j(x, y) is stationary at x = 2, Y = 0 n et
But r =/xx = 6 > 0
j{x, y) is minimum at x = 2, y = 0 and minimum value j(2, 0) = 0
(c) At x = I, Y = I or at x = I, Y = ~ I
rt ~ s2 > 0 and we can reject these points as they are not stationary points.

3.16.9 Example
Discuss the maximum and minimum of
f('(,y) = x2 ~ Y + 6x -- 12
P = 2x + 6, q = ~2y, r = 2, s = 0, t = ~2
p = 0 gives x = ~3, q = 0 ~ y =0

Downloaded From : www.EasyEngineering.net


Downloaded From : www.EasyEngineering.net

Mean Value Theorems and Functions of Several Variables 203

Stationary point is (-3, 0); and here r = 2 >0; and


rl-s~ == 4-0 == 4 > 0
) ) - 1,...
x- -.V + 6x - .!. is minimum at (-3,0) thc minimum valuc is
j'(-3,O)==-21.

Exercise -3(k)

I. Find the maximum and minimum valucs of:


I. x' + 3x/ - 3/ + 4 jAns : (0,0) max. its valuc 4,(2,0) min. its value 'W]
.... x'+3xy ) - 3.c)-3Y) +
7
ww [Ans :max . value 7 at (0,0) min.value '3' at (2,0))
')

3. x~ + / +6x+ 12 [Ans: mill. value '3' at (-3,O)j

w.E
4.
5. X4
3
x + x/ + 21x -12x2 - 2./
+x/ + /
l Ans : max.value 10 at ( 1,0) min. value -98 at (7,0) 1
jAns: minimum value '0' at (0,0)1
6. X4 + / _ x2 _ y2
asy
+1

En ~
lAns: max.value I at (0,0) min.value Y2 at t(Hlr pOints( ± ± ~)]

7.
) 0 3
Y -x Y -x Y
2 ~ 3 gin 1 (1 1)
jAns: max.valuc --at -,- ]

eer
X
324 2 2
) ) ) 3
Discuss the stationary values of II == x-y-3x- - 2y -4y + [Ans: u is a
ing
8.
maximum at(O,-I). The maximumvalue is 5, u is neither maximulll nor minimum

9.
at(±4,3)
.n
Find the maximulll and minimum values ofthc following functions.
(a) x'/(12-3x-4y) jAns:maximulllvalucat(2,1)] et
(b) Xl + / - 3axy( a > 0) [Ans: minimum value is _(/' at (a,a)1
1

(c) xy(a-x-y) (a>O) [Ans:maximum value is ~ at (a/3,a/3)]


27
10. Determine the values of x,y for which the following functions are maximum or
mllllmum ..
(a) x ' y 2(1-x- y) l Ans: maximum for x = 112 , Y = I/3 j
(b) (x 2 + y2)" -2a\x~ - /) [Ans: minimum for x == ± a,y = OJ
(c) sinx siny sin(x+ y) [Ans : maximum for x = y = 7[ /3]
(d)a[sinx+siny+ sin(x+y)] [Ans: maximum for x = y =7[ /31
(e) x/(3x+6y-2) [Ans: minimum at x = y = 1/6J

Downloaded From : www.EasyEngineering.net


Downloaded From : www.EasyEngineering.net

204 Engineering Mathematics - I

Lagrange's method of undetermined multipliers:

3.17.0Theorem: Find the maximum and minimum values of f(x"X 2'X3' ......xlI )
where x" x2 ' x 3 ' •••••• XII are connected by the following m equutions.
¢, (x, ,x2 ' •••••••• x,J= 0
¢2(X"X2' ....... .xJ = 0

ww¢m(X, ,X2 ,.·· .... .xll ) = O.


Proof: Let u = f(xl'x2, .........x,J .Since n variables are connected by the m given

w.E
relations, only n-m of the variables are independent.

The maximum and minimum values ofu can be found by the method of Lagrange's

asy
method of undetermined Multipliers .For u to be max or min of u , du = 0

i.e.
au au au
-dx, +-dx2 + ........ +-dxn =0
ax, aX2 aXil En
also from, we have gin
..... ( 1)
eer
ing
multiplying the above equations by 1, ~, ~, ....Am respectively and adding, we get

au a¢, a¢2 a¢m ) ( au a¢, a¢l a¢m )


( -+~-+~-+ ........ +AI/I- dx, + -+~-+~-+ ........ +AI/I- dx2
~ ~ ~ ~ ~ ~ ~ ~ .ne
+.................................... .
au a¢, ~ a¢2 1 a¢m) _
+ ( -+~-+"'2-+ ........ +AI/I-- dxn -0. . .... (2)
t
ax" aXil aXil Oxll
The values ~,~, .... Am are at our choice .We can, therefore choose them so as to
satisty 'm 'linear equations .
.. .. (3)

I(is immaterial which n - m of the n variables are regarded as independent.

Let these bexl/I+"x"H2' .......x".Then since the n - m quantities dxm+"dxm+2, .......dx" are
all independent, their coefficient must be separately zero. This gives additional equations.

Downloaded From : www.EasyEngineering.net


Downloaded From : www.EasyEngineering.net

Mean Value Theorems and Functions of Several Variables 205

..... (4)

au + A, atA + ~ a¢2 + ........ + Alii a¢m = 0


axil aXil - axil aXil
= 0, ¢2 = 0, .......¢III = 0
ww
Now we have m+n equations in all and given relations ¢,

The equations (3) and n-m equations (4) are sufficient to determine the m + n
quantities
wA,,~, .... AII/ and
.Eax"x2 ' ••••••••• x lI
for which maximum and minimum values of u

syE
are possible .The m multipliers are called Lagrange's Multipliers.
3.17.0 (a)Theorem: If u=<p(x,y, z), j; (x,y, z)=O., .t;(x,y, z) =0 discuss the
maximum and minimum values ofu
Proof: For a maximum or minimum, we have du ngi =0
I.e.
af" af" af"
-'-' d" + -'-' dy + _0_' dz = 0 nee
ax ay oz
rin
Also from j; = 0 andJ; = 0 g.n
FaJ; dx+ aJ; dy+ a.t; dz=O
ax iY az e '" .. ( 1)
t
aJ; dx+ aJ; dy+ aj; dz = 0
ax iY az
Multiplying these equations by 1, A" ~ respectively and adding

a¢ + A, aJ; + ~ aJ;) dx + ( a¢ + A, a.t; + ~ aj ;) dy + l( a¢ + A, a.t; + ~ a.t;) dz = 0


( ax ax ax liY iy iy fu fu fu

... ~ ... (2)

The lagrange's multipliers A" ~ are choosen such that

Downloaded From : www.EasyEngineering.net


Downloaded From : www.EasyEngineering.net

206 Engineering Mathematics - I

~ 01;
o¢ +:1/11 ()I; + /l, = 0 .... (4)
oy oy - ~v
then equation (2) becomes
o¢ +:1/11 ~/~ + /L1) ~/; =0 •.•• (5)
()z oz - oz
These equations (3) ,(4), (5) and given 1;<'t,y,z)=0, .I;(x,y,z)=O are
sufficient to find ~, ~ ,x,y,z which give maximum and minimum values of u ..

ww
3.17.0(b) Theorem: If u
minimum values ofu.
= ¢(x,y, z) and f(x,y,z) = 0 find the maximum or

w o¢ .Ea
Proof: For a maximum or a minimum
o¢ o¢
II ,we have du = 0

I.e. ~.dx+~.dy+~.dz
ox oy
syE
oz
=0 .... ( I)

and
from given
of
f(x,y,::) = o.
ol ~l
n gin
- .dx + -.Jy + -.Jz = 0
ox oy OZ
e eri
.... (2)

Multiplying (1) by 1 and (2) by A and adding

o¢ + A of)dx+( o¢ + A ol)dY+(o¢'+ A ol)dZ = 0 ng.


( ox ox oY oy OZ
Lagrange's multiplier A is choosen such that
oz
n .... (3)

et
o¢ +A ~l =0
ox ox
.... (4)

and o¢ +A ~l =0 .... (5)


Oy oy
The equation (3) gives
o¢ +A ~l =0 .... (6)
oz oz
The equations (4),(5),(6) and 1 (x,y, z) = 0 enable us to tind the values of A "x,y,z for
which u is a maximum or a minimum.

Downloaded From : www.EasyEngineering.net


Downloaded From : www.EasyEngineering.net

Mean Value Theorems and Functions of Several Vanables 207

Exercise - 3(1)

3.17.1 Example
Find the point upon the plane ax + by + cz = p at which the function
$ = x 2 +1+ z2 has a minimum value and find this minimum $.
Solution
p-ax-hy
ax+by+:::=p ~:::= ~---~ ..... (i)
e

ww
)

Again $= x~
') . .2
+ .V"" + z-
J
= r
') J
+ ), +
( 17 - axc- ·bV)-

w.Eox .
Then for min./max., we have

0$ = 2 x - 2a2 (p _ ax - by) = 0 ..... (ii)

asy ~$ = 2y- 2~
e

En uy e-
(p _ ax - hy) = 0 ..... (iii)

From (8) ~-=


x p-ax-hy
) gin
~-=
a

y p-lLr-hy
c-

.
e eri
h
x
c2
bx ng.
net
y
~-=-'--~ y=-
a . (lb
Now, substituting this value ofy in (ii) we obtain
2
X) =0
x- 2a ( P - l I Xb- -
e 1I

c 2x - (lP + a2x + b2x = 0


x( a 2 + b2 + ( 2) = (lP
ap bx bp
x = )2 ) and Y = - =) ) )
a +h- +c a a- +b- +c-

Then,

and

Downloaded From : www.EasyEngineering.net


Downloaded From : www.EasyEngineering.net

208 Engineering Mathematics - I

Hence 3 a min, when

- =-y =----:---=-::----c-
x p
2 2 2
a b a +b +c

=~[p_
2
Also Z = p-ax-by a p
c c a2 + b2 + c2

P
ww
Z
I.e., =

w Hence 3 a min. at

.Ea xy Z
-==-
abc
-
2
P
a +b 2 +c 2

4> . =.xl + y2 + z2 = s(ayE c r


a 2 p2 + b 2 + p2 +C2p2
----'---,---...:...--.---:'--

ngi + b2 +
mm 2 2

3.17.2 Example
nee
Divide 24 into the three parts such that the continued product of the first, square
of the second and cube of third is maximum.
Solution r ing
Let 24 be divided into parts x, y, z then x + y + z
Given 4>(x, y, z) x 2 y2z
= 24
.ne ..... (i)

t
=

then 4>(x, y) =.xl;? (24 .- x - y) from (i)


or 4>(x, y) = 24x3y2 - x4y2 - x 3y3
Differentiating, we get

p = ~! = n.xly2 - 4~y2 - 3x2y3

a 4>
r =-?
2
= 144xy2 - 12x2y2 - 6xy3
ax-

Downloaded From : www.EasyEngineering.net


Downloaded From : www.EasyEngineering.net

Mean Value Theorems and Functions of Several Variables 209


- - --- - - - - - - - - - - - - - - - - - - - - - - - - - -

and S =-02~
- = - (o~
oxoy oX ~Y
- a 1 = 144x2y-? - 8x3y - 9x 2y-?
.

For a maximum value of ~(x, y)we have

a~ = 0 => 72x 2y"2 _ 4x 3i - 3x2y3 = 0


ox
=> x 2i (72 - 4x - 3y) = 0
=> 72 - 4x - 3y = 0, x = 0, y = 0 ..... (ii)

ww and o~ = 0 => 48x3.v -


~
2x4y - 3x3 v2
.
= 0

w .Ea
=> x 3y (48 - 2x - 3y) = 0
=> 48 - 2x - 3y = 0, x = 0, y = 0 ..... (iii)
Solving (ii) and (iii) we get x
At (12, 8), we have
syE = 12 and y = 8 etc

n gin
r= 144, 12(8)"2- 12(12)2(8)-6(12)(8)3
= 12(8)2 (144 - 144 - 48) = 12(8)2 - (-48)
r=48(12)3_2(12)4_6(12)3.8
e eri
= (12)3 (48 - 24 - 48)
s = 144( 12)2 . 8 - 8( 12)3 . 8 - 9( 12)2 (8)2 ng.
:. rt - s2
== (12i. 8 (144 -96 -72) == (12)2.8(-24)
=12. (8)2 (-48). (12)3 (-24) -l (12)"2.8.
n
(-24)]2 et
== (12)4.8 2 .24 (48 - 24)

== (12)4.8 2 .242> 0;
Since rl - s2 > 0 and r < 0, therefore ~ (x, y) is maximum at (12, 8).
Putting x == 12, and y == 8 in (i), we get z == 4
The values of x, y, z are 12, 8, 4 respectively. This is the division of 24 for
maximum ~(x, y, z).

3.17.3 Example
A rectangular box, open at the top, is to have a volume of 32 c.c. Find the
dimensions of the box requiring least material for its construction.

Downloaded From : www.EasyEngineering.net


Downloaded From : www.EasyEngineering.net

210 Engineering Mathematics - I

Solution
Let I, band h be the length, breadth and height of the box respectively. Then,
wc have
v = Ihh = 32 ; surface = 2(1 + h) h + Ih = S (say)
32
S = 2(1 I- b)h + Ih and b = - .....(i)
III

32) h + I (32)
s = 2 ( 1+- - = 2111 + -64 +-
32
III liz I h

ww Now ~~ = 2/-(:;)
w .Ea
syE
For maximum and minimum ofS, we get

as =0= as
al ah

as = 0 ~ 2h _ 642 = 0 h = 32ngi
0/ 1 ' 12
nee
and
ah
as = 0 ~ 2/- 32 = 0
h2 rin
g.n
e t

and 82~ = +2 so 'S' is minimum for 1= 4,b = 4,h = 2.


8h-

Downloaded From : www.EasyEngineering.net


Downloaded From : www.EasyEngineering.net

Mean Value Theorems and Functions of Several Variables 211

I 1 1
3.17.4 Example: If u=a'x"+h'/+c3z1 where -+-+-=1 show that the
x y z
a+h+c a+h+c a+h+c
stationary point of u is given hy x =- - - , y = ,,"" - - - -
a h c
Solution: For a stationary value of 'u', du =0
1
I.e. a ' 2xdx+h'-2ydy+c 2:::dz =0 ..... ( I )

Also we have the given relation ~~ + l + l = 1


x y :::
1 1 1

ww
-) dx+-) dy+-) dz = 0
x~

..... (2)
y .. ~

I A
w.E
Equating the coefficients of dx , dy ,dz from (1)+ A(2) separately to zero
I A 1 A

1
x-
a
We get a x + -) = 0, h x + -) = 0, c x + -, = 0 .

:. a x ' =h ' /=c ' z'-=(-A)


y
syE --

or ax = by = cz = (-At] = k(say)
x = k I a; y = k I h; z = k I c ngi
..... (3)
nee
· . j'or x, y, Z
SU hstltutll1g .
In -
X
1 + -1 + ~~1 = I
Y z rin
abc
-+-+-= 1
k k k g.n
:. k = a+h+c
..... (4)
et
Substituting in (3) we get
a+b+c a+b+c a+b+c
X= ,y= , z = - - - for which u is stationary.
II b c

Exercise 3(M)

I. Find the maximum and minimum distances from the ongll1 to the curve
5x" + 6xy + 5 y2 - 8 = O. [Ans : max. 4, min .16,6,3]
2. Fin~the dimensions of the rectangular box, with out a top of maximum capacity
whose surface is 108 Sq. inches. rAns:6",6",3" J

Downloaded From : www.EasyEngineering.net


Downloaded From : www.EasyEngineering.net

212 Engineering Mathematics - I

3. If f = u 1 +j'1 +W1 where 11 + v + w = 1, show that f IS stationary when


1
U=V=W=~
3
4. Use Lagrange's method of multiplies to determine the minimum distance from the
origin to the plane x + 2y + 3z = 14. [Ans: Jl4"1
5. Find the maximum value of / = xyz when xy + yz + zx = k. [Ans: (k / 3)3/?]
6. If the distance from the origin of any point p(x,y, z) on the plane
°
ww 3x+2y+z-12 =
Lagrange's method).
is P =~X2 + y2 +Z1 ,find the minimum value of p(use the

7.
w.E
Find the maximum

x-, + y-) +z 2 = r.
'
volume of a parallelepiped inscribed III

[Ans:
8r
a sphere
2
]

8. a syE
Find the volume of the largest rectangular parallelepiped that can be inscribed in
) , , h
3J3
· ·d x- Y z- 1
t I1e e II IpSo! - 2 +-) +-) = .
a b- c- ngi 8a c]
[Ans: 3 (:;3
-V j

9. Given x+y+z=k,findthemaximumvalueof XII,yll,ZY


nee 1
rin
[Ans:
aa./3II.rY.k<at/l+Yl
[ (a+/3+rt l /l+ Y J

10. If r2 = x
2
+ l + Z2 g.n
and x+y+z =30 find the values of x,y,z for which r IS a

11.
mll111nUm
Find the dimensions of a rectangular box without a top of maximum capacity
whose surface area is 108 square inches. [Ans: 6,6,3 inches]
[Ans: x = y = z = 10]
et
12. Find the dimensions of a rectangular box with open top, so that the total surface
area S of the box is a minimum, given that the volume 'V' of the box is constant.
fAns: x = y = 2z = (2\,)1/3]
13. Show that the rectangular solid of maximum volume of that can be inscribed in a
sphere is a cube
14. I f the total surface area of a closed rectangular box is 108 sq.cm. Find the
dimensions of the box having maximum capacity [Ans: .Ji8,.Ji8,.Ji8]
15. Perimeter of a triangle is constant, prove that the area of this triangle is maximum
when the triangle is equilateral.
16. If u=¢(x,y)where/(x,y)=O, find the maximum and minimul.1 values of 11
using Lagrange's multipliers Method

Downloaded From : www.EasyEngineering.net


Downloaded From : www.EasyEngineering.net

4
ww
Curvature and Curve Tracing
w.E
asy
4.1.1 Curvature En
gin
The curvature of a curve (bending of a curve) varies from point to point on the
curve.
eer
Let P be a point on the curve and Q be c.l point nearer to P, are OQ = 0 s

ing
Let tangents at P and Q make angles, \.11, and \.11 + 0\.11 with the X- axis.

LT R T' = d\jJ is the angle through which the tangent at P turns as a point moves
along the curve form P to Q through a distance os along the curve and hence 0\.11 .ne
depends on the arc length os.
0\.11 is defined as the total bending or total curvature of the arc P Q.
t
0\.11
8s is defined as the average curvature of the arc P Q.

Now os ~ 0 as Q ~ P

Lt ~\.11
&s~O uS
= dd\.l1 is defined as curvature of the curve at the point P and is denoted
S

by K.
d\.l1
Thus K=-
ds

Downloaded From : www.EasyEngineering.net


Downloaded From : www.EasyEngineering.net

214 Engineering Mathematics - I

ww x' 0 T

w.E
Radius of Curvature asy Fig. 4.1

En
The reciprocal of the curvature of a curve at any point' P' is called the radius

gin
(?f curvature at that point and is denoted by 'p'.
ds
eer
Thus p = -d is called the radius of curvature of the curve at the point P.
\1' .

4.1.2 Theorem ing


.ne
If 'r' is the radius of a circle then the radius of curvature of the circle is same as
its radius.
Proof:
Let 'c' be the centre and 'r' be the radius ofa circle. P be any point on it and Q
t
be a neighbouring point, arc PQ = ds.
The tangents at P and Q make angles \If and \If + 8~, respectively.
From the figure

LT R T' = 8 \If

LP R Q' = 180 - 8\1f


LP C Q' = 8\jf, from the sector PCQ.

8s
~ = r and as Q ~ P, 8s ~ 0
u ~,

Downloaded From : www.EasyEngineering.net


Downloaded From : www.EasyEngineering.net

Curvature and Curve Tracing 215

Lt 8 \1' = ~
8s
01->0 r

1
I.e.,
P r
p=r

ww
w .Ea
x' o
syE x

y' n gin
Fig. 4.2 e eri
4.1.3 Cartesian form of the Radius of Curvature
ng.
tangent to the curve at a point P.
dy
n
Let y = j{x) be the Cartesian ~quation of a curve and tan (\lJ) be the slope of the

et
tan \lJ = dx

Differentiating w.r to 'x'

We know that

ds
dx

Downloaded From : www.EasyEngineering.net


Downloaded From : www.EasyEngineering.net

216 Engineering Mathematics - I

... ,.. ,

1+(ddxy )2

[I+(~~)' I+(~)' 1
p=

ww
w .Ea
syE
.
I.e.,p-
3/
_I[ + y( ) '2
whereYI
dy
ngi
d 2Y
= -d 'Y2=-2
y~ X dx
nee
is the radius of curvature of the curve (Cartesian form).

4.1.4 Parametric form of Radius of Curvature rin


Let x = fil), Y = get) be the Parametric form of the equation of a curve.
g.n
i.e.,
-d'(
dl
x'
= f\t() -dy
= /'(t),
' dl
y'
=g ,( )

= gl(t),
t
e t
dy
dy
-=-
dl
dx dx
dt
'., -,
dy y'
- = -I .
dx X

Downloaded From : www.EasyEngineering.net


Downloaded From : www.EasyEngineering.net

Curvature and Curve Tracing 217

Differentiating w.r. to 'x' on both sides

•,{2y " .- y·x


X 'Y '"
--=
{X')3
3/

ww dy d'y [l+(~)T
w .Ea
X
2 values in p =
Substituting -d ' - I
ex .
d2
Y
dx 2

sy[l+(Hr
En -
'----'..,..
p= gin x'y" - y'x"

e {X')3
eri
[(XI)2 + {y')2 y~ ng.
P=

I dx
x'y" - y'x"

dy
I
net
where x =- y=-
dt ' dt

is the parametric form of the radius of curvature,

4.1.5 Polar form of Radius of Curvature


Let P (r, 8) be a point on the curve r =f(O). If '0' is the pole, 0 X is the initial
line.

LXOP =0
The tangent at P makes an angle \I' with the initial line.
$ is the angle between the radius vector OP and tangent PT.

Downloaded From : www.EasyEngineering.net


Downloaded From : www.EasyEngineering.net

218 Engineering Mathematics - I

From figure \If=e+cj>

d\lf de d<l>
-=-+-
ds ds ds

1 de d<l> de
-=- +-.-
p ds de ds

!
p ds
[1
= de + d<l>]
de
..... (1 )

ww
w.E
asy
En
gin
eer
x

Fig. 4.3 ing


ds
r2+ (dr)2 .ne
We know
de

de
-
de
t
..... (2)

and tan = r-
<I>
dr
r
i.e., tan cj>-=-
dr
de

Differentiating w.r. to 'e' on both the sides

Downloaded From : www.EasyEngineering.net


Downloaded From : www.EasyEngineering.net

Curvature and Curve Tracing 219

2 d~
(1 + tan <1» de =

ww
w .Ea
d~
de

syE
ngi
d<l>
de nee
rin
The value of
I+ ~~ I + r' + t~ r-~~r g.n
e
de r2 + -(dr)2
de t
r2 +2(_dr)2 _rtl_2_r 2
d,h de de
1+ = --'-----'------:---
+(-:r ..... (3)
-'I'

de r2
ds , d~
Substituting the value of de' I+ de from (2), (3) in (I)

Downloaded From : www.EasyEngineering.net


Downloaded From : www.EasyEngineering.net

220 Engineering Mathematics - I

is the polar form of radius of curvature.

4.1.6 Pedal form of Radius of Curvature


Let P be any point on the curve. the tangent P T and P makes an angle \jJ with
the initial line OA, LP 0 X = e (see the figure in 4.1.5)

ww \jJ=e+~

Differentiating w.r. to s
..... ( I)

w.E d\jJ
-=-+-
d\'
de
ds
d~
ds

I asy I de d~ dr
-.r- + - -
En
- =
p r ds dr cis

I
-
p
I
= - (sin~)
r
+ -
dr
d~
gin
cos~,

We have sin~=r-, -
de I e
I [ sm~+rcos~-
.
eri
d~] ,cos~=-
dr

ng.
= -
~ p r ~ ~

p
I d(r sin ~)
r dr net
p = r sin ~

_ = ~dp :. p =
dr
r-
p r dr dp

where p is the perpendicular distance from pole to the tangent.

4.1.7 Examples

2
Prove that the radius of curvature of the curve y = a cosh (x/a) is L
a

Solution
y = a cosh (x/a)

Downloaded From : www.EasyEngineering.net


Downloaded From : www.EasyEngineering.net

Curvature and Curve Tracing 221

dy I
- = a sinh(x/a)-
dx a

d 2y I
-2 = cosh (x/a)-
dx a

dy d 2 y ( dy)2l·~2
[ I + ~j;
Substituting the values of - , - - ? in p = d2

ww dx dx-
dx 2
Y

w.E ?
P =1[ + sinh - x / a
]3 2

p asy
cosh{x / a ).~
= aeos h2(x/a)

~ a( ~:), En
~
p as y
gin
cosh(x/a)

p~ ( : ) e eri
ng.
4.1.8 Example
net
Prove that for the rectangular hyperbola xy = c2 the radius of curvature at any point
r3
is given by p = -2 where 'r' is the distance of the point from the origin.
2c

Solution
Differentiating xy = e 2 w.r. to 'x'

dy
x - +y=O
dx

Downloaded From : www.EasyEngineering.net


Downloaded From : www.EasyEngineering.net

222 Engineering Mathematics - I

.. dy y .
d 1+
2 [ ( ~b:dy)2l~~
Substltutmg for -d ,--? m P = /2
X dx- ( Y
dx 2

ww
w.E p=

asy
En
gin
e eri
as ,.2 = x 2 + rand xy = c 2

ng.
net
4.1.9 Example
Prove that the square of the radius of curvature at any point on the parabola
y = 4 ax varies as the cube of the focal distance of the point.
Solution
From y=4ax ..... (1 )

dy = ~
dx f~
d 2y I -3/
- = - lax /2
dx 2 2 -va.

Downloaded From : www.EasyEngineering.net


Downloaded From : www.EasyEngineering.net

Curvature and Curve Tracing 223

Hence p=

ww
w x'

.Ea
o x

syE
n
y'

gin
Fig. 4.4
e eri
p=
-1 r -~~ ng.
-~"a
2
x
n et
p = 1a(x+a)~ ..... (2)

The focal distance of the point p (x, y)

sp= ~(x-aY +(y-oy


= ~X2 +a 2 -2ax+ y2

= ~ x 2 + a 2 - 2ax + 4ax using(l)

sp= ~(x+aY
SP = focal distance = (x + a) ..... (3)

Downloaded From : www.EasyEngineering.net


Downloaded From : www.EasyEngineering.net

224 Engineering Mathematics - I

From (2) and (3)

-2
P = ~ [Focal distance]3/2

4
p2 = - [Focal distance]3
a
p2 00 = cube of the focal distance.

wwy2
4.1.10 Example
Show that the radius of curvature at an end of the major axis of the ellipse

w Xl
+---;;:
a2 =
.Ea
1 is equal to the semi latu~-rectum of the ellipse.

Solution
syE
ngi
i ..e., nee ..... (1 )
Differentiating (1) w.r. to 'x'
rin
dy
dx g.n
d y 2 b 2fy-xdyl
dx
e t
dx 2 = - --;; y2

Downloaded From
/ : www.EasyEngineering.net
Downloaded From : www.EasyEngineering.net

Curvature and Curve Tracing 225

y I

x' A'(-a,O) o (0,0) A(a,O)

ww y'

w.E
asyi
Fig. 4.5
d 2y b2 ( using ax'2 + b2 1
y' = I

En
--
2 ----
dx 2
a

[1+(:)']" gin
p=
d 2y eer
dx 2 ing
Hence p=
[ l+---~
h"
4
a i
r; (a 4i+b 4x2)
.ne
t
..... (2)
_b 4 a 4b 4
2
a i
The coordinates of the ends of the major axis are

A (a, 0) and A' (-a, 0)


Taking the end A (a, 0) we have from (2)

b2
pl(a,o) = -;; = Semi latus-rectum of the ellipse.

Downloaded From : www.EasyEngineering.net


Downloaded From : www.EasyEngineering.net

226 Engineering Mathematics - I

4.1.11 Example
J J' 5<
Prove that p at any point of the curve x;; 3 + y/3 =a 3 is three times the
length of the perpendicular from origin to the tangent at that point.
Solution
The parametric equations of the curve are
x = acos 3e, y = asin3e
dy
3a sin 2 e cose dy
ww dy
dx =
de
dx
-------,,---- - - = -
- 3a cos 2 e sin e dx
tan e

w.E d 2y
de

-sec2e-
de
and --
2
dx
=

asy dx
= - sec2e / (-3acos 2 e sin e)
En 3a cos 4 e sin e
gin
e eri
Hence gives
ng.
(I + tan 2e f2
net
p= I
4
3a cos e sin e
p = 3a cos e sin e ..... ( I)
The perpendicular distance from the origin to the tangent
dy
y-x- 2
a sin 2e - cos e(- tan e)
p = ---;===dx==
I+(:J 2
JI + tan e

P = a sin e cos e ..... (2)


From (1)&(2) p=3p

Downloaded From : www.EasyEngineering.net


Downloaded From : www.EasyEngineering.net

Curvature and Curve Tracing 227

4.1.12 Example
Find p at any point for the cycloid
x = a (8 - sin 8)
y = a(1- cos 8) and also find p at 8 = 90°
Solution
dy
dy _ de _ a(sin8) dy 8
- = cot-
dx - dx - a(l-cos8)' dx 2

ww 2
de

and
w.E -
d y
dx
2
=
2 8 1 d8
cosec - - -
2 2 dx

a d 2y
dx 2
=
1 28
-2cosec 2
syE
a(1-cos8) ,
-1

n
[1+(:)'r (ginat 1+cot
2
2
.. p=
d 2y eer p=
-1
dx 2
i 4a sin4 ~
2 ng.
p = - 4a sin
8
2 net
at 8 = 90°, P = - 4a sin 45° i.e., p = - 212 a
4.1.13 Example
Find p at (r, 8) for the curve r = a (1 + cos 8) and also find at 8 = ~.
Solution
r = a (1 + cos 8)
dr
de = r) = a(- sin 8)

d 2r
- 2 =r =acos8
de 2

Downloaded From : www.EasyEngineering.net


Downloaded From : www.EasyEngineering.net

228 Engineering Mathematics - I

[a 2(1 + cos e)2 + 2a 2 sin 2 e + a 2cos e(1 + cos e)]

[2a 2 (1 +cose)7i]
2
[2a .2cos
2
~~]7i
= .iacos e/
3a .2cos ~~
2 2 2 2 2 3 /2
a + 2a + 3a cose

ww at e=~
w .Ea 4
P = -acos 45°
3
=
2 r;:;
-,,2a
3

4.1.14 Example syE


Solution n
Find p for the curve r'" = if! cos me

gin
r'" = if! cos me
mlog r = m log a + log (cos m e) e eri
1 dr
m -- =
1
(- m sin m e) ng.
. r de
r) = -r
cos me
tan me n et..... ( I)

ar
r2 = -r m sec2 me - tan m e. de

r2 = - rm sec2 me + r tan 2 me ..... (2)


From (1)

3
= {r 2+ r2 tan 2me)2
r2 + 2r2 tan 2me + mr2 sec 2me - r2 tan 2me

Downloaded From : www.EasyEngineering.net


Downloaded From : www.EasyEngineering.net

Curvature and Curve Tracing 229

r
{m + J)cos me
r am
m
{m+J( m
a
e
ww For the curve ,-In = d" cos m

w.E P= (m + J)r m- 1

4.1.15 Example
asy
Find the radius of curvature of the curve p2 = ar
Solution
En
p2
gin
= ar ..... ( 1)
Differentiating w.r. to 'r'

2p dp a eer
ing
=
dr
dp
dr 2p
a
.ne
p = r dr
dp
= r. 2p
a
t
From(l)
2r
p=-j;;;
a

4.1.16 Example
1 J J r2
Find the radius ofcurvature at the point (p, r) on the ellipse -2 = -2 + -b2 - ~b2
P a a

Downloaded From : www.EasyEngineering.net


Downloaded From : www.EasyEngineering.net

230 Engineering Mathematics - I

Solution

Differentiating

2 dp -2r
p3 dr a 2b 2
2
dr a 2b
=> r-=--
dp p3

ww
w
4.1.17 Example
.Ea
Find the radius of curvature p of the curve ,2 cos28 = if
Solution
syE
,2 = cos28 = a2 ..... ( I)
Taking logarithms on both sides we get ngi
2 log r + log cos 2 8 = 2 log a
nee
Differentiating w.r.to '8'
1 dr rin
2 -;: d8 - 2 tan 28 = 0
g.n
d8
r-=cot28
dr
d8
e t
tan <\> = r dr = cot 28

tan <\> = tan (~ - 28 )


1t
<\>=2- 20

We have P = rsin<\> => p = r sin (~-28)


P = r cos 28
a2
Substituting cos28 = -2 from (1)
r

Downloaded From : www.EasyEngineering.net


Downloaded From : www.EasyEngineering.net

Curvature and Curve Tracing 231

dp = ~
dr ,.2

dp ,.
p=r-=---)

ww dr -(r

w.E
4.1.18 Curvature at Origin asy
(i)
En
When y can be expanded in powers ofx by some algebraical or trigonometrical
method from the equation of the curve as

gin dyl qxl d2y


eer
y = px + -
2!
+.... then p = -
dx x=O
y=o
q = --)
dx- x=o
y=o

.. (I + p).i
3
ing
p (at ongll1) = --'----~­
q
.ne
(ii) Newtonian Method: If a curve passes through (0, 0) and the tangent at (0,0)

IS X - axis, then we have x = 0, y = °and d,Y = °at origin.


{X
t
The expansion of y by Maclaurin's theorem reduces to
Xl
Y = (yo) + (YI)O + 2 (Y2)O

°
Taking upto terms of (x 2 )

Y= °+ °. + x
x2
q - +...
2!
Dividing each term by x 2 and taking limits
2y
q= L t -
x->o x 2

Downloaded From : www.EasyEngineering.net


Downloaded From : www.EasyEngineering.net

232 Engineering Mathel\latics - I

3
., (I + p)2
We have P = (at ongln) = -'-------=--<--
q
as p = 0 at (0,0)
2y
and Lt 2
q = x~Ox

I
and P (at origin) =
q

ww p (at origin) = Lt
x~o
(~)
2y

w.E(iii) Similarly, if a curve passes through (0, 0) and the tangent at (0, 0) is Y-axis
2
then
asy p (at origin) = Lt (Y2
x~o X
)

En
(iv) Curvature at pole: If the initial line is the tangent to the curve at the pole

then gin
p= Lt -
X2 )
x~o ( 2y
= Lt
x~o 2rsin8
2
r2 cos 8 (.: x = r c.os 8)
y=rsm8

eer r 8 2
P = Lt -·--·cos 8
e~o 28 sin8
ing
- Lt - r )
p - e~o ( 28
8
e~osm8 .ne
(as Lt - .- ~ 1.cos28 ~ I)

4.1.19 Example
Find p at the origin of the curve y4 + xl + a (xl + y) - a2y = 0 by Newtonian
t
method.
Solution
Equating to zero the terms of the lowest degree in the equation of the curve
- a2y = 0
=? y= 0
i.e., x - axis is the only tangent at origin
Then 'p' at (0,0) when X - axis is the tangent is given by
x2
p= Lt- ..... (1)
e~o2y

Downloaded From : www.EasyEngineering.net


Downloaded From : www.EasyEngineering.net

Curvature and Curve Tracing 233

Dividing the given equation by y


x-J x2
I + x· -
y
+ a. - + ay =
y
a2

Let x~O
So that y~O

2
x
and - =2p
y

a
o + 0.2p + a2p "2
ww From (1) + a.O = a2 => 2 a p = a2 => p =

w
4.1.20 Example

.Ea
Find p at the origin for the curve x = a (8 + sin 8), y = a (I - cos 8) by Newtonian
method.
Solution
syE
x = a (8 + sin 8), y = a( 1 - cos8) is a cycloid
X-axis is the tangent to the curve at (0, 0)
ngi
p at (0, 0) = Lt (~l
x---+o 2y nee
2
Lt a (8 + sin 8)2 rin
0---+0 2a(1- cos 8)
g.n
%) e t
a (8 + sin 8)2
L t - -'-----'-- (Of the form
0---+02 l-cos8 '

= Lt a (8 + sin 8)2(\ + cos 8)


0---+02 sin 2 8

= Lt !!.(~+ 1)2 (1 + cos 8)


0---+02 sm8

a 8
= -2 (1 + 1)2 (I + 1) using Lt - = I
0---+0 sin8

a
p= - [8]
2
P = 4a

Downloaded From : www.EasyEngineering.net


Downloaded From : www.EasyEngineering.net

234 Engineering Mathematics - I

4.1.21 Example
Find pat (0, 0) for the curve 2x4 + 31 + 4x2y + x Y - y2 + 2x = 0 by Newtonian
method.
Solution
Equating to zero the terms of the lowest degree in the given equation of the
curve 2 x = 0 => x = 0
y-axis is the tangent to the curve at (0, 0)
2

ww P at (0, 0) = Lt L
x->o2x

w So that

.Ea
Dividing the given equation by 'x'
..... ( I)

syE
2x3 + 3y2. L
2 y2
+ 4x y + Y - - +2 = 0
x
ngi x
Taking x~O,y~O

nee
2.0 + 3.0· (2p) + 4.0 + 0 - 2 P + 2 = 0; p = 1

rin
Exercise - 4(A)
g.n
I. Find p for the following curves :

1. y = 4 sin x - sin 2x at x =
n
2"
e t
(Ans:
515 ..
-I
4

3J2a
[Ans: - - ]
16

Downloaded From : www.EasyEngineering.net


Downloaded From : www.EasyEngineering.net

Curvature and Curve Tracing 235

5. E +JY =J; at %,~


a
lADs: J21

6. x = a (0 - sin 0), y = a (I - cos 0) at 0 = 1t

ww 7. x = a sin 2 t (I + cos 2t)


lADs: - 4 al

w
8.
Y = a cos 2 t (I - cos 2 t) at any point 't'

.Ea
x = a (cos 0 + 0 sin 0)
lADs: 4 a cos 3 tl

y = a (sin 0 - 0 sin 0)

9. ? = a 2 cos20 syE lADs: a 01

ngi
nee [ADS: );]
a2

10. r = ~ (\ + cosO) at (~,~) r ing


.ne J2

t
[ADS·. -3a ]

an
lADs: {n + 1)rn-1 1

4.2 Centre of Curvature, Circle of Curvature and Evolute


1. Definition: The centre of curvature at a point 'P' of a curves is the point 'C'
which lies on the positive direction of the normal at 'P' and is at a
distance 'p' (in magnitude) from it.
2. Definition: The circle of curvature at a point' P' of a curve is the circle whose
centre is at the centre of curvature 'C' and whose radius is 'p' in
magnitude.

Downloaded From : www.EasyEngineering.net


Downloaded From : www.EasyEngineering.net

236 Engineering Mathematics - I

x' o
y'

ww
w.E Fig. 4.6

4.2.1 Centre of Curvature


asy
Let P (x, y) be any point on the curve and C (X, Y) be the centre of curvature at

En
'P' which is on the normal PC at P. Then PC =: p = radius of curvature at 'P'. The
tangent P T at P to the curve makes an angle \jJ with X - axis. P B, C A are

gin
perpendiculars to X - axis, and P R is perpendicular to C A.

eer
L.PTX=L.RPT=L.PCR=\jJ
I
cos \jJ = - -
sec \jJ
ing
I .ne
(I + tan 2 \jJ2 )2"
t
..... ( I)

dy
dx ..... (2)

Downloaded From : www.EasyEngineering.net


Downloaded From : www.EasyEngineering.net

Curvature and Curve Tracing 237

and we have ..... (3)

x = 0 A = 0 B - A B = OB - PR
(i.e.,) x = x - P C sin \jJ (From ~ P R C, PR = PC sin \jJ)
X = x - p sin \jl

ww From (2) and (3)

w.E
X=
asy
En
g
d~[1 +(tfl)2] in
dx dx eer
X=x-
d y 2
ing
dx 2
Y=CA=RA+RC=Y+PCcoo\jJ .ne
From (I) and (3) t

Downloaded From : www.EasyEngineering.net


Downloaded From : www.EasyEngineering.net

238 Engineering Mathematics - ,

ww x' o
y'
x

w .Ea
syE
.. The coordinates of the centre of curvature of a curve

ngi
at p (x, y) is C
nee
rin
_ dy _ d Y .
2
[ _ Yl (I + Yl )
Denoting Yl - d 'Y2 - - - 2 we can WrIte C x ,Y +
2
1+ Yl ] ,g.n 2

x dx Y2 Y2

C (X, Y) is the centre and 'p' is radius ofthe circle of curvature


e t
.. The equation of the circle of curvature is
(x - xi + (y - Yi = p2
4.2.2 Example
Find the centre of curvature of the curve
Y = x 3 - 6x2 + 3x + 1 at (1, -1)
Solution
dy
- = 3x2 - 12x + 3
dx

Downloaded From : www.EasyEngineering.net


Downloaded From : www.EasyEngineering.net

Curvature and Curve Tracing 239

dy _
--6
dX(I,I)

=-6
dX2 (1,_1)

yn
ww X= X -
YI (I +
Y
2
I+ YI2
and Y = Y + - -
Y2

w .Ea X= I-
(-6XI+36)
-6
1+36
and Y = I + - -
-6

syE -43
X = - 36 and Y = -
6

ngi -43)
The coordinates of the centre of curvature are ( - 36, -6-

4.2.3 Example nee


Find the centre of curvature for the cycloid
ace - sine),
r ing
Solution
x = Y = a(1 - cos e)
.ne
dy = a sin e
dx
_ cot e/
a(l-cose) - 12
t
d 2y 2 e 1 de 1 1
-=cosec -.--=- x---,------,-
dx 2 2 dx 2· 2 e
Sin -
a(l - cos e)
2

. 4
4aSIn
e
--
2

Downloaded From : www.EasyEngineering.net


Downloaded From : www.EasyEngineering.net

240 Engineering Mathematics - I

• 4
4a SIO
e
-
2

. 4e e
4 aslO -cot--
X= ace - sine) + _ _-'2"'--------=-2
. e
ww SIO-
2

w.E X= ace - sine) + 2a sin e


X = ace - sine)

asy
Y=y+--
1+ YI
2

En
Y2

gin (l+cot2E!)
Y=a(l-cose)- 12

e eer
. 4
- 4 a SIO -
2 ing
Y = a(l- cose)- 4a sin 2 ~ .ne
2
Y = a(l - cose) - 2a(1 - cose)
t
Y = -a(l - cose)
.. The coordinates of the centre of curvature are
[ace + sine), - a(1 - cose)]

4.2.4 Example
Find the centre ofthe curvature at any point (x, y) on the eIlipse.
Solution

..... (1 )

Downloaded From : www.EasyEngineering.net


Downloaded From : www.EasyEngineering.net

Curvature and Curve Tracing 241

Diff. (I) w.r. to 'x'

2b 2x + 2Q2y dy = 0
dx

..... (2)

ww
w .Ea
..... (3)

Using (I) (2) and (3)


syE
X= x _ YI (1 + yO
Y2
ngi
nee
rin
g.n
e t
[~: (a' - h'), -;.' (a' - h' ij is the centre of curvalure,

4.2.5 Example

Find the circle of curvature of the curve fx + fY = fa at the point (~, ~)


Solution

..... (1)

Downloaded From : www.EasyEngineering.net


Downloaded From : www.EasyEngineering.net

242 Engineering Mathematics - I

Diff (1) w.r. to 'x'


1 -1 -I 1 --I
1
dy
-x 2 +_y2 -=0
2 2 dx
dy JY ..... (2)
dx--.r;

and
:10/ ai
14' /4
= ~1 ..... (3)

ww d 2y [
, I ~-I ----..jy-x
..;x-y
2
dy
dx
C 1 ~-I]
2

w .Ea
dx 2

d 2y
=

4
X

syE
..... (4)
dx 0/ (// a
/4' )14

p = [1 + y~ ¥= [1 + 1]% = ~ngi
i ..... (5)
Y2
a
.fi
nee
a a
is the radius curvature of the curve at ("4'"4 ) rin
X= x _ YI (1 + YI2 ) g.n
a 1(1+1)
X=-+--=-
3a
Y2
e t
4 4 4
a

(1 + y2)
Y=y+ 1
Y2
a (I + 1) 3a
Y="4 + -4- = 4
a
3a 3a)
Coordinates of the centre of curvature ( 4' 4

Downloaded From : www.EasyEngineering.net


Downloaded From : www.EasyEngineering.net

Curvature and Curve Tracing 243

Equation of the circle of curvature of the curve (x - X)2 + (v _ y)2 = p2

3a)2 ( 3a)2 _
( X -4- + y -4- -
(~)2
Ji

Exercise - 48

I. Find the coordinates of the centre of curvature of the curve y = x J at the point

ww 7 31

w.E 2
(3,
IAns: - - )
64' 38

(3, I;)
asy
2. Show that the centre of curvature for the curve y = x x+ 9 at 6) is

En 1
3. Show that the centre of curvature at the point 't' on the ellipse x = a cos t,

y=bSllltlS
. .
[( a2 , )cos-I,- ()
a - b- 1
b
gin
a- - b 2) .
Slll-'
1

3;1, 3;1) eer I


4. Find the centre of curvature at ( of the folium x 3 +
ing
= 3a xy

21(1 21a
IAns: - - --I
16 ' 16 .ne
5. Find the centre of curvature at the point '0' of the curve
x = (I - aO) cos 0 + a sin 0
t
y = (I - aO) sin 0 + a cos 0
IAns : a sin 0, - a cos 01
6. Find the coordinates of the centre of curvature of the cycloid x = a (0 + sinO),
y = a( I - cos 0)
(Ans: a (0 -sin 0), a (I + cos 0)
2
7. For the curve y = 111X + ~ show that the circle of curvature at the origin is
a
x2 +I = a 2 (I + 1112) (v - I1lx).

Downloaded From : www.EasyEngineering.net


Downloaded From : www.EasyEngineering.net

244 Engineering Mathematics - I

4.3
4.3.1 Evolutes
Definition .' Corresponding to each point on a curve we can find the curvature of
the curve at that point. Drawing the normal at these points we can find
centre of curvature corresponding to each of these points. Since the
curvature varies from point to point, centres of curvature also differ.
The totality of all such centres of curvature of a given curve will define
another curve and this curve is called the evolute of the curve.
Thus the locus of centres of curvature of a given curve is called the
evolute of that curve.

ww Notes:

w.E (i) Elimination of x, y from the coordinates of the centre of curvature (X, Y)
gives an equation in terms of X, Y which is called evo/ute of the given
curve y = fix).
(ii)
asy
When the equation of the curve is given in the parametric form (say t)
elimination of '1' from the coordinates of the centre of curvature (X, Y)

En
gives an equation in terms of X, Y which is called evo/ute of the curve
x = fit), y = get).

4.3.2 Example gin


Find the evolute of the parabolay = 4 a x
eer
Solution
Differentiating y = 4ax w.r. to 'x' ing
2ydy = 4a .ne
dy
dx
dx

=
Fa
Fx
t
..... (1)

d y
2
-Fa
dx2 =-3- ..... (2)
2x2

X= X _ YI (1 + yl2 )
Y2

X=x
-~(1
Fx
+~)
x
-Fa
3
2x 2

Downloaded From : www.EasyEngineering.net


Downloaded From : www.EasyEngineering.net

Curvature and Curve Tracing 245

X=3x+2a ..... (3)

1 + y2
I + Cl )
X
Y = Y+ _ _I = Y+ ~----=c'-
y)
I-Fa
,
2x 2

ww ..... (4\

w Squaring

.Ea y2 =
4
-.x3
a

syX-2aE
From (3) substituting x = --3- in (5)

y2= _
n
4(X-2a)' gin
a
27 ay2 =
3

4(X - 2 a)3
e eri
27 a r = ng.
4(x - 2 0)3 is the evolute of the curve.

4.3.3 Example n et
Solution

..... ( I)

Differentiating (I) with respect to 'x'


dy
2b 2x + 2a2y - = 0
dx
2
dy = -b x
..... (2)
dx 02y

Downloaded From : www.EasyEngineering.net


Downloaded From : www.EasyEngineering.net

246 Engineering Mathematics - I

_b 2 a 2b2

ww = a 2 y2· a 2;.

w .Ea
d 2y
dx2 =
_b 4
a 2y 3 ..... (3)

X =
sy(I E
x _ YI + yl2 )
Y2

n gin
e eri
x ng.
X

Substituting c?y2 =
= x - -4 -
a b2
(a4y2

a 2b2 - b2x 2 from( I)


+ b4;x2)
n et
..... (4)

b4 x 2
1+4""2
ay
=y+
_b 4
a
2
i
Downloaded From : www.EasyEngineering.net
Downloaded From : www.EasyEngineering.net

Curvature and Curve Tracing 247

Y = y - ~ (0 4; + b4x 2 )
a 2b 4
Substituting b2x 2 = 02b 2 - a 2; from (I)

Y = y -+ a b4
[a 4; + b
2 2
a (a 2 -;)]

a 2 _b 2
Y = y3 ..... (5)
b4
Eliminating x, y from (4) (5)

ww
w.E
asy
En
gin
e
(aX)32+ (bY)}2= (a 2 - b 2 )2:I is the evolute of the given curve.
eri
4.3.4 Example
ng.
Show that the evolute of the curve

2
X3 +y3 +a
2
3
2 2
= is(X+Y)3 +(x-Y)l =2a 3
2 ~ net
Solution
The parametric equation of the curve is x = acos 3S, y = a sin 3S
dy
dy = de 3 a sin 2 ScosS
dx dx - 3a cos 2Ssin S
de
dy
- =-tan S
dx
d2y de
- 2 =-sec 2 S -
dx dx

Downloaded From : www.EasyEngineering.net


Downloaded From : www.EasyEngineering.net

248 Engineering Mathematics - I

ww
w .Ea
30 cos 4 esin e

..... ( I)

syE
ngi
(I + tan 2
e)
1
nee
rin ..... (2)
From (1) and (2)
g.n
(X + Y) = a(cose + sine)3 and X - Y = a(cose - sine)3
')

{X+yt3 +{X-Y)"'3
I '),

= 2a 3
~ /
e t
J J 2
(x + y)~ + (x - y)~ == 2a:1 is the evolute of the given curve.

4.3.5 Example
Show that the evolute of the curve
x = a(cose + sine), y = a(sin e - e cose) is
x 2 +y=a2
Solution
x = a(cose + sine) and y = a(sin e - e cose)

Downloaded From : www.EasyEngineering.net


Downloaded From : www.EasyEngineering.net

Curvature and Curve Tracing 249

Differentiating w.r.t 0

dx .. dy .
-;-0 = a(- Sll10 + Sll10 + OcosO), -'- = {{(cosO - cosO + 0 SIIIO)
a £10
dx dy.
dO = aO cosO, dO = aO SII10

dy aOsinO
dx (JOcosO
ely

ww -, =
(X
tan 0 ..... ( I)

w .Ea d 2y
dx 2 -
syE
(l0 cos'O

y{l+y2) . tan{l+tan 2 0)
..... (2)

X = x- . 1
Y2
. 1

ngi
= a(cosO + 0 S1l10) _ ----"-------:-----"--
I
aOcos 1 e
X = a cos 0
nee(I ..... (3)

y =
I+ y2
y + --'_I = a(sin 0 - 0 cosO) +
rin
+ tan 2 0 )
I
Y2
aOcos '0
g.n
Y = (I

Eliminating '0' from (3) & (4)


X2 + y2
sin 0

= (/2
e t
:. x 2 + 1= (12 is the evolute of the given curve.

4.3.6 Example

Show that the evolute of the curve x a( cos t + log tan ~) ,y asin
= = t is

x
y = a cos h
a
Solution

x = aCcost + log tan '2t ) y = 1I sin t

Downloaded From : www.EasyEngineering.net


Downloaded From : www.EasyEngineering.net

250 Engineering Mathematics - I

dy t
1
-dx =a -smt+--.sec
t
r. t 1 -dy =acost
2 -.-
2 2 ' dt
j
an-
2

-dx = a -smt+
dt r·
. t1cos-t
2 sm--
=a -smt+-.-
IJ
sm t
2 2
j [.
ww dx
dt
acos 2 t
.
SIn t
dy
and -d
t
= a cost

w .Ea dy
dy
.J!L acost
= tan t
dx
syE
dx
--
dt
acos 2 t
sin t

n gin
e eri
n"2 g.
X = a( cos t + log tan
t
"2) - a cos t = a log tan
t
n et
..... ( 1)

1+ yf
Y2

1+ tan 2 t
y = a sin t+
sin t

a
y=- ..... (2)
sin t
t
From (1) tan - = eX/a
2
. a
From (2) SIn t= Y

Downloaded From : www.EasyEngineering.net


Downloaded From : www.EasyEngineering.net

Curvature and Curve Tracing 251

2
2 tan t 2 a
(i.e.,)
1+ tan 2 t . y
2
x
2e a ([

2X
I+e a Y

y = ~( / a + e - X,,) = (( cosh (%)


(%) is the evolllte of the given curve.
ww y = a cos II

w.E Exercise - 4(C)

a
I. Find the evolllte of the curve x
syE = c t and y
c
= -
t

2. Prove that the equation to the evolute of the parabola x 2 ngi = 4ay is 4(y - 2(/)3 =
27 a x 2
nee
3. Show that the evolute of the hyperbola
x2
rin I is (ax)2/3 - (by)213 =

(a 2 + b2 )2/3
g.n
4. Show that the evolute of the cycloid x
equal cycloid.
= a (q - sin q) y =
et
a (I - cos q) is another

4.4
4.4.1 Envelopes
Family of curves:
Let us consider the equation of a straight line x cosa + y sina = p. Where a is
a parameter. For different values of a, x cosa + y sina = p gives different straight
lines but all of them are at a constant distance 'p' from the origin. The set of all of
these straight lines is said to form a family of straight lines and 'a' is called the
parameter of the family. For a given p, different values of'a' give different straight
lines which touch the circle x2 + .Y = p2. The circle x2 + 1 = p2 is called the
envelope of the family of straight lines.

Downloaded From : www.EasyEngineering.net


Downloaded From : www.EasyEngineering.net

252 Engineering Mathematics - I

A curve, which touches each member ofa given family of curves and each point
is the point of contact of some member of the family is called the envelope of the
family of curves.
Letj(x, y, a) = 0 be the given family of curves and a is the parameter.

Now d ·f'" . h respect to "


. . WIt
I Jerentlatmg . II y
a partla a/(x,y,a) =
0
aa

EI· · · 0 f' a' f romJ'"(x, y, a ) = 0 an daf(x,y,a) = o·


Imlllatlon . .III
gIven an equatIOn
aa
terms of x, y which is called the envelope 0/ fhe family of curves.
ww If the given equation of the family ofcurvesj(x,y, a) = 0 is the quadratic in 'a',
say of the form Aa2 + Ba + C = 0 where 'a' is the parameter and A, B, Care

4.4.2
w.E
functions of x, y. Then the envelope of the family of curves is B2 - 4AC = o.

Example

a syE
Find the envelope of the family of straight lines y = mx + ~
m
where 'm' is the

parameter.
ngi
Solution

y=mx+ -
a nee ..... ( I)
m
rin
III is the parameter
Diff(l) w.r. to 'm' g.n
o=x--2
a
m
et
111= ~ ..... (2)

Substituting for' m' in (I)

y = 2.[;;;
Squaring we gety = 4 ax is the envelope of the family of curves

Downloaded From : www.EasyEngineering.net


Downloaded From : www.EasyEngineering.net

Curvature and Curve Tracing 253

a
Aliter: y = mx + -
III
m2x - 111 Y +a= 0
is quadratic in parameter '/11' (A = x, B = - y, C = a)
8 2 - 4 AC = 0 then gives the required envelop.
.. Envelope of the family of curves is i - 4(a)(x) = 0
i = 4 ax is envelope.

4.4.3 Example

ww Find the envelope of the family of curves

w.E
Solution
I J J J
y=mx+ va-nr+b- w here'm'ist heparameter.

asy
y=mx+ vlrm-+b-
I J J J

En
(y - I1lx)2 = ([2m 2 + b 2

m 2(x 2 - a 2) + m(- 2 y x) + (V
is quadratic in parameter '111' gin - b2) = 0

The envelope of the family of curves


e eri
(-2 x y)2 - 4(x 2 - a 2)(1- b2) = 0
i
b 2x 2 + a 2 = a 2b 2 ng.
2 2
.;. + -=;- = I is the envelope of the family of curves.
a b
net
4.4.4 Example
Find the envelope of the family of curves x cos 38 + y sin 3 8 = c when 8 is the
parameter.
Solution
Given x cos 3 8 + y sin 3 8 = c (8 is parameter) ..... ( I)
Diff. (I) partially w.r. to '8'
- 3 x cos 2 8 sin 8 + 3y sin 2 8 cos 8 = 0

Hence tan8= y
x

Downloaded From : www.EasyEngineering.net


Downloaded From : www.EasyEngineering.net

254 Engineering Mathematics - I

Substituting these values in (])

3 [ 13
X
[ ~ X2 Y+ y2 ] +y x
~ X2 + y2
j=c
x 21 = C(X2 + I) is the envelope of the family curves.
4.4.5 Example

ww Find the envelope of the ellipse -;- +


2 2
4 = I where

w .Ea
a b
a, b are parameters connected by the relation a2 + b 2
Solution
= c2

Given syE x2
-2+ -
y2
=] ..... (1 )

ngi
2
a b
a2 +b2 =c2 ..... (2)
Assume that' a' and' b' are functions of' t'
nee
Diff. (]) and (2) w.r. to '1'
2 (- 2) da + I (- 32)db = rin
0
x a3 dt b dt
g.n
Diff.
a dt
a2 + b2 = c2 w.r.
x 2 da
-3 - + -3 - =0
b dt
to '1'
y2 db
e t
..... (3)

da db
2a- +2b- =0
dt dt
da db
(i.e.,) a- + b- = 0 ..... (4)
dt dt

Eliminating da, db from(3),(4)


dt dt

Downloaded From : www.EasyEngineering.net


Downloaded From : www.EasyEngineering.net

Curvature and Curve Tracing 255

Using (I) and (2) we get


2
x /

£=~=-2
a b c

(i.e.,) [/4 c2

(14 =x2c2
(12= XC

ww /
-=-

w .Ea
Substituting (12, h2 , values in a 2 + h2 = ("2

xl + J,z = I syE
x 2c 2 + Ic 2 = c 2
is the envelope of the family of curves.

4.4.6 Example
ngi
Find the envelope of the family of curves nee
~ + Eh = I where (I, b are parameters

r
{/

connected by the relation ah


Solution
= ('2.
ing
Given ~+E = I .ne ..... ( I)
a b
ab = ('2 ..... (2)t
Assume that a, b are functions of 't'
Diff. (1) and (2) w.r. to 'I'

- I-) +
x( - da y.-( -I -) =
db O
a 2 dt h 2 dt
x da y db
- - - + 2- - =0 ..... (3)
(/2 £II h dl
Diff. ab = ('2 w.r. to 'I'

db da
a-+b-=O ..... (4)
dt dt

Downloaded From : www.EasyEngineering.net


Downloaded From : www.EasyEngineering.net

256 Engineering Mathematics - I

da db
Eliminating dt ' dt from (3), (4)

x y x y x y
2 -+-2 - -
!L.=lL~-.!L=JL=~
b a ab ab 2ab
Using (I) and (2) we get
x y
£=~=-2
ww b

x
a 2c

w.E a = 2c
ab
2

a syE
Using (2)
ngi
X
~= 2c 2
c 2

nee ..... (5)

a=2x
rin
g.n
a

y ab
et
-=--
b 2c 2
Using (2)

b=2y ..... (6)


Substituting a, b values from (5), (6) in (2)
(2 x)(2 y) = c2
4 xy = c2 is the envelope of the family of curves.

Downloaded From : www.EasyEngineering.net


Downloaded From : www.EasyEngineering.net

Curvature and Curve Tracing 257

Exercise - 4 (0)

1. Find the envelope of the family of curves:

(a) y = mx + a ~I + m 2 when 111 is parameter.

(b) ; = 2a(x - (I) where a is the parameter.


2; =
ww (c) x cos <p + y sin <p = p where <p is the parameter.
IAns : x 2 - 01

~ +~
w.E
(d)
a b
a
= I when a + b = e

syE
(a, b parameters).

(Ans : fx + .JY = -!c ]


~ + ~ = 1 when a2 + b2 = e2 ngi
(e)
a b
nee
(a, b parameters).

fx .JY = -!cl
ff H rin
IAns: +

(f) + = I when a + b = e (a, b parameters).


g.n
(Ans : x 3 + y3
2 2 et
=c 3 ]
2

In m
(g) Find the envelope of the family of curves ~ + L = I when dnb n = elll + n
ani bill

(h) Find the envelope of the family of straight lines ~+~ = 1 where a & bare

related by the equation d l + bn = en, e being a constant.


n II /J

(Ans : xn+1 + yll+1 = C"+ I ]

Downloaded From : www.EasyEngineering.net


Downloaded From : www.EasyEngineering.net

258 Engineering Mathematics - I

4.5 Curve Tracing


Generally a curve is drawn by plotting a number of points and joining them by a
smooth line.

If an approximate shape of the curve is sufficient for a given purpose then it is


enough to study certain important characteristics. This purpose is served by curve
tracing methods.

ww The points to be observed for tracing of plane algebraic curves are given below.

1. Whether the curve is passing through the origin, if so the equations of the

w.E tangents to the curve at the origin.

°
°a=>s
Suppose F(x,y) = is the algebraic form of the equation of the curve.

F(O,O) =
yE
the curve is passing through origin (i.e.,) If there
constant term in F(x,y) then origin lies on the curve.
IS no

n gin
The equations of the tangents to the curve are obtained by equating the
lowest degree terms in F(x,y) to zero.
e
If at 0(0,0) the tangents are:
eri
(i) real and coincident then '0' is called a cusp
ng.
(ii) real and different then '0' is called a node

(iii)imaginary then '0' is called a conjugate point


net
2. Whether the curve is symmetric about an axis or about other any line. If

(i) F(x, y) = F(x-y) => curve is symmetric about x-axis


(ii) F(-x, y) =F(x, y) => curve is symmetric about y-axis
(iii) F(-x,-y) =F(x, y) => curve is symmetric in opposite quadrants.
(iv) F(y, x) = F(x, y) curve is symmetric about y = x

(v) F(-y,-x) =F(x, y) curve is symmetric about y =-x

Downloaded From : www.EasyEngineering.net


Downloaded From : www.EasyEngineering.net

Curvature and Curve Tracing 259

y
y

----+-----------~~x
--------~~~----------.. x
y' =4ax

ww y
(i) (ii)

w .Ea
y

syE
----~~~----------------~x
ngi ------~~~----~x

nee x
3
+y' ~~V

(iii) y r (iv)
ing
.ne
-----..;:::.~"'-------~x
t
(v)

3. Weather the curve intersects the axes, if so the tangents to the curve at these points.
4. Find the region in which the curve exists (i.e.) the curve is defined. The values of x
for which y is defined gives the extent parallel to x-axis and the values of y for
which x is defined gives the extent in a direction parallel to y-axis.
If y is imaginary for values of x in a certain region then the curve does not exist in
that region.

Downloaded From : www.EasyEngineering.net


Downloaded From : www.EasyEngineering.net

260 Engineering Mathematics - I

Similarly with respect 10 values ofy .


5. Finding the asymptotes.
An asymptote is a line that is at a finite distance from (0, 0) and is tangential to the
curve at infinity (i.e.) the curve approaches the line at infinity.
(i) Sum of the coefficients of the highest degree terms in x equated to zero gives
the equations of the asymptotes parallel to x - axis.
(ii) Sum of the coefficients of the highest degree terms in y equated to zero gives
the equations of the asymptotes parallel to y-axis.

ww (iii) To find the asymptotes that are neither parallel to x-axis nor parallel to y-

w.E axis (i.e.) oblique asymptotes, the following method is suggested.


Substitute y =mx + c 111 F(x,y) = 0 and rewrite the equation as a polynomial

a
equation in 'x'as

$
n
(m)x
n +$
syE
n-
l(m)x
n-I + .... +$ =0
n

ngi
The slopes of the asymptotes are given by $I/(m) = O. Let the slopes be

nee = 0,$1/_2(m) = 0
The values of'c' can be obtained trom $I/-I(m)
rin (if necessary).

Let the corresponding values of c be c1' c2 , •••••


g.n
et
Note: if $1/ (m) is a constant then there are no oblique asymptotes to the curve.

6. Obtain dy from F(x,y) = 0 by differentiation.


dx

If in the interval for x, dy > 0 then the curve is increasing


dx

and : < 0, then the curve is decreasing in that region.

.
7. TIle po liltS 0f ·
maxima an dminima
·· .
are given by -dy = 0.
dx

Downloaded From : www.EasyEngineering.net


Downloaded From : www.EasyEngineering.net

Curvature and Curve Tracing 261

~o+-------------------~X

o.gIves
ww -
dy
dx
= XI,X)
-

(i)
w~ .E
If in an interval for x

el
2

a
> 0 then the curve is concave upwards (X), X 4 ' .... in the figure) and has
~- -

a minima in that interval.


syE
~
ngi
2
(ii) el < 0 the curve is concave downward (XI' xw .... in the figure) and has a
llx- -
maxima in that interval.

~
2 nee
(iii) If at a point X)
-
(say) in the interval d
dx-
=
rin
0 then that point is called a point

of inflection and at that point the curve changes it concavity to the opposite
direction. g.n
4.5.1 Example
X2 +2x
et
Trace the curve y = .... ( 1)
x+ 1
Solution:
Substitution of X =0 in (1) gives y = 0:. origin lies on the curve.
F(x,-y):f:- F(x,-y):f:- F( -x, y) etc :. The curve is not symmetric about any line

can be written as y (x + 1) - x
2
1. - 2x = 0 .... (2)
2
(ie) x - xy - (y - 2x) = 0 Lowest degree term is y - 2x = 0

Equation of the tangent at (0,0) is Y = 2x .


Substitutingy = 0 in (1) we have x(x+ 2) = o,x = O,x =-2

Downloaded From : www.EasyEngineering.net


Downloaded From : www.EasyEngineering.net

262 Engineering Mathematics - I

:. The curve crosses x -axis at (0,0) and (-2,0)


Since x =0 in (I) gives y = 0 only, there is no intercept on y-axis
The curve is not defined x at -I. Hence it IS discontinuous.
:. The curve is defined in the region R - {-I}
Coefficient of highest degree term in x «i.e.) of Xl) IS I which is a constant.
Hence there is no asymptote parallel to x -axis.
Coefficient of highest degree term in y is x + I. :. x + 1=0 is the asymptote parallel

ww to y- axis.
dy
= -1
w.E
From (2)
dx
Hence the curve is increasing in the regions.
which is > 0 always except at x

00
asy
< x < -2, -2 < x < -1, -1 < x < 0 and 0 < x < 00

En
\
\
\~

gin \..-
\j.:,
\

eer \
\
\

ing
\
I
\ /
\ /

1 2
.ne
t
Fig. 4.7
4.5.2 Example
2
x +1
Trace the curve y-
- Xl -1
Solution:
....... ( I)

Downloaded From : www.EasyEngineering.net


Downloaded From : www.EasyEngineering.net

Curvature and Curve Tracing 263

x= ° in (I) gives y
y intercept is -I.
= -I :. Hence curve does not pass through (0,0), and

r (-x,y) = r (x,y) curve is symmetric about y-axis

y= ° gives x
2
°
+ 1= => x = ± i curve does not cross x -axis.
Coefficient of highest degree term in x is y-I (from( I»
llence y = I is the asymptote parallel to x -axis

Coefticient of highest degree term in y is x2 -] x = ±] are asymptotes parallel

ww to y-axis

dy = Hence y
I
=0 .
gives x =0 II
-34 < 0 and =I:- 0
w.E
and Yx=o =
dx

asy
:. The maximum point on the curve is (0,-1) and there are no points of inflection.
y ~

~ gl~Y.,
En I'J
Ix
I
I
I

ine
--------1---_
I
I
----r---------
I

eri I
I
I
I
I

ng. I
I
I 0 I
I X

n
I
I
__ ..JI ________ _

et
I (0,-1) Y=-1
--------1--- I
I I
I I
I I
I I
I I
I

Fig. 4.8

The curve is decreasing in the interval ° < Ixl < ] and I< x < 00 as dy is -ve there
dx
an d ·· ..
IS IIlcreaslllg 111 -00 < X < 1 as -dy.IS +ve III
. t I"
l1S mterva
i
dx
4.5.3 Example:
3
Trace the Folium x + / == 3axy .... ( I)

Downloaded From : www.EasyEngineering.net


Downloaded From : www.EasyEngineering.net

264 Engineering Mathematics - I

Solution:
Clearly (0,0) lies on the curve .No intercepts on y-axis.

F (y,x)=/+x'-3ayx=0=F (x,y) :. curve is symmetric about x-axis

Coefficient of highest degree term in y is I. a constant, and hence no asymptote


parallel to y-axis. Lowest degree term equated to zero gives xy = 0, x = 0, y = 0
(i.e) the axes are the tangents to the curve at the origin.

y = x in (I) gives 2x' - 3ax 2 = 0 x = 0, 3;1 (ie) the line y = x meets the curve

ww at (O,O)and (3~, 3~)

wF (-x, - y) = x' - /
.Ea + 3axy = F (x, y) :. curve is not symmetric about (0,0)
To obtain oblique asymptotes: Substitute y = mx + c in (I) and rewriting in terms
3
syE 2 2
of 'x' we get (I+m )x'+(3m c-3am)x + ..... =0 (ie) ¢3(m)=I+m'and

¢2 (111) = 3m 2c - 3am, .....


ngi
¢, (m)
nee
= 0 gives m= -1, there is only one oblique asymptote with slope-I

r ing
, , .ne
,

,
x+y=3a
VI
t

Fig. 4.9
¢2(m)=0 gives 3m(cm-a)=0 => cm-(l=o (e) c=-a(-:m=-l)

:.Theasymptoteis y=-x-a or y+x+a=O .... (2)

Downloaded From : www.EasyEngineering.net


Downloaded From : www.EasyEngineering.net

Curvature and Curve Tracing 265


--------------------~----------------------------------------~

2
dy=
From (I) - ay-x
~ - )
:. (dY =-1
dx y -ax dx e~/,32")

Equation of the tangent to the curve at this point is y + x = 3£1 which is parallel to (2)
4.5.4 Example:

Trace the curve a 2y 2- x 2( a 2- x 2) = 0 ........ ( I )

Solution:

ww x =0 in (I) gives y = 0,0 (ie) double point on the curve

(-x, y) (x, -y) = F (x, y) curve is symmetric about both the axes.
w.E
F = F

Lowest degree terms, in (I), equated to zero gives a


2
(/ - x
2
) = 0 ~ y = ±x
are the tangents
as°y = ±a
to the curve at (0.0). y =

En in (I) gives x

(a, 0) (-(1,0)
gin
The curve intersects the x-axis at and From (I) it can be seen that

for Ixl > a


l is negative (ie) the curve does not exist for eer
Ixl > a
ing
° .ne
YI = gives

a
x = 0, ± .i2' points of maxima and III inima.
t
Tangents at the points x = ± J2 to the curve are parallel to x-axis .Further

ddx2y ) at x a .
= J2 .
IS negative. :. x
a .IS a maxImum
= J2 . . d" .
POlllt an Y IS IIlcreaslIlg
( 2

in 0< x <.i2 while decreasing in .i2 < x < a .Thus after tracing the curve in

the first quadrant, the symmetry about both the areas can be utilized for tracing the
complete curve.

Substituting y = mx + c in (I )gives X4 + a 2 (mx + c r -a 2 2


x =0

Downloaded From : www.EasyEngineering.net


Downloaded From : www.EasyEngineering.net

266 Engineering Mathematics - I

¢4 (m) = 1 which is a constant. Hence there are no oblique asymptotes to the curve.
y

~(_-a~,O~)~--------~----------+-~~X

ww ,
w.E
4.5.5 Example: asy Fig. 4.10

En
Trace the curve y2 ( x 2 - a 2) + a 2x 2 = 0 ..... (1)

Solution: gin
eer
x = Oin (1) gives y = 0 :. (0,0) lies on the curve.
F (x,-y) =F (x,y) =F (x,-y) :. curve is symmetric about both the areas.
ing
Highest degree terms of y equated to zero gives (X2 - a 2) l
.ne
= 0 => x = ±a
:. x = ±a are asymptotes parallel to y-axis. Highest degree terms of x equated to
zero gives x 2(y2 + a 2) = 0 => x = ±ia ... No asymptotes parallel to x-axis. t
Equating lowest degree terms to zero we get a 2(y2 - x 2) = 0 => y ± x are tangents at
(0, 0) writing y = mx + c in (1) and rearranging in terms of I x I

m2x 4+2mcx3+(c 2 _a 2m2+a 2)x2 -2a 2mcx-c 2a 2 =0

=> c is indeterminate since m = 0


¢2 (m ) = 0 gives c 2+ a 2 = 0 => c = ±ai
oblique asymptotes do not exist. Further there are no intercepts on either of the axes.

Downloaded From : www.EasyEngineering.net


Downloaded From : www.EasyEngineering.net

Curvature and Curve Tracing 267

2
a 2x
( 1) can be written as l = -2 - -
x _a 2
which indicates that curve does not exist

when Ixl > a :. The curve exists only in the region -a < x < a.

ww x
w.E
asy
En
gin
eer Fig. 4.11
dy a4 x
dn
=
y ing
and y> 0 in first quadrant. Hence curve is increasing in 0 < x < a .

4.5.6 Example: .ne


Trace the curve l (2 - x) - x2 (1 + x) = 0 ....... ( 1) t
Solution:
x=Oin (1) gives y=O :.(0,0) lies on the curve F(x,-y) = F(x,y) :.curve is
symmetric about x-axis . Lowest degree terms equated to zero gives

2l- x 2 = 0 => y = ± 12 tangents at (0,0) to the curve y = 0 in (\) gives -1,0

curve passes through (-1,0) in addition to (0,0)


Coefficient of highest degree terms of y equated to zero gives 2 - x = 0
(ie) x = 2 is an asymptote parallel to y-axis
2
No asymptote parallel to x-axis as coefficient of x is 1,a constant

Downloaded From : www.EasyEngineering.net


Downloaded From : www.EasyEngineering.net

268 Engineering Mathematics - I

Substituting Y = mx + c in (I) and rearranging


3 2 2 2 2 2
x ( m + 1) + x ( 1+ 2mc - 2m ) + ( c - 4mc ) x - 2c =0
¢3 (m) = m 2 + 1 => ¢3 (m) = 0 gives m = ±i :. There are no oblique asymptotes.

ww --~----~~--------~----~x

w.E ...
~..., y=-
-x

'" J2 I
I (2,0)
I

a syE
"'",. I
I
I
I
I

Fig. 4.12ngi
From (I) Y1 = y2 + (2x + 3x
2
h' h h nee
O· 0 < X < 2 were
) . T IS sows t at YI > 111
h t he curve
2y 2-x
is increasing and x = 2 is an asymptote. rin
5±J5j g.n
YI = 0 => x = 0, x =

..
In the secon d quadrant y IS maxImum at x
4
.

5-£1
=- --
et
where the tangent IS
4
parallel to x-axis

It can be seen that the curve forms a loop in [-1,0] due to symmetry about x-axis.

4.5.7 Example
Trace the curve / -(x-2)(x-4)2 =0 .. , .. (1)

Solution:
y = O. (I) gives x = 2, 4
Curve intersects the x-axis at (2,0) and (4,0)

Downloaded From : www.EasyEngineering.net


Downloaded From : www.EasyEngineering.net

Curvature and Curve Tracing 269

F(x, -y) = F (x,y) :. curve is symmetric about x-axis


For x < 2 , / is -ve :. therefore curve does not exist for x < 2
From (I) it can be seen that x = 2 is a tangent at (2,0)

Coefficient of x 3 = l,a constant :. No asymptote oarallel to x-axis


Differentiating (1) w.r.t 'x'
dy _ (x-4)(3x-8)
dx 2y

ww
At (4.0) dy
dx
= Lt ± (x-4)(3x-8)
H4 2.J x - 2 ( x - 4)
Lt± 3x-8 = ±J2
H4 2.Jx-2

w.E Equation of tangents at (4,0) are y = ±J2(x - 4)

asy
Further dy --}- 00 as x --}- 2 :. x = 2is a tangent at (2,0)
dx
y
En
gin
e eri
--~--~----~~~~~x
(2,0)
,, ng.
net
Fig. 4.13

dy =
dx
° gives x = 8/3. Thus the tangents at x = 8/3 are parallel to x-axis. For the

region.

2 < x < 8/3, dy >


dx
° above x-axis and dy <
dx
° below x- axis. At x = 8/3, y takes

extreme values. For the region 8/3<x <4 dy <


dx
° above x -axis and dy >
dx
° below

x -axis.
:. the curve forms a loop in 2 ~ x ~ 4

Downloaded From : www.EasyEngineering.net


Downloaded From : www.EasyEngineering.net

270 Engineering Mathematics - I

4.5.8 Example
2
2
Trace the curve y ( x + 4a ) - 8a 3 =0 .... (1)

Solution:
y * 0 when x = O. :. origin does not lie on the curve.

Curve is symmetric about x-axis ,since F (-x,y)=F (x,y)


y is +ve for all real x :. Curve completely lies above x-axis.

ww Substituting y = mx + c in (I)

m3 + cx 2 + 4a 2 mx + 4a 2 c - 8a 3 =0
w.E ¢3 ( m ) = 0 => m = 0 and ¢2 ( m ) = 0 gives c = 0

asy
(ie) y = 0 is an asymptote to the curve From (I) -
dy
= - ----
16a 3 x
t
En dx (X2 + 4a2

dx gin
In first quadrant dy < 0 and hence y IS decreasing. dy
dx
o at (0, 2a). y s
maximum at (0, 2a) .
eer
ing
.ne
----------~--------------~x
t
Fig. 4.14
4.5.9 Example:
Trace the curve y- c cosh x / c = 0 .... (1)
Solution:
x= 0 in (1) gives y = (c/2)[e O +e-D] = r

Downloaded From : www.EasyEngineering.net


Downloaded From : www.EasyEngineering.net

Curvature and Curve Tracing 271

:. Curve does not pass through (0,0) and intersects y-axis at (0, c).

F (-x,y)= y -(c/2)( e- x / c +e<i<) = F(x,y) curve is symmetric about y-axis

It can be seen from (I) that y>o for all real x .

. h
dy= Sill
- ( x /) e -x -Xlc]
C = -1 [x/c
dx 2
which is >0 for x >0 and < ° for x <0.
y

ww
w .Ea Y=c

syE (D,c)

------~~------------~x
o
ngi
Fig. 4.15 nee
Thus the curve is increasing in r
(0,00), and decreasing in (-00,0).
ing
°
When x = we have dy = 0. Further y = c for x = 0. .ne
dx
(i.e.) The tangent to the curve at (0, c) is parallel to x-axis. t
4.5.10 Example
Trace the curve (x / a )2/3 + (y /a )2/3 -1 = 0 .... ( 1)

Solution:
y :I; 0 when x = ° :. (0,0) does not lie on the curve.

It can be observed that F (-x,y)= F (x,y)= F (x,-y)


The curve is symmetric about both the axes.

Downloaded From : www.EasyEngineering.net


Downloaded From : www.EasyEngineering.net

272 Engineering Mathematics-I

--~---------+----------~----~x

ww
w.E (0, - b)

a syE
-
Fig. 4.16

x = 0 gives / = b 2 => Y = ±b y = 0 gives x = ±a. Thus the curve meets x axis.


at (-a,o),( a,o) and y axis at (o,b ),( o,-b). ngi
For Ixl> a or \YI >b (x/a) 2/3 +(y/b)2/3 -1> 0 nee Equation is not satisfied.

:. Curve does not exist for Ixl> a or IYI >b. rin


:. Curve completely lies in the region Ixl ~ a and Iy ~ bl g.n
From (I) dy
dx
-!!..(
a bx
y
a )I/3. In the first quadrant y is decreasing as et
dy < 0 for
dx
O<x<a.
4.5.11 Example
Trace the curve y2 (a -x) _x 3 = O,a > 0 .... (1)

Solution:
Clearly y = 0, 0 when x = O. Origin is a double point on the curve.

F (x, - y) = F ( x, y). Curve is symmetric about x-axis.

Downloaded From : www.EasyEngineering.net


Downloaded From : www.EasyEngineering.net

Curvature and Curve Tracing 273

Differentiating (I) w.r.t, 'x' we get -


dy Fx (3a - 2x )
= - 3jJ dy 1(0,0) = o.
dx 2 (a-x) - dx

So x axis is a tangent to the curve at origin.


x = a is an asymptote parallel to y-axis.
3
A" coefficient of x is a constant, there is no asymptote parallel to x -axis.

For x < 0 and x> a curve does not exist (as y2 is negative)

ww
Substituting y

x
3
= mx + c in (1) and rearranging

m 2 + 1) + x 2 ( 2mc - am 2 ) + x ( c 2 - 2ame ) - ac 2 =0
w.E
(

fA (m) = m 2 + 1=> m = ±i .

a syE I

ngi
I x=a
PI
I
I nee
I
--~~--------.---------~x
(a,O) rin
g.n
et
Fig. 4.17

Hence there no oblique asymptotes.


3x2 + y2
From (I) YI = ( )
2 a-x y

For 0 < x < a, YI is positive. Hence y is increasing in 0 < x < a .

. 4.5.12 Example
Trace the curve x
2
(a - x) = l (b + x) .. .,(\ )

Downloaded From : www.EasyEngineering.net


Downloaded From : www.EasyEngineering.net

274 Engineering Mathematics - I

Solution:
Clearly curve passes through origin
F ( x, - y ) = F (x, y) .'. Curve is symmetric about x-axis.
Equating lowest degree terms to zero

by2 _ax 2 = 0 => y = ±~.x


There are two real, distinct tangents to the curve at (0,0)

ww y =0 in (1) gives x2 ( a - x) = 0 => x = 0, a


(a, 0)
w The curve intersects x-axis at

.Ea 3
There is no asymptote parallel to x-axis as coefficient of x is a con

I
syE y

I
I ngi
I
--~I~--------~~--~~~~----~x nee
I
I
rin
X=-b
I
l
I g.n
Fig. 4.18 e t
Coefficient of highest degree term in y is b + x
:. x + b = 0 is the asymptotes parallel to y-axis
Substituting y = mx + c in (I) and rearranging

x 3(1 + bm 2) + .... --0'I.e., V'3"'(m)-0' - Jb


- gIves m -_+_i

Hence no oblique asymptote to the curve.

t!) can be written as y = ±xJa - x which shows that y is imaginary in the


b+x
regions x > a or x < -b

Downloaded From : www.EasyEngineering.net


Downloaded From : www.EasyEngineering.net

Curvature and Curve Tracing 275

Hence the curve completely lies in the region -b < x ~ a


dy ( - 2X2 - 3bx + ax + 2ab)
From (I) - = ± 3/2 1/) which shows that dy ---) 00 as x ---) a
dx 2(b+x) (a-x)- dx

The tangent x = a to the curve at (a, 0) is parallel to y-axis.


Thus the curve forms a loop between (0,0) and (a, 0) .
4.5.13 Example:

wwTrace the curve x 2(y2 - a 2) - b2/ =0 ....(\ )

w.E
Solution:

asy
x = 0 in (1) gives only y = O. Curve passes through origin.only and
does not intersect either of the axes:

En
F(-X,y)= F (x,y) = F (x, -y) :. Curve is symmetric about both the axes.

gin
Lowest degree terms equated to zero gives a 2x 2+ b y2
2 =0

eer
showing that there are no tangents to the curve at (0,0)
Coefficient of highest degree terms in y (i.e.) of y2 is x 2- b 2
ing
:. x = ±b are the asymptotes parallel to y-axis
Coefficient of highest degree terms in x (i.e) of x 2 is y2 _a 2
.ne
:. y= ±a are the asymptotes parallel to x- axis. t
Substitution of y = mx + c in (1) gives

x 4 .m 2 + x 3 (2mc) +X2 (c 2 _a 2 _b 2m2)_ 2b 2mcx-b 2c 2 = 0

¢3 ( m ) = 2mc does not provide value of c since m = 0

Downloaded From : www.EasyEngineering.net


Downloaded From : www.EasyEngineering.net

276 Engineering Mathematics - I

The asymptotes are y = ±a which are parallel to x-axis.


Hence there are no oblique asymptotes.

From (I) x ~ ±~ by For IYI < a , x is imaginary


y2 -a2

Similarly for Ixl < b, y is imaginary. Hence curve does not exist for Ixl < b,
Iyl<a.
Since the curve is symmetric about both the axes, it is enough to study the curve in

ww the first quadrant.

w.EJ y=a
"-
asy
E x =-b
ngi
(0,0)

y=-a
x=b

ne(e
From (I) Y = ± /
"
"x
ax
2 2
Fig. 4.19 r ing
.ne
..... (2)
-b
From (2) it is observed that y ~ 00 as x ~ ba and x ~ 00 as y ~ a
ax
t
For the first quadrant y = + ~
x2 _b 2
a 2b2
:. Yl = - ~ .Thus Yl < 0 for x> b . So y is decreasing when x> b 111
(x 2 _b 2 )2
first quadrant.

4.5.14 Example
Trace the curve y2 (x - a) - x2 ( X + a) = 0 .... (1)

Solution:
x = 0 gives y = 0 :. (0,0) lies on the curve.

Downloaded From : www.EasyEngineering.net


Downloaded From : www.EasyEngineering.net

Curvature and Curve Tracing 277

Lowest degree terms equated to zero gIves a x . (2 + y 2) = 0 => x- + y = 0 J J

:. There are no tangents to the curve at (0,0)


J
coefficient of highest degree terms in y (i.e.) of Y IS x - a

:. x = a is the asymptote parallel to y-axis


3
No asymptotes parallel to x - axis since coefficient of x is constant)
Substituting y = mx + c in (I)

x 3 (m 2 -1)+ x 2 (2mc-um 2 - a) + x(c 2 - 2amc)-ac 2 =0


ww ¢3(m)=m 2 -1=0 gives m=±l

w .Ea
¢2(m) = 0 gives c =
a(m2 + 1)
2m
= a when m = I and = - a when m =-1

syE I
I V'.Sl
/\/y=x+a

ngi / I (a,2a)
// I

--~.---~--------~x
1
I nee
(a,O)
I
1 rin
" " " l(a,-2a)

~~+a=o
g.n
Fig. 4.20
If"" et
:. The oblique asymptotes are y =x + a and y + x + a =0
2 2
From (I) 2d = x -ax-a
dx (x -a).J x 2 - a 2

When x ~ -a, dy ~ 00 :. Tangent at (-a, 0) is parallel to y-axis


dx

-dy = o.
gIves x 2 - ax - a 2 1±f5
= 0 => x =- -a
dx 2

Downloaded From : www.EasyEngineering.net


Downloaded From : www.EasyEngineering.net

278 Engineering Mathematics - I

Tangents to the curve at x = 1 + Fs a (A and B in fig) are parallel to x -axis.


2
For 0< x < a y is imaginary
a<x<oo yexists
-a<x<O y is imaginary
- 00 < x <-a yexists
(i.e.) The curve does not exist in the region -a < x < a.

ww 1-Fs
For x = - - a there is no tangent to the curve (since the curve itself is not
2

wdefined there).

4.5.15 Example .Ea


syE
Trace the curve /(a 2 _x 2)_x 2(a 2 +X2) = 0 .... ( 1)

Solution:
x =0 in (1) gives y = 0,0 ngi
:. origin is a double pt on the curve.

nee
The curve is symmetric about both the axes sinceF(-x,y) = F(x,y) = F(x,-y).
Lowest degree terms equated to zero gives y
rin
= ±x as the tangents at the origin.

(i) ·
can be written as y 2 = X
2(2 2)
a2 +x2 • Th"IS
a -x
III d'Icates that y
g.n
+
~ _00 as x ~ a.

Hence the curve lies only in Ixl < a.


x = ±a are asymptotes parallel to y-axis.
e t
Substitution of y = mx + c in (l) gives X4 (m 2 +1) + ... = 0
¢4 (m ) = m 2 + 1 = 0 gives m = ±i
There are no oblique asymptotes
4 2 2 4
From (1) dy = a +2a x -x
dx (a 2 _x 2)3/2(a 2 +X2yl2

In the first quadrant: dy > 0 for 0< x<a


dx
(i.e.) y is increasing in this region.

Downloaded From : www.EasyEngineering.net


Downloaded From : www.EasyEngineering.net

Curvature and Curve Tracing 279

ww
w .Ea
syE
ngi
Fig. 4.21
4.1.16 Example
nee
Trace the curve l (a 2 + x 2 ) - x2 ( a
2
- x
2
) =0
rin .... ( I)

Solution:
x = 0 gives y = 0,0 :. origin is a double point on the curve. g.n
F(-x,y) = F(x,y) = F(x,-y) shows that the curve is symmetric about both the
axes.
e t
there are no asymptotes parallel to x-axis, since
coefficient of highest degree terms in x ((ie) of X4 )is constant.

The coefficient of highest degree terms in y (ie) of l 2


is x + a 2 •
:. there are no asymptotes parallel to y axis.

a 2 -x 2
From (1) y = ±x 2 2
a +x
For Ixl > a, y IS Imaginary

:. The curve entirely lies in the region -a s x sa.

y = mx + c in (l) gives ¢J. (m) = m 2 + 1 :. m = ±i

Downloaded From : www.EasyEngineering.net


Downloaded From : www.EasyEngineering.net

280 Engineering Mathematics - I

There are no oblique asymptotes dy = 0 gives x = ±a~ -1 ± J2


dx

:. y values are extreme at x=±a~J2 -1 ,where the tangents to the curve are
parallel to x-axes.
4 2 2 4
dy = + a -2a x _x
..... (2)
dx - (2a +x 2)3/2 (2
a -x 2)1/2

ww
w.E
y

asy
En x

(-a,O)
gin
eer
ing
Fig. 4.22

(ddxY)
.
a: ne
.'. x = ±a are tangents to the curve at (±a ,0). Further
(0,0)
=
a
= 1 from (2)
t
Hence y = ±x are tangents to the curve at (0,0)
The curve thus forms a loop between (0,0) and ( a ,0) and another loop between
(-a,O) and (0,0).

4.1.17 Example
Trace the curve a/ - 4 x2 ( a - x) = 0 ..... ( t)
Solution:
F (x, - y) = F (x, y) :. curve is symmetric about x-axis
F (0,0) = 0 curve passes through origin.

Downloaded From : www.EasyEngineering.net


Downloaded From : www.EasyEngineering.net

Curvature and Curve Tracing 281

y= °in (1) gives x = 0, a (i.e.) the curve cuts the x-axis at (0,0) and (a ,0)
2
Equating the terms of lowest degree to zero we get al- 4ax = O::::? Y = ±2x

Thus Y ± 2x = 0 are the tangents to the curve at (0, 0)

dy = 4x(3a-x2) ::::?(dY ) ----).00

dx 3ay dx (up)
(ie) the line x = a is a tangent to the curve at (a ,0)

ww substituting y = mx + c in (1) and rearranging the terms

4x 3 +ax 2 (m 2 -4)+ 2amcx-ac 2 = 0


w .Ea
fA (m) = 4 constant. Hence there are no asymptotes to the curve.

For Ixl > a


sIxlyaE.
we find y is imaginary.

Curve does not exist for >


ngi
nee
rin
________ ~~--------~~~x
g.n
e t
Fig. 4.23
4.1.18 Example:
Trace the curve a
21 = x 2( 2a - x) ( x - a)
Solution:
F (+ x, - y) = F curve is symmetric about x-axis.

F (0,0) =0 origin lies on the curve (isolated point)

Downloaded From : www.EasyEngineering.net


Downloaded From : www.EasyEngineering.net

282 Engineering Mathematics - I

y = 0 in (1) gives x = 0,a,2a.


The curve intersects the x-axis at (a ,0) and (2 a ,0)
Terms of lowest degree are l + 2X2 which shows that there are no tangents to the
curve at (0,0) .

(y2 + 2X2 = 0 gives y = ±J2ix imaginary)

Proceeding as in 3.1.17 it can be seen that the curve has no asymptotes


y is imaginary for x < a and x> 2a(i.e.) the curve exists only in the region

ww
a:::;x:::;2a.

~ 00 as x ~ aor x ~ 2a
w
Further dy

:. x =a
dx
and x .Ea
= 2a are tangents to the curve parallel to y-axis.
y
syE
ngi
nee x
(0,0) 0
rin
I
x=a
I
x = 2a
g.n
Fig. 4.24 e t
4.1.19 Example
Trace the curve x( x 2 + y2) = a( x 2 - y2 ),( a> 0) ........... (1) (2001 s)

F (x, - y) = F (x, y) curve is symmetric about x-axis

Solution:
y = 0 in (I) gives x = 0,0, a . The curve meets the x-axis at (0,0) and (a ,0).
Equating lowest degree terms to zero we get
y2 _ x 2 = 0 => y = ±x which are the tangents at (0,0)

Downloaded From : www.EasyEngineering.net


Downloaded From : www.EasyEngineering.net

Curvature and Curve Tracing 283

dy ~ 00 as x ~a . Thus x = a is a tangent to the curve at ( a ,0)


dx
For x > a (1) shows that y is imaginary
No part of the curve exists for x> a.
y ~ 00 as x ~ -a :. x = - a is an asymptote to the curve.

ww
w.E
asy
En
gin
4.1.20 Example eer Fig. 4.2~

Trace the curve l (a 2+ x 2) = x 2( a 2- x 2) ing ..... (1)

Solution:
.ne
F(O,O) = 0 curve passes through origin

F (-x,y) =F (x,y) =F (x,-y) curve is symmetric about both the axes.


t
Lowest terms equated to zero gives a 2y2 - x 2a 2 = 0 => y ± x which are tangents
to the curve at (0,0)

(:) ~ 00 as x ~ ±a :. x = ±a are the tangents to the curve at (±a, 0)

y is imaginary when x < -a or x > a


:. curve completely lies within the region -a ~ x~a

Downloaded From : www.EasyEngineering.net


Downloaded From : www.EasyEngineering.net

284 Engineering Mathematics - I

x
(-a,D)

ww
w.E
4.1.21 Example asy Fig. 4.26

Yrace the curve


En
Solution: gin
Let F( x,y)= y( a
2
+ x 2 ) - a2x =0
e eri
F( -x,y) =I:- F(x,y) =I:- F(x,-y), Hence the curve is not
symmetric about both the axes. ng.
The powers ofy are odd. Therefore the curve is symmetric
In opposite quadrants.
net
F(O,O)=O. So the curve passes through the origin.

The coefficient of highest power of x i.e~ of x 2 is y


So y=O is an asymptote parreJ to x-axis.

The coefficient of highest degree of y is x 2 + a 2 = 0 => x = ±ai


Hence the extreme points are A(a,aJ2) B(-a,-aJ2)
Thus the tangents at A and B are parallel to x-axis.
The shape ofthe curve is as shown below.

Downloaded From : www.EasyEngineering.net


Downloaded From : www.EasyEngineering.net

Curvature and Curve Tracing 285

Exercise - 4(E)

ww Trace the following curves

w.E
3.
2 x-2
y = 2- - 4. x 3 + / = ax 2 x> 0
asy
;
x -1
5. 9ai = x(x-3a)2
En
7. y3 =a 2 x-x 3
gin 8. i(a-x)=x 3 ;a>O

9. y =2 - - -2
x +4a
8a 3
10.
eer
i(a+x)=x 2 (b-x)

11. i = (x-a)(x-b)(x-c) ; a,b,c>O 12. y=x


3
ing
13. i
2
= x (3a-x) .ne
15. y2 =(X-2)2(X-5)
x+a
t
17. xi =4a 2 (2a-x)

4.2.0 Tracing of curves (polar form)


The equation of the curve in polar coordinates may be in one of the following three forms
F (r,B)=O ,r=F(B), B=F(r)
A study of the following points will simplify the tracing
I. curve is symmetric
(a) About initial line B = 0 if F(r,-B) = F(r,B)

Downloaded From : www.EasyEngineering.net


Downloaded From : www.EasyEngineering.net

286 Engineering Mathematics - I

(b) About pole '0' if F(-r,B) = F(r,B) or


F(r,J[ + B) = F(r,O)

e = 1[/2

ww
w.E
0=0
Pole 0

asy
En
gin
eer
Fig.4.27(a)

(c) About 8 = J[
2
if F(r,J[-8) = F(r,8)
ing
(d) About 8 = J[ if F(r, J[ -8) = F(r,8)
4 2 .ne
(e)
3J[ . 3J[
About 8 = - If F(r,--8)=F(r,8)
4 2
t
2. (a) Values of 8 for which r =0 gives the equation of the tangents to the curve

at the pole

(b) F(r,O) = 0 gives the points of intersection of the curve with the initial line
J[
(c) F(r, -) = 0 gives the points of intersection of the curve with the line
2
8=J[
2

Downloaded From : www.EasyEngineering.net


Downloaded From : www.EasyEngineering.net

Curvature and Curve Tracing 287

3. (a) If 'i and r2 are the least and greatest values of r then the curve lies in the
annulus between circles of radii Ii and r2

(b) Curve does not exist if r is imaginary for values of () or if B is imaginary


for values of r

4. In t Ile region ' h -e1r > 0 I r I •mcreases -dr < 0 I r I decreases .111 that regIOn
. .111 wIlIC .
dB dB
() =rr/2 y

ww
w.E
a syE
ngi o_ _ _ _ _ _ _ _ _

n
__ ~~-..J... ~ . --. X

o
eer
0=0

Fig.4.27(b)

p(r,B)
Fig.4.27(c)
i ng.
5. Let
net
be a point on the curve and P T is the tangent to the curve at P, OR
radius vector and ¢ is the angle T P R. Then

(i) tan¢ =r dB gives equation ofPT


dr
(ii) ¢ = 0 gives tangent parallel to initial line

(iii) do
V' = -Jr .
gives tangent to Il1Itla I I'me
1./(/r. ..
2
6. If the curve meets the initial line at two distinct points and curve is symmetric
about the initial line then part of the curve forms a loop between these two points.
7. If for any value of B (sayB ,) such that It r = 00 then B = B1 is an asymptote to
0-->00
the curve

Downloaded From : www.EasyEngineering.net


Downloaded From : www.EasyEngineering.net

288 Engineering Mathematics - I

8. Sometimes it may be convenient to change the equation of the curve from polar to
Cartesian form by substituting respectively
x = rcosO, y = rsinO
y

9 = 1t/2

ww ! 9=0

w.E --nf~~Y===~)---7X

asy
Few solved examples are given below

4.2.1 Example En
Trace the cardioid
gin
r -a(l +cosO) = 0----(1)
eer
Solution:
F (r,O) = r -a(l +cosO) = F(r,O) curve is symmetric about the initial live. ing
When 0 = 1[ we get r = 0 :. curve passes through pole and .ne
o= 1[ is the tangent at the pole

Max IcosOI = I :. Max r = 2a


t
:. The curve is entirely within the circle of radius 'a'.
Hence no asymptote to the curve.

0 0 1[ 21[
1[/4 1[/3 1[/2 21[/3 41[/3 31[/2 51[/2
2a a a 2a
r
a(l + 1/J2) 3a/2 a/2 0 a/2 3a/2

r = 2 a when 0 = 0 and r = 0 when 0 = 1[

.'. The curve intersects the initial Iine at (2a, 0) and (0, 1[ )

Downloaded From : www.EasyEngineering.net


Downloaded From : www.EasyEngineering.net

Curvature and Curve Tracing 289


------------------~~--------------------------------------

. b
The tangents to the curve are gIven y tan
AI
'I' = rde
-
dr

tan ¢ = a(l + cosO) = -cot e = tan (1l2 + ( )...(2)


a(e-sine) 2 2
when e= 0 , (2) gives ¢ = ~ (ie)The tangents at (2 a ,O)is perpendicular to the
initial line ..

ww
w.E
asy
En
gin
Fig. 4.28 e eri
4.2.2 Example ng.
Trace the curve r

Solution:
= a(l- eose)
net
8 = n/2

8=0
(+2a,n) J--------..... ~---
(0,0)
........
~

Fig. 4.29

Downloaded From : www.EasyEngineering.net


Downloaded From : www.EasyEngineering.net

290 Engineering Mathematics - I

() 0, J[ 7J[ J[ SJ[ J[ 3J[ 2J[ 4J[ J[


--
2J[ 4' 4 3' 3 2' 2 3 ' 3
a
r 0
a(l- YJ2) ~ 3~ 2a

r = 0 when () = 0,2J[ and r = 2a when () = J[

:. The curve intersects the initial line at (0, 0), (2a, J[)

ww tan ¢ =
a(l-cos())
asin()
= tan-
()
2

w.E :. tan ¢ = 0 when () = J[ (ie) the tangent at (2a, J[) is perpendicular to the initial
line.
asy
(work out remain ing points as in 3.2.1)

4.2.3 Example: E°n


gin
Trace the curve r - a (1 + sin ()) = ..... (1)

Solution: r = a when () = 0, J[
eer
r=Owhen ()=-J[/2
J[ ing ...... (2)

and r = 2a when () = -
2 .ne
:. The curve intersects the initial line at

and intersects the line () = J[/2 at


(a, 0), (a, J[)
(2a"~-i) and (0, - J[/2)
t
F (r, J[ - ()) = r - a ( 1+ sin ( J[ - ())) = r - a (1 + sin ()) = F (r, ())
:. curve is symmetric about () = ~

The equation of the tangent at the pole is () = -~ (from (2))

Maxlsin ()I =1 :. Max r = 2a Hence curve completely !ie~ ".'!!hin the region of
the circle r = 2a and there are no asymptotes to the curve.

Downloaded From : www.EasyEngineering.net


Downloaded From : www.EasyEngineering.net

Curvature and Curve Tracing 291

0 0 7r/6 7r/2 57r/6 7r 27r/6 37r/2 117r/6 27r

r a 3(1/2 2a 3(1/2 II
a/2 0 (1/2 a

ww
w .Ea
--------~--~~--~------~.O=O

syE
n 0= -rr/2

gin
e
Fig. 4.30
4.2.4 Example:
eri
Trace the curve r - a sin 0
Solution:
=0
ng.
0 0 7r/12 7r/4 7r/3 57r/12 7r/2
n et
a 0
r 0
a
- J3
-a -
a
2 2 2
When 0 = 0 we have r = 0 :. pole lies on the curve

F (r,7r+O) = r-asin(27r+20)=r-asin20= F (r,O)


:. curve is symmetric about pole. ...... (1)

F (r, ; - 0 ) = r - a sin ( 7r - 20) = F (r, 0)

Downloaded From : www.EasyEngineering.net


Downloaded From : www.EasyEngineering.net

292 Engineering Mathematics - I

and F (r, 3; -8 ) = r -asin(3ff - 28) = F(r,8)

... curve is symmetric about the lines 8 = ff14 and 8 = 3ff14 ...... (2)
from (1) and (2) it follows that the curve is symmetric in all the four quadrants.
r =0 gives 8 = O,ffI2,ff,3ff/2 these four lines are tangents, to the curve, at the
pole.

Max r = Max la sin 281 = a :. The curve completely lies within the region of

ww
the circle r = a .

The line 8 = ffl4 meets the curve at (a,ffI4).

w .Ea 0= rc/2

syE
n gin
o =rc ----.-,;:~~rr_Illll:iiiii:::-----__1~ 0 =0
e eri
ng.
e =3rc/4
Fig. 4.31
n et
Note: Similarly trace the curve r = a cos 28 .
4.2.5 Example:
Trace the curve r2 - a 2cos 28 = 0 ...... ( I)
Solution:

8 0 ffl6 ffl4 ff13 to 2ff13 3ffl4 5ffl6 ff

r ±a ±a/J2 0 imaginary 0 ±a/J2 ±a

Downloaded From : www.EasyEngineering.net


Downloaded From : www.EasyEngineering.net

Curvature and Curve Tracing 293

Jr
r =0 gives eos2B = 0 => f) = ±-
4
Jr
curve passes through the pole and 0 = ±- are the two tangents at the pole
4
F(r,-fJ) = F(r,fJ) :. curve is symmetric about the initial line.

F(r,Jr -0) = r2 _a 2 eos(2Jr - 20) = r2 _a 2 eos20 = F(r,fJ)

curve is symmetric about 0 =~


ww F(,., Jr + fJ) = ,.2 - ([2 eos(2Jr + 2fJ) = F(r, 0)

w curve is symmetric about the pole

.Ea
syE
8=0
~ __ ~
n
________ ~~

gin
______ ~~~~x

e eri
ng.
Since leos 2fJI ~ 1 ,Max r =a
Fig. 4.32
n et
Curve lies completely within the circle r =a
B = 0 gives r2 = a 2 => r = ±a
curve intersects the initial line at (a,O) and (-a, 0)

r2 is -ve When Jr < 0 < 3Jr (": cos 20 < 0)


4 4
curve does not exist (above the initial line) in this region.
Differentiating (1) w.r.t I fJ I
2
dr 2 2 • 2B 0 dr _a sin 2B
2 r-+ a SIn - => -=----
dB dB r

Downloaded From : www.EasyEngineering.net


Downloaded From : www.EasyEngineering.net

294 Engineering Mathematics - I

dO r ff
tan¢ = r - = r. 2 = -cot20 = tan(-+ 20)
dr _a sin 20 2

Thus when O=O} tan¢ = co


O=ff
Hence the tangents at 0 =0 and 0 = ff are .lIar to initial line

ww When

w.E
(i.e.,) the radius vector 0 = %is tangential to the curve at the pole
(': 0 = asy
%gives r = 0)

Similarly the radius vector 0 En = 3ff


gin
also is tangential to the curve at the pole.
4
4.2.6 Example:
Trace the curve r - a cos 30 = 0 - - - -(1) e eri
Solution:
ng.
F (r, -0) = r -acos30 == F (r,O)

Curve is symmetric about initial line net


Line 0 =0 intersects the curve at (a, 0 )
Since Icos 301 ~ 1 , from (1), Irl ~ a .Curve completely lies

within the region of the circle r =a


r=O when 30=0 :.30=(2n+l)ff/2, n=0,1,2, .....

(i.e.) curve passes through the pole when 0 = (2n + 1)ff/6( n = O/oS)
These six lines are tangential to the curve at the pole

The part of the curve from 0 = to 0 to ff is AA, 0 .


6

Downloaded From : www.EasyEngineering.net


Downloaded From : www.EasyEngineering.net

Curvature and Curve Tracing 295

This has a reflection AA20 in the initial line. Thus AAIO A2A is a loop.
F (r,4n/3 -B) = r -acos( 47l' - 3B) = F (r,B)

The curve is symmetric about B = 27l'/3 OBIB is the part of the curve

from B = 27l'/2 to 27l'/3. This has a reflection OB2B in line 0 = 27l'/3

ThusOBI BB2 0 forms a loop. F(r, 8; -B ) = r -acos(87l' - 3B) = F(r,B)

ww Curve is symmetric about B = 4~

w .Ea B
0= 1t/2

syE
n gin ......... A2
(a,O)

e ..... .........
eri
() :::
.... ..... 111t/6
....
ng.
n
(-a,1t/3)

Fig. 4.33

OCIC is the part of the curve from B = 7l'/6 to 7l'/3 and


et
OC2C is its reflection in B = 4~

:.OCpC20 forms a loop.


It can be seen that OCPC20 is a reflection of OBI B820 in the line B = 7l'
acos3B -1
tan¢ = = -cot3B
-3asin3B 3
¢ ~ 00 when B ~ 0
:. The tangent at (a,O) to the curve is ..LIar to the initial line.

Downloaded From : www.EasyEngineering.net


Downloaded From : www.EasyEngineering.net

296 Engineering Mathematics - I

4.2.7 Example:

Trace the curve r2 - a 2 sec 2 f) =0

Solution:
The given curve can be written as F(r, 0) = r2 cos 2 f) - a 2 = 0 - - - -(1)

f) =0 gives r = ±a .Curve docs not pass through the pole.


F(-r,f) = F(r,f) :. Curve is symmetric about the pole

ww F(r,-f) =,.2 cos 2 (-f)-a 2 = F(r,f)

w.E
Curve is symmetric about the initial line f) =0

a (TC -f)-a 2 = F(r,f):. Symmetric about f) = TC


2
F(r,TC -0) = r2 COS

2
syE
Since cos 0 S; 1 (i.e.,) IcosOI S; 1 we get r2 2:: a 2
2

Curve does not exist in the region


ngi
,.2 < a 2

l' --) 00
TC
as f) --) -
2
TC
2 nee
:.0 = - is an asymptote to the curve.

f) 0 TC TC TC rin TC
-
6
-
4
-
3
g.n
-
2

l' ±a + 2a
-J}
±J2a ±2a 00
et
dl' J 1 df) l'
r=- = a- sec- f) tan f) - = -) cos 2 f)cotf)
df) dr a-

tan¢=rdf) =cotf)=tan(TC -f)) :. ¢=TC/2 when f)=O showing that


dr 2
the tangent ate a, 0) is ~/ar to the initial line .

Downloaded From : www.EasyEngineering.net


Downloaded From : www.EasyEngineering.net

Curvature and Curve Tracing 297

(-a,O) -\---~-~I-

ww Fig. 4.34

w
4.2.8 Example
Trace the curve
.Ea
,.2 - a 2
cos 2(} = 0
Solution:
() = 0 gives
syE
r = ±a :. curve intersects initial line at (±a,O)
,. = 0 gives cos 2fJ ngi
= 0 ~ B = ± 7r/4 pole lies on the curve

and () = ± 7r /4 are two tangents to the curve at the polenee


Icos201 S 1 ~
rin
IrI sa. Curve completely lies within the region of the circle I' =a

g.n
Curve is symmetric about the initial line.

F (-1',0) =(rr _a 2 cos20 = F (1',0) symmetric about the pole


e t
F (r,7r-(})=r 2 -a 2 cos(27r-20)= F (1',0) symmetric about 0=7r/2

0 ±Jr/6 ±7r/4 ±7r/3 ±Jr/2 ±37r/4


0
±a ±a/J2 0 imaginary 0
I'

Curve does not exist in the regions 7r/4 < B < 3Jr/4 and - 3Jr/4 < 0 < -Jr/4

Downloaded From : www.EasyEngineering.net


Downloaded From : www.EasyEngineering.net

298 Engineering Mathematics - I

The curve forms a loop bctween (a ,0) and (0,0) and has reflection in B = 1(/2

~ dO ~2
Diffcrentiating (1)2r- = -2a 2 sin20 tan¢ =r- = 2
= -eot20
dO dl' a sip 20
= tan (1(/2 + 20)

0= 0 gives ¢ = 1(/2

Thus the tangents at 0 = 0 (ie) (a ,0 ) and ( - ( l ,0) are perpendicular to the initial
line.
ww y

w .Ea
syE x

ngi
nee
rin
4.2.9 Example:
Fig. 4.35
g.n
Trace the curve r - {a + beosB} = 0 e t
Solution:
F (r, -0) = r - {a + beos ( -B)} = F (r, 0)
Curve is symmetric about the initial line.

Since leosBI ~ I we have IrI ~ a + b


The curve completely lies within the region of the circle. r = a+b
o =0 gives r=a+b :.curvemeets 0=0 in A(a+b,O)
o= 1( gives r = b curve meets B = 1( in B ( a - b, 0)
{l -

Downloaded From : www.EasyEngineering.net


Downloaded From : www.EasyEngineering.net

..
Curvature and Curve Tracing 299

Thus when a*- b curve meets the initial line at A and B

When a = b we get r = 0 curve passes through the pole.


Curve meets the line B = 1£/2 at (a, n/ 2) .

Curve passes through the pole when () = cos--! (-{l/ b) (since r = 0)


Hence the tangent to the curve at the pole is the line B = cos"! ( -a/ b) which

ww (i)

(ii)
does not exist when a > b

is B = cos-!(-l)=n when a=band

w.E(iii) exists when a < b

asy
0 0 En n/3 1£/2 2n/3 n

gin
--

r a+b a+b/2 a a-h/2 a-b

tan ¢ = r -
dB
= a+hcosB eer .
showmg that tan¢ = 00 when 0 = 0

:. ¢ = n /2
dr -hsinB
(ie) the tangent to the curve at (a + b, 0) is ing to the initiallinc.

.ne
.l/llr

Further ¢ *- 0 for any value of 0 and hence no tangent to the curve is parallel to

the initial line t


When a =h
l+cosO .
tan ¢ = . -cotB/2=tan(n/2+0/2).
smO
When 0 = n we get ¢ = n (ie) the initial line is a tangent to the curve at (0, n)

Wht1f\ a*- h we have tan ¢ ~ 00 as () ~ n (i.e.) The tangent to the curve at


(a - b, n) is also perpendicular to the initial line.

Downloaded From : www.EasyEngineering.net


Downloaded From : www.EasyEngineering.net

300 ____E_n--:9:::...ineerin9 Mathematics - I

I (a+b,O)
I (2a,0)
I
a>b a=b

ww Fig. 4.36 (i) Fig. 4.36 (ii)

w .Ea
syE
ngi
a<b

Fig. 4.36 (iii) nee


4.2.10 Example:
rin
Trace the curve r = a ( sin f) + 1/sin f))
g.n ..... ( I)

This example illustrates that changing the coordinate system from polar to cartesian
facilitates the tracing. e t
(I) can be written as ,.1 = a(r sin 0 + ~
r SIn f)
1 ..... (2)

Substituting r cos f) = x and r sin f) = y (2) reduces to

....(3)

No constant term in (3) :. curve passes through origin

Lowest degree terms equated to zero gives 2/ + x = 0 => x ± yJii


2

The tangents at (0,0) are imaginary.

Downloaded From : www.EasyEngineering.net


Downloaded From : www.EasyEngineering.net

Curvature and Curve Tracing 301


--------------------=-----~------------------------------------

x = 0 in (3) gives y = 0,0,2 a


y-axis cuts the curve at (0,2 a) and (0,0) is an isolated point

Equating coefficient of the highest dt'gree terms in y to zero gives y = ()


(i.e.) x-axis is an asymptotes to the curve.

F (-x, y) = F (x, y) :. curve is symmetric about y-axis


Differentiating (3) W.r.t 'x',

ww dy 2x ( a - y) . (dY )
dx = 3/ +X2 -4ay .. dx (O,la) = 0

w.E
The tangent at (0, 2a) is parallel to x - axis

asy (0,2a) y = 2a

En
gin o x

eer
x = ±yJ2a- y
Fig. 4.37
ing
From (3)
y-a .ne
when y < a or when y > 2a ,clearly x is imaginary (i.e.,) The curve exists only
in the region a < y ~ 2a
t
Exercise 4.2
Trace the following curves

1. r 2 cos20=a
l
2. r=l+J2cosO 3. r=asin 2 0secO
4. r=3+2cosO 5. r=asin30 6. r=a(cosO+secO)
l
7. r=asin40 8. r=a(coso+----) 9. r2 =asin20
cosO
a2
)0. r = a(I-sinO) 11. r2 = - -
cos20

Downloaded From : www.EasyEngineering.net


Downloaded From : www.EasyEngineering.net

302 Engineering Mathematics - I

4.3.0 Tracing of curves when the equation is given in parametric form:


Suppose the equation of the curve is in the tonn
x = ./; (t) and y = ./; (t) ..... (1)

where t is the parameter. The study of the following points are useful for tracing the
curve
I. If for some value for I , say I) , .I; (t) = 0 and .t; (I) = (l
then the curve passes through the origin.

ww 2. If.1; (I) is an odd function and .t; (I) is an even function

J;w.
then the curve is symmetric about y-axis.

E
3. If (I) is even and

asy
J; (I)
then the curve is symmetric about x - axis
is odd

4.
En
Intercepts on X(Y)a.xis are obtained by solving .1;(1)=0 (.t;(t) =0)

5.
respectively.
Greatest and least value of .I; gin (I) and .t; (I) give the region in which the curve
exists.
eer
6. If It (t) or 1; (I) tends to 00
ing
as I -) t) then t = t) is an asymptote to the curve.

7. (ddxY)I~II = 0 indicates that t = t) is a tangent to the curve parallel


.ne to the

x-axis. t
8. (: ) -) 00 as I -) ') then I = I) is a tangent parallel to the y-axis.

9. If It(t+a)=j;(1) and .t;(t+a)=.t;(t) then the curve is periodic of period


a
10. Sometimes it may be convenient to transform the equation of the curve from
parametric to cartesian form for tracing the curve

4.3.1 Example:
Trace the curve x = at 2 ,y = 2at

Downloaded From : www.EasyEngineering.net


Downloaded From : www.EasyEngineering.net

Curvature and Curve Tracing 303

Solution:
Eliminating t from x and Y

l = 4a 2t 2 = 4a.at 2 = 4ax
y

~~+- __ ~ ______-+x
(0,0)
l = 4ax
---...;..-
ww
origin.
w .Ea
Fig. 4.38
This is a parabola with vertex at origin y aXIs is a tangent to the curve at the

4.3.2 Example: syE


Trace the curve x = acos/,Y = bsint
ngi
Solution:
Eliminating t from both x and Y nee
(= J+( ; J~ rin
1 wh ich is an ellipse (standard fonn)
g.n
y (O,b)
e t
---------+----~~----~--------~.x
(-a,O) (a,O)

(O,-b)

Fig. 4.39
4.3.3 Example:
3
1
Trace the curve x = (2 ,Y = t--
3

Downloaded From : www.EasyEngineering.net


Downloaded From : www.EasyEngineering.net

304 Engineering Mathematics - I


--------------------------------------
Solution:
Eliminating't'

2
y2 =t (I-t/{f =x(l-73'f =i(3-X)2
(i.e.,) 9i = X(3-X)2 -----(1)
y =0 gives x =O,x =3.
The curve passes through origin and again intersects x - axis at (3,0).

ww F(x,-y) = 9y2 -x(3 _X)2 = F(x,y)

w.EThe curve is symmetric about x - Q.'Cis and hence it forms a loop between (0,0)


(3-x)Fx
and (3,0) y
3
asywhich shows that the curve does not exist for x < 0

En
(i.e.,) to the left of Y-axis
Lowest degree terms equated to zero gives x =0 (i.e.,) y-axis is a tangent to the
curve at origin.
gin i
eer
Coefficient of x 3 is I (a constant) and coefficient of is a constant.

Substituting, y = mx + c i
Hence, there are no asymptotes parallel to either x or y-axis
in (I) and rearranging ng.
3 2
_x + x 2 (9m +6) + ........ = 0
net
fA (m) =-1 a constant, fA (m) =0 gives m =±J{i
:. No oblique asymptotes exist

Fig. 4.40

Downloaded From : www.EasyEngineering.net


Downloaded From : www.EasyEngineering.net

Curvature and Curve Tracing 305


--------=------------------------------------------

dy =±![l-X]
£Ix 2 ~

Y 0:. The curve is extreme at x = 1


(d ) =
dx ,~I

elY)
( dx =+ ___ 1
(3,0) - J3
:. J3Y = ±(x - 3) are tangents to the curve at (3,0)
ww
4.3.4 Example:

w.E
Trace the curve x = a(t + sin I), Y = a(l + cos t)

Solution:
asy
For no value of f both x and Y simultaneously vanish

En
:. The curvc does not pass through the origin.

gin
x is an odd function while y is even

:. The curve is symmetric about y -llxis


eer
x(t) = 0 gives t =0 .Correspondingly x = 0 and y = 2a
ing
dy =dYidt =_ llsint =-tany,;=tan(TC-t/)
dx dx/dt a(i+cost) .2 12
.ne
When 1=0 slope tan¢ = tanTC ~ ¢ = TC

:. The tangent at (0,2a) is parallel to x - axis


t
t -TC _5% -~
0
~ 5% TC

X llTC
-a(~+ 5;) -a(; +1) 0 a(I+;) a(~+ 5:) (lTC

° a(l+ ~)
y 0 2a a (/(1+~)
°

Downloaded From : www.EasyEngineering.net


Downloaded From : www.EasyEngineering.net

306 Engineering Mathematics-I

y(t) = 0 gives cosl = -1 ~ I = ±7l",±37l", ..... .


(i.e.,) 1= ±(2n + 1)7l" n=1,2,3, ....
corresponding x is ±a7l",±3a7l", ....

Further x ~ 00 as I ~ 00 and IYI ~ 2a


:. The curve completely lies within the region 0 ~ Y ~ 2a

and is period of period 27l"

ww y

w.E
asy
En
gin Fig. 4.41

4.3.5 Example:
Trace the curve x = a(t -sinl);y = a(l-cost) eer
ing
Solution: .ne
X(I) =0 only when 1=0

y(t) = 0 gives 1= ±2n7l" n=0,1,2, ....


t
x(O) = 0 = y(O):. curve passes through origin.
Since Icos'l ~ I we have y 2 0

The curve is completely above x - axis and lies within the region 0 ~ y ~ 2a

(y = 2a when' = ±(2n + 1)7l",n = 0,1,2, .. )


The curve meets x - axis at x = ±2an7l", n = 0,1,2, ....
y is an even function and x is odd

Downloaded From : www.EasyEngineering.net


Downloaded From : www.EasyEngineering.net

Curvature and Curve Tracing 307

:. curve is symmetric about y-axis

LI x(t)
t~oo
= 00 but LI y(t) is finite
l~cIJ

:. There are no asymptotes to the curve

tan ¢ = : = cot ~ = tan ( ; - ~)


Jr I
~¢=---
2 2
ww
The tangents to the curve at t = ±2nJr, n = 0,1,2, ....... are parallel to y-axis

w.E
The curve is periodic of period 21f

asy y

En
gin
~------~------~--~--~--------~----~x
-4a1t -2a1t o e
2a1t
eri
4a1t

Fig. 4.42 ng.


4.3.6 Example:
Trace the curve x = aeos 3 e,y = bsin 3 e
net
Solution:

The functions cos 3 e and sin 3 e are periodic of period 2Jr .

Hence it is sufficient to trace the curve for one period.


For no value of e both x and y vanish. (0,0) does not lie on the curve

x( e) =0 gives e=±~ ,
yeO) =0 gives e = O,±Jr

Downloaded From : www.EasyEngineering.net


Downloaded From : www.EasyEngineering.net

308 Engineering Mathematics - I

:. The curve meets x - axis at x = ±a and y-axis at y = ±b

Since Icos
3
01 s:; 1 and Isin '01 s:; 1 we have Ixl s:; a, Iyl s:; b
e 0 Jr/6 Jr/3 Jr/2 2Jr/3 5Jr/6 Jr

3J3 -2J3a
x a --a -a
8 a/8 0 -a/8 8
1
~J3 b b

ww y 0
-b
8
3J3 b
8 h 8
-
8 0

w.E tan ¢ = -
dx
dy b
= - - tan
a
e
tan¢
asy e
=0 when = ±(2n+ l)Jr

when E
(i.e.,) The tangent to the curve at these points is parallel to x - axis tan ¢ ~ 00

n
e ~±
(
2n+l) gin
- - Jr
2
eer
:. The tangent to the curve at these points is parallel to y-axis

It can be observed that for corresponding to x for some ieng there is -x for Jr - e
and corresponding to x for some e there is - y for - e .
.ne
:. The curve is symmetric about both the axes
(O,b)
t
o =...nE-_ _-+ __~3!Jii-(a,o)
(-a,O) 0=0

0= -n12
(O,-b)

Fig. 4.43

Downloaded From : www.EasyEngineering.net


Downloaded From : www.EasyEngineering.net

Curvature and Curve Tracing 309

4.3.7 Example:

Trace the curve x = a[ cose + ~ log tan ~ J,:I = asin e


2

Solution:

yeO) = 0 but x(O):f:- 0 . (0,0) docs not lie on the curve

yeO) = 0 gives e= 0 and lyllll'Lx =a

ww x is an even function and y is odd. So the curve is symmetric about the x - axis

w.E
Corresponding to each x( e) there is -x for 1C - e

Since Isin el
a
:. Curve is also symmetric about y-axis

S; 1 syIYIE
we have S; a.:. The curve entirely lies in the region

-(I S; Y S; a .
ngi
x( ~) = a [ cos ; + log tan : ] nee
= a [0 + log 1] = 0

x[ -;] = 0 rin
Similarly,
g.n
Corresponding y values are y(; ) =a and y( -; ) =-£1 et
Thus the curve intersects the y -axis at (0, a) and (0, -(I)
1 l

tan~ = <!vdx
a -sine+ - .~
=
[
sinti = cos' 0
acosO
J
sinOcosO
= cot 0 = tan (1C _
2
e)
:.¢=1C_ e
2

Hence, ¢ =0 when e = ~ and ¢ = 1C when e = -~

Downloaded From : www.EasyEngineering.net


Downloaded From : www.EasyEngineering.net

310 Engineering Mathematics - I

(i .e.,) The tangents to the curve at e =:= ± 7r/2 are parallel to x - axis

Further e~ 00 or -00 according as e ~ 0 or 7r


.". X - axis is an asymptote to the curve.

ww
w.E (O,-a)

4.3.8 Example:
a syE
Fig. 4.44

3at 3al 2 ngi


Trace the curve x = - - , ,y = - - , a > 0
l+r' l+t'
nee
Solution:
rin
X
3
= (3a)3
- , .I,y
l+r
3 3
= (3a)3
-3
1+1
.1 6
g.n
.". x 3 + y3
3a )3
= ( __
1+1
3
1\1 +/
3
) =
3 3
27a t
3
(l+t
?
t
et
3at 3at 2
=3a'--3 ' - - 3 = 3axy
l+t l+t
Thus the Cartesian form of the curve is x +
3
i = 3axy

For tracing the curve see example 1.3

4.3.9 Example:
a(l-t 2 ) 2bl
Trace the curve x = y = - -2
1+ t 2 ' 1+ t

Downloaded From : www.EasyEngineering.net


Downloaded From : www.EasyEngineering.net

Curvature and Curve Tracing 311

Solution:

x
2
(1_12]2 i
a2 = 1+t 2 ' b2
2 2
Thus ~ + L -_ I ( see examp Ie 3.3 .L-'"')
2
a b

Exercise 40

ww I. x

2.
= aCt +sin t),y = a(l + cost)
x = a sin 21(l + cos 2t), Y = a cos 2t(l- cos 2t)
w .Ea
3. x = a[ cosO -log(l +cosO) ],y = asin 0
4. x = aU -sint),y = a(l-cost)

Exercise 3.1
syE
I . An asymptote to the curve ngi
i (a + x) = x (3a - x) is
2

(i) x-a=O (ii) x+a =0 (iii) x-3a=Onee (iv) none of these

2. The curve a 2y2 =x2 ( a2 - Xl) is symmetric about


r ing I ADS: (ii»)

(i) x-axis (ii) y-axis (iii) both the axes .ne


(iv) none of these

3. The curve x + 3
i = 3axy is symmetric about the line .............. .
t
I ADS: (iii) I

[ADS: y = x)

4. If the tangents to the curve i = (x - a)( x - b)2 ,( b > a,) at the point x = bare

inclined at angle 0, tan 0= .............. .

[ADS: ±.jb-a )
5. The two tangents to the curve i = x 3 at the origin are real and distinct.
I ADS: false)

Downloaded From : www.EasyEngineering.net


Downloaded From : www.EasyEngineering.net

312 Engineering Mathematics - I

6. For the curve r = a sec B tan B the origin is a cusp.


[Ans: true)
7. The curve x = a (t + sin t), Y = a (1 - cos t) is symmetric about x-axis.

[Ans: false)
8. x - 2a = 0 is an asymptote to the curve l (2a - x) = x 3
[Ans:true)

ww
w.E
asy
En
gin
e eri
ng.
net

Downloaded From : www.EasyEngineering.net


Downloaded From : www.EasyEngineering.net

5w
ww
.EofaIntegration to
Application
Areas, Lengths,sVolumes
yE and Surface areas
ngi
5.1
nee
LAGRANGE'S METHOD OF UNDETERMINED MULTIPLIERS
The definite integrals are useful in formulating important physical applications like

rin
(i) lengths (ii) areas and (iii) volumes and surgace areas of sol ids otrevolution.
5.1.1 (a) The area bounded by a curve y = f(x} , the axis of x and the two ordinates x = a
g.n
and x = b is given by the definite integral.

S:ydx= J:f(x)dx

y
et = f(x)

. x'-~----!--- "'---=--x
o
y'
Fig.5.1.1(a)
Similarly the area bounded by a curve x = fry) the axis of y and the two abscissae
y = c and y = d is given by the definite integral.

S:xdy = S:j(y)dy

Downloaded From : www.EasyEngineering.net


Downloaded From : www.EasyEngineering.net

314 Engineering Mathematics - I

x' ---:,-+----------------,
o x
y'
Fig.5.1.1(b)

ww
(b) If the equation of the curve is in prametric form,
then the area
f
w.E dx dy
l
=
fI,
ll
y - dt or
dt II
2
x - dt
dt
where tl' t2 are the limits of integration depending on the boundary of the curve.
(c)
a
The area enclosed by the polar form of the curve r = f(B) and two radii vectors
(} = a, (} = f3,, is
syE
= fPCd(}=~ 1
fP[r«(})Jd(}
ngi
2
(area of sector OPQ = - r d(})
a 2 2 a 2

nee
rin
g.n
o~~----------------
et
Fig.5.1.1(c)
5.1.2 Example
Find the total area within the curve c?y = c?x2 - x4
,
" Y ,,
Y = - x" " " "",
/ , / 'y =x
...........8
,,
A
x ' ---~---~~~~~~~-~x
,,
,,
,
,,
,
y' ,,

Fig. 5.1.2 --

Downloaded From : www.EasyEngineering.net


Downloaded From : www.EasyEngineering.net

Application of Integration to Areas, Lengths, Volumes and Surface Areas 315

Solution:
The curve has two equal loops between the lines x = a and x = -a and is symmetric
about x axis. The total area within the curve
= 4 (the area of the half of the loop i.e., 'OABO')
= 4 Laydx
ax 1~2--2
=4
fo -va
a
-x dx

=~x_l fa~2_x2j2(_2x}lx
a -2 0

ww ( 2
_ -2\a -x 2 )-2 4(
3
2~-
=--\O-a J2

w
- 3 3
a

.Ea 4
2 o

syE
2
= 3a Sq.units
5.1.3 Example:
Find the complete area of the curve a 2y2
ngi = :x3(2a - x)

y
nee x = 2a

x'
0
B
r ing
x
(0, 0)
A(2a,0)
.ne
Fig. 5.1.3
y'
t
Solution:
The curve is symmetrical about x-axis it cuts the x-axis at the points 0(0,0), A(2a,0).
The curve consists of a loop lying between x = 0, x = 2a
The required area
=20ABO
2
= 2 fo ;dx
2a X 3/ 2.J2a - x
= 2f dx
o a
Substituting x = 2a sin2 ()

Downloaded From : www.EasyEngineering.net


Downloaded From : www.EasyEngineering.net

316 Engineering Mathematics - I

We have dx = 4as in e cose


x=O=>O=O
x=2a=>O=Jr/2

Area =~
a
Jll
0
2 (2a)12 sin 3eEa cose.4asine cose de

= 32a 2 J: 2 sin
4
e cos 2 e de

= 32 a 23.1.1 /2 2
--Jr = 1111 Sq. units
6.4.2

ww
5.1.4 Example:
Find the area of the segment cutoff from the parabolay
y = 4x -1.
= 2x, by the straight line

w
Solution:

.Ea
The given curves are y = 2x
Y = 4x - 1
..... (i)
..... (ii)

syE
ngi
x'-----Ib-~_1_-=-------- nee x

rin
y' g.n
Fig. 5.1.4
Solving (i) and (ii) we get the points of intersection as
e t
J.! -.!}
A(.!2'I}'Lll8' 2
The required area = area AOBA (shaded region)
= area CBEAOOC - area BOADOCB
= J ! {y + 1)dy- J
1
-124
1
-12
y2 dy (from equations (i) and (ii) respectively),
2

~ ~[~2 +yL -[Y:L


9
= 32 Sq. units

Downloaded From : www.EasyEngineering.net


Downloaded From : www.EasyEngineering.net

Application of Integration to Areas, Lengths, Volumes and Surface Areas 317

5.1.5 Example:
Find the area between the curve.xlY = a2(y - x 2) and its asymptotes.

y
B

x' ---j===::;>+-:;::::=::::f-..!..--- x

ww 'y

w.E
Solution:
Fig. 5.1.5

asy
The given equation can be written as

y2{a 2 _X2)= a 2x 2 => y2 = ~2X2 2


En a- -x
The asymptotes to the curve are given by
a 2 _x 2 = O=> x::;:±a gin
The curve is symmetric about both the axes and passes through the origin.
The shaded portion is the required area.
The complete area = 4(areaOAB in the first quadrant)
e eri
= 4 Laydx ng.
=
4J" 12ax
0
va -x 2
dX=-2J" -2ax ,~dx
oi2 2)1/- ~ -x
net
12

= - 2a X
~2
1
2
- X ) a
10 = -4a[:2
-J a - x
2 Jl0I
2
= --4a[O - a] = 4a2 Sq.units.
5.1.6 Example:
Find the area bounded by the curves y = 9x, x 2 = 9y,
Solution:
Solving y = 9x and.xl = 9y
The points of intersection of the two curves are 0(0,0) A(9,9)

Downloaded From : www.EasyEngineering.net


Downloaded From : www.EasyEngineering.net

318 Engineering Mathematics - I

x'----+----~=F=-.:=--~-----x

Fig. 5.1.6

ww The required area = area OBACD


= area ODACO - area OBACO

w.E = S:=oydX - S:=oydx


(parabola y = 9x) (parabola x 2 = 9y)

asy X2
= f 93 J";dx- f9 dx = 27 sq. units
5.1.7 Example:
En 009

Find the area bound by the cissoid x gin


= asin 2t, y =
asin 3 t
cost
.
and Its asymptote.
Solution:
The equation of the curve is e eri
x = asin 2t, y
asin 3 t
= --- ng.
cost

y
x=a
net
x,-----l..~;;;;;;;~~~--x
o

y'

Fig. 5.1.7

a 2 sin 6 t
y = cos 2 t a-x

Downloaded From : www.EasyEngineering.net


Downloaded From : www.EasyEngineering.net

Application of Integration to Areas, Lengths, Volumes and Surface Areas 319

The equation of the given curve is transformed into cartesian form. The curve is
symmetric about x-axis and x = a is the asymptote.
The required area = 2(shaded area)
1/2
=2Jao ydx =2J: _x_
a-x 0 ( )
' dx

Writing x = asin 2 B, dx=2asinBcosBdB

= 4a
2fl! 2Sin. 4ede = 4a 2.-.-.-
3 I n
042 2

ww 4

w
5.1.8 Example:

.Ea
Find the area included between the cycloid x = a(e - sinO), y = a(1 - cosO) and its
base.
Solution:
The equation ofthe curve is syE
x = a(B- sinO) y = a(1 - cosO)
ngi
y
nee
rin
g.n
'y
: e = 2n
A(2an ,0)
e
x ' - - - - - - - - - : : ' I ' -.......~.........'-'-'~.........'---'-:::-...".----- x
e =0
t
Fig. 5.1.8
The required area = area OABO

= f02;, dx
= J2l! Y dx .dO
o dO
= f ;(1 -
0
2
cose )a(I - cose)de

= 4a 2f 2l! Sin. 4-de


e = 8a- fl!·Sin 4-de
?e
o 2 0 2

Downloaded From : www.EasyEngineering.net


Downloaded From : www.EasyEngineering.net

320 Engineering Mathematics - I

=
?
8a-.2 f
0
",2
sin 4 tdt
()
(taking -=t)
2

= 371a2 Sq.units
5.1.9 Example:
Find the area between the curve x = a(O + sinO), y = a(1 - cosO) and x-axis.

ww Solution:
The required area

w .Ea
=

=
2 area OAB

2f"ydx .dO =2f"aQ-cosO)aQ+cosO)dO

=
o dO
2
2a .2
syE
f
0
0

"/2
sin 2 0dO

ngi y

.-------f------. A nee
r ing
x,---=--~~~~~~l--- x .ne
o =-1t 0 0 =0 8
'y t
Fig. 5.1.9
2 I 1t
= 4a .-.-
2 2
= mJ Sq. units
5.1.10 Example:
Find the complete area of the curve given by the equations.
x = acos3 fJ, y = bs in 3 ()

Downloaded From : www.EasyEngineering.net


Downloaded From : www.EasyEngineering.net

Application of Integration to Areas, Lengths, Volumes and Surface Areas 321

Y
8(0, b)

x'---~?-+=~----x
A(a, 0)

Y'

Fig. 5.1.10

ww
Solution:
The required area

w = 4 area OABO
.°Ea
=4Jaydx =4Jo y dx de

=
syE
1t 2 de

4 I:'2bsin3B~3Bcos2BsinB)

= 12ab
1t/2 4
J° sin e cos e
2
de ngi
3 1 1
12abx-.-.-.-
1f nee
rin
=
642 2
3
= 81fQb Sq. units.
g.n
5.1.11 Example:
Find the area of loop of the curve -? = a 2 cos2f)
e t
e =1[/4

x'----~~--------~~------~~~~
e =0
8 x
c

Y'

Fig. 5.1.11

Downloaded From : www.EasyEngineering.net


Downloaded From : www.EasyEngineering.net

322 Engineering Mathematics - I

Solution:
The required area
= 4 areaOABO

= 2f"4~9 = f" 4r2d9


o 2 0

"!4 a2 ,,'4
= fo 2
a cos 29 d9 = -~in29]o
2
a2
2

ww
5.1.12 Example:
Find the area of the curve r a(l + cosO)

w.E
=

Solution:

a syE B

() = 1t
c
ngi A

nee
rin
Required area
Fig. 5.1.12
g.n
=2 . (area OABCO)

=2
f
r2
-d9
"
o 2
et
= f: a Q+cos9 )2d9
2

= J: a
2
4(cos %Jd~ 2

= 4a 2 f"o cos ~9
2
4

"/2
= 4a 2 f cos t .2dt
0
4
(where t = 0/2)

2 3 I 1r 3Jlll 2 •
= 8a ---=-- Sq. umts
422 2

Downloaded From : www.EasyEngineering.net


Downloaded From : www.EasyEngineering.net

Application of Integration to Areas, Lengths, Volumes and Surface Areas 323

5.1.13 Example:
Find the area of the portion included between the carbo ids , = a (l + cosB') and
, = a(l - cosB').
Solution:
Required area
= 4(OCBDO)

=4 [I 1t
0
2,2 d8
2
1
ww 4-~-~r::---_~ r = aQ + cas8 )
w .Ea A 8 =0

syE
ngi
Fig. 5.1.13
= 2
I
1t / 2
0 a-V -cas8 )2d8
1 {,

L1t/2a 2 Q- c~s8 )2d8


n eer
= 2a 2

~!2 +~.~}= a2 (3n -8) Sq.units


2 ing
.ne
2
= 2a { [8 - 2sin8]

5.1.14 Example:
Find the area afthe curve r2 = a2sin28. t
8=~
4

8 =n/4
x

Fig. 5.1.14

Downloaded From : www.EasyEngineering.net


Downloaded From : www.EasyEngineering.net

324 Engineering Mathematics - I

Solution:
The total area of the curve = 2 area of one loop of the curve

= 2 . -1 fll .!2r 2de = f1l 2?


1
a- sin 2ede
2 0 0
2 2
= ~ [- cos28] ~ 2 =~ n+ 1]=a2 Sq . units
2 2
Exercise 5 (A)

1.
x
2
Find the whole area of the ellipse ~ + [;2 = 1
i
ww 2. Find the whole area of the curve x 2(x 2 + .I) a 2(x 2 - .I)
(Ans: 1t ab]

w.E
=
[Ans: cl(Jr - 2)J
3. Find the area of the curve a2x 2 = y(2a - y)

4. a syE
(Ans: Jra2 ]
Find the area bounded by the curve xy = 4el(2a - x) and its asymptote.
(Ans: 4Jra 2 )
5.
ngi
Find the area included between .I = 4ax and y = mx

8a 2
nee
Find the area included between the parabolas .I
(Ans: - 33 )
m
6.
.I = 4b (b - x)
=

rin
4a(x + a) and

g.n (Ans: ~(a + b}Fab )


7.
3
Find the area common to the circle x 2 + y2 = 9 and parabola x2 = 8y et
[ADS: "31 {r;:; .
2,,2 + 27 Sin -1(2J2i}
- 3- ) I

2
3JTa
8. Show that the area of the loop of the curve ely = r(2a - x)(x - a) is -8-
9. Find the area of the loop of the curve r = asin2B.
JTa2
[Ans: - ]
8
10. Find the area common to the circles r = aJ2. - 2acosB
[Ans: el(Jr - 1)]

Downloaded From : www.EasyEngineering.net


Downloaded From : www.EasyEngineering.net

Application of Integration to Areas, Lengths, Volumes and Surface Areas 325

5.2.1 Lengths of plane curves:


(a) The length of the arc of the curve y = fix) included between the points whose
abscissae are 'a' and 'b' is

s=J h (d)2 dx
I+~ or
<J dx

(b) The length of the arc of the curve x = f(y) included between the points whose
ordinates are 'c' and 'd' is

ww d dx
2

w.E s=J
('
1+-
( dy )
dy

asy
(c)
En
The length of the arc of the curve x = fit), y = g(t) included between two points
whose parametric values are 'a' and '/3' is
gin
e eri
(d) The length of the arc of the curve r = j(B) included between two points whose ng.
vectorial angles are 8/ and 82 is

s= J- 9??
r- +( r -
dr
2

dt
net
9, de )

(e) The length of the arc of the curve 8 = j(r) from r = r/ to r = r2 is given by

s= J,,-,. , (de )2 dr
1+ r dr

5.2.2 Example:
Find the complete length of the curve
x2(a2 - x 2) = 8a2 y

Downloaded From : www.EasyEngineering.net


Downloaded From : www.EasyEngineering.net

326 Engineering Mathematics - I

Solution:
y

x'------4--------lIf-------+------ x
A'(a,O) A(a, 0)

'y

ww Fig. 5.2.2

wThe curve is symmetrical about both the axis, one loop is formed in between x

.Ea
and x = a and another loop is formed in between x = 0 and x = - a.
The total length of the curve 4(1ength OABO)
= 0

f: syE
=

J I+( : .... ( I)

ngi
Total length = s = 4 dx

The equation of the curve is


:x2(a2 - x 2 ) = 8a2y2 nee
Differentiating with respect to 'x'
rin
16a 2y dy =2xa2 _ 4x 3
dx g.n
dy _ x(a 2 _2X2)
dx - 8a 2 y
e t
dy )2
1+ ( dx =1+
x2(a2 _2X2
- -
r
64a 4/
Substituting for y2 from the equation of the curve
8a2y2 = x 2(a 2 - x 2 )

Downloaded From : www.EasyEngineering.net


Downloaded From : www.EasyEngineering.net

Application of Integration to Area!, Lengths, Volutnes and Surface Areas 327

9a 4 -12a 2 x 2 +4X4
8a 2{a 2 -x2)

.... (2)

ww
w.E
asy
En
gin
= .J2 2a2 sin- (I}=2ha. Jr
a
I

. 2 e eri
s= .J27«1
ng.
5.2.3 Example:
Find the length of the loop of the curve
3ay2 = x(x - a)2
net
Solution:

8
x'-------*J.U.jtJ.LL~~+_-- X
o
(0,0)

'y

Fit. S.l.3

Downloaded From : www.EasyEngineering.net


Downloaded From : www.EasyEngineering.net

328 Engineering Mathematics - I

The curve is symmetrical about x - axis, the loop is formed between lines x = 0
and x = a, differentiating 3ay = x(x - a)2 w.r., to x.

6ay dy = x.2{x-a)+ {x-a)2


dx

dy = (x-aX3x-a)
dx 6ay

dy )2 (x-a)2(3x-a)2
1+ (- = 1 + -'---------''------'::--::-----"-- ... (1)
36a 2y2
ww dx
Substituting 3ay2 = x(x - a)2 in (I) we get

w.E 1+ (-
dy )2
dx
{x-aY{3x-aY
= I + -'---------''--'--------'--
12ax{x-a)2

asy 12ax+9x 2 +a 2 -6ax

En 12ax

1+ (:r = (3~2+;)2 gin .... (2)

The length of the loop of the curve e eri


ng.
=2 r (3x+a) dx
net
o 2Fa-Fx

s= ~Ja3-Fx+ax-12dx
2,,3a 0

=
1 r:
r::;- L2x
32
+ 2ax
12] a
0
,,3a

= ~ [2a 3/ 2 + 2aFa]= 4~
,,3a ,,3
5.2.4 Example:
Find the perimeter of the loop of the curve 3ay = r(a - x)

Downloaded From : www.EasyEngineering.net


Downloaded From : www.EasyEngineering.net

Application of Integration to Areas, Lengths, Volumes and Surface Areas 329

Solution:

x'-------;~t..J.J,.t..J.J,."'f__:__-- x

ww Fig. 5.2.4

w.E
The curve is symmetrical about x-axis and the loop of the curve lies between x = 0
and x = a.
Perimeter of the curve asy = 20ABO

En
gin
s= 2
f
a (4a-3x)
dx
e eri
o 2J3a.Ja-x
ng.
s = _1-
afj
fa a+3{a -X~lx
0 .Ja-x net
= -1-[-2a{a-x) 2 -2{a-x)'2] ~
afj

= Ir:;- [0 + 2afa + 2a 3!2]= 4a/ fj


a-v3
5.2.5 Example:
Find the length of the arc of the parabola y2 = 4ax cut off by the line 3y = 8x.

Downloaded From : www.EasyEngineering.net


Downloaded From : www.EasyEngineering.net

330 Engineering Mathematics - I

Solution:

x'-----~!OII"I'!J ...------x

Y'
ww Fit. ,.2.6
w
Solving the equations of the J>araboLa anct the fine gives the points of intersections

0(0,0) and A
.Ea
(~: ' 3; )
y=4ax syE
2ydy =4a
dx ngi
dy
-=-
2a
nee
dx y
rin
1+ (dy)2
dx
=1+L
40 2
g.n
The length of the arc OBA of the
e t
s=

. a

= _1 [y ~y2 +4a2+ 40 log~~~y24a2 }T2


2

2a 2 2.. Jo
= a[~~ + IOg2]

Downloaded From : www.EasyEngineering.net


Downloaded From : www.EasyEngineering.net

Application of Integration to Areas, Lengths, Volumes and Surface Areas 331

5.2.6 Example:
Find the length of an arc of the cycloid x = a(t - sin t), y = a( I - cos t).
Solution:
y

ww ------~o~--~a-n--~~a~n----A~------x

w.E (2a1t,0 )

asy Fig. 5.2.6

dx
dt
En
= a(l- cost), dy = asint
dt

gin
(~~J +(~J =~2Q-costY+a2sin2t] 1/2
e eri
The length of an arc of the cycloid
= 2asint/2
ng.
net
= 2 Io" 2asin 7i dt
= 4a[- 2cost/2] ~
= 8a

5.2.7 Example:
Find the total length of the curve
X2/3 y2/3

a 2/ 3 + b 2/ 3 =I

Downloaded From : www.EasyEngineering.net


Downloaded From : www.EasyEngineering.net

332 Engineering Mathematics - I

Solution:

y
B(O,b)

A(a,O)
x'-----E:-~-~------x

B'(O,-b)
y'

ww Fig. 5.2.7

w.E
The parametric form of the curve is
x = acos 3 (J, y = bsin3 0
asy
The total length of the curve = 4. length OABO

En
gin
s=4 0
eer
f1t2[(-3asinecos2e)+~bsin2ecoseJ 2 :\2J12
de

ing
.ne
Substituting
clcos 2o + b 2sin 2 0 = z2
t
and (b 2 - a 2) 2sinO cosO dO = 2z dz

h z 12 Z3/ h
s = 12 fz. ( 2
a ,b -a
2) dz = 2 2
b -a 3 a

~ 4 ~ (b
2 2
= 3 _ a3 )= 4(a + b + ab)
b--a- a+b
5.2.8 Example:
Find the length of the arc of the curve given by x = asin21(J + cos21),
y = acos2t(J-cos2t) measured from the origin to any point.

Downloaded From : www.EasyEngineering.net


Downloaded From : www.EasyEngineering.net

Application of Integration to Areas, Lengths, Volumes and Surface Areas 333

Solution:
x = asin2t(1 +cos2t)
dx
dt = 2acos2t(1 + cos2t) + asill2t( -2sin2t)

dx
dt = 2a(cos2t + cos4t)
dx
dt = 4acos3tcost .... ( I)
and y = acos2t(1 - cos2t)

ww dy
dl = - 2asin2t(1 - cos2t) + acos2t(2sin2t)

w dy
.Ea
dl = 2a(sin4t - sin2t) = 4acos3tsint .... (2)

s
The length of the curve from origin to any point

L (~; r yrE
= +(; dt
ngi
Using (I) and (2)

s= f~ J(4a cos3/COS/)2 + (4acos 31 sin 1Ydl nee


s = f'4a cos 3tdt = sin 3/11
lIa rin
0 3 0

4a . 3 g.n
5.2.9 Example:
s= -Sin I
3
e t
Find the perimeter of the cordioid r = a(1 + cosO).
Solution:

e = 1t 0=0
------~~------~-----------x

Fig. 5.2.9

Downloaded From : www.EasyEngineering.net


Downloaded From : www.EasyEngineering.net

334 Engineering Mathematics - I

dr
r = a( I + cos 0) :=) dB = -a sin 0
The perimeter of the cordiod
=20ABO

= 2 fox 1'2 +(~ J dO

= 2f" Ja 2 Q+ COSO)2 + (- asinO )2dO


o
L" J2a
ww f" = 2
2

4a 2 COS2~ dO
Q+cosO )dO

w.Ef = 2
x
o
0
2

asy = 2 o 2a cos-dO
,..,
X
L

En = 4a.2.sin-
B/
20

gin = 8a
5.2.10 Example:
Find the perimeter of the curve r = 2acosB. eer
Solution:
ing
The equation of the curve is r = 2acosB.
.ne
It is a circle passing through pole whose centre is on the initial line at a distance
8/2 from pole.
t
0=0

Fig. 5.2.10

Downloaded From : www.EasyEngineering.net


Downloaded From : www.EasyEngineering.net

Application of Integration to Areas, Lengths, Volumes and Surface Areas 335

The perimeter of the curve


=20ABO

=2J n'2 r2+(dr)2dB


o dB

= 2 f: ~(211~-;;~OJ ~-(-
12
2asinB)1 dO

ww = 4a-
J(
2

5.2.11 Example w.E


= 2aJ(

Solution: asy
Find the length of the arc of the parabola Ijr = 1 -t cosOcut off by its latus rectum.

En
LSL' = 21 is the latus rectum of the parabola.

gin
e eri
ng.
net
Fig. 5.2.11
The length of the arc L' AL = 2AL

n
(dr)2 dB
=2
Io
;2 1
r+ -
(IB
Equations of the curve is Ijr =1 + cosOc
logr = logl + log(l + cosO)
~ dr = 0 _ - sin 0
rdO l+cosO

Downloaded From : www.EasyEngineering.net


Downloaded From : www.EasyEngineering.net

336 Engineering Mathematics - I

dr
-=rtanB/2
dB
1t/2 ~ 2
The length of the arc = 2
f 0
? ?
r + r- tan-9/2d9
1t/2
= 2
f rsec9j2d9
0

1t2 I
= 2
fo 1+ cos9 .sec9j2d9
ww = 2
1t 2

fo 2cos 9 2 .sec9j2d9
/
2

w.E = I
1t!2

fo 2
9
sec-.sec 9j2d9
2

asy
= / L1t/2 ~I + tan 2 9j2 sec 2 9j2d9
writing tan Bj2
En= t

sec 2 B/2.-dB = dt
I
gin
2
e eri
ng.
net
= 2{~F2 +~log(1 +F2)]

= l[ F2 + log(1 + F2)]
Exercise - 5(8)
1. Find the length of the arc of the parabola y = 4ax cut off by its latus rectum.

[ADS: 2a[F2+log(l+F2)]}

2. Find the perimeter of the loop of the curve 9ay = (x - 2a}(x - 5ai

(ADS: 4.fja J

Downloaded From : www.EasyEngineering.net


Downloaded From : www.EasyEngineering.net

Application of Integration to Areas, Lengths, Volumes and Surface Areas 337

3. Find the length of the arc of the parabola x2 = 4ay from vertex to one extremity of the
latus rectum.
[Ans: a[ 12 +Iog(l + 12)]J
4. Find the length of an arc of the parabola y = x2 measured from the vertex.

[Ans: ~Jl+4x2 +-.!.-sin- I {2x)]


2 4
5. Find that the length of the arc of the curve y = log tanh (x/2) from x = 1 to x = 2

2
I)
ww e +
[Ans: log ( -e- ]

6.
w.E
Find the length of the arc of the curve y = x(2 - x) as x varies from 0 to 2.

J5)+ J5 ]
7. Find the length of the curve x
asy a(B + sinO) y a(l- cosO)
[Ans: -.!.-log(2 +
2

En
= =
[Ans: 8a)
8.
B = 1[/2 gin
Find the length of the arc of the curve x = eOsinB, y = eOcosB from B = 0 to

eer
9. Prove that the loop of the curve
ing
X = (2, Y
(3
= (- - is of length 4fj .ne
t
3
10. Find the perimeter of the cardioid r = a(l- cosO)
[Ans: 8a)
11. Show that the arc of the upper half of the curve r = a(l - cosO) is bisected by
e= 21[/3

5.3.1 Volume and Surface of solids of revolution:


(a) Volume of the solid generated by the revolution of the area bounded by the curve

y = f(x), the x-axis, the ordinates x == a and x == b about the x-axis is J:1t)12 dx.

(b) Volume of the solid obtained by the revolution of the curve x = fry), the y-axis the

abscissae y = c and y = d about the y-axis Ld 1tX


2
dx.

Downloaded From : www.EasyEngineering.net


Downloaded From : www.EasyEngineering.net

338 Engineering Mathematics - I

(c) Volume of the solid obtained by the revolution of the curve x = }(t), y = $(t) about

x-axis is f '2 ny2 dx dl = f '2 n ~ Cr)r dfCr ).dl


'] dt '] dt
(d) Volume of the solid obtained by the revolution of the curve x = j{t), y = ¢i...t) about

y-axis is f'21t\"2 dy dl = f'2n[f'Cr)Y d$Cr).dl


'] dt" dl
(e) Volume ofthe solid obtained by the revolution ofthe cure r = j{ fJ) about the initial line
IS

ww
w .ESa =
OJ
02 f.
n\"sIno
. f\)2 d{rcose )d0
de
.

(f)
syE
Volume of the solid obtained by the revolutuon of the curve r
perpendicular to initial line
= j{fJ) about the line

=
f rrx- dy = 1°2 n \" cose )2 -"-.de
h? dv f. ngi
a 81 dO

0)2
nee
=
f
0]
82 f.
n \" COSo
d{rsine),lO
de
.U'

rin
(g) The volume of the solid generated by the revolution about the initial line (x-axis) of
the area bounded by the curve r = j{ fJ) with the radii vectors B = a, B = {3 is g.n
=
IO=a
O=~ 2nr 3 SIlIO
-
3
'_0 d{\
0
e t
(h) The volume of the solid generated by the revolution of the area about the line
B = ni2 (y-axis) of the area bounded by the curve r = f(e) with the radii vectors
B=a, B={3is
o=~ 2n
=
I -r3 cos OdO
8=a 3

5.3.2 Example:
Find the volume of the solid formed by the revolution of the loop of the curve
y(a + x) = x 2(a - x) about x-axis.

Downloaded From : www.EasyEngineering.net


Downloaded From : www.EasyEngineering.net

Application of Integration to Areas, Lengths, Volumes and Surface Areas 339

Solution:
y = 0 gives x = 0, a
:. The loop is formed between x = 0 and x = a x = - a is the asymptote to the curve.

y
x=-a

ww
w.E
asy Fig. 5.3.2
:. The volume formed by the revolution of the loop about x-axis

En
gin
= 1(
(Jr x2 (a-x)l
ex eer
o a+x
ing
.ne
~ 1{- ~ + ax' -2a'x+2a'log(a+x)I t
= 21(a 3 [ log 2a - log a -1 ]

3
= 2Jra [IOg2-1] cubic units.

5.3.3 Example:
Find the volume of the solid generated by revolving the curve xy = 4(2 - x) about
y-axis.

Downloaded From : www.EasyEngineering.net


Downloaded From : www.EasyEngineering.net

ww
w.E
a syE
ngi
nee
rin
g.n
et

**Note : Other Websites/Blogs Owners we requested you, Please do not Copy


(or) Republish this Material.
This copy is NOT FOR SALE.
**Disclimers : EasyEngineering does not own this book/materials, neither created nor
scanned. we provide the links which is already available on the internet. For any quarries,
Disclaimer are requested to kindly contact us. We assured you we will do our best. We DO
NOT SUPPORT PIRACY, this copy was provided for students who are financially troubled but deserving
to learn. From : www.EasyEngineering.net
Downloaded Thank You and God Bless!
Downloaded From : www.EasyEngineering.net

340 Engineering Mathematics - I

Solution:
y

x' -------,o..-+----f.,-,,;-,~O)r- x

ww Fig. 5.3.3

w The given curve can be written as

.Ea
xcY+4)=8

syE
The volume of the solid obtained by revolving the given curve about y-axis

ngi
= 2n J y+4 )2
OC!

(~
64 nee
o
dy
rin
Substitutingy = tanO ~ dy
dO
= 2sec 2
OdO g.n
e t
1t / 2
= 8n f 0 I +cos2e de

1 ] 1t 2
= 8n [ e +2 sin28 0

= 4n2 cubic units.


5.3.4 Example:
The part of the parabola y2 = 4ax cut off by the latus rectum revolves about the
tangent at the vertex. Find the volume of the reel thus generated.

Downloaded From : www.EasyEngineering.net


Downloaded From : www.EasyEngineering.net

Application of Integration to Areas, Lengths, Volumes and Surface Areas 341

Solution:
y
L(a,2a)

S
x'------------+-M+--+-------------x

J?--+--4.I L'

y'

ww Fig. 5.3.4

w
Volume of the reel generated

.Ea
=2fo 211

[ s'}yY E
1[-
y-
4a

Jf Y-1 ]2<l ngi


= 8a 2 [ 5 0
nee
= --
4Jfa 3
cubic units rin
5.3.5 Example:
5
g.n
Find the volume ofa sphere of radius 'a'.
Solution:
e t
Equation of the circle of radius 'a' is x 2 + Y = a2 .
y

x'------------~~~~~~~~---------------x
B A
(-a,O) (-a,D)

y'

Fig 5.3.5

Downloaded From : www.EasyEngineering.net


Downloaded From : www.EasyEngineering.net

342 Engineering Mathematics - I

By revolving this circle about x-axis we get a sphere of radius 'a'.


a

f
1
Volume of the sphere = ~"rry-dx

= rr
f ll (2
~11 \q - x-' ) dx

= "34 JlU 3 cubic. Ul1lts.


ww
5.3.6 Example:
Find the volume of the solid generated by the revolution of the curve x
3
= bsin () about x-axis.
= (l em;3 (),
y
Solution: w.E y

asy B(D,b)

En
x'
gin (a,D)
x

'y
e
B'(O,-b)
eri
Fig. 5.3.6 ng.
The volume of the solid generated by the revolution of the curve x

y = hsin3 () ((i.e.,) x: ',33 + <~: = 1) IS


net
= aeos 3 (),

a- .h-·
dx
=2fO 21tJ' de
7t
.dB 2

= 2fO rr(bsin e y~3acos2esine )de


3
11/2

32JlUh 2
- - - cubic units.
105

Downloaded From : www.EasyEngineering.net


Downloaded From : www.EasyEngineering.net

Application of Integration to Areas, Lengths, Volumes and Surface Areas 343

5.3.7 Example:
Find the volume of the solid fonned by the revolution of x = a( e- Sillf/),
y = a(l - cosf/) about its base.

Solution:
y
o = n/2

ww
w.E
a syE Fig. 5.3.7
Volume obtained by the revolution of the area OABO about the base (x-axis).

ngi
= n
f
O=21t dx(10.
y2- nee
()=o dO
rin
= na
3f8=21t1J
8=0 ~ - cosB
)~
dB
g.n
3 J 21t
et
= na" .8 2sin 6 O/2dO
0=0

(e12 = t => dB = 2dl)

= 16na3.2 J1t 2 sin6 tdt = 32u3n(6 -I 6 - 36 - 5 .n I 2)


o 66-26-4
= 5,(2a3 cubic units.
5.3.8 Example:
Find the volume generated by revolving the curve r = a( 1 - cosq) about the initial
line.

Downloaded From : www.EasyEngineering.net


Downloaded From : www.EasyEngineering.net

344 Engineering Mathematics - I

Solution:

O=~
2

o = It
A

ww Fig. 5.3.8

w.Ef1t
The volume generated by the curve

= 2n r3 sin8 d8

f1tas
3 0

yE = 2n
3 0
a 3 Q-cas8)1 sin8 d8

J1t
ngin 3
1- cosO = t
sin OdfJ = dt
1
y =
eer
2na
--
3 0
Q-cas8 sin8 £18 [
and():O~ 2

ing
.ne
8Jfa 3
= - - cubic units.
3 t
5.3.9 Exam'pre:
Find the volume orthe solid formed by the revolution of the curve r = a + hcosO
(a<b) about the initial line.
Solution:

Volume =2n
- f1t r3 sin8d8
3 0

= ~
')n f1t {,a+hcos8 )3 sin8d8
3 0

(a+bcOS()=t ~ sint1f() = - ~ ,t :a+b ~ a-b)


Downloaded From : www.EasyEngineering.net
Downloaded From : www.EasyEngineering.net

Application of Integration to Areas, Lengths, Volumes and Surface Areas 345

= 21t
3
J U

u+h
-
h
p(_ dt)
b

= - 21t SU-h t3d!


3b u+b

=- ~~[,;[
= ~[(a+bt -(a-b)4]
ww 6b

41l'Q(2 2)
-3-\a + b cubic units.
w.E
=

5.3.10 Example:

a
Find the volume of the solid generated by revolving the lemniscate?

2 sy
tr
= a2 c:os2B

about the line B=


En
Solution:
gin
e eri
ng.
net
Fig. 5.3.10
The curve is symmetric about q = tr/2

Volume = 23. 1t I1t 4,3 case de


3 0

4 1t 3
= ~ S4(a 2COS2e)2 cosede
3 0

= i3 1ta 3
I1t4 (cos2e l2 case de
0

Downloaded From : www.EasyEngineering.net


Downloaded From : www.EasyEngineering.net

346 Engineering Mathematics - I

(taking .J2 sinO = sint => .J2 cos(}dO = cos/dt)

ww - 2.J27lZJ3 [4--I
3
--
4 4-22
7C]
4 --3 -

w.E .J27C 2a3


- - - cubic units.
8
asy
En Exercise - 5(C)

~: = I
1.
gin
Find the volume of the solid generated by revolving the clips :: x

(a) About the major axis


e eri
(b) About the minor axis ng.
[ADS:
net
4 2
37lZJ b J
2. The curve yea + x) = r(3a - x) revolves about the x-axis. Find the volume generated
by the curve.
[ADS: ~(8Iog2 - 3)J
3. Show that the volume of the solid generated by the revolution of the curve (a -x)y

7C 2a3
= a2x, a~out its asymptote is -2-.
4. Find the volume of the solid generated by the revolution ofthe curve y(a2 + x2) = a3
about the asymptote

Downloaded From : www.EasyEngineering.net


Downloaded From : www.EasyEngineering.net

Application of Integration to Areas, Lengths, Volumes and Surface Areas 347

5. Find the volume of the solid obtained by revolving the loop of the curve
1
a 2 == x 2(2a - x)(2a - x)(x - a) about x-axis.

(Ans:
60
6. Find the vloume generated by the portion of the arc y == JI + x" lying between
x ~ 0 and x = 4 as it revolves about the x-axis.
(Ans: 767[/3)

7.
ww
Show that the volume of the solid generated by the revolution of the cycloid

. . . 3 , 1 87['

8.
x ==

w.E
a( 8 + SinO), Y == aU -- cosO) about the y-axIs
IS - 7[" £l - -
2 3
Find the volume of the solid generated y revolving the cycloid x = a(8 + sinO),
y = aU + cosO) about its base.
asy IAns: 5~£l31
9.
En
Find the volume of the solid generated by revolving r = a2cos28about initial line.

gin (Ans:
7[(1' ( h + 1)-"\/2h] I
Ii rl3log"\/2
10. Find the volume of the solid generated by revolving r e eri
=
12
acos38 between 8 == -7[/6
and e == 7[/6 about the initial line.
ng.
5.4.1 Area of the Surface of revolution
(Ans: net197[(1'
968 )

(a) Surface area of the solid generated by the revolution about the x-axis of the area
bounded by the curves y = fix), the x-axis the ordinates x = a, x = b is

h
21ty £Is ==
Ii> ds
21ty - £Ix
I X=Q X=U dx

(b) Surface area of the solid generated by the revolution about y-axis of the area bounded
by the curve y = fix), the y-axis and the abscissae y = c, y == £I is

I
Y=J 21tX cis == IJ
X=Q Y={
cis
21tX -
dy
(~V
[ d~==
dy
1+(dX)2]
dy

Downloaded From : www.EasyEngineering.net


Downloaded From : www.EasyEngineering.net

348 Engineering Mathematics - I

(c) The parametric form is

f
'=/2

/=/1
ds
.21ty-dt
dt [ d~
dt
== (dX)2 + ({~y)2l
dt dt

(d) The polar form is

I - 21trsine (dS-de )de


O?
91

5.4.2 Example:

ww ? '0
Find the surface area of solid generated by revolving the curve x-i-' + y'? '3 == a-I?',

w
about x-axis.
Solution:
.Ea
The parametric form of the curve is x acos3 e, y asin 3 B.
syE
= =

ngi B(O,a)

x'
A'(-a,O)
nee x

rin
Y'
8'(0,-a)
g.n
Fig. 5.4.2
The surface area of the solid due to revolution about x-axis
e t
== 2(surface area of the solid generated by revolving an arc in the first
quadrant of the astroid is)
" 21ty-.de
ds
f° de
2
= 2

41t f Y - ) +(2) .de


?
dx -
lt/2 d
=
° de' de (

asin 3 e ~(\.3acos 20 sine )2 +( 3asin 2 e cose )2de


= 41t
I0
It/2

Downloaded From : www.EasyEngineering.net


Downloaded From : www.EasyEngineering.net

Application of Integration to Areas, Lengths, Volumes and Surface Areas 349

"n
= 12na 2
J
0' - sin-le cosO dO

ffn
2sinSe - 12JlU 2
= 12JlU [ - 5 - j0 = - 5 - Sq. units.

5.4.3 Example:
Find the area of the surface formed by the revolution of the ellipse x2 + 4y = 16
about its:
(a) major axis (b) minor axis

ww Solution:
2 2

w
Equation of the ellipse is

ely.Ea x
~ +L
16 4
= I

dx 4y
syE
ngi
16y2
nee
rin
+X2

16/
(a) Surface area formed by the revolution about major axis
g.n
= 2
o
-l
J
ds
2ny-dx
dx e t

Downloaded From : www.EasyEngineering.net


Downloaded From : www.EasyEngineering.net

350 Engineering Mathematics - I

ww
.Jl[~(~ r = -x' + *
~: Sin-{ )I
w = Jl] .Ea 8"[1 + 4"9 Sq. units.

(b)
syE
Surface area formed by the revolution about minor axis

ngi
nee
= 41t L2 J Xl + 16yl dy r ing
= 41t J: J16-4 y 2 +16y dy
2
.ne
= 8Jl. s: (1 r + y' dy
t

Downloaded From : www.EasyEngineering.net


Downloaded From : www.EasyEngineering.net

Application of Integration to Areas, Lengths, Volumes and Surface Areas 351

5.4.4 Example:
Find the surface area formed by revolving cycloid x = a(B+ sinO),y = a = (/ - cosO)
about the tangent at the vertex.

Solution:

e =-1[
y e =-1[

a1[ a1[

ww 0=0

w.E
x' - - -.........-.....;~"""'"--..I---- X

y'

a syE Fig. 5.4.4


Surface area required
ngi
= 2(surface area generated by revolving the arc

2
I
x ds
21ty- dB
nee
in the first quadrant about OX)

rin
=
o de
= 4 IX (dX)2
de
+(dy)2 de
de g.n
et
II 1t)'

=161t a-. IX sm-


. -cos
e -e de
ry ?

o 2 2

(%=t => dB = 2dt)


= 321ta- ry I 0
X/2
sin 2 t cost dt

32JlU 2
= -- Sq. units.
3

Downloaded From : www.EasyEngineering.net


Downloaded From : www.EasyEngineering.net

352 Engineering Mathematics - I

5.4.5 Example:
Find the area of the surface of reel thus generated by revolution of the part of the
y
parabola = rax bounded by the latus rectum about the tangent at the vertex.
Solution:
y

x'------+-~-+_------ x

ww
w.E
y = asy
4ax
Fig. 5.4.5

dy 2a
En
dx y
gin
Surface area = 2
fo
a ds
21tX-dx
dx eer
d )2 dx ing
=2fa 21tX 1+.1:'.
o ( dx
.ne
t

= 4n fo" x.J x 2 + ax dx
a

= 4n J:[(x+~r -(~rl2dx

Downloaded From : www.EasyEngineering.net


Downloaded From : www.EasyEngineering.net

Application of Integration to Areas, Lengths, Volumes and Surface Areas 353

= 4lT

o
2
= JZa [3J2 -log(J2 + 1)] Sq.units.

5.4.6 Example:

ww
Find the area of the surface of revolution formed by revolving the curve r
about the initialline~
= 2acosB

w
Solution:
Equation of the circle is
.Ea
r =

dr= -2
-
2acosB

' B
asm
syE
dB
y
ngi
nee
x'-------~I-----
o ....--xrin
g.n
y' e t
Fig. 5.4.6

The surface area = J" 21ty -dO dO


o
2 ds

2
= L lIi2 dr
21trsinO r2 + ( dO )
dO

= J:/ 21trsinOJ4a 2cas 0 + 4a sin 0 dO


2
2 2 2

"/2
= 81ta J sinO cosO dO
2
0

= 4mr Sq. units

Downloaded From : www.EasyEngineering.net


Downloaded From : www.EasyEngineering.net

354 Engineering Mathematics - I

5.4.7 Example:
Find the surface area of the solid formed by the revolution of the cardioid
r= a(J + cosO) about the initial line.

Solution:
Equation of the curve r = a(J + cosO)

dr . B
-=-asm
dB

ww
w .Ea
0=0

syE
ngi
The surface area = Io"2ny-de ds
Fig. 5.4.7
nee
L" rsine
de

J r ing
= 2n

I:
r2 +(* de
.ne
= 2n

= 2n L"
2
rSineJa (i +cose y 2
+a 2sin e de

a(i + cose)sine .Ja 2(i + cose)2 + a 2 sin 2e de


t
= 16na 2 I"o COS3~2 sin ~2 cos ~2 de
= 16na 2 I" cos "2e sm"2
0
4 . e d8

= [25 2eJ"
16a 2n - - cos 5 -
0

32mi
= -- Sq. units
5

Downloaded From : www.EasyEngineering.net


Downloaded From : www.EasyEngineering.net

Application of Integration to Areas, Lengths, Volumes and Surface Areas 355

5.4.8 Example:
Find the surface of the solid generated by revolving the lemniscate,.2 = a 2cos2B
about the initial line.
Solution:

ww
w.E
a syE Fig. 5.4.8

Equation of the curve is ,.2 = a 2cos2B ngi


Surface area = 2
" ds
fo 21ty-dO
4
nee
dO
rin
41t In 11/4 dr
rsinO r2 + ( dO )
2
dO
g.n
= 41t
f
1[/4
0 rsinO
~ a4 • ~
r- +-~ sm-20 dO
et
r-

= 41ta 2 [_ cosO ] ~ 4

Downloaded From : www.EasyEngineering.net


Downloaded From : www.EasyEngineering.net

356 Engineering Mathematics - I

Exercise - 5(0)
? ?

I. Find the total area of the surface obtained by revolving the ellipse x~ + y~ = 1 about
a- b-
its major axis.

2. Find the surface ofa sphere of radius 'a'.


(Ans: 4Jlu 2 ]

ww
3. Find the surface of the solid generated by revolving the arc of the parabolay
bounded by its latus rectum about x-axis.
= 4ax

4.
w .Ea (Ans:
3
Find the area of the surface generated by the revolution of the cycloid x
~7lU2(2J2 -I)J
a(t - sint),

syE
=
y = a(l - cost) about x-axis.

ngi [Ans:
64
3 7m )
2

5.
nee
Find the area of the surface of the solid formed by the revolution of the cardiod
r = a(l - cosO) about the initial line.

rin (Ans:
32
SJl"a )
2

6.
surface area generated is 47lU2 •
g.n
The lemniscate? = a2cos2B revolves about a tangent at the pole. Show that the

5.5.1 Double and triple integrals: e t


Letj(x,y) be a continuous and single valued function of x and y within a region
R bounded by a closed curve 'c' and upon the boundary c. Let the region R be
subdi vi doo in rrty mrrtne- into n subregi ons of areas 8R\, 8R2 .... 8Rn •
Let (x" y,) be any point in the subregion of area 8Ri .
Consider the sum

The limit of this sum as n ~ 00 (i == 1,2, .... ) is defined as the double integral ofj(x,y)
over the region R and is written as
fff(x,y}iA

Downloaded From : www.EasyEngineering.net


Downloaded From : www.EasyEngineering.net

Application of Integration to Areas, Lengths, Volumes and Surface Areas 357

Hf(x,y}ixdy
II
= Lt
n~oo ,=\
I f(x" yJ) R,
(a) Suppose the region R is described by the inequalities
c ::::; y ::::; d and g(y) ::::; x ::::; heY)

y
y=d
...--_ _.,....::--....f-

x = g(y) X = h(y)

y-c
x'---------+---------
ww o x

w.E Y'

Then
a syE
Fig. 5.5.1 (a)

t ="
ff f(x,y )dydx=Jff(x,y )dxdy = f fX=II(Y)f(r,y)dXdy
II II
ngi Y~
x=g(y)

(b) Jfthe region R is described by inequalities


a ::::; x ::::; band g, (x) ::::; y ::::; h,(x) nee
y
rin
g.n
x ' - - - - -.......
et
oof----I----""----x
y'

Fig.5.5.1(b)
x=h y=lI\ (.)

Then ff r(x,y}ixdy = fff(x,y}iydx = f f y=g\ (.) f(x,y)dx.dy


II II x=a

(c) If the region R is bounded by the lines x = a, x = b, y = c, y = d (rectangle)


h " " h
Then Hf(x,y}dxdy = f ff(x,y}dydx= f ff(x, y}ix dy
II a c ( a

Note: The order of integration is immaterial for constant limits

Downloaded From : www.EasyEngineering.net


Downloaded From : www.EasyEngineering.net

358 Engineering Mathematics - I

5.5.2 Example:
I 2
2
Evaluate I I{x + y2 }Ix dy
o I

Solution:

ww =
8
03 I I
-+2y 2 ---y
3
2dy

w.E il
2 Y7 7 I
Y +- dy=-+-yl
J
3

asy o
3 3 3 0

En =
1 7 8
-+-=-
333
5.5.3 Example:
gin
Evaluate
4

I
K-~

Ixydydx e eri
Solution:
o

ng.
4

I
I
J4-x

0
fxydydx = f·
4

x=l
I'=J4-x

f [xydy] dx
0
net
4 y=K-~

f fxydy.dx
x=l 0

2
= 4f~(4 -X}tx = 4x _~14 9
2 4 6 I 2
x=l

Downloaded From : www.EasyEngineering.net


Downloaded From : www.EasyEngineering.net

359
Application of Integration to Areas, Lengths, Volumes and Surface Are."'''

5.5.4 Example:
1 2- r

Evaluate J Jx dXdY
o J~'
2

Solution:

v=1

J1(2- Y )' -l(J.i~)\~v


ww y~O

J8 - -
)'=1 3

w .Ea
= -1
3
\'~O
V1 v2
-12)1 + 6- V- l IV
--') ~

syE
ngi
1[S-1-6+2-lJne
=

eri
67
60 ng.
5.5.5 Example:
Find the value of Hxy(x+ y}/xdy taken over the region enclosed by the curves
net
y = x and y = x2.
Solution: y
y=x

x' _ _ _ _ _ ~~:-:.....----- x

y'

Fig. 5.5.5

Downloaded From : www.EasyEngineering.net


Downloaded From : www.EasyEngineering.net

360 Engineering Mathematics - I

7
R is the rogion bouuded by the curves y = x and J' = x~

1= ff'"y(x + y')dxcly
II

ww x5 x5 x7 x8 I
w.E = 10+]5-14- 24 10

asy
=
I I
10
I 1
-+-----
15 14 24
=-
3
56
5.5.6 Example:

IfR
En
Evaluate
A 1- Y gin
dx{1y where A is the area in the positive quadrant of the circle

x2 + y2 = 1.
e eri
Solution:
y ng.
net
x'----------~r-~~~~~~~------x
8(1,0)

y'

Fig. 5.5.6

Downloaded From : www.EasyEngineering.net


Downloaded From : www.EasyEngineering.net

Application of Integration to Areas, Lengths, Volumes and Surface Areas 361

ww ±fx[~ -/'2-~--] £Ix


= -
_I ~ 1- (2

w.E I
o

fX{I[-Q-X 2)]2 -I}dx


I
()

asy = --
o

f{x x}It E
I

ngi
= -
o
2
-

- [
n
-x 3 X-J ]1
- --+-
3 2 () eer
-I 1 1 ing
= -+-=-
3 2 6
.ne
5.5.7 Example:

Evaluate ffe
tlHhY
dt dy over the triangle x = 0, y = 0, ax + by = I
t
Solution:

x:: : Ol----t---"
x'------------~----~~----~~----x
~ AU,a)
Fig. 5.5.7

Downloaded From : www.EasyEngineering.net


Downloaded From : www.EasyEngineering.net

362 Engineering Mathematics - I

1 = ffea<+hY dx dy

1= I I'"
,=0
lY: '-S-f:/\
e
y=o
j
+hI' I
-(Y(X
I

ww
w.E-;-R~ ,(I-a<) 1
asy = _
I
bo
e
llX+) -----
h _e lLt +O dx

En
gin
eer
ing
.ne
= ~[.:.
b a a
-.:. - 0 + ~l
a
t
ab
5.5.8 Example:

Evaluate ffydxdy where R is the region bounded by the parabolas y = 4x and


R

xl = 4y.

Downloaded From : www.EasyEngineering.net


Downloaded From : www.EasyEngineering.net

Application of Integration to Areas, Lengths, Volumes and Surface Areas 363

Solution:
y

x'--------~~--~------x

ww y'

w .Ea
Solving the curves i' = 4x, x 2 = -/y
Fig. 5.5.8

X2 )2
-
syE
=4x
( 4
ngi
x= O,x= 4
(0,0), (4,4)
nee
1= ffydxdy
11 rin
g.n
e t

48
5

Downloaded From : www.EasyEngineering.net


Downloaded From : www.EasyEngineering.net

364 Engineering Mathematics - I

5.5.9 Example:

Evaluate ff(f; - y2 }iydx where R is a triangle with vertices at (0, 0), (10, )),
II

(I, I).

Solution:

ww =1
w.E 8(1.1) Y
,....-.,------""!:II... A(10.1)

asy________________---------x
x· ____________ ~

En y'
0(0.0)

gin
eer Fig. 5.5.9

ing
The triangle OAB is within the limits y ~ x ~ lOy and 0 ~ y ~)

II
- J J(FxY -
ff(J~ y dydx) = 2
I

y=o x=y
.n
x~IOy

y2)dxdy
et
JelY ~~~y- /y
I [ 3 ]IOY
=
o y Y

Downloaded From : www.EasyEngineering.net


Downloaded From : www.EasyEngineering.net

Application of Integration to Areas, Lengths, Volumes and Surface Areas 365

Exercise -: 5(E)
I. Evaluate the following double integrals:
2 2

1. f f(x 2 + y2 ~xdy
0 0

32
IAns: - I
3
3 x+2

f f~vdx
2.
ww
-I x2

w.E IAns:
8
-I
3

3.
} Jdxdy
J 4
J
asy
1 3 (x + y)-
En
gin 25
IAns: log-I
24

4.
II

f
~1I2_x2

fidydx
e eri
o 0
ng.
II h
net
25(/5
(Ans: -15- 1

5. f f(x 2+ y2 )ixdy
o 0

ah (J
IAns: 3\U- + h- ]
J)

3Jru 4
[Ans: --I
4

Downloaded From : www.EasyEngineering.net


Downloaded From : www.EasyEngineering.net

366 Engineering Mathematics - I

a 2a-x

8. f fxy{O;dx
o );2

ww 3a 4
[Ans: - ]
8

w.E
9. Find the value of ffxydxdy taken over the positive quadrant of the ellipse
J J

~: + ;~ = I

asy
a Ja 2 -y2 En
10. f f ~a2 _x 2 - 2
y dxdy
gin
o 0

e eri 7fa
(Ans: - ]
3

x=1 y=x y ng. 6

II. f
x=o y=O
f eX dxdy
net e-\
[Ans: - )
2

12.

[Ans: 7f log(.J2 + 1)1


4

13. j fSin(x+ y)dxdy


x 0
o
[Ans: I]

Downloaded From : www.EasyEngineering.net


Downloaded From : www.EasyEngineering.net

Application of Integration to Areas, Lengths, Volumes and Surface Areas 367

14. Evaluate H-,ydxcZV where R is the quadrant of the circle x 2 + y2 = {/2 when

x~O,y~O.

{/4
[Ans: -I
8

15. Evaluate Hxydx((v whose R is the region bounded by x-axis, ordinatex ~ 2a and the
?
curve x- = .Jay.

ww [Ans: 3
a4
1

16. Evaluate w
H(x
.E + )')d'({~}' whose R is the triangular region bounded by y = 2x, y = ~
and y =
/I
3 ~ x.
asy
17. Evaluate Hxydx(~Y
En
where R is the triangular region with vertices A(~6, 2), B(~ 1,3)
and C(9, ~7).

gin ~1025
----:n- 1
e eri
[Ans:

~ 1
18. Evaluate Hxydn'y over the region in the positive quadrant for which x + y
ng.
5.5.10 Change of variables:
netI
[Ans: 241

The variables x, y in Hf(x, y ~/xdy are changed to It, v with the help of the relations
II

x = j(u, v), y = filt, v) then the double integral is transferred into

fIt!; (11, v)f2 (It, v)]!JI}illdv


1/'

dx dx
du elv
where J = dy cZY and 'R' is the region in the uv plane corresponding to the region
du dv
R in the xy plane.

Downloaded From : www.EasyEngineering.net


Downloaded From : www.EasyEngineering.net

368 Engineering Mathematics - I

5.5.11 Changing from Cartesion to Polar co-ordinates:


x = reosO, y = rsinB
dx (be
J = dr dO =ICOSB -rsinBI =r
dy dy sinO rcosB
---
dr dB

Jff(x,y)dxdy = Jff~·cos9,rsinO] /Jld9dr


/I /I'

ww = Jff[rcosO,rsinOp'dltlr
/I'

w.ENote: In polar form dxdy is replaced by rdBdr


5.5.12 Example:
Evaluate
asy
Jf~ a 2 - x 2 - y2 dxdy over the semi circle xl + Y = ax in the positive
quadrant.
En
Solution:
Substituting x = reose,
Y
g ine
y = rsinB, in xl +
,.2eos2 0 + ,.2sin2 0 er =
= ax
an'osO
which is the equation ofthe given circle in polar co-ordinates.
r =

ing
aeosO

.ne
y
t
x' --------~I'-.-lo._~-_+-_- X
o A(a,O)

Y'

Fig. 5.5.12

From the figure it can be sedn that


r = OP = aeos8

Downloaded From : www.EasyEngineering.net


Downloaded From : www.EasyEngineering.net

Application of Integration to Areas, Lengths, Volumes and Surface Areas 369

for the upper halfofthe circle, Ovaries form 0 to "27T where as for any intermediatary
value of 0, r varies from 0 to or i.e., () to acm·O.
1[

2 acosO,--_ _ _ __

JJ~a2-x2-idx(ZV= f fJa2-r2cos20-r2sin20rdrdO
II I) ~O r~O

where R is the shaded region.

ww 1t

w.E = ~ f~
3 2

3 0
-sin 0)dO
2

asy
En
gin
5.5.13 Example: eer
{/ I 2 J
\ a - y- ing
Evaluate f
o
f{x 2 + y2 }Ix~v by changing into polar coordinates.
0 .ne
Solution:
From the given limit x = ~ a 2 - i it is clear that one of the boundaries is the circle x 2
t
+ y = (r, taking x = rcasO, y = rsillO. The given rigion is the first quadrant of the
7T
circle. Here r varies from '0' to '(I' and 0 varies from '0' to "2.
1t

£l Ja 2 -y2 2 a

f f~2 + y2 )dxdy = f fr2.rdrdO


o a O~~O

1t

Downloaded From : www.EasyEngineering.net


Downloaded From : www.EasyEngineering.net

370 Engineering Mathematics - I

(14 1l
=-x-
4 2

8
5.5.14 Example:
(2 2)
ww
00 W

Transform the integral I Ie - '+Y dxdy to polar coordinates and evaluate it.
o r=d

w Solution:

.Ea
The limits of x and yare both from 0 and' 00 '. Therefore the region is in the first

syE
1l
quadrant where r varies from '0' to ' 00 ' and B varies from '0' to 2
Substituting x = rcosB, y = rsinB, and dxdy = rdrdB.

n 1t

gin
1t
e eri
1t

=
2 [2jW
I --2
e-r
de =-
1 ?-
J'
1
ng.
rde =-[e]
1t
2

o 0
2
0
2 0
n et
4
Changing of order of inte8,ration:
The double integration can be integrated with respect to y first and then with respect
to c or it can be integrated with respect to 'x' first and then with respect to y'. In the
former case, the limits of integration are determined for the given region by drawing
strips parallel to y-axis while in the second case by drawing strips parallel to x-axis.
5.5.15 Example:
, a 2a-x
Evaluate by changing the order of integration I Ixydx«:v
o "
a

Downloaded From : www.EasyEngineering.net


Downloaded From : www.EasyEngineering.net

Application of Integration to Areas, Lengths, Volumes and Surface Areas 371

Solution:
2
The given limits are x:O~a,y:~~2a-x
a
2
i.e., the region is bounded by x = 0, x = a and y = ~ y = 2a - x i.e., x + y = 2a.
a

ww
w.E
x' _ _ _ _ _ _.-.."""'....._ _ _ _ _---::a...-_ _ X

asy Y'

a 2a-x

J Jxydxdy En Fig. 5.5.15

o x2/a
=
OA(,O gin
Hxydxdy + Hxydxdy
('AIl(,

y=a x=[:ry

J
y=2ax=2a-y

J
e eri
= Jxydxdy + Jxydxdy
ng.
net
a 1 2(/
= ~ Ji dY+2" JY(2a- yydy
o (/

5.5.16 Example:

Change the order of integration and evaluate


aH
J J(x 2
+ y2 }txdy
o x
a

Downloaded From : www.EasyEngineering.net


Downloaded From : www.EasyEngineering.net

372 Engineering Mathematics - I

Solution:

The given limits are x = 0 x = 0 and y = ~a and y = .[; i.e.,


a
ay = x.
y

x'------------~~-------------x

ww Y'

w.E Fig. 5.5.16

f 2J~2asy2)1xdy
+ yE=:tEX2 y~dx}y +
a ngi
nee
f ~3 i
r ing
=
I (

o
3
- a 3 l + ai - ay4 } y
3 .ne
a
=-+-
a
28 20
3
t
5.5.17 Example:
a 2&
Evaluate f fx 2dxdy by changing the order of integration.
o 0
Solution:
The given limits are x = 0, x = a and y = 0, y = 2.[;;; i.e., y = 4ax.

Downloaded From : www.EasyEngineering.net


Downloaded From : www.EasyEngineering.net

Application of Integration to Areas, Lengths, Volumes and Surface Areas 373

y x=a

x' -----...",..jl--""-::-I------ x

ww
w
y'

.Ea Fig. 5.5.17

=
I
3" f
2a [ 3
a - sy]3]E [ 7
[')
Y
4a dy =
I
3"
3 I
a y - 64a 3 x
Y7]2(1
0

ngi
nee
5.5.18 Example:

Change the order of integration and evaluate


ri f yfx
4 4

2 X 2 dxdy ng.
.

Solution:
o +y
n et
y x=4

x'------..,t"f'---~~--- x

y'

Fig. 5.5.18

Downloaded From : www.EasyEngineering.net


Downloaded From : www.EasyEngineering.net

374 Engineering Mathematics - I

The given limits are y = °y = 4 and x = y, x = 4.

4 4
r -I dx= I7r-dx=-7r [x ]4 =7r
= ian

ww
0
o 0 4 4
5.5.19 Example:

w.E a '22
vaL _yL

asy
Evaluate by changing the ord¢r of integration f
o
fxydxdy
0

Solution:
En y
gin
x'------------+---~+_~--~-----------x
e eri
ng.
y' net
Fig. 5.5.19

The given limit are y = 0, y = a and x = 0, x = ~a2 _ / i.e., x 2 + y2 =a2

2l~ dx
xL
X
o 2 0

Downloaded From : www.EasyEngineering.net


Downloaded From : www.EasyEngineering.net

Application of Integration to Areas, Lengths, Vo'lumes and Surface Areas 375

ww 8

w.E Triple integration:


Let u = j(x,y,z) be a single valued function of the independent variables x, y, z defined

asy
through the region V. Divide the region V into 11 subregions t5V I' t5V 2 ..•. t5V nand
P(x l , YI' z) be any point inside or on the boundary of the subregion t5V /' then the sum

under the limit Lt En II

If(x"ypzJdv, is defi·ned as triple integral.


11 ~oo ,~I

II gin
Lt
11 ~ 00
If(xpy"z, )dv,
,~I
=
e
HJf(x,y,z}1v
/'
eri
5.5.20 Example:
\ z x+z

JJ J(x + y + z }ixdyd::. ng.


Evaluate

Solution:
-I 0 x-z net
\ z x+z

I = Jdz Jdx J(x + y + z }ixdydz


-I 0 x-z

\ z
Jaz fi2z(x + z)+ 4xz}ix
-\ 0

Downloaded From : www.EasyEngineering.net


Downloaded From : www.EasyEngineering.net

376 Engineering Mathematics - I

JZ 3 + 2z 3 + 2z 3dz
-I

1 [ 4]1
=SJ.::\lz=S 24 =0
-I -I

5.5.21 Example:

ww J J
Evaluate
2 r x+y

JeX+Y+ZdXdydz
w.E 000

asyH'LT+[[e'.(e' r
Solution:

HJe'[J'dZ En
t"dy

gin }IY}b: fe<[ feY{e-HY -\

eer
o 0

ing
.n et
2 4<
e - 3 7< +e-rdx
J---e--
o 2 2

Downloaded From : www.EasyEngineering.net


Downloaded From : www.EasyEngineering.net

Application of Integration to Areas, Lengths, Volumes and Surface Areas 377

5.5.22 Example:

Evaluate

1 1

I= f f[xz t~dnly
\'=0 r::: 1,2

ww
w
1 ) 1 \=1

f .~ - X3 dy

.Ef (Ia-
\'=-0 t:..::y2

y=()
1

6 syy6lE
y4
_0_+-
2
dy
3

~ [~< +~;I 3~ n gin


5.5.23 Example:
e eri
Evaluate ng.
Solution: n et

4[-~+-S'11 ]2.[; dz
f
x 42 2 ° ~1 X
{-}
z=O 2 2 J4; <=0

4In::dZ [z22rJo
= 1f

=81f

Downloaded From : www.EasyEngineering.net


Downloaded From : www.EasyEngineering.net

378 Engineering Mathematics - I

5.5.24 Example:
Evaluate HJryzdzdydx over the volume enclosed by three coordinate planes and the
I'
plane x + y + z = I.
Solution:
The plane x + y + z = I meets the coordinate axes in A( I ,0,0),8(0, I ,0) and C(O,O, I).
z
C(O,O,1)

ww
w .Ea
x

syE Fig. 5.5.24

WX;adZdyd!~ ,1, :I('JpIz }!YdX ngi


1(2 ~ r' }n nee
I I-x
XY[ dy
r ing
= JJ X
;[(I-X- y )2_ 0 }/YdX
.ne
x=o y=o

= ~
I I-x

J JXy(I+X2+y2_2X-2y+2Xy~ydx
x=O ),=0
t

Downloaded From : www.EasyEngineering.net


Downloaded From : www.EasyEngineering.net

Application of Integration to Areas, Lengths, Volumes and Surface Areas 379

5.5.25 Example:

Evaluate ff~X2 + i + z2 }Izdydx: where V is the volume of the cube bounded by the
I'

coordinate planes and the planes x = y = z = {[,

Solution:
z

ww
w.E
asy
y

Fig. 5.5.25

ff~x2 En'f 'f 2 )Iz(zvch


I'
gin
+ y2 + z2 )lzdydx =
x~o \'=0
'f(x + y2 + z2
Z~O

e eri
ng.
net

Jra-))
(/ 4 4
a a
+-+-dx
x=o
3 4

= [ ~' a' + ';' x+ ~' x I


=d

Downloaded From : www.EasyEngineering.net


Downloaded From : www.EasyEngineering.net

380 Engineering Mathematics - I

5.5.26 Example:
Evaluate by tripple integration the volume of a hemisphere of radius o.
Solution:

ww
w .Ea
The volume of the hemisphere
Fig. 5.5.26

syE
ngi
a r;;C;'1-
J J [z]ga -r
,----
2 2 nee
V=4
X~O y~O
-1'2 dyd"(
rin
g.n
e t

2 3
V = -1rG
3

Downloaded From : www.EasyEngineering.net


Downloaded From : www.EasyEngineering.net

Application of Integration to Areas, Lengths, Volumes and Surface Areas 381

Exercise - 5(F)
1. Evaluate the following integrals by changing the order of integration:
3 [4-:':;'
I. J
0
J(x+ y}lxdy

241
IAns: 60 I
5 x2

2. J fx{x + i }Ixdy 2

ww
0 0

29 56

"w
,2,,1,,2_x 2
.Ea
IAns: 24· I

3.
0
f 0
fy 2dx((V
syE
2-x
n gin
({~
IAns: - + - 1
16 32
lW4

e
I

J J!...dxdy
4.
0
v x .
eri
j; ng.
IAns: 2 log2 - II

n
I

f
5.
0 I'
fxy(X + y}lxdy
et
3
IAns: 1
56
4£1 2\'at
,.
6. f
o x2/4a
Jdydx

2
16a
IAns: -I
3
2 4

7. Jfx 2 2
+ y dxdy
I 3

IAns: 10)

Downloaded From : www.EasyEngineering.net


Downloaded From : www.EasyEngineering.net

382 Engineering Mathematics - I

II. Evaluate the following tripple integrals:



u x y

I. f f fxyzdxdydz
000

a6
[Ans: 48]

2. f 'J 1-](> + i + Z1 }lwlydz


o 0 0

ww I
[Ans: 20]

f II' w.E
, ,
J-XIY~ dydz
3.
() 0 asy
En I
3"]
gin [Ans:

e
122

eri
2
4. f f fx yzdxdydz
U I

ng.
(J

[Ans: I]
III
5. f f f(x + y + z)dxdydz
() () ()
net
3
[Ans: 2]
I JI-x 2 ~~2_y2
6. f f fxyz dtdydz
o 0 0

I
[Ans: 48]

Downloaded From : www.EasyEngineering.net


Downloaded From : www.EasyEngineering.net

Application of Integration to Areas, Lengths, Volumes and Surface Areas 383

3 I Fv
7.
JJ
I I 0
Jxyz dxdydz
x

13 1 1og3 )
(Ans:] ( 9-6 ]

I I-x x+y

8. JJ z
Je dxdydz

ww
o 0 0

w.E !BY
(Ans: -]
2

asy
4 x

9. JJ
o
Jz dxdydz
En
0 0

(Ans: 16]
o

J J
J 02_x 2 niX
2 gin
10.
o _J02_x2
Jz dxdydz
o eer
ing
.ne
t

Downloaded From : www.EasyEngineering.net


Downloaded From : www.EasyEngineering.net

ww
w.E
a s yE Left Blank"
"This page is Intentionally
ngi
nee
rin
g.n
et

Downloaded From : www.EasyEngineering.net


Downloaded From : www.EasyEngineering.net

ww
w.E
6 asy
E
Sequences of Series n gin
6.0 Sequence
e eri
A function fN ~ S, where S is any nonempty set is called a Sequence
ng.
i.e., for each nE N, ::l a unique elementj{n) E S. The sequence is written asj{I),j{2),

net
j(3), ..... j{n).... ,and is denoted by (j(n)}, or <j(n», or (/(n». If j{n) =an ,the sequence is
written as ai' a 2 •••••an and denoted by , {an} or < all > or ( all ). Here j{ n) or all are the
d h terms of the Sequence.
6.1.1 Example: 1 ,4,9, 16, ......... n 2 , ••••• (or) < n2 >
s

Downloaded From : www.EasyEngineering.net


Downloaded From : www.EasyEngineering.net

386 Engineering Mathematics - I

6.1.2 Example: ~3 ,~,-;-, ~ (or)(~)


••••• ••••
I 2- J n n
N s

ww
w.E
6.1.3 Example: 1, 1, 1. ..... I.. ... 0r<l>

asy
En
gin
eer
ing
6.1.4 Example: 1 ,-1, 1, -1, ......... or (( - I r- I
) .ne
t

Note 1. If S c R then the sequence is called a real sequence.


2. The range of a sequence is almost a countable set.

Downloaded From : www.EasyEngineering.net


Downloaded From : www.EasyEngineering.net

Sequences of Series 387

6.1.5 Kinds of Sequences


1. Finite Sequence :A sequence < all > in which an = 0 Vn > mEN is said to
be a finite Sequence. i.e., A finite Sequence has a finite number of terms.
2. Infinite Sequence: A sequence ,which is not finite is an infinite sequence.
6.1.6 Bounds of a Sequence and Bounded Sequence
1. If :3 a number 'M' ~ an:::; M, Vn E N, the Sequence < an > is said to be
bounded above or bounded on the right.

I,.!.), ,....... here all :::; 1Vn EN


ww
Ex:
2 3
2. If :3 a number 'm' ~ a" ~ m, Vn E N, the sequence < a" > IS said to be

w.E bounded below or bounded on the left.


Ex : 1 , 2 , 3 ,..... here a" ~ 1 Vn EN

asr (1y+;)
3. A sequence which is bounded above and below is said to be bounded.

Ex: Let an = (-1


En
n 2 3 gin 4
3/2 -4/3
e5/4
eri
ng.
2--
net
1 __

o 2 3 4 5 n
-1 -

From the above figure (see also table) it can be seen that m = -2 and M = l.
2
:. The sequence is bounded.

Downloaded From : www.EasyEngineering.net


Downloaded From : www.EasyEngineering.net

388 Engineering Mathematics - I

6.1.7 Limits of a Sequence


A Sequence < (Ill > is said to tend to limit 'I' when, given any + ve number' E "

however small, we can always find an integer 'm' such that Ian -II <E, Vn ;:::: m,
and we write Lt all =I or (an ~ I)
n->co

n2 + 1 1
Ex: If an = 2 then <a >~-.
2n +3 n 2
6.1.8 Convergent, Divergent and Oscillatory Sequences

ww ). Convergent Sequence: A sequence which tends to a finite limit, say' I' is called
a Convergent Sequence. We say that the sequence converges to 'I'
2. Divergent Sequence: A sequence which tends to ±oo is said to be Divergent

w.E (or is said to diverge).


3. Oscillatory Sequence: A sequence which neither converges nor diverges ,is

Examples asy
called an Oscillatory Sequence.

En
1. Const'der the sequence 2, -,
3 -4 ,-,.....
5 here 1
an =1 +-
gin 234
The sequence < all > is convergent and has the limit 1
n

1
eer
1 1
an -1 = 1+ - -1 = - and - < E whenever n > -
nnE n
1

1
Suppose we choose E= .001, we have - < .001 when n> 1000. ing
1
n
.ne
2. If all

3. If an =n
=3+(-lr--<a/l > converges to 3.
2
'n'
+(-lr .n,<an > diverges.
t
4. If an = ..!. + 2 (-1
n
r '< an > oscillates between -2 and 2.
6.2 Infinite Series
6.2.1
If < UII > is a sequence, then the expression u1+ u2 + u3 + ........ + un + ..... is called an
co
infinite series. It is denoted by I Un or simply I Un
11=1

The sum of the first n terms of the series is denoted by sn


i.e., sn = U, +u2 +u3 + ...... +un;sps2's3' ....sn are called partial sums.

Downloaded From : www.EasyEngineering.net


Downloaded From : www.EasyEngineering.net

Sequences of Series 389

6.2.2 Convergent, Divergent and Oscillatory Series


Let Iu" n
be an infinite series. As ~ 00, there are three possibilities.
(a) COl1vergent series: As n ~ oo,s" ~ a finite limit, say's' in which case
the series is said to he convergent and's' is called its sum to infinity. Thus
Lt
11----)00
8" =S (or) simply LIs" =8
00

This is also written as III + 1I} + II] + ..... + 11" + .. .1000 = s. (or) I UII = S
11=1

(or) simply IlIlI = s.


ww (b) Divergent series: If 8 11 ~ 00 or -00, the series said to be divergent.

w (c) Oscillatory Series: If

.Ea
8 11

infinite it is said to be an Oscillatory Series.


does not tend to a unique limit either finite or

Note: Divergent or Oscillatory series are sometimes called non convergent series.
6.2.3 Geometric Series
. 1
TIle senes, +x+x- + ..... x + ...
1 II-I syE •
IS

(i) Convergent when Ixl < 1, and its sum isngi _I_
I-x
(ii) Divergent when x ~ 1.
(iii) Oscillates finitely when x =
nee
-I and oscillates infinitely when x < -I.
Proof:
rin
The given series is a geometric series with common ratio 'x'

=- -
I-x"
when x *1 [By actual division - verify] g.n
e
sn
I-x
(i) When Ixl < 1:
1
t
LI s
n;->oo n
=
1- x
n->oo
( 1)
Lt - - - Lt (XII]
- - =--
n->oo 1- x 1- x

[ since xn ~ 0 as n ~ 00 ]
. 1
:. T he senes converges to - -
I-x
x" -1
(ii) When x ~ 1: sn =- - and sn ~ 00 as n ~ 00
x-I
:. The series is divergent.
(iii) When x = -1 : when n is even, sn ~ 0 and when n is odd, sn ~ 1
:. The series oscillates finitely.

Downloaded From : www.EasyEngineering.net


Downloaded From : www.EasyEngineering.net

390 Engineering Mathematics - I

(iv) When x < -1, s" ~ 00 or -00 according as n is odd or even.


:. The series oscillates infinitely.
6.2.4 Some Elementary Properties of Infinite Series
1. The convergence or divergence of an infinites series is unaltered by an
addition or deletion of a finite number of terms from it.
2. If some or all the terms of a convergent series of positive terms change their
signs, the series will still be convergent.
3. Let Iu" converge to's'
Let 'k' be a non - zero fixed number. Then Ikll" converges to h.
ww I I Also, if u" diverges or oscillates, so does I ku"

w.I E
4. Let
(i)
II" converge to 'I' and I
(u" + v,,) converges to ( 1+ 111 ) and
v" converge to '111 '. Then

a (ii)
syE
I(u" -v,,) converges to (/-m)
6.2.5 Series of Positive Terms
ngi
Consider the series in which all terms beginning from a particular term are +ve .
Let the first term from which all terms are +ve be ul •
Let I nee
Un . be such a convergent series of +ve terms. Then, we observe that the
convergence is unaltered by any rearrangement of the terms of the series.
rin
6.2.6 Theorem
If Iu" is convergent, then It un = o. g.n
e
n->oo
Proof:
Sn=u l +U2 +······+U"
sn_1 = ul + u2 + ...... + U,,_I,' so that, un = sn - sn_1
t
Suppose IU n =/ then It sn =/ and Lt sn_1 =/
n---+oo n~oo

It un = It (Sn-Sn_I); It s,,- It S,,_I =/-/=0


n~oo n---+oo n~oo n~oo

Note: The converse of the above theorem need not be always true. This can be
Observed from the following examples.

Downloaded From : www.EasyEngineering.net


Downloaded From : www.EasyEngineering.net

Sequences of Series 391


,
1 1 1 1
(i) Consider the series, 1+ - + - + ....... + - + .... ; lin = -, It lin =0
2 3 n n n~oo

But from p-series test ( 2.7) it is clear that I!n is divergent.


. . 1 1 1 1
(ii) ConsIder the sertes, + -) + -) + ..... + -) + ..... .
-2
1 2- 3- n-

lIl/ = ~, Itlin = 0, by p series test, clearly I ~ converges,


Ir n~OCJ n-

ww Note: If Lt lin ::t:- 0 the series is divergent;


IJ-too

2n -1
w.IE
Ex: 1'" = - 2n ,here Lt Il---Ht)
lin =1

asy
lin is divergent.

Tests for the Convergence of an Infinite Series

En
In order to study the nature of any given infinite series of +ve terms regarding
convergence or otherwise, a few tests are given below.
6.2.7 P-Series Test gin
Th e '111f illite
i ' sertes,
. ~1
~- = -1+ 1
- +1
- + ...... ,
n~1 n" 1" 2" 3"
e IS
eri
(i) Convergent when p > I, and (ii) Divergent when p ~ 1. [JNTU Dec 2002, A 2003]
ng.
Pro(~r:

Case (i) Let p> I . J) > 1 3" > 2'" => _1 < _1_
, , '3" 2"
net
1 1 1 1 2
-+-<-+-=-
2" 3" 2" 2" 2"
111111114
Similarly, -+-+-+-<-+-+-+-=-
4" 5" 6" 7" 4" 4" 4" 4" 4"
1 1 1 8
-+-+ .... +-<-, and soon.
8" 9" 16" 8"
Adding we get
1 2 4 8
"-<1+-+-+-+ ....
~ n" 2" 4" 8"
1 1 1 1
I.e., I n" < 1+ 2(,,-1) + 2 2(,,-1) + 2 3(,,-1) + ..... .

Downloaded From : www.EasyEngineering.net


Downloaded From : www.EasyEngineering.net

392 Engineering Mathematics - I

The RHS of the above inequality is an infinite geometric series with common ratio
1
2,,-1 < 1(since p > 1) The sum of this geometric series is finite.

Hence f ~n
11=1
is also finite.

:. The given series is convergent.


1 1 1 1
Case (ii) : Let p = I; I - = 1+ - + - + - + ......
n" 2 3 4
1 1 1 1 1

ww We have, -+- >-+- =-


3 4 4 4 2
111111111

w.E -+-+-+->-+-+-+-=-
567888882
11 111 11

asy -+-+ ....... - >-+-+ ..... - = - and so on


9 10 16 16 16 16 2

En I_I = 1+(!+!)+(!+!+!+!)+ .....


n" 2 3 4 5 6 7

gin 1 1 1
~1+-+-+-+ .....

~ ee
2 2 2

= n; 1and /1~'" SII = 00 )


1
The sum of RHS series is 00

r(Since sn = 1 + n

f_l-n ing
:. The slim of the given series is also 00;:.
11=1
P
(p
.ne
= I ) diverges.

Case(iii):Let p<l, I7=1+1

1 1 1 1
1 1
+ +.....
2P Y t
Since p < 1,- > - - > - ...... and so on
21' 2' JP 3'
1 1 1 1
I->I+-+-+-+
P
.......
n 2 3 4
From the Case (ii), it follows that the series on the RHS of above inequality is
divergent.

I Jl
n
is divergent, when
P
P< I

Note: This theorem is often helpful in discussing the nature of a given infinite
series.

Downloaded From : www.EasyEngineering.net


Downloaded From : www.EasyEngineering.net

Sequences of Series 393

6.2.8 Comparison Tests


1. Let I
Un and I Vn be two series of +ve terms and let I VII be convergent.
Then I Un converges,
1. If un ~ Vn' \:;f n EN
U
2. or -.!!.. ~ k\:;fn EN where k is> 0 and finite.
vn

3. or un ~ a finite limit> 0

ww I
Proof:
vn

w.E 1. Let
Then, u l
Vn =I (finite)
+u2 + ....• +lln + ...... ~VI +v2 + ..... VII + ..... ~l >0

2.
a Since I is finite it follows that

un ~ k ~ UII ~ kv \:;fn EN, since syE


,
I UII is convergent

I VII is convergent and k (>0) IS

ngi
ll
vn

finite, Ikvll is convergent:. IU is convergent.

3. Since Lt un is finite, we can find a +ve constant k,3


n

nee ~ < k\:;fn EN


n-'>oo vn

IU rinvn

2. Let
:. from (2), it follows that
I UII and I Vn
II
is convergent
be two series of +ve terms and let I VII g.n
be divergent.

Then I Un

* 1.
diverges,
If un ~ VII' \:;fn EN
et
or * 2. If un ~ k, \:;fn EN where k is finite and"* 0
vn

or * 3. If Lt ull is finite and non-zero.


n-'>OO Vn

Proof:
1. Let M be a +ve integer however large it may be . Science I VII is divergent,
a number m can be found such that
VI +V2 + ..... +vn > M, \:;fn > m

u +u2 + ...•. +un > M, \:;fn > m(un ~ vJ


l

Downloaded From : www.EasyEngineering.net


Downloaded From : www.EasyEngineering.net

394 Engineering Mathematics - I

I 1I" is divergent
2 11, 2:: kv,,\:;In
I v" is divergent ~ I kv" is divergent

I u" is divergent

3. Since Lt ~ is finite, a + ve constant k can be found such that un > k, \:;In

(probably except for a finite number oftenns )

ww :. From (2), it follows that I Un is divergent.

w.E
Note:
I. In (I) and (2), it is sufficient that the conditions with * hold \:;In> mEN
Alternate form of comparison tests : The above two types of comparison tests

Iu" I
asy
2.8.( I) and 2.8.(2) can be culbed together and stated as follows:

If and Vn
EnI I
are two series of + ve terms such that Lt
"->00
un =
vn
k,

where k is non- zero and finite, then


gin u" and Vn both converge or both
diverge.
eer
Note:
I. The above fonn of comparison tests is mostly used in solving problems. ingI
2. In order to apply the test in problems, we require a certain series Vn whose

I .ne
nature is already known i.e., we must know whether
divergent. For this reason, we call I Vn
Vn

as an 'auxiliary series'.
is convergent are
t
3. In problems, the geometric series (2.3.) and the p-series (2.7) can be
conveniently used as 'auxiliary series'.

Solved Examples
6.2.9 Example
Test the convergence of the following series:
3456 4567
(a) -+-+-+-+..... (b) -+-+-+-+ .....
1 8 27 64 1 4 9 16

~ (n +1) 1/4 -n ]
00 [ 4
(c)

Downloaded From : www.EasyEngineering.net


Downloaded From : www.EasyEngineering.net

Sequences of Series 395

Solution
(a) Step 1:
To find "ul/" the d" term of the given series. The numerators 3, 4, 5,
6 ...... ofthe terms, are in AP.
n'" term tl/ = 3 + (11- 1).1 = n+2
· n+2
are 1 ,-,
3 2' 33 43
Denomlllators , ..... n ,,, term = n \ ; 111/ = --3-
n
Step 2:
To choose the auxiliary series I VI/ . In 1I11 , the highest degree or 11 in the

ww numerator is I and that of denominator is 3.

w.E :. we take,

Step 3 :
VII = ----:l=I = 2
n 11

a~=sy
It
vl/
It n+2xn
I/~OO3
n
2
= Lt n+2 = It
I/~OO 11 I/~OO
(1+~}=1
11 '

En
1/-->00

which is non- zero and finite.


Step 4:
Conclusioll: gin
It ~=1
n-----}oo VII eer
:. IUI/ and I v" both converge or diverge (by comparison test). But ing
I VI/ ,,-?
= ~n-
1
is convergent by p-series test (p =
.nIll"e
2 > 1); :. is

(b)
convergent.
4 5 6
-+-+-+-+ .....
7
t
1 4'-9 16
n+3
Step 1 : 4 , 5, 6, 7, ..... in AP , tn = 4 + (n -1) 1 = n + 3

Step 2 : Let "~ v" = -1 be tIle auxi'1'lary series


.
11

Step 3: Lt
I/~'"
~
v"
= Lt (11~3)xn = It (1
n- ,,~oo I/~OO
+l) = 1,
n
which is non-zero and

finite.

Downloaded From : www.EasyEngineering.net


Downloaded From : www.EasyEngineering.net

396 Engineering Mathematics - I

Step 4 : :. By comparison test, both I utI and I v" converge are diverge
together.

But Iv" =I-n1 is divergent, by p-series test (p = 1); :. Iu" is

divergent.

ww
w =n .E1+-'a-.+ ~G-l)~+ . . _1 =n[-'-.-~+ . . .J
4n 2! n 4n 32n

syE
= 4~3 ~3 n
[~-
- 3;n 7 + .... = .J
gin 3;n 4 + ....

=~
Here it will be convenient if we take v"

(! - !,
e eri n
Lt utI = Lt
n~oo
-1-4 + ..... ) =
vn n~oo 4 32n 4
ng.
which is non-zero and finite

:. By comparison test,

by p-series test IV
I I
n =-3
Un

1
and
n
Iu"et
is convergent. (p
VII both converge or both diverge. But

= 3 > 1); :. is
n
convergent.
6.2.10 Example
{j'--3n-2-+-1
If u
n
=
~hn3 +3n+5
show that I Un- is divergent

Solution
As n increases, utI approximates to
r:::-? II II
~ 3n 2
l 2/
Jl3 n73 3/ 3 1
~2n 3 = 2Y.; x nYt = 2}~ . nX2

Downloaded From : www.EasyEngineering.net


Downloaded From : www.EasyEngineering.net

Sequences of Series 397

Ii
U )13
If we take vn = - 1-/ , Lt -.-!!.. =-I which is finite.
nl12 n-->""VII 274

1
[(or) Hint: Take vn = ~ , where II and 12 are indices of 'n' of the
n' ,
largest terms in denominator and nominator respectively of ull • Here
1 1
VII =-3-2 =-1 ]
n4 3 nl2

I I
ww By comparison test,

"L..,. VII =,
VII

, 1 .IS d'Ivergent by p -
and U II converge or diverge together. But

. test (smce
. p = -1 < 1)

Lw.
L..,.-I,-' senes
Wl2 12

E
6.2.11 Example
un is divergent.

asy
Test for the convergence of the series. EnJf ~ J! ~ + + + + ......
Solution gin
Here, un = ~ n: 1; eer
Take
1 1
vn =-1-I =-0=1, Lt u.
----
~ n
i Lt ) 11 ng.
= 1 (finite)

I VII
n2 2

is divergent by p - series test. (p = 0 < 1)


n-->"" V
n
11-->""
1+-
n n et
:. By comparison test, I Un is divergent, (Students are advised to follow the
procedure given in ex. 1.2.9(a) and (b) to find" un" of the given series.}
6.2.12 Example
1 1 1
Show that 1 + IT + l2 + ....... + l!:! +..... is convergent.

Solution

un = l!:!1 (neglecting 1sl term)

1 1 1
- - - - < -----==,--- =
1.2.3 ......n 1.2.2.2 .....n -ltimes (2n-l)

Downloaded From : www.EasyEngineering.net


Downloaded From : www.EasyEngineering.net

398 Engineering Mathematics - I

1 1 1
"1I" < 1+-+-? +-, + ..... .
~ 2 2- 2'
which is an infinite geometric series with common ratio ~ <1
2
" 2,,-1
:. ~ 1 IS. convergent. (1.2.3(a)). Hence I "/I is convergent.

6.2.13 Example
Test for the convergence of the series, _1_ + _1_ + _1_ + ...... .
1.2.3 2.3.4 3.4.5

ww
Solution
u = 1
.
w.E" n(n+1)(n+2)'
1
Take V"=-3
n
asyn)( ) 3

Lt
,,~ao V
U" = n~ao
Lt
En
3
( 1
n 1+-- 1+-
2 = 1 (finite)

I gin
n
n n
I
By comparison test,

series test, I v" = I ~ e U" ' and v" converge or diverge together. But by p-

is convergent (p = 3> 1 ); :. Iu" eri is convergent.

6.2.14 Example
n
ng.
If u" =
Solution
-J n4 + 1 - -Jn4 -1, show that I Un is convergent. [JNTU, 200s]net
1 I

u" = n2 ( 1+ n1)2
4 - n 2 ( 1- n 4 1)2
~n2[(1+ 2~4 - 8:' + 16~12 - ....)+- 2:4 - 8:' -16~12 - .....)]
=n
2
[-;-
n
+ ~
8n
+ .... J= ~ [1 + ~ + .... J
n 8n
1 u'
Take vn = -2 ' hence Lt -E... =1
n ,,~ao v"

Downloaded From : www.EasyEngineering.net


Downloaded From : www.EasyEngineering.net

Sequences of Series 399

:. By comparison test, I lin and I v" converge or diverge together. But

I Vn =~ is convergent by p -series test (p = 2 > 1):. I Un is convergent.


n
6. 2.15 Example
· 1 1 1 I-'
Test the senes - - + - - + - - + ..... lor convergence.
l+x 2+x 3+x
Solution
1
un =--;

wwTake =-,
n+x

w.E
VII
n
UII _ n
then =
asy
n+x 1+~
n

Lt (_1_] 1; I EI!
ngi = VII =
n
is divergent by p-series test (p =1 )
n~""

:. By comparison test,
1+-
n
X

nI is divergent. eer
ing
Un

6.2.16 Example

Show that fSin(!)n is divergent. .ne


Solution
n=1

Take
t
U n
. (1) sint
SIn -
Yn
Lt --E...
n~"" V
= n~""
Lt ( )
1
= Lt -
I~O t
(where t = 1n) = 1
n _

n
Iu I n
, VII .both converge or diverge. But I VII = I ~ is divergent

(p -series test, p = 1); :. I Un is divergent.

Downloaded From : www.EasyEngineering.net


Downloaded From : www.EasyEngineering.net

400 Engineering Mathematics - I

6.2.17 Example

Test the series L sin -I ( : ) for convergence.

Solution
• _I 1
u =sm _.
n
n'
1
Take vn =-
n
ww u sin -{~)
( ()) ( . . _ 1
Lt 2..= Lt ( ) ;= Lt - . - =1 Takmgsm '-=()
)

wBut L v" .Ea


v
/1---*00tHoo

is divergent .Hence
/I
1 8---*0 sm ()

L u"
_

n
n

is divergent.

6.2.18 Example
syE
1 22 33
ngi
Show that the series 1 + - 2 + - 3 + - 3 + ..... is divergent.

Solution
234
nee
1 22
Neglecting the first term, the series is 1 + - 2 + - 3 + - 4 + .....
234 r 33
ing.Therefore

U -
n"
-
nn
n-
"~(n+l)"" ~ (n+l)(n+l) ~ n(l+~).n"(I+~J
nn
.ne
1 1
t
Take vn =-
n

1
e

which is finite and L vn = L!n is divergent by p -series test (p = 1)

L un is divergent.

Downloaded From : www.EasyEngineering.net


Downloaded From : www.EasyEngineering.net

Sequences of Series 401

6.2.19 Example
. 1 3 5 .
SIlOW t Ilat t Ile series - - + - - + - - + .......00 IS convergent.
1.2.3 2.3.4 3.4.5
Solution
1 3 5
--+--+--+ .......00
1.2.3 2.3.4 3.4.5
1
Iii 2n-1 ( 2- n )
n term~ u" ~ n(n+l)(n+2) ~ n' ·(I+~)(I+~)
ww 1
w
Take Vn = - 2
n
.Ea _I (2-~ ) (_1)
sy(1 E 1 2n)
it un
n~oo VII
= it
HOO n
2
+ ~'~)( +
+
n
2

it Un ngi=( 2)(
0
) =2 which is finite and non-zero
HOO

:. By comparison test
Vn

Llln nee1 +0 1+0


and LVII converge or diverge together

L n1 LUll rin
But

6.2.20 Example
LVII = -2 is convergent. :.
g
is also convergent.

.ne
Test whether the series

Solution
LII~I J;;n + ,J;;+l
00

n+ I
1
is convergent t
1
The given series is L-==---==
00

J;; +.j;;+1
11=1

Downloaded From : www.EasyEngineering.net


Downloaded From : www.EasyEngineering.net

402 Engineering Mathematics - I

Take

ww which is finite and non-zero

wI .EI l I I Ii)
Using comparison test Un and Vn converge or diverge together.

But

I
a syE
Vn =

Un
is divergent (since p

is also divergent.
=

6.2.21 Example n
f[ n nJgin
Test fo~ convergence ~
eer
3
+1- [JNTU 1996,2003, 2003s]
n=1

nih n[(1 +~))~


term
3n Jj U~in
n -1] n[1 +~+
Un = -1) .~+
g.n
n -1] =
1.2
.....

3:2- 9:5 :2 (1- 9:3 = et + ...... = + ...... ) ; Let


1
Vn =-2
n
Then f~L X=n~Jlj - X n 3 + .... )= lj *0
:. By comparison test, I Un and I Vn both converge or diverge.
But I Vn is convergent by p -series test (since p = 2 > 1)
convergent.
6.2.22 Example
Show that the series, ~ + ~P + ~P + ....... is convergent for p > 2 and divergent
JP 2 3
for p 5, 2

Downloaded From : www.EasyEngineering.net


Downloaded From : www.EasyEngineering.net

Sequences of Series 403

Solution

III •• n +1 n ( 1+ In) ( 1+ 1~)


n term of the given senes = U = --~ = =
I n" n" 11
,,-1

Let
I = _1_.
liS take v Lt u
n,,-I '11-"') /v
= l:;t: O· n/
lI
'

... L 1111 and L VII both converge or diverge by comparison test.


BlIt LVII = L }~>-I converges when p -I> 1 ; i.e., p >2 and diverges when

ww p -1 ~ 1 i.e p
6.2.23 Example
~ 2; Hence the result.

w.EL (?II7-3J
Test for convergence
OCJ

3 +1
+ 1
2

asy
11=1

Solution

"" J 2"(1+ 3~,,)r'En


l3 '~3 1 ) J gin 11

(
1
+

" I )1,- eer 1+ 3


Take
Jf. VII 3 V
ing
-
n -
_.
3"'
U
--'!...-
- [ 1 1,'
+ n
2

L L .ne
Lt !!A = 1 :;t: 0; :. By comparison test, un and behave the same way.

t
VII
v
1l-7r:iJ
ll

"
BlIt L... vn = ~(2)"2 = ~- + -2+ (2)3
L... - -
2
I. I. a geometflc sefles Wit
+ ..... , W lIC 1 IS ···1 1
11=1 3 3 3 3
common ratio )34, «I) :. LVII is convergent. Hence LUll is convergent.
6.2.24 Example
. 1 4 9
Test for convergence of the sefles, + + + ..... .
4.7.10 7.10.13 10.13.16
Solution
4,7, 10, .............. is an A. P; til = 4 + (n -1) 3 = 3n + 1
7, 10, 13, ............ is an A. P; 'II = 7 +(n-l)3 = 3n+4
and 10, 13, 16 , ............. is an A. P; 'II = 10 + ( n - 1) 3 = 311 + 7

Downloaded From : www.EasyEngineering.net


Downloaded From : www.EasyEngineering.net

404 Engineering Mathematics - I

n2 n
ll= =---:-----:------:-----;----;----;-
II (3n+1)(3n+4)(3n+7) 3n(1+ V ).3n(1+4 ).3n(1+ 7/ )
" 3n 3n ·3n
1
27n(1+ 1, )(1+ 413n
/3n
;' )(1+ 7/
/3n
)
Taking vn =! ,we get
n
Lt ~ = _1_:;t: 0; :. By comparison test, both LUn and L vn
ww
behave in the
IHOO vII
27
same manner. But by p -series test, L is divergent, since p I. :. Lilli is

w
VII =

divergent.
6.2.25 Example .Ea
Test for convergence syE
",hn2 -Sn+1
~ 3 2
4n -7n +2
Solution
ngi
Iii
n term of the gIVen senes = un
••
=
.J2n2 - Sn + 1
3
4n -7n +2
2
nee
Let vn =-2
rin
n

~/ ~/? g.n
L lin

nloo ~ = IH~·
L

l n 2 - / n + / n- n 2
n3 (4-/~ +~<3) x-I

~/ V J
J
e t
:. By comparison test,
convergent. [p series test - p
L
nloo
l 2 - / n + / n2
(4 _ 7/ +
/n
L un
=
2/)
'n
and
3

L vn
2 > I] :.
r;;
'112
= 4 :;t:
0

both converge or diverge. But


L un is convergent.
L VII is

6.2.26 Example
·"
Test tI1e senes ~ un ,w hose n Iii term • (
IS I .)
4n 2 - I

Downloaded From : www.EasyEngineering.net


Downloaded From : www.EasyEngineering.net

Sequences of Series 405

Solution
1
Ull = (4n 2 -/.);

=Lt lrn
2
1
Let VII =-\,
n-
Lt
11->00
_llil
VII Il->ao
2
(
n /'n
4- "
o
)1=-4 *0
:. I Ull and I Vn both converge or diverge by comparison test. But I VII IS

I
ww convergent by p -series test (p = 2 > 1) ; :. UII is convergent.

6.2.27 Example

If un
w.E
= (; ).Sin (;), show that IU II is convergent.

Solution asy
Let VII =
1
-?
n-
,so that En '" VII is convergent by lJ -series test.
~

J g(1)in (
- LI sin
(1 ) - e n - It -sin I )
ll
Lt _"
11->00 ( V
n
- n->ao
n
eri 1l~<X' I

where t = lin, Thus Lt (~J =1 * 0 ng.


:. By comparison test, I
Jl~<t)

is convergent.
VII

Un
net
6.2.28 Example

Test for convergence I 1tan(l;>


Take vn = II ~/; Lt [ulIl ] = 1 * 0 (~s in above example)
In 2 n--->oo I VII
Hence by comparison test, IIIIl converges as I VII converges.

6.2.29 Example

Show that fsin


2(~) is convergent.
11=1 n

Downloaded From : www.EasyEngineering.net


Downloaded From : www.EasyEngineering.net

406 Engineering Mathematics-I

Solution
. ?(I) ; Take vn =-?1
(
Let un =sm- -
n n-

l
sin 1/ .
(~)= Lt ')12 =Lt(smt) 2

Lt
I/~Cf) V n~oo 1,
<n t I~O
n ,'n

where t = II ;Lt (lin/v/ ) = 12 = 1 0


In n~oo n
:;t:

ww
:. By comparison test, I
But I
and I behave the same way.
UI/

is convergent by p- series test, since p 2 > 1; :. I


Vn

w.E
6.2.30 Example:
Vn = UI/ is convergent.

Show that f
n=2/ asy
Vlog (n1/) is divergent.
Solution
En
U
II
= Inlogn'
II .
gin log 2 < 1 => 2log 2 < 2 => 1/ > II .
12log2 72'
Similarly
eer
}jIOg3> X,.····Xlogn> Yn,nE N
L In
1/ Iogn > L In
II ; But L II
In
ing
is divergent by p-series test.

I
By comparison test, given series is divergent. [If
u/I ~ vn"i/n then IU
Vn

.ne
is divergent and

n is divergent.]
(Note: This problem can also be done using Cauchy's integral Test. See ex 1.6.2)
6.2.31 Example
t
Test the convergence of the series I
00

n=1
(c + n rr (d + nr' , where c, d, r, s are all

+ve.
Solution

Th e n Ih term 0 ft h ·
e senes = un = r
1 ,
(c+n) (d+n)
1 U/' _ 1
Let vII = -r+.\- Then --------=------
n Vn
n
r
( 1+ ~ J +~- J (1+~J (1+ ~J
.n" ( 1

Downloaded From : www.EasyEngineering.net


Downloaded From : www.EasyEngineering.net

Sequences of Series 407

_
Lf 1111 -_ 1 ---
-;-- 0 :.
," L)III and "
~ v" both converge are diverge, by comparison
11-)00 vJl
test.
But by p-series test, L VII converges if (r + .\) > 1 and diverges if (r + s) ~
:. L II" converges if ( r + s ) > I and diverges if (/"1 s) ~ I.

6.2.32 Example
~ -(1+ 1 )
Show that ~n "is divergent.

ww Solution
1

-(1+ 1 ) 1

wTake
11

.Ea
VII
"
=n

1 1I
"=--

=- ; Lt -" = Lf -
1
n.n 1n
= 17:- 0

1
n
syE
11---)00

1
V
"
1
11-+00
n
In

For let
n
Il-)ct) II ,,-too n 1 ngi
Lt -v = y say; log y = Lt - - .log n = - Lt -.-!1 = 0
11-)00

y = eO = 1 «(:) nee
using L Hospitals rule)

By comparison test both


L
L 1I" and LV"
L
r ing
converge or diverge. But p-series

test,

6.2.33 Example
VII diverges (since p = I); Hence lIlI diverges.
.ne
Test for convergence the series ~
~ (n+ar
II~I (n + b)" ( n+ c )"
, a, h, C
t
,p, q, r, being +ve.

Solution

Downloaded From : www.EasyEngineering.net


Downloaded From : www.EasyEngineering.net

408 Engineering Mathematics - I

1
Take v
II
=- - . It u = 1"* 0 .'
I1P+lJ-r' II~OO V
_II

II

Applying comparison test both I lin and I VII converge or diverge.


Rut by p-series test, I VII converges if (p+ q -r) > I and diverges if (p + q -r)
~ I.
Hence I lIlI converges if (p + q - r) > 1 and diverges if (p + q - r) ~ I.

6.2.34 Example

ww Test the convergence of the following series whose nl" terms are:

(a)
(311+4)
(b) tan-;
1

w(c)
( 211 + 1)( 211 + 3)( 211 + 5)

.Ea (d)
11

syE
11 11
(3 +5 )'

(e)

Solution
11. 311
ngi
(a) Hillt:Take VII =~;Iv"
11-
is convergent;nee(u,,)=~"*o It
v 8
(Verify)

rin
,,~oo
"
Apply comparison test:
I lIlI
g.n
is convergent [the student is advised to work out this problem fully]
Iu"
(b)

(c)
Proceed as in 1. 2.16;

1
Hint: Take v" = - 2 ; It -" = It (
is convergent.

(u) (l+J~lr = 3e = 1 "* 0 )11 -?


et
11 ,,~oo VII II~OO 1 + 3/~ e e-

v" = -\- is convergent (work out completely for yourself)


11

(d)
U" ~ 3" +5" ~ 5"
1 1
II 1
+(~J] ;Take v" ~~ ; "If.
1 (Un)
~ ~l ,,0

Downloaded From : www.EasyEngineering.net


Downloaded From : www.EasyEngineering.net

Sequences of Series 409

LUll and L VII behave the same way. But L v" is convergent since it is

a geometric series with common ratio ~<1


5
:. LUll is convergent by comparison test.
1 1
(e) - - ~ - \;fn EN, since n.3";:::: 3n ;
n.3" 3" '
1 1
L -3;;-
n. <L -:;-; _1
.....( I)

ww The series on the R. H .S of (1) is convergent since it is geometric series with


1
r=-<1.
w 3

.Ea.
:. By comparison test I -I-.
n. 3"
IS convergent.

6.2.35 Example syE


Test the convergence of the following series.
(a) 1+ ~+2J
21+~+3 J + J 1+~+32+4 2 +.............. .
+ ngi
1 +2-
1 +2-+3- 1-+2 +3 +4
12+22+3 2 12+22+3 2 +42
12+22 nee
(b) 1+ 3 + 3 3
1 +2 3 1 +2 +3
3
+
C+2 3 +3 3 +4 3
+............. .
rin
Solution
(n+ 1)
n-- g.n
(a)
U _
n -
1+2+3+ .... +n _
12 + 22 + 3 2 + .....wJ - n ( n + 1) _on
(2 -t-_
6
2
1) - (2 n + 1)
_ 3
e t
Take Vn =- U = Lt ( -
Lt _" 3n- ) =-*0
3
n n ..... '" 2n + 1
n .....'" VII 2
I Un and I VII behave alike by comparison test.
But I Vn is diverges by p-series test. Hence I Un is divergent.
(2n+ 1)
12+22+ .... +n2 n(n+1)~6-- 2(2n+1)
(b)
un = 13 + 23- + .....n 3 = (
2 n +1
)2 = 3n (n + 1)
n ----
4

Downloaded From : www.EasyEngineering.net


Downloaded From : www.EasyEngineering.net

410 Engineering Mathematics - I

Hillt: Take vn =!
n
and proceed as in (a) and show that IU II
is divergent.

Exercise 6(a)

1. Test for convergence the infinite series whose nih term are:
1
(a) [Ans: divergent)
n-Fn
.Jn~I-Fn
ww
(b)

(c)
n
.Jn2 + I-n
[Ans : convergent)

[Ans : divergent)

(d)w Fn
n 2 -1 .Ea [Ans : convergent)

(e) .Jn3 +1-N syE [Ans : divergent]

(f)
1
~n(~+I)
ngi [Ans : divergent]

Fn nee
(g)
n 2 +1
2n3 +5
r ing
[Ans : convergent]

(h)
4n 5 +1
.ne
[Ans : convergent]

2. Determine whether the following series are convergent or divergent.

(a)
1 2 ·3
1 + 3- 1 + 1 + T2 + 1 + T3 + ............ [Ans : divergent] t
12 22 32 2+ IOn
(b) r+23+]T+ ........ + n3 +..... ....... [Ans: convergent]

1 1 1
(c) J1 +.J2 + .J2 +J3 + J3 + J4 +...... ........ [Ans : divergent]

234
(d) 2+-2 +2+ ............... [Ans : divergent]
345
1 1 1
(e) 2+-3 +4+ .......... [Ans : convergent]
123

Downloaded From : www.EasyEngineering.net


Downloaded From : www.EasyEngineering.net

Seq uences of Series 411

00 ~n2 + 1
~ ~4n2 +2n+3
(f) (Ans : divergent]

(g) f(8 Yn -1) .................... (Ans : divergent]


J
3
3n +8
(h) L 5n
00

J +9
5 ...................... .. (Ans : convergent]

123
(i) - + - + - + .......... . (Ans : divergent]

ww
6.3
1.3 3.5 5.7

wLU.E
6.3.1 0' Alembert's Ratio Test
Let (i) beaseriesof+vetermsand(ii) Lt un +J =k(~O)
asy
n
II~OO un
Then the series L un is (i) convergent if k < 1 and (ii) divergent if k > I.

Proof:Case(i): Lt un + J =k«I) En
n~oo un
From the definition of a limit, it follows that gin
3m > 0 and 1(0 < I < 1) 3 ~ < IV n ~ m
U

un
e . eri
i.e., U m+ J <I , U m+ 2 < I ,.......... ng.
Um

Um
U m+ J

+ Um+J + Um+ 2 + .............. = Um [1 + uu m


+
J
+ um+ 2 + ..... ]
net
m um

< um(1 +1 +/2 + ...) = um.-I-(I < 1)


I-I
1
But Um . - -
I-I
is a finite quantity:. L un
00

is convergent

By adding a finite number of terms u J + u2 + ...... + um- J ' the convergence of the
00

series is unaltered. L un is convergent.


n=m

Downloaded From : www.EasyEngineering.net


Downloaded From : www.EasyEngineering.net

412 Engineering Mathematics - I

U
C(l!t'e (ii) Lt ---.!!.±l. = k > 1
n-->oo 11n

There may be some finite number of terms in the beginning which do not satisfy

the condition ull +' 21. In such a case we can find a number 'm' 3 u +' 21, 'lin 2 m
ll

Ull UII

Omitting the first 'm' terms, if we write the series as u, + 112 + llj + ......... , we have
!!2. 2 1, u3 21, u4 2 1 .......... and so on

ww u, 112 u3

+ 112 + ...... + till = u, (1 + u + 2


.!!2. + ..... J
w.E U,

2 ull (1 + I + 1.1 + ....... to n terms)


ll,
113
tl2 u,
(to 11 terms)

asy
Lt
II

Iu" En IU 2 Lt n.u, which ~ 00; :. is divergent

gin
Il
n~oo 11=1 Il~CO

6.3.2
eer
Note: 1
inIg ~
The ratio test fails when k = 1. As an example, consider the series,
n=' n
.ne
t
p
u n P 1
Here Lt ---.!!.±l. = Lt - - = Lt - - =1
n-->oo U n-->oo ( n + 1) n-->oo ( 1+ 1/ )
n ,In

i.e., k = I for all val ues of p,


But the series is convergent if p > 1 and divergent if p::; 1, which shows that when

k = 1, the series may converge or diverge and hence the test fails.
Note: 2 Ratio lest can also be stated as follows:

If I Ull is series of +ve terms and if Lt ~ = k ,then I Un is convergent


11-->00 u +'
ll

If k > 1 and divergent if k < 1 (the test tilib '.I. hen k = I).

Downloaded From : www.EasyEngineering.net


Downloaded From : www.EasyEngineering.net

Sequences of Series
-~---~ ~----- ----~---- ----- 413

Solved Examples

Tests for convergence of Series


6.3.3 Example
\
X X X
(a) -+--+---+ ................ .
1.2 2.3 3.4
Solution
x" Xl/II

= n (11 + 1) ;111/+1 = (11 + 1)( n + 2) ,

r.
111/

ww ~/!.!..'.l= Xl/+I

r
n(n+1)
~ (I + ~
1

w.E 1/" (n+ 1)(n + 2 x"

a
Therefore LI
11----)00
II
_1/_,1
1111
syE
=X

IUI/ Ixl
:. By ratio test
Ixl> I; ngi
is convergent When < and divergent when

When x = I, ul/ = J (
1,
n- 1+ lin
. n
1
) ; fake vn = - 2 ;
II
Lt _1/ = 1
v1/ nee 11->00

:. By comparison test IUI/ is convergent.


rinIxl
Hence I Un is convergent when Ixl ~ 1 and divergent when
g.n >1

Solution
(b) 1+ 3x + 5x 2 + 7 x + ...... .

Un = (2n-l)xn-l;ul/+
3

=(2n+l)x"
et
1

L1-= -+
ul/+ I L t (2n -1) x=x
1/-->00 lin 1/-->00 2n-l
:. By ratio test I Un is convergent when Ixl < 1 and divergent when

Ixl > 1
When x = 1: ul/ = 211 -1; Lt un = 00; 11----)00
:. I UI/ is divergent.

Hence IlIl/ is convergent when Ixl < 1 and divergent when Ixl ~ 1
00 1/

(c)
I-+-···········
n- + 1
1/;1

Downloaded From : www.EasyEngineering.net


Downloaded From : www.EasyEngineering.net

414 Engineering Mathematics - I

Solution
x" I
X"+
U =2
-- . U =----
n n +1' n+1 (n+l)2 +1
2
Hence un +1 ( n +1 )
-;;: = n 2+ 2n + 2 x

2
Lt un +1 = Lt n (
1
+ >;:2) (x)=x
ww n~'" Un II~'" n2(1+~+2)
2
n n

w :. By ratio test,

When .Ea
x=I'u 1
L un
= - - . Take
is convergent when

v =-
1
Ixl < 1 and divergent when Ixl > 1

n2
• n n2 + 1 '
:. By comparison test, syLUE n
n

is convergent when Ixl ~ 1 and divergent when

Ixl >1
6.3.4 Example
ngin
Test the series f (n: -1}n, x>
n~'" n +1 e 0 for convergence.
eri
Solution
ng.
U" =(:::: )X";U"" = [~:: :i: :: ]X'HI n et
2 2
Lt un+1 = Lt [( 2n + 2n ) ( n2 + 1)] x
n~'" un n~'" n + 2n + 2 (n - 1) .

4
n (1+2/n)(1 +l/n) 1
=nIf", [ n4(l+2/n+2/n2)(1-l/n) =x
:. By ratio test, LU n is convergent when x< 1 and divergent when x> 1 when
x = 1,
n2 -1 1
U = 2- - Take vn =-0
n n +1 n

Downloaded From : www.EasyEngineering.net


Downloaded From : www.EasyEngineering.net

Sequences of Series 415

Applying p-series and comparison test, it can be seen that L un is divergent when
x= 1.
.'. L un is convergent when x < 1 and divergent x ~ 1

6.3.5 Example
2P 3P 4P
Show that the series 1+ ~ + l] + l1 + ..... , is convergent for all values ofp.
Solution

ww
w Lt .un +1
n~oo un .Ea
= Lt [(n+IY x
n~oo In + 1 nP
l.fl]=
n~oo
Lt { 1
(n + I)
(n+I)P}
n

= Lt (
n~oo
s
n + 1y
1 ) x Lt (I
1+ - )P = 0 < 1 ;
En
n
n~oo

L lll/ is convergent for all 'p'.


gin
6.3.6 Example
Test the convergence ofthe following series
e eri
1 1 1
-+-+-+-+
1
F 3P 5P 7 P
........... . ng.
Solution
1 1
n et
u = ·u =---
1/ (2n-IY' (2n+ly1/+1

ul/+ 1 = (2n-Iy _ 2P.n P(I-l/2nY. Lt un+1 =1


un (2n+ly - 2PnP(I+l/2ny , HOO un

.'. Ratio test fails.


1 nP 1
Take v = _. U
-E... = = U
. Lt -E... = -1P
n nP' vn (2n -I)P ( I)P , I/~OO V 2 ,
2 P 1-- n
2n
which is non - zero and finite 7
.'. By comparison test, LU n ar.d L vn both converge or both diverge.

Downloaded From : www.EasyEngineering.net


Downloaded From : www.EasyEngineering.net

·H6 EngineerinG M~th~ni1!!l'S~

But by p - series test, I VII -= I -I 11"


'~onverges when p > I and di\ c, :'t_ '

when p ~ I
:. I iI" is convergent if /' I and divergent if p ~ I .

6.3.7 Example
" ' (n -1- l).\""
Test the com crgence of till' "cric~ L.. - - -,--.X >0
1/, I 11

ww
Solution

lin =
( n + I) x"
1
( n + 2) X"-l I
;1I1Itl - - - - , - , -

w.E n (11+1)

asy
En ;
I ; ~
1I 1

LI
11->''0
illi rl
--
111/
= ..:
L,
" ,.w II
'
! ,,- _.. ,

gin
: , "" - - - ,

+ 11 ( 1+ 1 )'
i
.x = X

By i'".1i j() test, I UII


n, \.. n
e
converges when x < 1 and diverges when x > 1 . eri
When ..x= I ' IIII = -
n3-
n+ I
ng.
Take v" = -~ ; By comparison test
n
I UII .is convergent ( give proof)
net
.'. I UII is convergent if x ~ 1 and divergent if x > I.

6.3.8 Example
Test the convergence of the series
. ~(n2
(I) L.. -+-2
1) (.. 1 2.5.8 2.5.8.11
II) + - - +
... ) 1 1.2 1.2.3
+ ... (III - + - + - - +
II_I 2" n 1.5.9 1.5.9.13 3 3.5 3.5.7
Solution
(n2 1) n 2 1
(i) I ---;+-2 I-II + I-2
00 00 00

= Let
2
II-I n 2 n 11=1 11=1

Downloaded From : www.EasyEngineering.net


Downloaded From : www.EasyEngineering.net

Sequences of Series 417

U = (n+1)2 .1l11+1 = (n+1)2 ~


11+1 2"rl' 2"tl· 2
U" 11

Lt1111+1
-- _ L
t-. 1(1 +-1)2 --<
_1 1
I1~OO lin 2 11-+00 n 2

By ratio test I Un is convergent .By p -series test, I v" IS

convergent.
(I I v,,)
ww(ii)
:. The given series Un +
Neglecting the first term, the series can be taken as,
is convergent.

w.E 2.5.8 2.5.8.11


--+
1.5.9 1.5.9.13
+
Here, 1sl term has 3 fractions ,2 nd term has 4 fractions and so on .
asy
:. nih term contains (11 + 2) fractions
2.5. 8....... are in A. P.
(n + 2 t En
term = 2 + ( 11 + 1 ) 3 = 3n + 5 ;
1. 5. 9, ....... are in A. P.
( n + 2t
gin
term = 1 + ( n + I ) 4 = 4n + 5
2.5.8 ..... (3n+5)
e eri
U
n
=----'--~

1.5.9 ..... ( 4n + 5)
ng.
2.5.8 ..... (3n+ 5)(3n+ 8)
u + = 1.5.9 ..... (4n+5 )( 4n+9 )
n 1 net
un +1 _ (3n+8) .
-;,:- - (4n + 9) ,

n(3+~)
Lt u +1 = Lt =l
n
!) <1
l1

HOO un HOO n ( 4 + 4

By ratio test, I Un is convergent.


(iii) 1,2,3, ........ are in A. P nih tenn = n ; 3.5. 7 .......... are in A.P. d" term =
2n+ 1

Downloaded From : www.EasyEngineering.net


Downloaded From : www.EasyEngineering.net

418 Engineering Mathematics - I

[
1.2.3 .....n 1
un = 3.5.7 ..... (2n+l)

n 1
U +
1.2.3 .....n(n+l)
= [ 3.5.7 ..... (2n+l)(2n+3)
1
Il
U +
I
=(~)
U l 2n+3

ww ! Lt
n~oo
UIl +I = Lt
n.(l+!)

n(2+~)
n = <1

w.EL:. By ratio test,


Un /HOO

un is convergent.
2

asy
6.3.9 Exercise

EnL
Test for the convergence of the series 1.2.3.(....n )

Solution gin 3.5.7 ...... 2n+ 1

e 1.2.3 ...... n
un = 3.5.7 ...... (
2n+l) ; U n+1 =
1.2.3 ...... (n+l)
3.5.7 .....: ( );
2n+3 eri
= n+l . Lt = Lt n(l+ /~) =!<l . ng.
net
U n +1 U,,+I

un 2n+3' /HOO un HOO n( 2+ %) 2 '


:. By ratio test, LUll is convergent.

6.3.10 Example
~ 1.3.5 .... (2n-l) n I( )
Test for convergence L.. .x - x> 0
11=1 2.4.6 .... 2n
Solution
1.3.5 .... (2n-l) nl
The given series of +ve terms has u" = .x -
2.4.6 .... 2n
1.3.5 .... (2n+l) n
and U,,+I = 2.4.6 .... (2n+2 ) x

Downloaded From : www.EasyEngineering.net


Downloaded From : www.EasyEngineering.net

Sequences of Series 419

2 1 x 2n(1 + 1 )
Lt 1111+1 = Lt ( ~ = Lt 2n .x=x
IHOO U// //-->00 2n + 2 ) IHOO 2n( 1+ 22n)

By ratio test, I u// is converges when x < 1 and diverges when x > I when
x = ), the test fails.
1.3.5 .... (2n -1)
Then U = <1 and Lt u// ;f::. 0
II 2.4.6 ..... 2n 11-700

:. I U/I is divergent. Hence I UII is convergent when x < ), and divergent

ww
when x ~ 1

w.E
6.3.11 Example

Test for the convergence of 1+ -2 x + -6 x 2 + ........ + ( -


2// --2 ) x /I-I + ..... ( x > 0)

Solution asy 5 9 2/1+1

~: :~ E
Om itting 1" tcnn, II, =(
n) x'-' ,(n " 2)

= (2"+1 - 2) x/l. Lt (~) = Lt (211+1 - 2) x ( 2// + 1) X


gin
and 'II,' arc all eve.

U
IHI ( 2//+1 + 1) '/1-->00 u ll e 11-->00· 2"+1 + 1
e1 ri 2/1 - 2 .

2n+1 (1- 12/1) 2// ( 1+ 12/1) n


x ; g.
Hence, by ratio test,
= //~~ [ 2//+1 (I +
I Un
l''2/1+1 )·2/1 (1- 2, 2/1 ).x
converges if x < 1 and diverges if x> 1.
=
net
2/1-2
When x = ), the test fails. Then u/I = -n - ; Lt
2 +1 IHOO
un = 1 ;f::. 0; :. I Un diverges

Hence I Un is convergent when x < ) and divergent x > 1

6.3.12 Example
(3 -4i)"
Using ratio test show that the series I 00

converges
/1=0 n!
Solution
_ (3-4i)" I . _(3 -4i),,+1 / .
un - In! ,un+1 - /(n+l)! '

Downloaded From : www.EasyEngineering.net


Downloaded From : www.EasyEngineering.net

420 Engmeerlng MC3tht::rnatl~~


---------------------------------=--

1II1tl)_ 1 I (3-4i)_0
1 t ( --- ---- <1
II ',,,,llil - ":'h n + 1 - II

Hence, by ratio test, l: 1111 l:onverges.

6.3.13 Example
.
DlsclIss t Ile nature 0 f t he series,
. - 2 x + - 3 x 2 + - 4 x 3 + ........00 (x0
>)
3.4 4.5 5.6
Solution

ww Since x > 0 , the series is of +ve terms;


(n + 1) (n + 2) X/HI
w.E
II

llil = (n+2)(n+3)x > Zln+1 = (n+3)(n+4)

asy[(n
Lt IIn+ I -
II~OO - - ; ; : - (
+ 2)2.X ]
n + 1)( n + 4)

Er n
l gin
2
= Lt n (1 + 7;;)2 .x ] = X.
2
n (1+ Yn + j~2 ) ,
e
HOO

Therefore by ratio test, I Un


eri
converges if x < 1 and diverges if x> 1

When x = 1, the t(.st falls; Then


.
un = (
(n+l)
)( ) ; ng.
Taking v
n
=!. Lt = un = 1 :;t: 0
n n~oo vn
'
n+2 n+3
net
:. By comparison test IU n and I Vn behave same way. But I Vn is divergent

by p-series test. (p = 1);


.. I Un is diverges when x =1
:. I Un is convergent when x < 1 and divergent when x ~ 1

6.3.14 Example
. . ~ 3.6.9 .... .3n.5n
DIscuss the nature of the senes ~ ( )( )
4.7.10 ..... 3n+l 3n+2

Downloaded From : www.EasyEngineering.net


Downloaded From : www.EasyEngineering.net

Sequences of Series 421

Solution
3.6.9 ..... 3n 5"
Here, U = .
n 4.7.10 ..... (3n+l)(3n+2)'
3.6.9 .... .3n(3n + 3)5"+ 1
U II + I = 4.7.1O ..... ( )(
3n+l ) (3n+5
3n+4 );

Lt
11 I
~ = Lt (3n+2)(3n+3).5
--'-----_-'--c-'---,-----_'---:-

11->00 llil 11->00 (3n+4)(3n+5)

ww = Lt
5.9n2 (1 + 2 3" )(1 + 3/3'
Inn
)1 = 5 > 1
2
(1
9n + 4 3n )( + 5, 3n) 1
w.EI f
11->00

6.3.15 Example a
:. By ratio test, lin

syEI
is divergent.

ngi
00

nl -
II
Test for convergence the series

Solution
1I11 = n ; u +1 = (1)-11
n+
I-II
; nee
rin
lI

':;,:' = = (n;I,!" n(n:l)" =~C:J g.n


Lt un+1 = Lt
n~'" U
II
11->00
!.[_l_,]" =O}=O<1
n 1+ 1,'
,'n
e
et
:. By ratio test I Un , is convergent

6.3.16 Example
2 3
I ~ ,for convergence.
00

Test the series


11=1 ~
Solution
2n 3 2(n+ 1)3
Un = ~ ; U II +I = In + 1

Downloaded From : www.EasyEngineering.net


Downloaded From : www.EasyEngineering.net

422 Engineering Mathematics-I

lln+1 = 2(n+l)3 x ~ = (n+l)2 = (1+;~r


un In + 1 2n 3
n3 n
u
Lt --.!!..±l = 0 < 1;
n-+oo Un

:. By ratio test, L u/l is convergent.

6.3.17 Example
2/1 ,
L~
ww Test convergence of the series

Solution
nn

w.E
a un +1 = 2
1111
n 1

syE
+ (n + I)!
~=2 /1
(n + 1)"+1 . 211 n!
( ~ )11
n+ 1
Lt
U
--.!!..±l = 2 Lt
1 ngi =-
2
< 1 (since 2 < e < 3)
IHOO ull Hoo(l+ )-~r e
nee
:. By ratio test, LUll is convergent.
rin
6.3.20 Example
Test the convergence of the series L un where u/I is g.n
(a)
n2 + 1
3/1 +1 (b)
x n- I
--,(a>O)
(2n+lf
et
(c)
1.2.3 ....n )2 (d)
~
( 4.7.10 .... .3n+3
~1+3n
(e) 3n + 7n
3 2
)xn
( 5n 9 + 11
Solution

(a) Lt
n-+oo (~
ull n-+oo
I)
u = Lt [(n+l)2 +1 x3/1-+1]
3n+1 + 1
-
n2 + 1

Downloaded From : www.EasyEngineering.net


Downloaded From : www.EasyEngineering.net

Seq uences of Series 423

1
= -<1
3
:. By ratio test, LUll is convergent.

Lt (u )= Lt [ (2n + 3r x~(2n+lf]
n+ I

l
(b) xn

ww 11-->00 un x
n-->oo

aa
n I
-

w .Ea = Lt
IHOO
2 n (I+!' )a
. 2n .x = X
[ 2a na (l+ }Snf
By ratio test, LUll syE convergence if x < 1 and diverges if x> 1.

When x = 1, the test fails; Then,


ngi If
1I = 1
(2n+
; Taking v = _1a
n
awe have,

ne*e
/I II

Lt (Un) = Lt (_n_)a = Lt 1 0 and finite (since a > 0).


n-->OO Vn n-->oo 2n + 1 n-->oo (
r = _1
2 + ;!~r 2
a

ing
:. By comparison test, L un and L VII have same property
.ne
But p -series test, we have
(i) LVII convergent when a> 1
t
and (ii) divergent when a ~ 1
:. To sum up, (i) x < 1, LU n is convergent Va.

(ii) x > 1, L un is divergent Va.

(iii) x = L
1, a> 1, un is convergent, and

(iv) x = 1, a ~ 1, L un is divergent.

Downloaded From : www.EasyEngineering.net


Downloaded From : www.EasyEngineering.net

424 Engineering Mathematics - I

U I [ I.2.3 ....n(n+I) 4.7.1O .... (3n+3)]2


(c) Lt~= Lt x-----'--------<-
IHoc> ltll IHoc> 4.7.IO .... (311+3)(3n+6) 1.2.3 ....n

= Lt [ (n + I) X]2 =! < I
IHoo 3 ( n + 2) 9
:. By ratio test, IlIlI is convergent

(d)

ww
w.E = LI

aI sy
:. By ratio test,
En r U" is convergent.

tgin
9
(e)
U I
~=
LI LI
lr 3 ( n + 1)3 + 7 ( n + I x
Sn + II ]
xx
3
HOC> lIlI Hoc>

~
eer S( n + 1 + II 3n + 7

= Lt
U
--"..±.!.. = Lt
f3n3(1+
.n
9
)3 +7n2(1+ 1.1)2 Sn9(I+l!'s/ 9) ]
.n x
3n + 7;1
n xX in(I g 3) 1

.
3
Sn (I + V)9 + II
ne
n->oc> U" n->oc>
.n / 3n

= Lt
3n
3
{(I + .1/)3
n
+2_(1 + 1/)2}
3n n i
x
Sn 9 + (1 IV )
/ S 9
n xx =X
t
n->oc>
Sn
9
{( I+ }~r + SI~_} 3n
3
(1 + .%n 3)

:. By ratio test, I Un converges when x < 1 and diverges when x > 1.


When x = 1, the test fails,

Then U = 3n3(1+ jjn) =~ (1+ l~n)


6
" Sn
9
( 1+ IYsn9) Sn (I + IYsn 9)

Taking Vn = ~, we observe that Lt U" = l;f: 0


n "->OC>~ S

Downloaded From : www.EasyEngineering.net


Downloaded From : www.EasyEngineering.net

Sequences of Series 425

:. By comparison test and p series test, we conclude that L un is convergent.


:. L u" is convergent when x::::; 1 and divergent when x > 1.

Exercise - 1(b)

1. Test the convergency or divergency of the series whose general term is :

x"
(a)
n
I Ans: Ixl < legl, Ix ~Ildgt ]
ww (b) nx n-I .......................... . I Ans : Ixl < legt, Ix ~Ildgt I

w:n.E
(~:.
(c) X
"' ••••• I Ans : Ixl < legl, Ix ~Ildgt I

(d) (:::: )X" ......... . asy [ Ans : Ixl < legl, Ix ~Ildgt I

lfl En
(e)

4"·lfl
gin [ Ans: cgt.]

(f)
nil e eri
[ Ans: dgt.]

(g)
( n 3 +1 )"
(3 + 1)
n
[ Ans: cgt.] ng.
2. Determine whether the following series are convergent or divergent :
net
x x2 x3
(a) - + - + - + ..............
1.2 3.4 5.6
(Ans: Ixl::::; legl,lxl > ldgt ]

x2 x3
(b)
X
1+-2+-2 +-2 + ............. .
234
I Ans : Ixl : : ; legt, Ixl > Idgt ]

1 x2
(c)
1.2.3 4.5.6 7.8.9
X
- - + - - + - - + ..... . ( Ans : Ixl::::; legt, Ixl > Idgl ]
2 3 n
x
(d)
X

2
X
1+-+-+-+ ..... - 2- + ....
5
X

10 n +1
(Ans: Ixl::::; legt,lxl > ldgt J

1.2 2.3 3.4


(e) -+-2 +-3 + .............. . [Ans: Ixl > legt,lxl::::; Idgt]
x x x

Downloaded From : www.EasyEngineering.net


Downloaded From : www.EasyEngineering.net

426 Engineering Mathematics - I

6.4 Raabe's Test

Let I u" be series of +ve terms and let E!. {n(:.:, - I)} ~ k
Then
(i) If k > 1, ~>n is convergent.
(ii) If k < 1, L un is divergent. (The test fails if k = 1)
Proof:

ww Consider the series

w.E
asy
En
gin
eer
Case (i): ing
In this case,

Lt n{~-l} =k .ne
n-+oo U +
n 1

We choose a number 'p' 3 k > p > 1 ; Comparing the series


>1

L U" with L Vn
t
which is convergent, we get that L un will converge if after some fixed number
of terms
un Vn (n+ l)P
U +
n 1
> Vn+1 = -;;-
i.e. If.,
Un)
n ( - - 1 > p+
p(p-l) 1
.-+ ......... from (I)
Un+1 L2 n

i.e., If Lt n(~-I» P
n-+oo U +
n 1

Downloaded From : www.EasyEngineering.net


Downloaded From : www.EasyEngineering.net

Sequences of Series 427

i.e., If k > p, which is true. Hence Llln is convergent .The second case also
can be proved similarly.

Solved Examples

6.4.1 Example
Test fe>r convergence the series
1 x3
x+-.-+-.-+--.-+ .....
1.3 x 1.3.5 x 7

2 3 2.4 5 2.4.6 7
Solution
ww Neglecting the first tern ,the series can be taken as ,
1 x 3
1.3 x5
1.3.5 x
7

w.E -.-+-.-+--.-+ .....


2 3 2.4 5 2.4.6 7
1.3.5 .... are in A.P. nih term = 1+(n-1)2=2n-1

asy
2.4.6 ... are in A.p. n'h term = 2 + (n -1) 2 = 2n

En
3.5.7 ..... are in A.P nih term = 3+(n-1)2 = 2n+ I

gin
uti ( nih term of the series) =
21l 1

eer 1.3.5 .... (2n-1)


=----'----;------::--"--
2.4.6 .... (2n)
X

2n+1
+

=
1.3.5 .... (2n-1)(2n+ 1)
2.4.6 .... (
2n)2n+2
() .
X
-
2n
+
3
ing
1l1l+1

1.3.5 .... (2n+1) X 2n +3 2.4.6 ... .2n (2n+1)


2n+3
.ne
lIn+1 _

-;;:- 2.4.6 .... (2n+2)"(2n+3)"1.3.5 .... (2n-1)" X 2n + 1


(2n + 1)2 x 2
t
( 2n + 2) ( 2n + 3)

4n-1 ( 1+-- 1)2


II 2n 1
LI ~ = LI x
2
= X-
IHOO Un n~oo 4n 2
( 1+ 22~ )(1+ ~~ )

:. By ratio test, LU" converges if lxl < 1 and diverges if Ixl > 1
If Ixl = 1 the test fails.

Downloaded From : www.EasyEngineering.net


Downloaded From : www.EasyEngineering.net

428 Engineering Mathematics - I

x2 =1 and ~ = (2n + 2)( 2n + 3 )


Then
1111+1 (2n+ 1r
~ -1 = (211 + 2)( 2n + 3) _ 1 = 6n + 5
111/+1 (2n+1)2 (2n+l)2

LI
tHOO
{ (
n -
ul/ -
ul/+
1
1J} = l I ( 6n
2
+ 5n
I/~OO 4n 2 + 4n + 1
1
~)
ww
2
n ( 6+ 3
= Lt =- > 1
(4 + ~n + _n-\ ) 2
w
By Raabe's test, .EI a UI/
11-->00 n 2

converges. Hence the given series is convergent when


Ixl ~ 1 an divergent when
syE Ixl > 1 .
6.4.2 Example
Test for the convergence of the series
ngi
3 3.6 2
1+-x+--x +
7 7.10
3.6.9 3
7.10.13
x + ..... ;x> 0
nee
Solution
Neglecting the first term, rin
ul/ =
3.6.9 .... 3n
7.10.13 .... 3n+4
.x
1/
g.n
ul/+ 1
3.6.9 .... 3n(3n+3)
= 7.10.13 .... ( ) ( 3n+7 ).x
3n+4
1/+1
e t
lIl/+1 = 3n + 3.x ; Lt lIll+1 =X
lIl/ 3n + 7 1/-->00 lIlI

:. By ratio test, I UII is convergent when x < I and divergent when x > I.
When x = I The ratio test fails. Then
u 3n + 7. lin 1 _
ll _ 4
1I1/+1 - 3n+3' lIn+1 - - 3n+3

LI {n(~-l)}= Lt (~)=i>l
11-'>00 lln+1 11-->00 3n+3 3

Downloaded From : www.EasyEngineering.net


Downloaded From : www.EasyEngineering.net

Sequences of Series 429


--~-----------------------------------------------------------

:. By Raabe's test, LUll is convergent .Hence the given series converges if x ~ 1


and diverges if x> 1.
6.4.3 Example
Examine the convergence of the series
f 12.5 2.92.... (4n-3)2
11=1 42.g2.122 .... (4n)2
Solution
12.5 2.92•••• ( 4n - 3)2
ww U
II
= ---------'-------'-
~ ~
4-.8-.12-.... 4n )2 7 (

w.E litH
12.5 2.9 2.... ( 4n -
3)2 (4n + 1)2
1= -~--~---2--'---(-----')2'---(--'--------'-)2-
4-.8-.12 .... 4n 4n+4

Lt u 1
asy t
Lt
( 4n + 1)2
~
En = 1 (verify)
lI
+ =
n->oo un 11->00 ( 4n + 4

:. The ratio test fails.


Hence by Raabe's test, LU gin is convergent.

eer
n

6.4.4 Example

Find the nature of the series L (l!:!)2 xn


, (x > 0) ing
Solution
~
.ne
ull =
(l!:!)2
~
II (l~±l)2 11+1
.x ; un + 1 = 12n+2 .x t
ulI +1 _ (n+l)2 X'

ull (2n+l)(2n+2) '

n2 ( 1+ 1/ ')2
Lt u n +1 = Lt ;' n .x = ~
11->00 U
n
n->004n2(1+}/
, 2n
)(1+2/)
2n
4
:. By ratio test, L un converges when:! < 1, i. e ; x < 4; and diverges when x >4;
When x = 4, the test fails.

Downloaded From : www.EasyEngineering.net


Downloaded From : www.EasyEngineering.net

430 Engineering Mathematics-I

x = 4 ~ = (2n + 1)( 2n + 2 )
When
- '1l11+1 4(n+l)2
U -2n-2 -1
_" -1=
lln+1 4 (n + 1 r = .
2 ( n + 1) ,

LI
II-->if.>
[n(~-11]= -12 <1
li
ntl

:. By ratio test, I is divergent

ww I
lin

Hence Un is convergent when x < 4 and divergent when x> 4

w.E
6.4.5 Example

Test for convergence of the seriesasy ,,4.7 .... (3n+ 1)


~
n
x-

En 1.2.3 .... n
Solution
gin
_ 4.7 .... (3n+1) n. _ 4.7 .... (3n+l)(3n+4) n+1
lin -
1.2.3 ....n
X 'U n + 1 -

eer ()
1.2.3 ....n n + 1 .
X

II
Lt ~ = Lt [(3n+4) .x]
n-->oo Un
n-->oo ( n + 1)
= 3x ing
:. By ratio test I converges if 3x <1 Le., X <! and diverges if x .n>!e;
t
Un
3 3
If x= !, the test fails
3
1 [ [(n+ 1)3 ] [-1 ] 1
When
x ~:3' n u.:, -1
II ]
~n 3n + 1 -1 ~ n 3n + 4 ~- (3+ ~ 1
Lt
n--><Xl
n[~-I] = -!3 < 1
II
11+1

:. By Raabe's test, IU n is divergent.

:. " lin
~ . convergent wl
IS len I - and 'Ivergent w Ilen x
x <d 1
~-
3 3

Downloaded From : www.EasyEngineering.net


Downloaded From : www.EasyEngineering.net

Sequences of Series 431

6.4.6 Example
1
, 3x 4x 2 5x
1 est for convergence 2 + - +- + ~ + ........... (x > 0 )
2 3 4
Solution

The nih term II =(n+l)x tl - I •


H '
11

_ (n+2) .tl. II'HI _11(11+2)


Hel
II - (11+1) X '--;;,:- (11+1)2 .x

ww Lt
11
_H_' I = LI
1(1+ 2)
lr

11 + )-
(1 1 11 .x = 1 X

w.IUE
:. By ratio test,
H-N, 11
tl
H~-'UJ 2

is.convergent if x < I and divergent if x > I


11

Ifx= I, the test fails. asyII

1]
[ E
II (11 + 1)-
Then Lt
1n [
11 _tl - 1 = Lt 11 -1
II--'>UJ

g[in1] IIl/el ,I/---W' n(n+2)

III//
e eri
=LtI1
11->00 11 ( 11 + 2 )
=0<1

:. By Raabe's test

:. I
is divergent
ng.
6.4.7 Example
.
Zlil is convergent when x < I and divergent when

.
3 3.6 3.6.9
x 21
n et
FlI1d the nature of the series - + - + + ......00
4 4.7 4.7.10
Solution
3.6.9 ..... 3n 3.6.9 ..... 311(311 + 3)
1I 11 = 4.7.10 ..... ();1I1/+
3n+l = 4.7.10 ..... ( )(
311+1 )
311+4
1

3 3 3n(I+3 )
1111+1 =~. LI llll+1 = LI 3n = 1
ZlII 3n + 4' /1--'>00 llll 11->00 3n( 1+ 4 )
3n
Ratio test fails.

LI 11 { - -
[ ,lll/+1
tl--'>oo
Zltl I}] - LI [ n (3n+4
/1--'>00
---
3n + 3
I)]
Downloaded From : www.EasyEngineering.net
Downloaded From : www.EasyEngineering.net

432 Engineering Mathematics - I

11 n 1
= Lt = Lt =- <1
,,->003(n+1) 11->003n(1+,]n) 3

:. By Raabe's test L U" is divergent.

6.4.8 Example
If p, q > () and the series
1 p 1.3.p(p+1) 1.3.5 p(p+1)(p+2)
1+"2 q + 2.4.q(q + 1) + 2.4.6 q(q + l)(q +2) + ....

ww is convergent, find the relation to be satisfied by p and q.


Solution

w .Eaun =
1.3.5 ..... (2n-1) p(p+l) ..... (p+n-1)
2.4.6 .... .2n q(q + 1) ..... (q + n-1)
. I
[negle"tmg l' term 1

lI"tl =
syE
1.3.5 ..... (2n-1)(2n + 1) p(p+ l) ..... (p + n -l)(p+ n)
2.4.6 .....2n(2n+ 2) q( q + 1) ..... ( q + n-1)( q + n)
U,,+l _

--;z- -
(2n+1) (p+n). ngi
(2n + 2) (q + n) ,
nee
~ = Lt [2n ( + 12n ) n (1+ ~'~ )rJ= 1
1
Lt
lI" 2n( 1+ 1"2n)" n( 1+ ~~) ing
.ne
11->00 11->00

:. ratio test fails.


Let us apply Raabe's test
t
,~.H::':, ~1)]~ ,~.H ~::~)~~::~~ ~1}]
Lt [n {2q ( n + 1) - ~ (2n + ) + nIl
1
11->00 n 2 ( 1+ ~/~ ) ( 2 + },~)

r
Lt 2q(1+ )~)-p(2+ J~)+11=2q-2p+l
II->ool 2 2

Downloaded From : www.EasyEngineering.net


Downloaded From : www.EasyEngineering.net

Sequences of Series 433

~ . 2q-2p+l
Since L..,.llil IS convergent, by Raabe's test, >1
2
=> q - p > Yi, is the required relation.

Exercise 1 (c)

00

1. Test whether the series I Un is convergent or divergent where


I

ww II =
2 2.4 2.6 2..... ( 2n - 2 )2
X
2n
[ADS: Ixl:::; legl, Ixl > Idgl ]
w
f .E
II 3.4.5 ..... (2n-I)(2n)
2. Test for the convergence the series

lfl as
4 .7 . 10 ..... (3n+l) x" 1 I
[ADS: Ixl < -egl,lxl ~ -dg/]
I
yE 3 3

ngi
n eer [ Ans : divergent]

(ii) 3.4
- x +4.5
1.2 2.3
5.6 3 + ..... ( x> 0)
- x 2 +-x
3.4 ing
~ 1.3.5 ..... (2n -1)
.ne
[ ADS: cgt if x :::; 1 dgt if, x > 1 ]

(iii) L..
2.4.6 ..... 2n
xn (
. ( ) x>O
2n + 2
)

[ ADS: cgt if x :::; 1 dgt if, x > 1


t
2 3
(iv) 1+ (U)2
II
X + (1l)2 x
l1
+ (lJ)2 x
l2
+
..... .
(x > 0)

[ ADS: cgt if x < 4 and dgt if, x ~ 4

6.5 Cauchy's Root Test


I'
Let L un be a series of +ve terms and let LI u/n = I . Then
n-->co
L un is convergent

when I < 1 and divergent when I> 1

Downloaded From : www.EasyEngineering.net


Downloaded From : www.EasyEngineering.net

434 Engineering Mathematics - I

Proof:
I,' II
(i) Lt u/" =1<1=>3a +venumber 'A'(!<A<1):HI,/" <A,Vn>rn
I1~OO

(or) u ll <A",Vn>rn
Since A < 1, I A" is a geometric series with common ratio < I and
therefore convergent.
Hence I UII is convergent.
II
/11 = I> 1
ww (ii) Lt
11-->00
UII

u/ > 1Vn > r


'II

w :. By the definition of a limit we can find a number r 3


i.e.,
.Ea 2>11
ull > Vn > r
i.e., after the 1sl 'r ' terms, each term is> 1.

Note:
syE Lt
11-->00
= 00 :. LUll is divergent.

When Lt{ In)


n-->oo
n Un =

gin
1, the root test can't decide the nature of LUll' The fact

of this statement can be observed by the following two examples.

I Yn3n :-LfU'/" =Lt 1/


e (
eri
1 ) J~
= Lt ( V1 )3 =1
1. Consider the series

L Yn,
11-->00 11-->00
-3
n
ng.
11-->00 nl II

2. Consider the series

In both the examples 'given above,


in which LfUlly" = Lt ~ = 1
11-->00

1/
11-->00

LfU/" = 1. But series (1) is convergent


n11l net
(p-series test)
And series (2) is divergent. Hence when the limit= 1, the test fails.

Solved Examples

6.5.1 Example
Test for convergence the infinite series whose nIh terms are:
(i) 1 (ii) 1 (iii) 1
n'" Oogn)" [1 +~r

Downloaded From : www.EasyEngineering.net


Downloaded From : www.EasyEngineering.net

Sequences of Series 435

Solution

, 1
. Lt u 1n
Lt -n2 = 0 < 1·
= lI~oo
, n~co II '

By root test I Un is convergent.

1 ~. 1 ,. 1
(ii) u = ·u n = - - . Lt u 'n = Lt --=0<1·
n (log n)'" n log n ' n->oo n n->oo log n '
By root test, I Un is convergent.

ww
(iii)

w.E
asy
En
gin
6.5.2 Example
eer
Find whether the following series are convergent or ,) i vergent.

(i)
'" 1
I-n-
-1
n=13 ing
Solution
.ne
t
1/
In

1/
7n

1 1
Lt un Yn = n----+oo
Lt =-<1·
n~ct)
3 '

By root test, I Un is convergent.


111
(ii) 1+-2 +3+-4 + .....
234

Downloaded From : www.EasyEngineering.net


Downloaded From : www.EasyEngineering.net

436 Engineering Mathematics - I

Solution

=-1 ' Lt U 1/ = Lt ( - 1 )}~ = 0 1


1I in <
11 nil 'Il~OO n Il~OO nil

By root test, I lin is convergent.

(iii)

Solution

ww U =
[(n+1)x r
-'=--'----':--=----
n"+1
w.E
n

r
II

Lt u};, = Lt [{(n+1)x ],'n


n---j.c:/J

asy
n Il~OO n"+ 1

X}"E ~
Lt [{ ( n + ] )
n-->oo n n 1]),
.-
n
_

gin
-
( n + 1)
Lt -
n-->oo
1
- x ' I-'
n nl n

(l+!)X.+, = Lt x.+,=x
Lt
n n'II n-->oo n/ n
n-->oo
e eri
:. I Un is convergent if Ixl < 1 and divergent if Ixl > 1 and when Ixi = 1 the test ng.
fails.

Then Take vn =-
1 net
n

vn n"+ 1 nn
r
Un = (n + 1 .n = (n + 1 = (1 + !)n ;
n
r U
Lt-"=e>l
11-->00 VII

:. By comparison test, I Uti is divergent.


(I Vn diverges by p -series test)
Hence I Un is convergent if Ixl < 1 and divergent Ixl ~ 1
6.5.3 Example
,/
If un = n ,,2' show that I Ull is convergent
(n+l)

Downloaded From : www.EasyEngineering.net


Downloaded From : www.EasyEngineering.net

Sequences of Series 437

Ill

l
2
I n n II n
Lt u" " = Lt /1
2 ; = Lt = " = Lt (-)-"
"~a) II~OO n + 1)" ,,~oo
( n + 1)
J ,,~oo n + 1 (

=
,,~OO l I e
+-
L.
~
Lt [_1_]" = < 1 ;:. "u" converges by root test.
n

ww
6.5.4 Example

w.E
Establish the convergence of the series r
~ + ( % + ( % + .' ...... J
Solution

U
11
= as . . . . .
(_n_)"
2n+l
yE (verify);

Lt u };,
,,~oo n II~OOn
= Lt (_n_)=!<1
2n + 1
gin
2
I
By root test, UII is convergt:nt.
e eri
6.5.5 Example

Test for the convergence of Ioog


--.x
/I ng.
Solution
11=1 n+ 1
net
U/I = [_11 ]~ .x"; Lt u"·v,,
,,~oo
= Lt
Il---+oo
[_11 JI x =x
1+- 1+-
n n
:. By root test,IU is convergent if Ixl < 1 and divergent if Ixl > 1.
n

When Ixl = 1 : un = J n ,taking vn = ~ and applying comparison test, it can


n+I n
be seen that is divergent
I U/I is convergent if Ixl < 1 and divergent if Ixl ~ 1 .

Downloaded From : www.EasyEngineering.net


Downloaded From : www.EasyEngineering.net

438 Engineering Mathematics - I

6.5.6 Example

Show that ~ ( n~ _ I) n converges.


Solution

Un =( nYn -If
Lt
n~oo
unYn = Lt
n . . . . . ct)
(n~-I)=I-I=O<I(since Lt
1l~Cf)
n'~:' =1);
:. L is convergent by root test.

ww
Un

6.5.7 Example

w.E
. the convergence 0 f the series whose nI h
Examine term·IS ( n + 2 )n .Xn
--
n+3
Solution
asy
Un = ( n+ 2 )n .xn; (nn+3
2
n+3 En Lt
n~
unYn = Lt
n~<XJ
+ ) X = X

:. By root test, L un converges when


gin2)n
Ixl < 1 and diverges when Ixl > 1

( e
When Ixl = 1 : un = (n + 2) ; Lt Un
n

= Lt
1+-
nn eri
n+3 n~<XJ n~<XJ ( 1+-3 )
ng.
e2
- 3 = - '" 0
e
I
e
n

and the terms are all +ve .


net
:. L un is divergent. Hence L un is convergent if Ixl < 1 and divergent if
Ixl~l.
6.5.8 Example
Show that the series,

[~: -HI +[~: -H' +[:: -H +. . . 3

isconvergent

u" ~ [(n :":t -n: r~ (n: T[( n: r r 1 1


-1

Downloaded From : www.EasyEngineering.net


Downloaded From : www.EasyEngineering.net

Sequences of Series 439

~ 1 1 1
Lt un II =-.--=--<1
IHOC) 1 e-l e-l
LUll
ww :. By root test, is convergent.

w
6.5.9 Example

Llllll .E
00

for convergence when um =


e
-Ill

asy (1+2 mfll


Test 2

111=1

Solution En
gin
eer
Hence Cauchy's root tells us that LU is divergent. ing
m

.ne
6.5.11 Example:
Test the convergence of the series '" ~
~ 112
e
t
Solution
II
II n/n
Lt
11-)-00
U /n
n
= n--.r:1J
Lt - = 0 < 1
en
:. By root test, LU n is convergent.

6.5.12 Example
2 32 2 .
Test the convergence of the senes, -2 x + - 3 X + ...
( n + 1 .x
I
r l
+ ...... , x > 0
1 2 n'H

Downloaded From : www.EasyEngineering.net


Downloaded From : www.EasyEngineering.net

440 Engineering Mathematics - I

Solution

n+I ) 11 .xn ]);, n+I 1


Lt unYn
I
= Lt [(
= Lt [ (- - ) .-/.x]
n~oo II~OO n"+ 1 II~OO n n)'n

= Lt
n~oo
[( + -1). 1]
1
n
I/.x
n l II
= [.
I.I.x = x SInce Lt n/Vn = 1]
II~OO

:. By root test, L un converges if x < 1 and diverges when x> 1 .


When x = 1 , the test fails.

ww Then II11
(
I)n _.
= 1 +- 1
n
1
. , Take vn =-
n n

w.E Lt un = Lt (1 + !)n
n
=e 0 :;t:

asy L
n~oo Vn II~OO

By comparison test and p-series test, un is divergent.


Hence L un En
is convergent when x < 1 and divergent when x ~ 1.

gin Exercise 1 (d)

eer d h terms are:


1. Test for convergence the infinite series whose

(a)
1
.............................. ing
[ Ans : convergent]
2" -1
1 .ne
(b) ---'2::--n • (
(log)
3n+ 1 )n
n :;t: 1) [ Ans : convergent]

4 4
t
(c) ( 4n+3· x ................................ . [Ans: Ixl < -cgt,lxl
3
~ -dgt ]
3

(d) [ Ans: cgt for all x ~ 0)

(e) [ Ans: convergent)

(t) [ Ans : convergent]

Downloaded From : www.EasyEngineering.net


Downloaded From : www.EasyEngineering.net

Sequences of Series 441

(g)
(2n2 -I r [ Ans: convergent]
(2n )211

(h) (n}~ -I rn [ Ans: convergent]

-n) . . . . . . . . . . . . . . . . . .
_112

n-I
(i) (
[ Ans : divergent]

ww (j) (n:l)"'(x>o) ........................ . [ Ans : x < 1 cgt, x ~ 1 dgt ]

w.E
2. Examine the following series for convergence :
x x2 x3
(a)
asy
1+2+)2+43 + ..... ,x>O ................. . [ Ans: x ~ 1cgt,x > Idgt ]

~+(H +UoJ +
. . . .E. . . . . . . . . .. [ Ans : convergent]

6.6
(b)
ngi
6.6.1 Integral Test
+ve term series, nee
¢(1)+¢(2)+ ..... +¢(n )+ .....
rin
where ¢ (n) decreases as

r g.n
n increases is convergent or divergent according as the
integral
Proof:
¢ (x) dx is finite or infinite.
e t
Let Sn = ¢(l) +¢(2)+ ..... +¢(n)
From the above figure, it can be seen that the area under the curve y = ¢ (x)
between any two ordinates lies between the set of exterior and interior rectangles
formed by the ordinates at
n = I, 2, 3, .....n, n + I , ..... .
Hence the total area under the curve lies between the sum of areas of all interior
rectangles and sum of the areas of all the exterior rectangles.
Hence
{¢(l) + ¢(2) + ..... + ¢( n)} ~ r+ ¢(x)dx ~
1
{¢(2)+ ¢(3) + ..... + ¢( n+ I)}

.. Sn ~ r ¢(x)dx~Sn+I-¢(l)

Downloaded From : www.EasyEngineering.net


Downloaded From : www.EasyEngineering.net

442 Engineering Mathematics - I

v= O(n)

P,

P2
0
1

ww Q
Bntl
Cn +1

w
°3
°11~1
°Il
.Ea
0
AI
1 2
A2
3
A)
n
An
n
~
+1

As n ~ 00, syE
Lt SI1 is finite or infinite according as [ ¢ (x)dx is finite or infinite.
Hence the theorem. n gin
6.6.2 Example
Solved Examples
e eri
Test for convergence the series I ---
00 1
ng.
Solution

r 1 It [ r' 1 dx]
dx - n~oo
n logn
n=2

= It [log log
n
x]~ = 00
et
- xlogx 1 xlogx 11~00
By integral test, the given series is divergent.
6.6.3 Example

Test for convergence of the series

Solution

dx - It [f-
p 1

[ _1
xP x
1
dx]
11~00 P
It [ x- + ]11
n~oo - p + 1 I

= _1_ Lt [ n l - p -1 ]
1- P IHOO

Downloaded From : www.EasyEngineering.net


Downloaded From : www.EasyEngineering.net

Sequences of Series 443

ClIse (i)
,,1 .
If P > I, this limit is finite; L..J - IS convergent.
n"
ClIse (ii)
,,1 . .
If P < 1, the limit is in finite; L..J- IS dIvergent.
n"
Ca.\·e (iii)

Ifp= 1, the limit Lt logxl;' = Lt (logn)=oo;:. I_l- IS divergent.


11...--.)00 11-)00 n"

ww Hence I_l-
n"
is convergent if p > I and divergent if p :S

6.6.4 Example
w.E"-2
Test the series
00 n
for convergence.

Solution asy ~e"

¢ (n)
E1111 =
n
-2

e"
= ¢(n){say);
ngi
n increases. So let liS apply the integral test.
is +ve and decreases as

I e-,2 =! le- dt{t=x


nee 2

r
1
1¢(x){lx= x dx ,dt=2xdx}

1 iI.nh·
I I 2I

= -"21e 00
-I 1
I = -"21(0 --;;1) = 2e ' W lIC g.n
IS fiillIte.
.

By integral test,
6.6.5 Example
IlIl1 is convergent. e t
1
Apply integral test to test the convergence of the series "-sin - 00 (7f)
~n2 11

Solution

Let ¢ (n) = ~2 sin ( : ); ¢ (n) decreases as 11 increases and is +ve .

J¢(x)dx =
00 00
J-21 sin
(}x
7f
2 2 X X

Downloaded From : www.EasyEngineering.net


Downloaded From : www.EasyEngineering.net

444 Engineering Mathematics - I

Letix =t

TC
J
-! sintdt = !costl~/=!
~ TC /2 TC
finite, _TC/
/ x
2dx = dt

Yx2dx=- ~dt
:. By integral test, I Un converges x = 2 => t = ~ x = 00 => t = 0
6.6.6 Example
Apply integral test and determine the convergence ofthe following series.
ww
w.E
(a) (b) (c)

Solution
(a) ¢ (n) - asy
3n is +ve and decreases as n increases
- 4n 2 + 1
En
~x dx (4X +1=t=>Xdx= Ysdt )
j 2
j¢(x)dx=
I 14x +1 gin
x=l=>t=5,x=00=>t=00

[3 [3 ee]
I¢ ( x ) dx = Lt
00

I
n~oo

I
r
85 t
dt] = Lt -log t -log 5 = 00
- If-
n-->oo 8
ing
:. By integral test,

~n3
Un diverges.
.ne
(b) ¢ ( n) =

j¢(x)dx =
3n +2
decreases as n increases and is +ve

1~X3 dx
t
I 13x +2
1 OOfdt 1
= (; t
= (;[logt]5 = 00
00 4
[where t= 3x + 2]
5

By integral test, I Un is divergent.

(c) ¢ (n) = _1_ is +ve, and decreases as n increases.


3n+l
00 1 1 dt
00 1 00

f¢(x)dx= f-dx= f--[t=3x+l]=-logtl~=00


I I 3x+ 1 4 3 t 3

Downloaded From : www.EasyEngineering.net


Downloaded From : www.EasyEngineering.net

Sequences of Series 445

:. By integral test, I Un is divergent.


6.7
6.7.1 Alternating Series
A series, u l - u2 + u3 - u4 + - + ..... + ( -1 ),,-1 un + ..... , where un are all +ve,
is an alternating series.
6.7.2 Leibneitz Test

If in an alternating series I (-1 ),,-1 .un ' where un are alI +ve,

ww
(i)
(ii)
un > un+I ' Vn, and
Lt ull = 0, then the series is convergent.

w.E
Proof:
Let u l
n ........ oo

u2 + u3 - u4 + ...... be an alternating series (' un 'are all +ve)


asy
-

Let ul > u2 > u3 > u4 •••••• , Then the series may be written in each of the following
two forms:
(UI -U 2)+(U3-U4)+(U5-u6)+····· En ......... (A)

ul gin
-(u 2 -U3 )-(U4 -u5 )-····· ............ (B)
(A)
(B)
Shows that the sum of any number of terms is +ve and
Shows that the sum of any number of terms is < ul . eer
Hence the sum of the series is finite. :. The series is convergent. ing
Note:
If Lt ull -:t:- 0, then the series is oscillatory. .ne
t
n--.<XJ

Solved Examples

6.7.3 Example
Cons!·derthe series
. 1--+---
1 1 1 .....
234
In this series, each term is numerically less than its preceding term and nih term
~O as n~oo.
:. By Leibneitz's test, the series is convergent.
(Note the sum of the above series is Loge2 )
6.7.4 Example

Test for convergence L (-I ),,-1


2n-1

Downloaded From : www.EasyEngineering.net


Downloaded From : www.EasyEngineering.net

446 Engineering Mathematics - I

Solution

The given series is an alternating series I (-1 ),,-1 un ,where U


n
= _I_
2n-I
We observe that (i) un > 0, Vn (ii) un > Un+ I ' Vn (iii) Lt
/1---700
Un =0
:. By Leibneitz's test, the given series is convergent.
6.7.5 Example
111
Show that the series S - 1- - + - - - + ...... converges.
3 9 27

ww
Solution

~
( ),,-1
w.E The given series is

series in which 1.
~
-1 1
3n-
> 0, Vn 2.
= "'(_1)/1-1
L.J
u"
I
where

> U/I+1' Vn and 3. Lt


u, = _1_1 is an alternating
I 3/1-
= 0;

asy
un un un
ll---7oo

Hence by Leibneitz's test, it is convergent.


6.7.6 Example
En
gin
X 2
Test for convergence of the series, - - - ~ + - - 3 - +..... , 0 < x < 1
1+x 1+x- 1+x
x3

Solution
n-1
( -1 ) .xn
e
n-1 eri
The given series is of the form'"
L.J
= "'(-1) u,
1+ XII L.J
ng.
n

where UII=--II
. I+x'
x"
Since O<x< 1, ulI>O,Vn;

x" X"+1
net
Further, U -u =--
n ----
II n+1 l+x I+XIl+1
x" _X"+1 x" (I-x)
=--:-----:--:------;-

( 1+ x" ) ( I + x"+ 1 )
0< x < 1 ~ all tenns in numerator and denominator of the above expression are
+ve.

Again, x" ~ 0 as x" ~ 00 since 0 < x < 1; .'. Lt U = --.


o =0
II~OO II 1+0
:. By Leibneitz's test, the given series is convergent.

Downloaded From : www.EasyEngineering.net


Downloaded From : www.EasyEngineering.net

Sequences of Series 447

6.7.7 Example
(-1 ),,-1
L ----r========
00

Test for convergence


1I~2 ~11(11+1)(n+2)
Solution
The given series is an alternating series "(-1)"-1
L..J ull
1
where
1111 = ~11 ( 11 + 1)( 11 + 2) ; u > 0, \1'11 ;
ll

wwAgain, ~(11+1)(n+2)(n+3) >~n(I1+1)(n+2)

w~(11+1)(n+2)(11+3)
.Ea
1 1
< JI1(I1+1)(11+2) ,\1'11
I.e.,

Further, Lt u
syE = Lt
1
=0
IHOO

By Leibnitz's test,
II

n 11-->00 J 11 ( 11 + 1)( 11 + 2 )

f( -1)"-1 un gin
is convergent

6.7.8 Example
2
e eri
Test for the convergence of the following series,
1 2 3 4 5 ng.
Solution
---+---+--+
6 11 16 21 26 ....
net
11
Given series, L (-1)
00

n=1
n-I
- - = L (-1)
511 + 1
II-I
Un is an alternating series

11
u =-->0\1'11 .
n 511 + 1 '
11 11+1 -1
----- ( )( ) ~ Un < Un+ I' \1'11
511+1 511+6 511+1 511+6
11 1
Again, Lt u = Lt --=-:;t:0
n-->oo n IHOO 511 + 1 5
Thus conditions (ii) or (iii) of Leibnitz's test are not satisfied. The given series is
not convergent. It is oscillatory.

Downloaded From : www.EasyEngineering.net


Downloaded From : www.EasyEngineering.net

448 Engineering Mathematics - I

6.7.9 Example
Test the nature of the following series.

(a) I
00 ( -1 r- I

(b) I --'--::--'r---
( -1
2
I

(c)
1 f;z +J;+i n +1
Solution
1
(a) u,,= I ~>OVn
vn+vn+l
1 1
f;z + J;+i - J;+i + J n + 2
ww U" - U,,+I =

w.E
asy
:. By Leibnitz's test the series converges.
1 1 1
(b)
n +1 En
u" =-2->0,Vn; - 2 - > 2
n +1 (n+l) +1
=>U" >un+I,Vn;

Lt
n-->oo
UI/
gin=0 :. By Leibnitz's test, given series converges.

(c) un = I ~.l
~ eer
1
1 > 0, Vn ;

In + 2 > In + 1 => I ~
1 1
< I ~.1 1 => UI/ > lln+P V n ing
.ne
I ')

~~
By Leibnitz's test, given series converges.
6.7.10 Example

Test the convergence of the series


111
sJ2 - sJ3 + sJ4 - +..... .
t
Solution
(_1)"-1 1
I
00

The series can be written as -'-;===== U =---;==


n=1 sJ;+i /I SJn+l
(i)

(ii)

(iii) Lt un
n-->oo
=0 .
'

By Leibnitz's test, the given series converges.

Downloaded From : www.EasyEngineering.net


Downloaded From : www.EasyEngineering.net

Sequences of Series 449

6.7.11 Example
Test for convergence the series, 1- h + ~ - ~ + .....
Solution

The given series can be written as


(-If (omitting \'1 term)
I--
2n
1 1 1 1
->OVn; ->--=>u >lll/+I'Vn; Lt -=0 lI
2n 2n 2n + 2 2n IHOO

ww I--
:. By Leibnitz's test,
(-If
2n
is convergent.

6.7.12
w.E
Example
1 1
Test for convergence the series, 1- - + - - - + ..... .
1

Solution asy 3! 5! 7!

En
General tenn of the series is /-1)"-;
2n-I !
gin
The series is an alternating series; (
2n-I ! e
1 ) > OV n

eri
1
( 2n - I!
)
>(
1
) => Un> Un+ I' Vn EN; Lt (
2n - 1 ! IHOO
1
) =0
2n - 1 ! ng.
By Leibnitz's test, given series is convergent. net
6.8 Absolute convergence
A series IUII is said to be absolutely convergent if the series I IU I IS
II

convergent
6.8.1
Consider the series
1 1 1
IU =1--3 +3"--3 +-+ ......
II
234
1 1 1 1 00

Ilun l=1+-3 +3"+-3 + ...... = I-3


2 3 4 I n

By p - series test, I I
IUn is convergent (p = 3 > 1 )

Downloaded From : www.EasyEngineering.net


Downloaded From : www.EasyEngineering.net

450 Engineering Mathematics - I

Hence I Un is absolutely convergent.


Note:
1. If I Un is a series of +ve terms, then I lIlI = I IUI/ I·
For such a series, there is no difference between convergence and ahsolute
convergence. Thus a series of +ve terms is convergent as well as absolutely
convergent.
2. An absolutely convergent series is convergent. But the converse need not he
true.

ww Consider I (_1)"-1.-!-n =1-1..+1..


1 2 3 4
-1..+ ..... .

wI .I(E )~I ~ ~ ~
This series is convergent (1.7.3)

-1 ),,-1 = 1+ + + + ....... is divergent (p-series test).


But

Thus I asy Un II I
is convergent need not imply that UII is convergent (i.e.,
I Uti En
is not absolutely convergent).
6.9
gin
I e
6.9.1 Conditional Convergence
Ilu,,1
[fthe series
eri
is divergent and Un is convergent, then I Un is said
to be conditionally convergent.
ng.
6.9.2 Consider the Series
1-1..+.!-1.. .......
234
IU II
is convergent by Leibnitz's test. (Ex. 1.7.3) net
But I IUn I= 1+ ~ + ~ + ~ ..... is divergent by p - series test.

:. I Uti is conditionally convergent.

6.10
6.10.1 Power Series and Interval of Convergence
A series, ao + a1x + a2 x 2 + ..... + anx n + ..... where 'an' are all constants is a
power series in x.
It may converge for some values of x .

Downloaded From : www.EasyEngineering.net


Downloaded From : www.EasyEngineering.net

. Sequences of Series 451

Lt UII +I = Lt a/HI •x (I st term is omitted.)


n~<Xl till n~<Xl all

=kx (say) where Lt a ll +1 = k


n~'" all

Then, by ratio test, the series converges when Ikxl < 1 .


i.e., it converges '\Ix E ( ~1 , ~ )(k *- 0)

ww The interval ,)
(~1 ~ is known as the interval of convergence of the given

power series.

w.E Solved Examples

6.10.2 Example
asy
En I \
'" II

Find the interval of convergence of the series


n

gin
11;1

Solution

eer
Ixl
By ratio test, the given series converges when
i n
"Lf.( U~:, ) "Lf.(n: l)'·x H.[ ~.~ xg.n
=
et
= 1 I
J: =

< 1, i.e., x E (-1,1)


When x = 1, L un = L in 3' which, is convergent by p series test.

:. L un is convergent when x =1
Hence, the interval of convergence of the given series is (-1, I]
6.10.3 Example
Test for the convergence of the following series.

(a) 1- YJ2+ YJ3- YJ4+··········


(b) 1+ X2- }i2- ~2+ Ys2+ ~2- (;2- h2+ . . . .

Downloaded From : www.EasyEngineering.net


Downloaded From : www.EasyEngineering.net

452 Engineering Mathematics - I

(c) I-X~+X~-~+ ......... .


n
(d) I(-I)"(n + l)x , with x <-.!..
o 2
Solution
(a) The series is of the form L (-I),,-I un where un = y.;;;
It is an alternating series where (i) un > OV n (ii) Un > 1l,l+IVn and
un = 0;:.
ww (iii) Lt

Again
n~oo

the series
By Leibnitz test, the series is convergent.

1+ YJ2 YJ3 YJ4


+ + + ...... is divergent, by

w .Ea
p - series test.

(b) L lu,,1 = L;;:2 syE


Hence the given series is conditionally convergent.
00

which is convergent by p - series test.


p=1

ngi
The given series is absolutely convergent.

It is convergent. nee
(c) The given series is rin
"(_I)n-l. x
2n-2
. ="(-1 ),,-I U · . Iu I=_x__ g.n
2n-2

~ (2n-2)! ~ n' •• n

-U n +1 = 1
(2n-2)!

1 21 ; Lt -lln+1
.x
e =0 < 1 t
u" (2n -1 )(2n) tHoo ll"

By ratio test, the series Llu,,1 converges Vx; i.e., L u" is absolutely
convergent Vx;
(d) Here, Iunl =(n + I)xn;1 un+1 1= (n + 2)x n+1 (neglect 1 sl term)

lu I
Lt ~ = Lt
(n + 2)
Ixl = Lt
(1 + 7n)~ Ixl =Ixl < 1 }i
(": x < 12 )
IHOO Iunl IHOO (n + 1) IHOO (1 + Yn)

:. Llunl is convergent Vx, i.e., given series is absolutely convergent

and hence convergent.

Downloaded From : www.EasyEngineering.net


Downloaded From : www.EasyEngineering.net

Sequences of Series 453

6.10.4
Show that the series 1 + x + x/6 + x/6 + ..... converges absolutely \;Ix

Solution

Lt IU'HII =13 =0 < 1 when x*-O [since Ix"-II ·Iu 1= Ix"l ]


111" 1= (n-I)!'
'1->00 lUl I n . n!
IItl

:. By ratio test, Ilunl is convergent \;Ix *- O.


When x = 0, the series is (1 + 0 + 0 + ...... ) and is convergent

ww:. Ilu,,1 converges ~ Ill" is absolutely convergent \;Ix.


6.10.5
w.EExample
Show that the series, 1 - ~ + ~ - ~4 + ........
3 3- 3
is absolutely convergent.

Solution
a syE
, which is a geometric series with common ratio ~ <1
ngi
:. It is convergent. Hence given series is absolutely convergent.
3

6.10.6 Example
nee
Test for convergence, absolute convergence and conditional convergence of the
series,
1 ] 1 rin
1--+---+ ......... .
5 9 13 g.n
Solution
The given alternating series is of the form I (-1 r- I
Un' where, un
et
= _1_ .
4n-3
1 1
Hence, u" > O\;ln EN; U,,+I = 4 (n+ 1) -3 = 4n+ 1
1 1
u -u =-----
n ,,+1 4n - 3 4n + 1
4n+I-4n+3 4
= - - - - - - > 0, \;In E N
(4n - 3)( 4n + 1)
(4n - 3)( 411 + 1)
1
I.e., un > un+I ' \;In EN Lt II = Lt - - = 0;
n-+oo 11 fl~a) 4n - 3

All conditions of Leibnitz's test are satisfied.


Hence (-1 I I
r-
un is convergent.

Downloaded From : www.EasyEngineering.net


Downloaded From : www.EasyEngineering.net

454 Engineering Mathematics - I

lu 1=-1-.
/l
4n - 3 '
Take v =! .
11'
n
Lt
//--+00
lunl = Lt
v" ,,--+00
n I
n( 4 - 3/)
=! *- 0
4
and finite.
n
By comparison test, I 111// I and I v" behave alike.
But by p - series test, I v" is divergent (since p = 1) .
Ilu// I is divergent and :. The given series is conditionally convergent.

6.10.7 Example

I (_1)"-1. I, ,for absolute / conditional convergence.


ww
Solution
Test the series
//=1 }\1 n

w.E
The given series is an alternating series of the form I (-1 r-Iu// .
Here

(i) un = 3vn"
1
Vn EN asy
En
(ii) 3(n+l»3n=>3-Fn+l >3,Jn,Vn.
1 1. gin
\-I N
:. 3.Jn+l <3,Jn ,I.e.,

un = 1
ll'/+I <u//,vnE
e eri
And Lt
n~oo
Lt
n---)-oo
, =0
3" n
ng.
But
:. By Leibnitz's test, the given series is convergent.

I (_1)//-1.__ =
3,Jn
1
I_I-
3,Jn
is divergent by p- series test (since net
1
p= -<1)
2
:. The given series is conditionally convergent.

6.10.8 Example
Test the following series for absolute / conditional convergence.

(a) I(-If-I , sin(~a) (b) I(-I)"-I.~


//=1 n n=1 n +1

(c)
I (-1)" (d) " (_1)"-1.
L..J
nJr
e3n+1
n

,,=0 (2n)!

Downloaded From : www.EasyEngineering.net


Downloaded From : www.EasyEngineering.net

Sequences of Series 455

Solution

(a) lu,,1 = Isinnal


112
1 [
< n 2 since Isin nal < 1
]
considering VII
II
= 1112 and using

comparison and p - series tests, we get that I lUl I is convergent I lin is


absolutely convergent.
(b) By Leibnitz's test, the series converges.
.
Tak1l1g v" =-
1
n
.
,by companson and p- series tests, . "n. 2
~ - ,- , IS
n- +1
seen to

be divergent.

ww lu,,1 Hence given series is conditionally convergent.

II I I= 0 < 1; By ratio test, I lUl I is convergent;


w.E(c) Take = _1_; Lt
2n! ,,->0')
III +
lIlI

asy
Hence given series is absolutely convergent.
Jt'''
(d) lu,,1 = e~)"+I By root test, is convergent, given senes is absolutely

En
;

convergent.

gin
[In problems (a) to (d) above, hints only are given. Students are advised to
do the complete problem themselves]
6.10.9 Example
eer
(b):t (_1)"-1 n(x:i3n2)"g
Find the interval of convergence of the following series.

:t (_1)"-1 xX
(a)
,,~I
3
II~I
.ne
Solution
(c) log(l+x)

\ "\ \"+1\
t
(a) Let the given series be I lIlI ; Then Iunl = ~ ; lun+,1 = X 3
n (n+ 1)

/3 [_11J' ·lxHx:
00

1+--
n
:. By ratio test, Ilu,,1 is convergent if Ixl < 1
I Un is absolutely convergent if Ixl < 1 ;
i.e.,
:. I u" is convergent if Ixl < 1

Downloaded From : www.EasyEngineering.net


Downloaded From : www.EasyEngineering.net

456 Engineering Mathematics - I

1 1 1
Ifx= I,thegivenseriesbecomes 1-23"+ 33 - 4 3 + ....... .

W h·IC I1 IS
. ."
convergent, sIllce 1 IS
~ -3 . convergent.
n
Similarly, if x = -1, the series becomes I -~ = -I ~ which is also
n n
convergent.
Hence the interval of convergence of I UII is (-1 ~ x ~ 1)

ww
(b) Proceeding as in (a),
lulI+11
Lt-=--
Ix+21
w .Ea
IHOO Iu'" 3
:. I U is convergent if Ix + 21 < 3, i.e., if -3 < x+ 2 < 3, i.e., if
II

-5 < x < 1.
If x = -5, I lIlI syE
=I (-llll-I .n ,and is divergent ( in both these cases

Ifx= I, IU = I( -1)"-1 .n, and is divergent


II
ngi Lt
ll--i.rJJ
UII:I; 0 )

x2 n
Hence the interval of convergence of the series is (-5 < x < I)

x--+---+ ...... . ee
x3 X4
(c) log (I +x)=
00
234
I( -1)"-1 ~ = IU
r
II ing
=
n=1 'n
II
(say)
.ne
Ixnl
IUnl=-;-; lun+1= :+1 1
111+11
t
lun+11
Lt - - = Lt 1 I x I= I I X
Iunl
/1-+00 (1 +~) /1-+00

By ratio test, I lUl I is convergent when Ixl < 1


i.e., I Un is absolutely convergent and hence convergent when -1 < < 1. x
When x = -I, "~ u = "(
~ -1 )11-1 . -1 = 1- -1 + -1 - -1 + ....... ,
ll
n 2 3 4
which is convergent by Leibnitz's test. (give the proof)

Downloaded From : www.EasyEngineering.net


Downloaded From : www.EasyEngineering.net

Sequences of Series 457

Whenx= 1, :L>n = -(1+~+!+~+


234
...... ) which is divergent,

since L ~n is divergent by p -series test (prove).

Hence I Un is convergent when -1 < x ::; 1


Interval of convergence is ( -1 < x ::; 1 ).

Exercise -1 (e)

ww
1. Use integral test and determine the convergence or divergence of the following
series:

Ln w
1
[ Ans : convergent]

2.
.E
00
2

~ n{logn )2
1
asy [ Ans : convergent]

En
2. Test for convergence of the following series:
1 1 1
l-~
gin
+l:!: -l2 + ...................................... [ Ans : convergent]

'"
~(2n-I)(2n)
{-If-l e eri [ Ans : convergent]
2.
ng.
I
3. Classify the following series into absolutely convergent and conditionally
[ Ans : convergent]
net
convergentseries:

[ Ans : abs.cgt ]

[ Ans : abs.cgt ]

{-If
[ Ans : abs.cgt ]
3. I n{logn)2
4. Find the interval of convergence of the following series
2n x n
1. L ~ [ Ans : -00 < x < 00 ]

Downloaded From : www.EasyEngineering.net


Downloaded From : www.EasyEngineering.net

458 Engineering Mathematics - I

[ Ans : - 1 ::;; x ::;; 1 ]

r Ans : -1 < x ::;; 1 ]

111
5. (a) Show that 1- - 2 + - ? - - 2 + .... is absolutely convergent.
2 3- 4
111
(b) Show that 1 - J2 + j j - J4 + .... is conditionally convergent.

ww Tests of Convergence - A Summary

w
I. The geometric series
.Ea
00

Lx
n=1
n I
- converges if 1x 1< 1 , diverges if x ~ 1, and oscilates

2. If
when x::;;-1
L u" is convergent, Lt ull syE =0 [The convergent need not be necessary ]

ngi
n->oo

3. P - series test:- f ~n is convergent if p > 1 and divergent if p::;; 1

4.-C-omparison lest :- The series


11=1

LUll and nee


L vn are both convergent or both

divergent if Lt ~ is finite and non - zero. rin


n->oo VII

5. D 'A/ember! 'sRatio test:- L un converges or diverges according as g.n


Lt
n->oo
U
2:':l.
u ll
< 1 or > 1 e t
(
Alternately, if Lt
n->oo
~
u
>1
+
or < 1) . If the limit = 1, the test fails
ll 1

6. Raabe's test: L un converges or diverges according as

.~HU::1 -I}]> lor <I .


7. Cauchy's root test: LU n converges or diverges according as Lt
n->oo
(U}) < lor> 1

(Iflimit = 1, the test fails.)

Downloaded From : www.EasyEngineering.net


Downloaded From : www.EasyEngineering.net

Sequences of Series 459

8. Integral test: A series L ¢ (n) of +ve terms where ¢ (n) decreases as n increases
co

is convergent or divergent according as the integral J¢ (x) dx is finite or infinite.


1
co

9. Alternating series - Leibnitz 's test: An alternating series L (-1 ),,-1 un convergent
n=1

if(i) un = ull +1' \In and (ii) Lt un


Il---+CO
=0
10. Absolute / cO':lditional convergence:
L L Iun I is convergent.
ww (a)
(b)
un is absolutely convergent if
LUll is conditionally convergent if LUn is convergent and Llun I is

w (c)
divergent.

.Ea
An absolutely convergent series is convergent, but converse need not be true
. i.e., a convergent series need not be convergent.

syE
Miscellaneous Exercise - 1 (e)

1. Examine the convergence of the following series:


111 ngi
1. - + - + - + ........................................... .
1.2· 3.4 5.6
e 32
nee ( cgt.]

22
2. -3-+-3-+-3-+ ..... .
1 +1 2 +1 3 +1 rin ( dgt.]

2 22 23
3. -+-+-+ ...... . g.n ( dgt.]
1 2 3
1 2 3
4. 1+2 + 1+22 + 1+23 + .....
e t
( cgt.]
3
X x2 x
5. --+--2 +--3 + ..... (x>O) ........ ( cgt. if x~l dgt. ifx> I]
l+x l+x l+x
2 3
3x 4x
6. 2x+-+-+ ..... (x > 0) .......................... ( cgt. if x ~ 1 dgt. if x > 1)
8 27
7. 1+!+ 1.3 71.3.5 +.... ................................. (dgt.]
22.4.6 2~4-
32 32.5 2 32.5 2.7 2
8. 2" + -2-2 + 2 2 2 + ... ... ............................... [ cgt ]
6 6 .8 6 .8 .10
9. 3.4 + 4.5 + 5.6 +.... ................................ (dig.]
1.2 2.3 3.4

Downloaded From : www.EasyEngineering.net


Downloaded From : www.EasyEngineering.net

460 Engineering Mathematics - I

(l!)2 (tJ)2 2 (lJr ~


10. tJ .X+ l1 x + 12 x + .... (X>O) ............. [ege ifx<4,dgt. if x24 ]

X x2 x3
II. 1+ 22 +j2+4T+ ..... (x>O) ........................ . [ egt.if x::::; 1, dgt. if x > I]

12. 4+ (4 5
3x 5)2 x 2+(6)3 +x3+ ..... (x>o) ............ . [ egt if x < 1, dgt. if x 2 1 ]

13. L(l+~r [ dgt. ]

ww [ egt. ]

15.
wa"
2:--2,a<1
l+n .Ea [egt.]

111
16. 1 - - + - - - + - + -.....
2.2 3.3 4.4
syE [ Abs. egt.]

ngi
2. Examine for absolute and conditional convergence of the following series:
n
"(_I)n 33 n
I. L.,;
3

.......... ......... ............. .... nee [ Abs. egt.]

rin
2

(-l .n
2. 2: 2f n
g.n
[ Abs. egt. ]

e
[Cond. cgt]
t
[ Cond.egt.]

3. Determine the interval of convergence of the following series:


x 2 x 3 X4
x+-+-+-+ ..... .............................. .
234
" (x+lf [-3<x<1 ]
2. L.,;
n.2"

Downloaded From : www.EasyEngineering.net


Downloaded From : www.EasyEngineering.net

Sequences of Series 461

Exercise - 1 (g) (Objective type questions)

1. The infinite series

(i) convergent (ii) divergent


(iii) oscillatory (iv) ndne of these I ADS :(i) ]
. 1+n
2. T he senes - - 2 IS
1+n
(i) convergent (ii) divergent
(iii) oscillatory (iv) none of these I ADS :(ii) ]
. 1 1 1 1
ww
3. The senes 1.J}- 2J2 + 3J3 - 414 + .....

4.
w (i)

1-.!. .E
(iii)
oscillatory
conditionally convergent

-.!. ......
(ii)
(iv)
absolutely convergent
none of these [ADs :(ii)]

The series

(i)
asy + ..!.
234
oscillatory
IS

(ii) divergent
(iii)
E
convergent

5. The interval of convergence of the series ngi


(iv)
X
none of these
X
3
X--+---+ ...... ,IS
X
4
.
I ADS :(iii) ]

(i) -oo<x<oo (ii)


234
nee
-1 < x < 2
(iii) -1 < x :-::; 1
6. The series _1_+~+~+ .........00
(iv) none of these
rin I ADS :(iii) ]

1.2 3.4 5.6


IS

g.n
(i)
(iii)
· 123
convergent
oscillatory
.
7. T he senes - + - + - + .........00 , IS
(ii)
(iv)
divergent
none of these
e
I ADS :(ii) ]
t
1.3 3.5 5.7
(i) conditionally convergent (ii) convergent
(iii) divergent (iv) none of these I ADs :(iii) ]
.2345
8. The senes - + - P + - + - P + ........00 is convergent if
V 2 JP 4
(i) p < 2 (ii) p= 2
(iii) p> 2 (iv) none of these I ADs :(iii) I
9. The series 6 - 10 + 4 + 6 - 10 + 4 + 6 - 10 + 4 + 6............. 00 is
(i) convergent (ii) oscillatory
(iii) divergent (iv) none of these [ADS :(ii)]

Downloaded From : www.EasyEngineering.net


Downloaded From : www.EasyEngineering.net

462 Engineering Mathematics - I

111
10. The series - + - + - + ......... IS
2.4 4.6 6.8
(i) convergent (ii) divergent
(iii) oscillatory (iv) none of these [Ans :(i) ]

2. Indicate whether the following statements are true or false:


1
I. The series " is convergent. [ False]
LJ 1+ Til
2
n +5
I
ww U 12 lJ
2. The series

1
2n2 + 7 is not convergent.

1 1 . d'
.. ................... . [ True]

w.E
3. The series +

x
3
+

x
+ ........ IS Ivergent... ................ . [ False]

asy
4. The series x - - + - - + - ....... , converges when -1:s x :S 1..
3 5
[True]

E
5. The series "
LJ n.5/l
(_1)"-1
ngi
is absolutely convergent. ........................

6. The series x + 2X2 + 3x 3 + 4X4 + ....00 is convergent if x> I........


[ True]

x2 x3
nee [ False]

7. The series x + -
3 2 r
+ - + ........00 is divergent if x ~ 1 ............... [True]
ing
x 2! 3 3! 3 4! 4
8. The series 1+-+2"x +-3 x +-4 x + ...... +00 is convergent
2 3 4 5 .ne
if x> e

9. The series I ( 1 + .);; )


-"J;;
is divergent ............................ .
[ True]

[ False]
t
10. The series "21 +"32 x + ( 4"3)2 x 2+"5
( 4 )3 x 3+ ....... IS. convergent
ifx< I . [ True]
II. The series 1- 2x + 3x 2 - 4x 3 + .....00 ( x < 1) is divergent... ........ . [ False]
2 3 4
. x
12. The senes - - - --2
x +-- x 3 - --
x . T]
4 + ......00 IS convergent ...... [ rue
l+x l+x l+x l+x
. sin x sin 2x sin 3x
13. The senes -3- - --3- + --3- + ......00 converges absolutely ...... [True]
1 2 3

Downloaded From : www.EasyEngineering.net


Downloaded From : www.EasyEngineering.net

Sequences of Series 463

14. The series I (~1-1 is conditionally convergent. [ True]

2
• Ih. 3n + 5
15. The sertes whose n term IS is convergent. [ False]
(n+2)"
3. Fill in the Blanks :

L ar l -
00
I
1. The geometric series converges if_ _ _ __ [ Ans: 1r 1< 1]
11=1

ww
2. If a series of +ve terms ""
L.. un Lt
is convergent, n->oo un =- - - [Ans: 0 ]

I w
3. {{!n 3 +I-n} IS _ _ __ [ ADs: convergent. ]

4. If
.E
11=1

L 3n3-4
00

asy
is divergent ,value ofp is _ _ __ [ Ans: ::;; 4]
1/=1 (n+5Y

5. The interval of convergence of I En ~


gin u" where u" ( :: :~ r x, is _ __

e
(un er
[ Ans: -1 < x < 1]
6. L un is convergent series of +ve series. Then Lt
11-»00
11
1( ) is
ing
----

.ne ( ADs: < 1]

7. The series 8 - 12 + 4 + 8 - 12 - 4 + ....... is

8. If un > 0, V nand LUll is convergent, then Lt [n {~-I}] is _ __


( ADs: Oscillatory]
t
11->00 u +
n 1

[ADS: > 1]

9. If the series I(-Ifan,(all >OVn) is convergent, then for all values of n,!!JL
~ aMI

is- - - ( ADS: > 1]

10. If un =(I+!)-n2 , Lt u l/ n= _____ [ ADS: 1/e]


n n->oo
lI

Downloaded From : www.EasyEngineering.net


Downloaded From : www.EasyEngineering.net

464 Engineering Mathematics - I

Tests of Convergence - A Summary


00

1. The geometric series I X"-I converges if Ixl< I, diverges if x~ 1 and oscilates


n~1

when x~ 1.
2. IfIll" is convergent, Lt u"
Il-~OO
=0 [The converse need not be necessary ]

3. p - series test:- f ~n
,,~I
is convergent if p > I and divergent if p ~1
4. Comparison test: The series L u" and I Vn are both convergent or both

ww divergent if LI u" is finite and non - zero.

w .Ea
n~oo

5. D'ALEMBERT'S Ratio test :


v"
IU n converges or diverges according as

I, the test fai Is.


syE
< l o r > 1 (Alternately, L t1l,,+1
- <lor >1). If the limit

ngi
=

6. Raabe's test: I U" converges or diverges according as Lt [n{~ -I}] I I


> or <

nee n-)oo Unt-I

(u~ J< 1 or> 1 ;


7. Cauchy's root test: I U" converges or diverges according as Lt
rin n~oo

(If the Limit = I, The test fails).


8. Integral test: A series L rj> ( n) of +ve terms where rj> ( n) decreases as n increases g.n
is convergent or divergent according as integral
00

1
e
frj> ( x ) dx is finite or infinite. t
00

9. Alternating series - Leibnitz's test: An alternating series I (-1 ),,-1 un (where


n~1

Un> OVn ) is convergent if(i) un > lin+1 ' Vn (ii) Lt


n~oo
Un =0
10. Absolute/ conditional convergence:
(a) LUn is absolutely convergent if II Un I is convergent.
(b) IU n is conditionally convergent if Iu" is convergent and II U" I is
divergent.
(c) An absolutely convergent series is convergent but converse need not be true.
i.e., a convergent series need not be convergent.

Downloaded From : www.EasyEngineering.net


Downloaded From : www.EasyEngineering.net

Sequences of Series 465

Solved University Questions

1. Test the convergence of the series:


123
- - + - - + - - + ........ .
1.2.3 2.3.4 3.4.5
Solution
Let ull be the d h term of the series;
n 1
Then, U - -

ww II - n(n+1)(n+2) - (n+1)(n+2)
1 U n2
Let
w.E VII = -2 ;
n
then, Lt --'!..
11->00 V
II
= Lt (
tHoo
)(
n +1 n +2
1
)

a syE
= Lt
11->00 ( 1) (
1+;; 1+;;
2)
=1
,

Which is non-zero and finite.


:. By comparison test, both LUll and nLgi converge or diverge together.

nLUllee
VII

But L VII is convergent by p-series test (p > 1):. is convergent.

2. Show the every convergent sequence is bounded


rin
Solution
(all) g.n
Let
Lt
1l~C()
all = I =>
be a sequence which converges to a limit '/ 'say.
given any +ve number E, however small , et
we can always find an integer' III ' , ) , Ian -II <E, 'ifn ~ m
Taking E = jaIl -II < 1;
1, we have,
I.e., (I - 1) < all < (i + 1), 'if n ~ m
Let A = min {a"a 2 , •••••••• am _,,(1 -1)} ,and fl = max {a"a 2 , •••••••• am-,,(I + 1)}
Then obviously, A ~ an ~ fl, 'ifn EN;
Hence (an) is bounded.

3. Show that the series,


1 1 1
S =1--+---+ .......... converges.
3! 5! 7!

Downloaded From : www.EasyEngineering.net


Downloaded From : www.EasyEngineering.net

466 Engineering Mathematics - I

Solution
The given series is an alternating series 2)-1 ),,-1 u" '
1
where u" =( ) We observe that,
211-1 !
(i) ll" > 0 and u" > 1l,,+1' '1111 and
1
(ii) Lt II = Lt =0
,,--+00" IHoo ( 211 - 1) !

:. By Leibneitz's test [7 .2 ] the given series converges.

ww
4. Show that the geometric series I
00

qlll = 1+ q + q2 + ....... converges to the slim

w.E 1
- - when /q/ < 1 and diverges when /q/ ~ 1
111=0

l-q
Solution asy
5.
En
See theorm 2.3 (replace' x ' by 'q') .
Define the convergence of a series. Explain the absolute convergence and

gin
conditional convergence of a series. Test the convergence of series

I[l+ J,;r'
Solution
e eri
For theory part, refer 2.1,2.2, 8.1 ,9.1, and 9.2
, n
Lt (1 _1_)-ng.
Problem: Let u" = (1 + _l_)-n
J;;
1
; Lt (u~{,) =
11--+00

1
,,--+00
+
J;; net
~ H-[l+ J,;J =7<1

By Cauchy's root test, I Un is convergent.


1 1.3 2 1.3.5 3
6. Test the convergence of the series, 1+-x+-x +--x + .....
2 2.4 2.4.6
Given that x> O.
Solution
1.3.5 ..... (211-1) n
Omitting the first term of the series, we have, ------'--------'- x and
2.4.6 .... .211

Downloaded From : www.EasyEngineering.net


Downloaded From : www.EasyEngineering.net

Sequences of Series 467

1.3.5.(2n-l) 1/ 1.3.5 ..... (2n+l) n+1


UI/ = 2.4.6 .... 2n
X ; UI/+ I = (
2.4.6 ..... 2n + 2
).x;

L u n+ 1 L (2n + 1 )
1/100 ---;;: = 1/100 2n + 2 .x = x
By ratio test, LUI/ is convergent when x < 1, and divergent when x > 1

The ratio test fails when x = 1


Whenx= 1, ~-1 = 2n+2 -1 =_1_

ww U n +1 2n + 1

Lt [n(~-I)]= Lt (_n
2n + 1

w.ELUI/
:. By Raabe's test,
n~oo 2n +
U Il +1

diverges.
IHOO 1
)=!<1
2

a sy2"E"3 +"4)2 +"5(


:. The given series converges when x < 1 and diverges when x 21 .

7. Test the convergence of the series, 1 + ( 2 ) x


ngi (3 x
2 4 )) x 3 +......... x > 0

Solution
Neglecting the 1st term,
nee
=[(=:~H' rin
u"

[1+ 1/]
g.n
u 1n
1/
II
= (n+l)
- - X= _ _
n+2
n x
1+2/
n
et
Lt uny.n =
ll----too
x; By Cauchy's root test, "~u// is cgt. when x < I and dgt. when x >

1; when x = 1 , the test fails.

(1 + l/~r
When x = 1, un =( , )n;
1+ 3/~
Lt Un = ee = -e1* 0
IHOO
-2

:. LU n is divergent.

:. is cgt. when x < 1 and dgt. when x 2 1 .

Downloaded From : www.EasyEngineering.net


Downloaded From : www.EasyEngineering.net

468 Engineering Mathematics - I

8. Test the series whose nih term is (3n -1) / 2n for convergence.

Solution
(3n-l) {3(n+l)-I}
U
n
= 2"
.
'
U
IHI
=-'--------'-
2"+1
U,,+I _ (3n + 2) - - -- 1
<1
L (Un+1
un 2(3n-l) n->oo Un 2

ww L :. By ratio test, un is convergent.


1
w.E L <X)

9. Show by Cauchy's integral test that the series


n;2 n(logn r converges if p > 1

asy
and diverges if 0 < p ~ 1
Solution

En
Let ¢ ( x ) =
1
; x ~ 2 ; Then ¢ ( x) decreases as x increases in [2,00]
x(logxy
<X) 00 ax ginI <X) d I-p

f¢(x)dx=f x(logx) e
00
U II 1
= --;;; = ~
eri
p log2 ;
2 2 log2 P
. I
[Takmg log x = u, - ax = du x = 2 => u = log 2 and x
x ng.
= 00 => U = 00 ]
Case (i) : p > 1 => 1- P < 0 => Integral is finite, and
Case (ii) : 0 < p ~ I => Integral is infinite.
Hence, by integral test, the given series converges if p > I and diverges when
net
O<p~1 .
JI

10. Test the convergence ofthe series I (I + J,;)"""


Solution

Downloaded From : www.EasyEngineering.net


Downloaded From : www.EasyEngineering.net

Sequences of Series 469

1 1
L1 U;,II = - <1 [2 <e <3I .
n~oo e
By Cauchy's root test, I u" is convergent.

11. Test the convergence of the se'ries, I,,~2 -(1)"


00 (

n-l
11
"
.x)' 0 < x < I

Solution
xn
I (-1)" 11" ,where u" = ( )
ww
The given series is of the form
n n-l
> 0 and > 1l1l+IVn EN.
w.E
This is an alternating series in which (i) lin 1I"

further L1 ll" = o. Hence, by Leibnitz test, the series is convergent.


11-»00

12.
.
DISCLISS
a .
the convergence of the series,
syE 2'1'1
1
J. +
x2 X4
r;; + r:; + t. + ......... .
3'1'2 4'1'3 5'1'4
x6

Solution
ngi2n
n Iii
term 0 f th ·
e series = u =

X
ll
x r--:-:;
(n+2)'I'n+l
21
1+2 "1I+1,J;;+2
nern+i
( . . I sl term )
omlttmg

eri 2

U
llt
1= (n + 3).J n + 2 ; ---;;: =
n
(n + 3) .x
g.n
/HOO
LI ~= I/~OO lrVn.M.
It" (+ 3.~) 2l=x
LI
1
X
2
;
et
By ratio test, I converges if x "II
2
< 1 , i.e., if Ixl < 1, and diverges if
x
2
> 1 , i.e., if Ixl > 1 ;

When x
2
= 1, u = ll
1rn+i ;taking vn = -3-' ,
(n+2) n+l n i2
]I
U n/ 2
Lt = Lt =1
(1 + ,'n
2/) \fIT?n
hl
_II

n~oo vn n~oo n%

By comparison test, I Un and I VII both converge or diverge together;

Downloaded From : www.EasyEngineering.net


Downloaded From : www.EasyEngineering.net

470 Engineering Mathematics - I

But L vn is convergent by p-series test.


:. L un is convergent if Ixl ~ 1 and divergent if Ixl > 1
00 211
13. Test the convergence
<::1 of the series " " x
L..--~=
n~1 (n+ 1)J;;
Solution
X 211 x2n+2

ww U
n
=
(n+l)J;;'
. U
11+1
=----==
(n+2)rn+i

w.E u n+1 =
,r-:-; ~
vnvn+l. x2 = \jIT /~ .x 2 ; Lt u n +1 =x2;

a
By ratio test,
un

syE
n+2 (l+;ln)

L un converges when Ixl < 1 and diverges for Ixl > 1


11-+00 Un

When Ixl = 1 , un =
n
3/ (
2
1
1 + 1/
.) taking
ngi ~ VII =
n/ 2
and applying the comparision

test, we observe that LUll


.n
nee
is convergent.
Hence L lin converges when Ixl riIxln
~ 1 and diverges when >1 .

14. Find the interval of convergence of the series, g. 2 3


~+~+~+ .........oo
234
4

net
Solution
11+1
· .
For tIle gIven serIes, un
x
= --; un +1
n+1 n+2

Lt un +1 = Lt ~
n~'" U
1+
n~oo ( 1+ 2/
II) X=X

n /n

By ratio test, L un converges when Ixl < 1 i.e., -1 < x < 1


1
When x = I U =--
, n n+1

Taking
un
;-=--.,
Vn 1+;11
n

Downloaded From : www.EasyEngineering.net


Downloaded From : www.EasyEngineering.net

Sequences of Series 471

and, Lt un = 1 *- 0 and finite.


II~'" VII

:. Both IU II
and I VII converge or diverge together.
But I Vn diverges:. IU n also diverges when x ~ I.

When x = -1, the given series is

-1 --I + -1 --1 + wI·h· I . .


lIC IS a ternatll1g senes \\ 1111
2 3 4 5

ww Ut •

By Leibneitz's test
> un+IVn and un
IU II
~ 0 as n ~

converges when x =-1


00

w.E.. Interval of convergence is [(-1, 1) i.e., -1


~ 1.3.5 .... (211+1)
~ x <1

15.
asy
Test the convergence of the series L.
11=1 2.5.8 .... (311+2)

Solution
En
un gin
1. 3.5.... ( 2n + 1)
= 2.5.8 .... (3n+2 );
1.3.5 .... {2n+3)
un +1 = 2.5.8 .... (3n+5 )

eer
~ = Lt r2 + (
3
~
Un +1 _

Un
2n+3.
3n+5' n~'"
Lt
Un n~'" ing3
l3 + ( 5
11 )]

)
= <1

.ne
11

By ratio test, I UII is convergent.

16. Prove that the series


(-If
I---
n(logn)3
converges absolutely. t
Solution

Iu 1- n(1ognr
n -
I ; "'f
2 X(logxf
dx = "'f d!
log2 r'

(where t = log x) -11


= - : -2 = - - 1 ,which is finite.
t g log 2
By integral test LlulIl is convergent.
" L un converg..:s absolutely.

Downloaded From : www.EasyEngineering.net


Downloaded From : www.EasyEngineering.net

472 Engineering Mathematics - I

17. Test the convergence of the series L.J


'" (2n3 + I) X ,x> 0 II

n +1

Solution

n /I! term 0 ft l . .
le given series, U = -2n+
3-X
I II

n n +1
_[2(11+1)+1] n+1 _ 2n+3 n+1
un + 1 - 1 X - ,x
(n+1) +1 (n+1} +1

ww Lt
II
~I/~t
1
= 1,1
(2n+3).x"H
r
n + 1 + I}
(113+1)
x --'-----'---
xn ( 2n + I)
w.E
IH"O 1111 11->00 { (

asy x=.x

En
l1y ratio test, IlIlI converges if x < I and diverges if x> I. If x
gin = I the test fails.

Whenx = I,
2n+ I .
un = - , - ; Takmg
n +1
VII =
n
1
2
e '

eri
II~'OO
It2n + I ?
n +1

LI _" = LI - 3- X n- = 2"* 0 and filllte
ng.
net
VII 11->00

I lin and I Vn converge or diverge together.

But I Vn converges:. I Un also converges.

Thus, I lIlI converges when x ~ 1 and diverges when x > 1.

(-I)" (log n )
18. Test the series I00
2 ' for absolute/conditional convergence
II~I n

Solution

Un = (-1)" (;ogn) ; IUIII=(lo~n)


n n

Downloaded From : www.EasyEngineering.net


Downloaded From : www.EasyEngineering.net

Sequences of Series 473

"'flO~, x dx -- ' 'f le-'dl I/


[taking logx=l, x=e ' , /xdx=log/]
2 x- log2

~ -Ie -, + e-, I:.2 ~ 0 - [I-log 2].e -'"" ~ i(!Og 2 -I),


which is finite.
:. By integral test ~]un is convergent => I I 1I11 converges absolutely.
(Note that I UII is cgt. by Leibnitz's test).

19. ww
Test the convergence of the series I I
(log log n )"

Solution w .Ea
Given that u,
I
=
syE
[I]
I
(log log n)"

LI u'1/n
n-->oo n
= LI
n-->oo n
log log n
=0 < 1
gin
I
By Cauchy's root test, Un is convergcnt.
e eri
20. Find the interval of convergence of the series,
I x 1 3 x 1.3.5 x 3
x+-.-+-.-.-+--.-+ ..... .
5 7
ng.
Solution
2 3 2 4 5 2.4.6 7
n et
1.3.5 ..... (2n-l) Xlll'l
Term of the series, ull = .( ) (neglecting I st term)
2.4.6 ..... 2n 2n + 1
1.3.5 ..... (2n-I)(2n+l) X211 +1
u lI
+1 = 2.4.6 .... .2n (
2n +)
2 . (2n + 3)

~/n
2
4
Lt u
11-->00 un
lI +1 = Lt [
n-->oo
(2n + 1)2
(2n + 2)( 2n + 3
X2]
r
Lt [ n ( + + I n2) X2] = x2
I/-'>OO 19
n 2( 4 + ~ + 6 n2 r
By ratio test, I Un converges when x < 1 , i.e.,
2
Ixl < 1 => -1 < x < 1
2
When x = 1 ,the test fails;

Downloaded From : www.EasyEngineering.net


Downloaded From : www.EasyEngineering.net

474 Engineering Mathematics - I

Then ~_1=(4112?+1011+6 -1)= ~11+5


ull
+1 411- + 211 + 1 411- + 211 + 1

Lt [11(~-1)]= Lt n2(8+1~) =2>1


IHOO lIn+1 IHOO n ( 4 + 7~ + /~~2 )
2

By Raabe's test, LUll converges when x


2
= 1, i.e., x = ±1.
L (-1:::; X:::; 1)
ww
:. Interval of convergence of lin is

w.E
asy
En
gin
e eri
ng.
net

Downloaded From : www.EasyEngineering.net


Downloaded From : www.EasyEngineering.net

7ww
w.E
Vector Differentiation asy
En
gin
7.1.1 Vector Point function and vector field
e eri
ng.
Let P be any point in a region 'D' of space. Let r be the position vector ofP. Ifthere
exists a vector function F corresponding to each P, then such a function F is called
a vector point function and the region D is called a vector field.
Note: In what follows i,j, k are unit vectors along X, Y, Z axes respectively net
For example, consider the vector function
F = (x - y) i + xyj + yzk ..... (1 )

Let P be a point whose position vector is


r = 2i + j + 3k in the region D of space.
At P, the value of F is obtained by putting x = 2, Y = I, z = 3 in F.
I.e. At P, F = i + 2j + 3 k
Thus, to each point P of the region D, there corresponds a vector F given by the
vector function (I).
Hence F is a vecotr point function (of scalar variables x, y, z) and the region D
is a vector field.

Downloaded From : www.EasyEngineering.net


Downloaded From : www.EasyEngineering.net

476 Engineering Mathematics - I

Scalar point junctioll and scalar fielit


Ifthere exists a scalar 'f' given by a scalar function 'f' corresponding to each point P (with
position vector r) in a region 0 of space, of' is called a scalar point function and 0 is called
a scalar field.
As an example, let P be a point whose position vector is r = 2i + j + 3k.
Consider f= xyz + xy + z
Then the value of f at P is obtained by putting x = 2, y = I, z = 3
i.e., At P, f= 2.1.3 + 2.1 + 3 = II

wwHence the scalar' II ' is attached to the point P.

w
The function 'f' is a scalar point function (of scalar variables x, y, z), and 0 is a
scalar field.

.Ea
Note : There can be vector and scalar function of one or more scalar variables.
7.1.2 Differentiation of a vector
syE
Ifr (u) = r,(u)i + r2(u}j + riu)k, (where rl' r2, r3 , are scalar functions of 'u') be a
vector function of' u' , then,
ngi
dr
du ou
Lt
~Oou
Or
ou ~O
Lt r{ u + ou) - r{ u)
ou nee
rin
g.n
~ dr,.
= L.,-l=-l+-}+-
du
dlj.
du
dr2 .
du
dr3 k
du
e t
Example

If r(u) = (3u 2 + 5u + 6) i + 3U7 - 4uk, Find dr , when u = 1


du

dr = {~(3U2 +5U+6)li+{~(3U2)}j+{~(-4U)}k
du du 1 du du

Note: We can apply the above rule of derivative to the case of partial derivatives also
ex : If A = (ryz)i + (xyz}j - (3;Jyz2)k

a
2
A
find - - at the point (I, -I, 2)
axOv

Downloaded From : www.EasyEngineering.net


Downloaded From : www.EasyEngineering.net

Vector Differentiation 477

aA
-
ay
= { - a (2
ay
x yz )t. a {xy 2Z
1+-
Oy
}. j -a- {3x·yz-
Oy
3 ') ')} k

= (x 2z)i + (2xyz)j - (6x1yz2) k

= 2xz i + 2yzj - 18x2yz2 k

ww
a
At the point (I, -1,2)
2
A

w ax~v =

7.1.3 Application to space curves


4i - 4j + 72k

.Ea
space curve whose equations are given by xsyE
Let ret) = x(t)i + y(t)j + z(t) k represent the position vector of a point (x. y. z) on a
= x(t), y = yet), z = z(t), where 't' is
time.

dr dx. dy. dz
n gin (K + M, Y + ~Y, z + 6Z)
Then -=-I+-J+-k
dt dt dt dt
e eri
and
x ng.
(i) dr represents the velocity vector v (or tangent vector) of the point (x, y, z)
dt
n et
(ii) d 2 r represents the acceleration vector a at the point (x, y, z)
dt 2

Ex: If a particle moves along a curve x = e-t, y = 2 cos 2t, z = 2 sin 2t, where 't' is time.
I) find velocity and acceleration at time t = 0, and
2) find also their magnitudes
Sol: r = xi + yj + zk
= (e-t)i + (2 cos 2t)j + (2sin 2t)k

Downloaded From : www.EasyEngineering.net


Downloaded From : www.EasyEngineering.net

478 Engineering Mathematics - I

dr d d d
v= - = - (e-~i + - (2cos 2t}j + - (2sin 2t)k
dt dt dt dt
= (-e-t)i -(4sin 2t}j + (4 cos 2t)k

d 2 r dv d d. d
a= -=-=-(-e-t)i-
2
-(4sm2t}j+ - (4cos2t)k
dt dt dt dt dt

= (e-t ) i-(8cos 2t)j-(8sin 2t) k .... (2)


Putting t = 0 in (I), velocity as t = 0 is v = -i + 4k

ww Magnitude = 10

w putting t = 0 in (2) acceleration at t = 0 is a = i-8j


Magnitude =
.Ea J65
7.2 Gradient of Scalar Function
7.2.1
syE
The Vector differential operator 'DEL' or 'NABLA', denoted as 'V' ' is
defined by
a .a k a
n gin
V'
=i -+ }-+ -
ax ay az
e
(i,j, k are unit vectors in x, y, z directions)

eri
ng.
This operator' V' ' is used in defining the gradient, divergence and curl.

vector and scalar functions.


7.2.2 Gradient
n
Properties of' V' ' are similar to those of vectors. The operator is appled to both

et
If ~ (x, y, z) is a scalar function, defined at each point (x, y, z) in a certain region of
space and is differentiable, the gradent of ~ (shortly written as grad ~) is defined
as,

.a .a
grad ~ = V' ~ = 1-+ }-+k-- ~
a)
( ax ay az

(which is a vector function)

Downloaded From : www.EasyEngineering.net


Downloaded From : www.EasyEngineering.net

Vector Differentiation 479

:. If ~ defines a scalar field, 'grad' ~ or V ~ defines a vector field.

7.2.3 Physical significance of 'grad ~' :

If ~ (x, y, z) = c (c being a constant) represents a surface, then 'grad ~'represents


the normal vector to the surface at the point (x, y, z)
For, if r = xi + yj + zk, is the position vector of the point (x, y, z) on the surface, we
have, dr = (dx) i + (dy) j + (dz) k which is in the tangent plane to the surface of
(x, y, z)

ww
w.E
asy (.,' ~ =c)

En
:. The vector' V ~' which is 1.- r to the tangent plane is the normal vector to
~ = c at(x, y, z)

7.2.4 Directional Derivative gin


If a be any vector, V~.a e er
which represents the component of V ~
ing
in the direction of
lal .'~
a is known as the directional derivative of' ~ , in the direction of a,
.ne
(1) Physically the directional derivative is the rate of change of' ~ , in the direction
ofa.
(2) The directional derivative will be maximum in the direction of V ~
t
(i.e.,

a= V~) and the maximum value of the directional derivate = It~~ = IV~I·
7.2.5 Some basic properties of the gradient
If ~ and \jJ are two scalar functions,
'J) grad (~+\jJ) = grad ~ + grad \jJ(or) V(~+\jJ)= V~+V\I'

Proof: grad (~+ \1') = ~


V( + \jJ) = {! (~+ \jJ)} i + { ; (~ + \jJ)} j + {! (~+ \I')} k

Downloaded From : www.EasyEngineering.net


Downloaded From : www.EasyEngineering.net

480 Engineering Mathematics - I

o~. ~. ~ ) + (Oljl.
(
= -I+-j+-k -I+-j+-k 8ljI. 8ljI )
ox oy OZ ox ay OZ

= Y'~+Y'ljI

(2) grad (~ljI)= ~ (grad\fl) + ljI(grad~) (or) Y'(~ljI) = ~ (Y'ljI)+(Y'~)\jJ

Proof: grad (~ljI) = Y'(~ljI)

ww{! (~ljI)}i (~ljI)} {! (~ljI)}k


= +{; j +

w.E (8ljI (8ljI


oljl
ox
a~).1+
= ( ~-+ljI-
ox
~-+-
ay
O~).j+
oy
~-+ljI-
OZ
<.

~)k
OZ

~(Oljl =
a syE(o~; o~ o~ ~(Y'ljI) ljI(Y'~)
i + oljl j + oljl k) + ljI + j + k) = +

=
ox
ngiay oz

~(grad \fI) + ljI(grad ~)


ox oy OZ

~)nee ljI (grad~ ) - ~(gradljl )


(3) If ljI :;t: 0, grad ( ljI
rin = (ljI)2

(Proof is left to the reader.)


g.n
e Solved Examples
t
Ex. 7.2.6 Iff= x2yz, find gradfat the point (I, -2, 1).
o~. o~. o~
Sol: f= x2yz; ": grad ~ = Y'~ = 0/ + ay } + OZ k,

o 0 0
gradf= -8 (x2yz); + - (x2yz}j + -8 (x2yz)k
x ay 'Z

= (2xyz)i +(x2z}j + (x 2y)k

:. At the point (1, -2, ]), grad ~ = - 4; + j - 2k

Ex. 7.2.7 Find the unit normal to the surface xy +yz + zx = 3 at the point (I, 1, I).

Sol: If ~ = c is a surface, Y'~ is the normal to it.

Downloaded From : www.EasyEngineering.net


Downloaded From : www.EasyEngineering.net

Vector Differentiation 481

Here f= xy +yz + zx

= (y + z)i + (x + z)j + (y + x)k

ww :. normal at(l, 1,1) = 2i + 2j + 2k

w .
:. Umt normal = ~

.Ea
2;+ 2j + 2k
22 + 22 + 22
;+ j+k
J3
Ex.7.2.8
syE
(a) Find the directional derivative ofl= 2e2x-ytz at (1,3, I) in a direction
towards the point (2, I, 3).

Sol : 1= 2e 2x-ytz;
n
Vf = 2e 2x- ytz (2i - j + k )

gin
Vll(1,3,1) = 2.e 2- 3+1(2i - j + k) = 4i - 2j + 2k
Let A = (1,3, I) and B = (2, 1,3), e eri
AB = (2-1)i + (l-3)j + (3-1)k = i - 2j + 2k = a (say)
ng.
. .
DlrectlOna I denvatlve
. . JI1. the d'IrectlOn
. 0 f a = Vf. a ¥ n et
(4i-2j+2k).(i-2j+2k) (4+4+4)
= = =4
,Jl +4+4 3

(b) In the 'Problem (a)' find the maximum value of the directional derivative

Ans: Maximum value of directional derivative = IV11


= I 4i - 2j + 2k I = ,J16 + 4 + 4 = 2 J6

Ex.7.2.9 Find the acute angle between the surface xyz = 2 and:x2 + y + z3 = 6 at the
point (2, 1, 1).
Sol : Let 1 = xy2z = 4 be the surface (1)

Downloaded From : www.EasyEngineering.net


Downloaded From : www.EasyEngineering.net

-
482 Engineering Mathematics - I

Normal vector to (1) at (2, 1, I) = Vfl(2 1,1) = 1(Vz); + (2xyz}j + (xy)k 1(2, I, I)

= ; + 4j + 2k = a (say).
Let g = (.xl y + .?) = 6 be the surface (2)

Normal vector to (2) at (2,1,1) = Vg 1(2,1,1)= (2x; + 2yj + 2zk)i(Z,I,I)

= 4; + 2j + 2k = b (say)
:. Angle between the surfaces

ww = Angle between the normals to them


= Angle between a and b
w._1(a,b)1
I Ea
-II
- cos I - =cos 4+8+4
-
s y
ab .Jl+16+4.J16+4+4

J2t16) En16) 8)
-1 (
J24 -I ( -I (
= cos
gin = cos 6.J14 = cos 3.J14

7.2.10
eer
Find the constants p and q such that the surfaces px2
4x2y + z3 = 4 are orthogonal at the point (1, -1, 2)
- qyz = (p + 2)x and

Sol:
in
Letf= px2 - qyz - (p + 2)x = 0 be surface (1), and
Let g = 4.xly + z3 = 4 be surface (2) g.n
Normal to (1) at (1, -1, 2) = Vf (1-1
" ax ay az
e
2)=(81 ; + 81 j + af k 11(1_12)
y" t
= [(2px- P - 2); -{qz)j - (qy}k]I(I,-1,2)= (p - 2); - (2q)j -(q)k = a (say)

Normal to (2) at (1, - 1, 2) = Vg 1(1,-1,2)

= [(8xy); + (4x 2)j + (3z 2)k]1 (1,-1,2) = -8; + 4i + 12k = b (say)

Since the surfaces (1) and (2) are orthogonal, a.b = 0


:. -8(p - 2) + 4(-2q) + 12(q) = 0
=> -8p + 16 - 8q + 12q = 0

Downloaded From : www.EasyEngineering.net


Downloaded From : www.EasyEngineering.net

Vector Differentiation 483

=> -8p + 4q + 16 = 0
=> 2p - q = 4 ..... (i)
Since the point (1, -1,2) lies on (1), we have,
p + 2q - p - 2 = 0 => q = I
from (i) we get,p = 5/2 :. p = 512, q = 1

7.2.11 If r = xi + yj + zk and r = Irl = ~ x 2 + y2 + Z2 show that grad (r3) = 3r r

Sol:
ww Let ~ = r3

a~
= (x 2 +

2"3
T + z3)3/2

w.E Then, ax = (x 2 + T + z2)3/2-1 . 2x = 3x r

similarly 8y
a~
=
asy
3yr; and az 3zr
a~

grad ~=
a~. ~.
- l + - ) +- k En ~
ax 8y az
gin
= (3xr) i + (3yr) j + (3zr) k
= 3r (xi + yj + zk) = 3r r eer
Aliter: Ifr2 = xl + T + z2, we have, 2r ar = 2x => ar _~ ing
ax ax r
.ne
..
Slmtiarly
ar
8y
=- .-
yarz
=-
r'az r t
grad ~ = Vr 3

a 3. a 3 . a 3 ..1 ar 2 ar. 2 ar
= -(r )l+-(r )}+-(r )k =3r-i+3r -}+3r - k
ax 8y az ax 8y az

..1
=3r y.[x.
-l+-)+- zk] =3rr
r r r
7.2.12 Evaluate grad r"
Sol: Let ~= rn = (xl + T + z2)nl2
~ =!!{x 2 + y2 + Z2
ax 2
r/ 2
-
t
.2x =nx{x 2 + y2 z2 r/z- t

Downloaded From : www.EasyEngineering.net


Downloaded From : www.EasyEngineering.net

484 Engineering Mathematics - I

..
sImilarly -o~ = ny (X 2 + y2 + Z2 )'?z-/,I and -o~ (222)";-1
= nz x + y + z
ox oz

o~. ~. ~k
grad rn = grad ~ = - I + - j +-
ox 0' oz
= n, {x 2 + y2 + Z2 )'fi- I • (xi + yj + zk) = n rn-2.r

Aliter: g
rad ~
(-o~).
ox
1+ (o~l.
- j+ (~)
oy OZ
o~ or. o~ or. o~ or
- k=-.-I+-.-j+-.-k
orox or0' oroz

ww = nrn- 1 . !.. i + nxn- I . y j + nxn- I . !... k = nrn-2.r

w
r r r
Ex.7.2.13
Sol: .Ea
If A is a constant vector prove that grad (r . A) = A
Let A = A I i + Aj + A3k (A I' A 2 , A3 being constant functions)
r = xi + yj + zk
syE
r. A
of
= A1x + A2Y + A3z = f(say)
81 81 ngi
ox = AI' 0' = A2, OZ = A3
nee
:. grad (r.A) = gradf= 81
ox
i+ 81 j+ 81 k
0' oz I
rin
=A i+Aj+A k=A
3

Ex.7.2.14 Prove V (~(r»=--


~I(r).r g.n
Sol: Let f= ~ (r)
r
e t
81 ar I'X or x
- = ",I (r) - = ~ (r).-
ox 'Y ox r ox r

81 =~I(r)ar =~I(r).y or y
0' 0' r 0' r

81 = ~I(r) or = ~I(r).!... or z
-az =-r (see Aliter of ex. 7.2.11)
OZ OZ r

81. of. 81
.. V (~ (r» = V f= 0/ + 0' ) + oz'''
<j>1(r) <j>1(r)r
= W (xi+yj+zk) = W

Downloaded From : www.EasyEngineering.net


Downloaded From : www.EasyEngineering.net

Vector Differentiation 485

Ex.7.2.15 Find the equations for the tangent plane and normal line to the surface
.r
z = x 2 + at the point (2, I, 5).
Sol: Let r = xi + y) + zk be the position vector of any point P(x, y, z) on the surface.
Let r l = xli + yJ + zl k be the position vector offixed point A(xI'YI' z,) on the
surface.
Then AP = (x - xI) i + (y - YI») + (z - z\) k = r - r,
Let n be the normal to the surface at A.
Then, since AP is perpendicular to n, we have,

ww (r-r\)n=O ..... (I)

w.E which is the equation to the tangent plane at A.


Here, in the given problem

a.rs-y
r-r, =(x-2)i+(y-I»)+(z-5)k

and 11 = V (xl +
En z) at A (2, I, 5)

= (2xi + 2y) - k)I(2.).5)


gin
= 4i + 2) - k

4(x - 2) + 2(y - I) - I(z - 5) ~4x


e
:. The tangent plane at (2, I, 5) is, (from (I»,
=0 eri
+ 2y - z = 5 .... (2)
From (2), the direction ratios of the normal Iine at A are 4, 2, -I ng.
:. Equation 10 tpt( pormal line at A are,
x-x)
-- =
y- Y) z-z) he \
w re
~,=--
net
abc
(xl'YI' zl) = (2,_1,5) and (a, b, c) = (4; 2, -I)
:. The equations of normal line are x - 2 = Y -I = z - 5
4, 2 -I
From the above example. we have to remember the following :

Let + = c be any given surface and (xl' YI' zl) be a point on it; then

(I) Equation to the tangent plane to +=cat(xI'Y"z,) is

Downloaded From : www.EasyEngineering.net


Downloaded From : www.EasyEngineering.net

486 Engineering Mathematics - I

(2) Equations to Normal line at (xl,y\, zl) are


x-x, = Y- y, = z-z,
8<1>/8x 8<1>/ay 8<1>/8z

Exercise 7(a)

I. If <jl = 2xz3 - 3xlyz, find V <I> and IV<I>I at the point (2, 2, -I)
[Ans: (i) 22i+ 12}-12k(ii)2M3]

ww
2. If V = 2x i - 3y) + z 3k, and <jl = 2xyz - 3z2, find V. V<I> and Vx V<I> at the point
(1,2,3)

3. w.E [Ans . (I) - 426 (2) 6i + 352} t 156k]


Iff=2xyzandg=xly+z,fing V (f+g) and V (fg) at the point(I,-I,0)

4.
a
Evaluate V (3r
2
- syE
4.,Jr + 1 ~)
~r
[Ans : -2i + ) - k; 2k ]
7
[AI:1S : (6 - 2~/2 - 2r -3) r ]

5. r
ngi
If <I> = r2 e- , show that grad <I> = (2-r) e-r r
Find a unit normal vector to the surface z = xl + Y at the point (I, -2, 5)
6.
nee I

rin
[Ans: 51 (2i - 4j - k)

7. Finq the equations to the tangent plane and the normal line to the surface
xz2 + xly = z - I at the point (I, -3,2) g.n "
8.
.
°
.. x-I y+3 z-2
[Ans: (I) 2x- y- 3z + 1 = (n) -2 =-1- =-3-
Find ~uations to the Tangent plane and normal line to the surface y
et = xl + z2
at the point (1, 5, -2)
x-I y-5 z+2
[Ans: (i) 2x-y-4z= 5 (ii) -2-=-=1=--=-4

9. Find the directional derivative ofU = 4xz3 - 3x2yz at (2, 1, -2) in the direction of
(3i - 2) + 6k).
384
[Ans: 7]
10. Find the directional derivative of <I> = 4e2x- y+z at the point (1, I, -1) in a direction
towards the point (2, 3, 1)
[Ans : 8/3]

Downloaded From : www.EasyEngineering.net


Downloaded From : www.EasyEngineering.net

Vector Differentiation 487

II. Find the values of the constants a, b, c so that the directional derivative off= axl
+ byz + cz2x 3 at (1,2, -I) has a maximum magnitude 64 in a direction paraIlel to
z-axis.
[Hint: /0:./ at (I, 2, -I) is IlleI to z-axis.
:. Equate coefficients of i and j to zero and IV ~ = 64. Thus get 3 equations in a,
b, c and solve them]
[Ans: a =6, b = 24, c = - 8]
12. Find the acute angle between the surfaces xlz = 3x + z2 and 3x2 - l + 2z = I at

ww thepoint(I,-2, I)

J3
w.E [Ans : cos- I - - ]
7J2
13.
asy
Find grad ~ if r = xi + yj + zk, r = Irl and

(i) ~ = Log r (ii) ~ En


= -
I
(iii) ~ =r
r
gin [Ans: (i) r/~ (ii) -r/~ (iii) r/r]
14. e eri
Find the directional derivative of g = x 2l + lz2 + z 2x 2 at the point (I, I, -2) in the
direction of the tangent to the curve x = e-t, y = 2 sint + I, z = t - cos t at t = o.
[Hint: Tangent vector to the given curve is ng.
dx. dy. dz k
-l+-}+-
dt dt dt
net
2
[Ans: J6]
15. Find the acute angle between the normals to the surface xy = z2 at the points
(1,9,3) and (3, 3, -3)

16. If r = xi + yj + zk and 4> = x 3 + Y + z3 - 3xyz, show that r, grad 4> = 34>.

Downloaded From : www.EasyEngineering.net


Downloaded From : www.EasyEngineering.net

488 Engineering Mathematics - I

7.3 The Divergence of a Vector Function


7.3.1 If A = A)i + A2k is a vector function, defined and differentiable at each point
(x, y, z) in a certain region of space [i.e., A defines a vector field], then the divergence
of A (abbreviated as 'Div A') is defined as,
DivA= V.A

. a }-+
= (1-+ . a k a)
ax ay -az' (A )1'+A21'+A3k)

ww (aAax aA = I i+
ay
2 + aA3)
az
w.E (since i.i = j j = k.k = 1)
Note: (1) Div A is a scalar field

a (2) V.A 7:- A.V


syE
7.3.2 Physical significance of the divergence

ngi
If A represents the velocity of fluid in a fluid flow, Div A represents the rate offluid

nee
flow through unit volume. (or) Div A gives the rate at which fluid is originating at a
point per unit volume.

r ing
Similarly if A represents the Electric flux or heat flux, Div A represents the amount
of electric flux or heat flux that diverges per unit volume in unit time.
7.3.3 Some properties of Divergence
.ne
(1)
If A, B , are vector functions and '/' is a scalar function, then, prove that
Div (A + B) = Div A + Div B (i.e) V. (A + B) = -V . A + V .8 t
Proof Let A = A)i + Aj + A3k
B = 8 1i + B~ + B3k
A + B = (A) + 8) i + (A2 + 8 2 )j + (A3 + 8 3) k
a a
Div A = -(AI +B l )+-(A 2 + 8 2 )+-(A 3 + 8 3 )
a
ax ay az
= (aA1 + aA2 + aA3)+(aBI + aB2 + aB3)
ax ay az ax ay az
~

= Div A + Div B.

Downloaded From : www.EasyEngineering.net


Downloaded From : www.EasyEngineering.net

Vector Differentiation 489

(2) Prove that, Div (fA) = (grad.f) .A + .f{Div A) i.e. V .(fA) = (V .f). A + .f{ V A)

Proof: Let A = Ali + A'll + A3k then fA = fAli + fA'll + fA3k


a a a
V .(fA) = ax (fA.)+ ay (fA2)+ az (fA3)

= fQ~+A. af + f aA2 +A2 af + f aA3 +A3 8j


-Ox ax ay ay az az

ww ..... (1)

w .Ea
syE ..... (2)

n gin
e eri
..... (3)

(1), (2), (3) => V .(fA) = (v.f).A + f(v .A)


ng.
7.3.4 Solenoidal vectors: A vector A is said to be solenoidal if Div A = 0

Solved Examples
n et
Ex. 7.3.5 If A = (.x2y) i + (xy2z)j + (xyz)k, find div A at the point (1, -1,2).
Sol: A = (.x2y) i + (xyz)j + (xyz)k
. aA. aA 2 aA 3
DIVA=-+-+-
ax ay az
a 2 a 2 a
= -(x y) + -(xy z) + -(xyz)
ax ay az
= 2xy + 2xyz + xy
= xy (2z ,+ 3)
:. At (1, -1, 2), Div A = (1) (-1) [4 + 3] =-7

Downloaded From : www.EasyEngineering.net


Downloaded From : www.EasyEngineering.net

490 Engineering Mathematics - I

Ex. 7.3.6 If V = 2xyi + 3x2yj - 3pyz k is solenoidal at (1, 1, 1), find 'p'.
. a a 2 a
Sol: Dlv V = -(2xy)+-(3x y)+-(-3pyz)
ax oy oz

= 2y + 3xl - 3py
At (1. 1, 1). Div V = 5 - 3p
5
Since V is solenoidal, Div V = 0 :. p = 3"

ww
Ex.7.3.7 Ifr = xi + yz + zk, and r = Irl, show that Div (r3 r) = 6 r3

Sol:
w r= ~X2+y2+Z2

.Ea
:. r3 r = (xl+y +z2)3/2 (xi+yj+zk)

= Ali + Aj + A3k (say)

A2
syE
then, A I = x(xl + y + z2)312
y(x2 + Y + z2)3/2

ngi
=

A3 = z(xl + y + z2)312

nee
rin
g.n
oA2 .1. .. l
similarly ~ =3y r+r~
oA 3
-
OZ
2 .. l
=3z r+r e t
. oA I -
DlvA= - +
oA2 oA3
-+-
ax oy oz

= 3r (xl + Y + z2) + 3r3 = 3r3 + 3r3 = 6r3


Aliter: r3 r = r3 xi + r3 yi + r3zk.
a Or x
-8 (r3 x) = r3.1 + x . 3r2 -8 = r3 + x . 3r2 . -
x' x r

= r3 + 3xlr

Downloaded From : www.EasyEngineering.net


Downloaded From : www.EasyEngineering.net

Vector Differentiation 491

o 0
Similarly ay (~y) = ~ + 3yr and oz (r3z) = ~ + 3z2r

DivA=6~

l
Ex. 7.3.8 Evaluate Div [ r grad (r-3) ] or V. {rvC 3 )}

ww ar . --4 -or k--3r (x. k)


w.E _
--3r4 or .
-1-3r4 -j-3r
ox c:y oz
_ --4 -i+-j+-
r
y. Z
r r
=

asy
-3r-5 (xi + yj + zk)

r grad (~) = -3r--4 (xi + yj + zk)

En
= Ali + Aj + A3k (say)

gin
where AI = -3r--4x, A2 = -3r--4y , A3 = -3r--4z

aA I = ~ [-3r--4 x]
ax ax e eri
= -3r--4 . 1 + x. -3. -4 r- 5
ng.
= -3r--4 + 12 x r- 5 ~
= -3r--4 + 12 x 2 r-6
. :: .

net
oA
Similarly, c:y2 =-3r-4 + 12yr-6

:. Div (r grad r-3) =


oA
_I + __
oA
2 +_3
oA
ax Oy az
= -9r-4 + 12 r-6 (x 2 + y + z2) = -9r-4 + 12 r--6 . r2 = 3r-4
(Alternate method is left to the reader).

Downloaded From : www.EasyEngineering.net


Downloaded From : www.EasyEngineering.net

492 Engineering Mathematics - I

Ex.7.3.9 Show that V .(x" r) = (n + 3) r". Hence show that r/f3 is solenoidal.
Sol: r" r = r" (xi + yj + zk)
a a a
V .(r" r) = -a (xr")+ ~, (yr") +- (zr")
x v)' az

z)
ww y + z.-
= 3r" + n ['1-1 ( x.-X + y.-
r r r

w.E
= a
Ifn = -3, (r-3 r) = (-3 + 3) r-3
syE
3r" + n r"-2 . r2 = (n + 3) r"
=0

r ngi
:. 3 is solenoidal
r
n eer
Ex.7.3.10 Prove that Div (CIA + C 2 B) = C I Div A + C 2 Div B, where CI' C 2 are
constants.
i ng.
Sol: Let A = A I i + Aj + A3k
B = Bli+ Bj+ Bl net
CIA + C2B = (CIA I + C 2B I) i + (C IA2 + C 2B2U + (C IA3 + C 2 B 3 )k
a a
Div (CIA + C2B) = ax (CIA I + C2B I) + Oy (C IA2 + C 2B2)

=C (MI+M2+M3)+C(aBI+aB2+
I ax Oy az 2ax ay
aB
az
3)

Downloaded From : www.EasyEngineering.net


Downloaded From : www.EasyEngineering.net

Vector Differentiation 493

Ex.7.3.11 If A = 2xi + 3yj + 5zk and f= 2xyz, find div (fA) at (1,2,3).
Sol: fA =2xyz (2xi + 3yj + 5zk)
= (4x 2yz)i + (6xyz)j + (10xyz2)k

a a a
Div (fA) = ax (4x2yz) + ay (6xyz) + az (10xyz2)

= 40xyz
:. At (\,2,3), div (fA) = 240
Aliter:
ww div (fA) = D.(fA) = (V f). A + f( V A)

w Vf

V fA
= 2yzi + 2xzj + 2xyk

.Ea
= 4xyz + 6xyz + \ Oxyz = 20xyz

syE
f(v .A) = 2xyz (2 + 3 + 5) = 20xyz

Hence V .(fA) = 40xyz and at (\,2,3) V .(fA) = 240

ngi
Ex.7.3.12 Iff, g are scalar fields show that V fx V g is solenoidal

nee
rin
g.n
j k
e t
vfx V g = a.fjax aljay atjoz
agjax agjay agjaz

(Suffixes denote partial derivatives)


a
Div (V fx V g) = L ax Vygz - fzgy)

= L (!y gxz + gzfxy - fz gxy - ~fxz)


=0
:. V fx V g is solenoidal

Downloaded From : www.EasyEngineering.net


Downloaded From : www.EasyEngineering.net

494 Engineering Mathematics - I

Exercise 6(b)

I. If V = (xlz)i - (2y3z1)j + (vz)k, find div A at the point (I, -I, I)


[Ans. - 3]
2. Ifr = xi + yj + zk, find div r [Ans.3]

3. If F = (3,xyz2)i + (2xy3)j - (xlyz)k, and ¢ = 3xl - yz,


find (i) Div F (ii) Div (¢F) and (iii) Div (grad¢); at the point (I, -I, I)

ww4.
[Ans. (i) 4 (ii) I (iii) 6 ]
If V = (3xly - z)i + (xz 3 + y4)j - 2x3z 2k, find grad (Div V) at the point (2, -I, 0)

w.E Evaluate: (I) Div (r2r) (2) Div (r r) (3) grad Div (r/r) (4) div (r/r3)
[Ans. - 6i + 24j - 32k]

asy
5.
[ Ans. (1) 5r2 (2) 4r (3) -2r/r3 (4) 0 ]
6. Show that V =
En
3y4z2i + 4x3z7 + 6x2y3k is solenoidal
7.
gin
Show that the vector F = (2xl + 8xyz)i + (3~y - 3xy)j - (4yz2 + 2x3z)k is not
solenoidal, but G = xyz2 F is solenoidal.
8. If a is a constant vector and V =
eer
a x r, where r = xi + yj + zk, show that V is a

9.
solenoidal vector.
ing
Determine the constant 'b' such that the vector, V = (2x + 3y)i + (by - 3z)j +
(6x- 12z)kis solenoidal
.ne
10.

11.
Ifr, and r2 are vectors joining fixed points A (x"YI' z,) and B(x2'Y2' z2) to a variable
point P(x, y, z) prove that r, x r2 is solenoidal.
If r = xi + yj + zk and r = Irl show that, Div (grad rn) = n( n+ I )rn- 2 .
t
12. If g = r-2n , find div (grad g) amd find 'n' such that 'g' is solenoidal.
2n(2n -I) I
[A ns. ?n+2 ·n--
,- 2 ]
r-

7.4 Curl of a vector function


7.4.1 If A is a differential vector function, then curl A is defined as, curl A = V' x A

j k

If A = Ali + Aj + A3 k , then Curl A = a/ax a/ay a/az


AI A2 A3

Downloaded From : www.EasyEngineering.net


Downloaded From : www.EasyEngineering.net

Vector Differentiation 495

= (aAJ _ aA 2 )i + (aA I _ aA3)j + (aA 2 _ aA I )k


0' az az ax ax 0'

Note: The curl of a vector is also a vector


7.4.2 Physical significance of curl

ww Let r = xi + yj + zk be the position vector cf a point P(x, y, z) of a rigid body rotating


about a fixed axis about the origin 0 with an angular velocity ffi = ffili + ffi.j + ffi)k.

wThen the velocity V of the particle P is given by,

.Ea
i j k
V = ffi x r = ffil

X
ffi2

Y syE
ffi3

ngi
Curl V= a/ax
j
8/ay
k
a/8z nee
rin
g.n
e t
= i (ffi 1+ ffi I) + j (ffi2 + ffi 2) + k ( ffi 3 + ffi3) = 2ffi
Thus the curl of velocity vector is twice the angular velocity of rotation.
7.4.3 Irrotational Vector: A vector V whose curl is zero is said to be an irrotational
vector.
7.4.4 Properties :(1) V x (A + B) = V x A + V x B (or) curl (A + B) = curl A + curl B
Proof: Let A = Ali + A.j + A)k and
B = Bli + B.j + B)k so that

Downloaded From : www.EasyEngineering.net


Downloaded From : www.EasyEngineering.net

496 Engineering Mathematics - I

i j k

Curl (A+B) =
ajax ajay ajaz
AJ+BJ A2 +B2 A3 +B3

j k j k
= ajax ajay ajaz + ajax ajay ajoz

ww = curl A + B
(2)
w.E If ¢ is a scalar function and A is a vector function
Curl (¢A) = ((curl A) + (grad¢) x A

a (or)
V x (¢A) = ((V x A) + (V {P) x AsyE
ngi
Proof: If A = Ali + Aj + A3k, then cj>A = cj>Ali + cj>Ai + cj> A3k

L\ x (cj>A) = ajax ajay ajaz


j k
nee
cj>AI cj>A2 cj> A 3
r ing
i[~(q,A3)-~(cj>A2)]+ j[~(cj>AJ)-~(cj>A3)J+ k[~(cj>A2)-~(cj>AJ)]
=
ay az az
.ne ax ax ay

= "i[.I.
~
aA3 + A aq, _.I. OA2 - A aq,]
"'ay 3ay "'az 2az
t
= .I. L( aA 3 _ aA2)i + L(A aq, _ A acj»i
'" ay az 3ay 2az

i j k i j k
= cj>ajax ajay ajaz -?> aq,jax aq,jay acj>jaz
AI A2 A3
=cj>( V x A) + (V cj» x A
7.4.5 Conservative vector field:
A vector field F, which can be derived from a scalar field q, such that F = Vcj>, is
called a conservative vector field and q, is called the scalar potential ofF.

Downloaded From : www.EasyEngineering.net


Downloaded From : www.EasyEngineering.net

Vector Differentiation 497

Solved Examples

Ex.7.4.6 If A = (xy)i + (yz}j + (zx) Ii; find (a) curl A and (b) curl curl A at (1,2, -3)
Sol : A = xyi + yzj + zxk
j k
(a) curl A = V x A = a/ax a/ay a/az
xy yz zx

ww.[ aya a].[


= I -(zx)--(yz) a a ] +k[-(yz)--(xy)
+ } -(xy)--(zx)
&
a
&
a]
& & ay
w = i(O -
=
y) + j(O - z) + k (0 - x)
.Ea
-yi - zj - xk

syE
:. curl A at (1, 2, -3) = -2i + 3j - k
(b) curl A= V x (V xA)

=V x (-yi-zj-xk) n gin
=
i
a/ax a/ay a/az
j k
e eri
-y -z -x ng.
[aya a].[
= i -(-x)--(-z) a a ] +k[-(-z)--(-y)
+ } -(-y)--(-x) a a ]n et
& fu fu fu ay
= i(O - (-I) + j(O - (-1» + k(0 - (-I»
=i+j+k
:. curIAat(I,2,-3)=i+j+k
Ex.7.4.7 Show that V = xi + Ij + z3k is irrotational -
Sol: Curl V = V x V

a 3)---(y
= I.[-(z a 3]) +k[-(y
a 2]) + }.[-(x)--(z
a a 2)--(x)
a ]
ay az oz & fu ay
=0 :. V is irrotational

Downloaded From : www.EasyEngineering.net


Downloaded From : www.EasyEngineering.net

498 Engineering Mathematics - I

Ex.7.4.8 If F = (4x + 3y + az)i + (bx - y + z)j + (2x + cy + z)k is irrotational, find the
constants a, b, c
i j k
Sol: Curl F = ajax ajay ajaz
(4x+3y+az) (bx- y+z) (2x+cy+z)

= i[~(2X+CY+Z)-~(bX-
ay az
y+Z)]+ j[~(4x+3y+aZ)-~(2X+CY+Z)J
az ax

k[~(bX- y+Z)-~(4x+3y+az)]
ww +
ax ay

w = (c - l)i + (a - 2)j + (b - 3)k

.Ea
Since F is irrotational, curl F = 0

syE
:. c - I = 0, a - 2 = 0, b - 3 = 0
i.e., a = 2, b = 3, c = 1

n
Ex.7.4.9 If r = xi + yj + zk, and r = Irl, find curl (rn r)

r = xi + yj + zk, r = Irl = ~ x 2 + y2 + Z2
gin
Sol :
rn r = (.xl + Y + :?)n/2 (xi + yj + zk) e eri
j
ng. k
curl (rn r) = ajax
III
x(x 2 + y2 + Z2Y2 Y(X2 +
ajay
l + Z2Y2
1/ 1
l
Z(X2 + n
ajaz
+ Z2 )"2
et
= I.[ z."2
n x- x + Y 2 +z 2) 2z J
(? + y-?+z-?hJLY- Y."2n (2

+ ).[ x."2(X n 2 + y 2 +z 2 ).2x ]


n 2 + y 2 +z 2 ).2z-z."2(x

= i(O) + j(O) + k(O)


=0

Downloaded From : www.EasyEngineering.net


Downloaded From : www.EasyEngineering.net

Vector Differentiation 499

Aliter: rn r = rn (xi + yj + zk)

j k
curl (rn r) = ajax ajay ajaz
xrn yr" zr"

= "'.[
L..,l - a ( zr n) - -
a ( yr n)] = "'.[ ar - y.nr n-I -ar]
L..,l, z.nr n-I -
ay az ay az

z]
ww_
-
'" .[
L..,l
Y - y.n.r II_I -;
f/zr n-I -;

w.E
= i( 0) + j( 0) + k (0) = 0
Ex. 7.4.10 Prove that, ifF = (x + Y + I)i + j - (x + y) k, F. curl F = 0

asy
j k
Sol: Curl F = ajax
En
a/ay ajaz
x+ y+1 -x-y
gin
= i

= -;
[-I -0] + j[O + I] + k[O -1]
+j - k e eri
:. F curl F = -I (x + y + 1) + 1.1 + 1(x + y) =0
ng.
Ex. 7.4.11 P(x, y, z) is a variable point and Q(xl' YJ, zJ)' R(x2, Y2' z2) are fixed points.
If U = QP and V = RP; Prove that curl (U x V) is equal to 2(U - V). net
Sol: P(x, y, z), Q = (xl' Yl' zJ)' R = (x 2, Y2' z2)
:. QP = (x -xJ); + (y - YJN + (z - zJ)k = U
RP = (x - x 2); + (y - Y2 N + (z - z2)k =V
j k
UxV= (X-XI) (Y-YI) (Z-Zl)
(x-x 2) (Y-Y2) (Z-Z2)

= ~{(y - YJ) (z - z2) - (z - z\) (y - Y2)} ;


=~ {y(z\ - z2) - z(yJ - Y2) + (yJ z2 - Y2 z J)}i

Downloaded From : www.EasyEngineering.net


Downloaded From : www.EasyEngineering.net

500 Engineering Mathematics - I

Curl (U x V) = I[ ~{x(y, - Y2)- Y{X, -X2)+{X'Y2 -x2 y,)}

-! {Z{X, -X2)-x{Z, -Z2)+{Z,X2 -Z2XJ}}

= L(-(XI - X2) - (xI - X2);


= L -2(x l - X2 ); = 2L(X2 - xI);
= 2(U - V)

ww
Ex. 7.4.12 If F is a conservative vector field show that curl F = 0
Sol. F is a conservative vecor field.

w.E :. There exists a scalar field '~' such that F =


V'"
'I'
8$. 8$. 8$ k
= ax 1 + ~ J + az
asy ; j k
:. Curl F = ajax En ajay ajaz
a~jax
gin a~j~ 8$jaz

i( a ~ a ~
== L
2
eer
_
2
) == 0
~az az~
ing
Ex. 7.4.13 Show that F == (6xy + iJ)i + (3r -z)j + (3xz 2 - y)k is irrotational. Find ~ such
that F == V ~ .ne
Sol: Curl F = ajax
i
j
aj8y k
ajaz J t
6XY+Z3 3X2 -z 3xz2-

== i (-1 + 1) + j (3z2 - 3z2) + k(6x - 6x)


==0
:. F is irrotational

8$. 8$. a~k


L tF== V~== -I+-J+-
e ax ay az

:. : == 6xy + iJ ..... (I)

Downloaded From : www.EasyEngineering.net


Downloaded From : www.EasyEngineering.net

Vector Differentiation 501

a~
- = 3x2 -z ..... (2)
Oy

a~ ?
- = 3xz~ - y ..... (3)
az
Integrating equation (1) with respect to x we get
~ = 3.x2y + xz 3 + fly, z) ... :. (4)
Differentiating (4) partially w.r.t 'y'

ww a~
ay = 3.x2 +
af(y,z)
ay ..... (5)

w.E From (2) and (5) we have

asy
af(y,z)
ay =-z ..... (6)

En
Integrating (6) w.r.t 'y' we get
fey, z) = -yz + h(z) gin
Hence ~ = 3x2 + zx 3 - yz + h(z) e eri ..... (7)

ng.
I
difffrentiating (7) w.r.t 'z' we get

net ..... (8)

Comparing (3) and (8) we have


h l(z) = 0, :. h(z) = constant, 'c' say
Hence ~ = 3.x2y + xz 3 - yz + c (from 7)

Aliter: -~ =6xy+z3 .... (I)


ax
a~
ay = 3.x2 - z .... (2)

..... (3)

Downloaded From : www.EasyEngineering.net


Downloaded From : www.EasyEngineering.net

502 Engineering Mathematics - I

Integrating the above equations respectively w.r.t x, y, and z, we get


= 3x2y + xz3 + f(y, z)
~ .... (4)
~ = 3x2y - yz + g(z, x) .... (5)
~= xz3 - yz + h(x. y) ..... (6)
~ should satisfy all the above three equations simultaneously
(i.e.) (4), (5) & (6)
3x2y + xz3 - yz + c, where c is a numerical constant.
:. ~ =
Note:
ww [Herej= -yz, g = xz3, h = 3x2y will satisfy (4), (5) & (6)
Ex.6.4.14 Iff(r) is differentiable, show that f(r)r is irrotational
Sol:
w.E
fir) r = fir) (xi + yj + zk)

j
a
Curl (f(r) r) = %x
syE
j
o/Oy
xj{r) y j{r) zj{r
%z
k

I{; {zj{r)}- ! {y j{r)})ng


=

ine
= I{z./{r)~ - y./{r):) eri
I{z./{r)~ -y./{r);) =0
ng.
..
=

f{r)r is irrotational
net
Ex. 7.4.1SlfU and V are irrotational, prove that U x V is solenoidal
Sol: Let U = Uti + U2i + Ui
V=V t i+V2i+ Vi
U is irrotational :. curl U = 0

=> Li[OU 3 _ OU 2 ] = 0 ..... (1)


oy oz
..
sImilarly, '" .[OV
L-l - 3 oV 2] = 0
Oy oz
- - ..... (2)

Downloaded From : www.EasyEngineering.net


Downloaded From : www.EasyEngineering.net

Vector Differentiation 503

} k
VxV= VI V2 V3 =1:;(U 2V 3 -U 3 V 2)
VI V2 V3

..... (3)

ww au 3 _ au 2 • au I _ au 3 . au 2 _ au I
From (I) ,we have, - - - - - , - - - , - - - -
0' az az ax ax 0'
..... (4)

w.E ,
av3 _ av2 • aV _ av3 . av2
an d from (2) we have - - - -I- - - - -I
'0' az' az ax' ax 0'
_ aV
..... (5)

asy
Substituting the six equations of(4) & (5) in (3), we observe that all the 12 terms of
(3) will get cancelled. Hence Div (U x V) = 0
~ U x V is solenoidal En
gin
Exercise - 7(c)

e eri
I. If V = (2xz2);_ (yz)j + (3xz3)k, andf= xlyz, find the following at the point (I, I, I)
(a) curl V (b) curl (jV) (c) curl (curl V)
ng.
[Ans. (a) ; + } (b) 5; - 3} - 4k (c) 5; + 3k]
2. If 'g' is a scalar function, show that curl (g grad g) = 0
net
3. Find the value of the constant 'p' for which the vector V = (pxy - z3) + (p - 2) xl}
+ (I - p) xz2k is irrotational.
[Ans: 4]
4. Find the constants a, b, c so that the curl of the vector A = (x + 2y + az);
+ (bx - 3y - z)j + (4x + cy + 2z)k is identically equal to zero
[Ans. 4, 2, -1]

5. If r = xi + y) + zk, r = Irl, show that curl (;Z ) = 0 and find a scalar function of' such
r
that 2"= -v J,.f(a) = 0 where a> o.
r

[Ans :f= log (~)]

Downloaded From : www.EasyEngineering.net


Downloaded From : www.EasyEngineering.net

504 Engineering Mathematics - I

6. If r = xi + yj + zk, and p, q are constant vectors, show that (I) curl [(r x p) x q]
= ( p x q) and (2) curl [(p.q)r)] = o.

7.5 Laplacian Operator: V2

7.5.1 V2 = V.V = (i~+


ax j~+k~).(i~+ j~+k~)
ry az ax ry az
a2 + -a2' + -a22 ) .IS called the Laplacian
. Operator
=
( ax 2 ay2 az
-

ww 'V2' can be applied to both scalar and vector functions as shown below.

w.E 2
V~=-+-+-
a2~ a·2~
ax 2 ry2 az2
a2~

where '~' is scalar function


asy
If A = Ali + Aj + A3k, is a vector function, then

a2 a2 a2 ) En
V2 A =
gin
( ax2+ ay2 + az2 (Ali + Aj + A3k)
= (V2AJ+(V2A2)j+(V2A3~
eer
7.5.2 Vector Identities: we shall give below some vector identities with proofs.
\. If '¢' is a scalar function curl grad ~
ing
= 0, (or) V x V ~ = 0
acjl acjl. acjl )
(-i+-}+-k
.ne
Proof: Curl grad cjI= V x
ax ry az
j k
t
ajax ajry ajaz
acjljax acjljay acjljaz

= L>[~(acjl)-~(a~)l
ay az az ry

= 0, assuming that '~' possesses continuous second order


partial derivatives.

Downloaded From : www.EasyEngineering.net


Downloaded From : www.EasyEngineering.net

Vector Differentiation 505

(2) IfY is a vector function, Div (Curl A) = 0 (or) V.(Vx A) = 0


Proof: Let A = Ali + A-j + A3k

CurIA= " .(-


~I aA-3 -aA-, )
ay az

Div (curl A) = "~- a {aA3 aA)}


ax -ay- - -az-
a~A3 a2 A2 a2A4 a~A3 a~A~ a~AI
ww = -----+-----+-----
axay axaz ayaz ayax azax azay

(3) w.E =0
If A is a vector function, curl (curl A) grad (Div A) - V2 A (or)

asy
Proof: curlA= "~l.(-
aA-3
ay -az-En
aA, )
gin
eer
Curl (curIA)= a/ax a/ay
j
a/az
k
ing
aA3_ aA2) aA I _ aA3) aA2_ aAI)
( ay az ( az ax ( ax ay .ne
= I[~(aA2 _ aA I)_~(aAI _ aA 3)]i
t
ay ax ay az az ax

Downloaded From : www.EasyEngineering.net


Downloaded From : www.EasyEngineering.net

506 Engineering Mathematics - I

ww
w.E
= -'1 2A + '1('1.A)
(4) asy
'1.(AxB)=B.('1xA)-A.('1xB), (A, B are vector functions) (or) Div(AxB)=B.
curl A-A. curl B.
En
Proof: Let A = A) i + A?J + A3k and B = B) i + B?J + B3k

i j k gin
A x B = AI A2 A)
eer
I
BI B2 B)
ing
= i(A2B) - A)B2)
.ne
Div (A x B) = "-(A2B)
LJ -A3B 2)
a
ox t
= " (A OB3 + B OA2 + A OB2 + B OA3)
L:. 2 AX 3 AX 3 AX 2 ax ..... (1)

'1xA = a/ax
AI

..... (2)

Downloaded From : www.EasyEngineering.net


Downloaded From : www.EasyEngineering.net

Vector Differentiation 507

Similarly,

A. (V' x
" (BB3
0' - BB?)
B) = L.J AI Bz- ..... (3)

Expanding the summations of (I), (2) & (3), we observe that,


Div (A x B) = B. (V' x A) - A. (V' x B)

7.5.3 Operation of V' on product of two functions


Suppose ~ and 'P, are two scalar or vector point functions. When V' is operated on
the product of ~ and \j1, the following rule is useful.

ww V' (~'P) = V' (~o \{J) + V' (~'P 0) wherein the suffix '0' indicates that the function
is not to be varied, that is, V' is not to be operated on that function. After the

w completion of operation of V' the suffixes are dropped.

.Ea
While proving the identities the following are useful.
(a)
(b)
a.b = b.a
syE
a x b = -b x a
(c) axa=O
ngi
(d)
(e)
a.(bxc)=(axb).c
a x (b x c) = (a. c)b - (a. b) c nee
(f) (a. c) b = a x (b x c) + (a . .b}C
rin
(g) a. a = a2
g.n
(h) x operation is always between vectors only
vector identities using V' operation: e t
(i) V' .(FG) = V' . (FG o) + V' . (FoG)

= V' F.G o + Fo V' .G


:. div (fG) = G. grad F + F div G

(ii) V' x (FG) = V' x (FG o) + V' x (FoG)

V' x (FG o) = V' F x Go due to (h)

V' x (FoG) = (V' x G)Fo=Fo(V' xG)

:. V' x (FG) = V' F x G + F( V' x G)


(i.e.) curl (FG) = (grad F) x G + F curl G

Downloaded From : www.EasyEngineering.net


Downloaded From : www.EasyEngineering.net

508 Engineering Mathematics - I

(iii) V'. (F x G) = V' .(F x Go) + V'. (Fo x G)


V' . (F x Go) = (V' x F) .G o due to (d)
= Go .( V' x F) due to (a)

V' . (Fox G) = - V' . (G x F0) due to (b)


= - V' x (G. F0) due to (d)
=-(V' x G). Fo
=-Fo(V' x G) due to (a)

ww :. V'. (F x G) = G. (V' x F) -F. (V' x G)


Thus div (F x G) = G. curl F - F. curl G

w.E
(iv) V' x (F x G) = V' x (F x Go) + V' x (Fo x G)

a
V' x (F x Go) = (V' . Go) F -
syE
= (Go· V') F - Go (V' . F)
(V' . F) Go due to (e)
due to (a)
V' x(FoxG)=(V'.G)Fo-(V'·Fo)G
= Fo (V' . 0) - (F o · V') G ngi due to (e)
due to (a)

nee
:. V' x (F x G) = (G . V') F - G( V' .F) + F (V' .G) - (F. V' ) G
Thus curl (F x G) = (G. V' ) F - (F. V' ) G + F (V' .G) - G (V' .F)
"I- rin
(v) V' (F . G) = V' (Fo . G) + V' (F .G o) {
V'(Fo.G)=Fox(V' xG)+(Fo. V')G g.n
due to (t)
V' (F . Go) = V' (Go. F) = Go x (V' x F) + (Go. V') F
:. V' (F . G) = F x (V' x G) + G x (V' x F) + (F . V') G + (G . V') F
et
due to (t)

Thus grad (F . G) = F x curl G + G x curl F + (F . V') G + (G . V') F


(vi) curl (grad F) = V' x (V' F) = (V' x V') F = 0 due to (c)
(vii) div (curl F) = V' .( V' x F) = V' x (V' . F) due to (d)
= ( V' x V' ). F = 0 due to (c)
(viii) curl (curl F) = V' x (V' x F)
= (V' . F) V' - (V' . V' ) F due to (e)
= V' ( V' . F) - V' 2 F (since V'. must not appear at the end)
= grad (div F) - V' 2 F

Downloaded From : www.EasyEngineering.net


Downloaded From : www.EasyEngineering.net

Vector Differentiation 509

Solved examples

Ex.7.5.4 If/= x 21z2, find V 2/ at (1,2, 1)


2 2 2
0 / 0 / 0 /
Sol: V Y = --J +--+-J-
ox- 0/ oz-
= 21z2 + 6x2yz2 + 2x21

:. at (1, 2, 1), v2j= 16+ 12+ 16=44.

ww
Ex.7.5.5 Show that, if r = xi + yj + zk, r = \rI, then V 2r ll = n(n + 1) rll 2

Sol:
w.E
~ = (x2 + Y + z2)n12, (·:r = ~x2 + y2 +Z2 )

asy
En
gin
e eri
ng. ... (1)

2
Similarly -02 (r n) = n ( n- 2)y 2r n-4 + nr n-2
, oy
net... (2)

2
and -a2 I!
( n)
=n ( n- 2)z-r
J n-4
+nr n- 2 ... (3)
oz

= 3n,.n-2 + n(n-2) ~-4 (x 2 +y + z2) adding (I), (2) & (3)


= 3n~-2 + l1(n - 2) r"-2 (": x 2 + y + z2 = r2)
= n(n + 1) ~-2

Downloaded From : www.EasyEngineering.net


Downloaded From : www.EasyEngineering.net

510 Engineering Mathematics - I

Aliter: -a (r") =nr 11-1 .-=nr


ar 11-1 .-
x
ax ax r

= n,.rr2 . x

~~II)=
2 n(r ll
-
2.1 + x.{n _ 2)r"-3. ar)
az ax

~ n(,"-' + (n-2}r"-'. x:)


ww = nr"-2 + n(n-2) r"--4. xl ... (I)

w.E 2
Similarly -a2 (r ") =nr 11-2 +n (n- 2)r n-4 .y 2
, ay ... (2)

anda az syE
~2 ~n ) = nr n- 2 + n{n - 2 )r n-4.Z2 ... (3)

Adding (I), (2) & (3) we get,


ngi
nee
=> V2rn = 3nr"- 2 + n{n - 2)r n- 4.r2
= ,.rr2[3n + n2 - 2n] rin
= n(n + I) r"-2
g.n
Note: Ifn =-1, we have
et
vt~ )= 0, which means that (.; ) satisfies the Laplace's equation

av au
Ex.7.S.6 If V .U =0, v .v =0, v xU=-at' v xV = at' show that Uand V satisfy
a2u
the wave equation V 2U = at 2

Sol: vx(vxU)= v x (_av) = -a{vxv)


at at
2
-a(au)_ -a u
= at at - ----ai2 ... (1 )

Downloaded From : www.EasyEngineering.net


Downloaded From : www.EasyEngineering.net

Vector Differentiation 511

But V x (V x U) = V (V .U) - V 2U

= - V 2U (": V U = 0) ... (2)

from ()) & (2), a,2 = V2U , which shows that U satisties the wave equation.
i'iu

Similarly;

Vx (Vx
au a
v)= Vx- =-(Vx u)=--
-a~v
... (3)
at at2
at
and vx(vxY)=V(V.Y)-V 2 Y=-V 1 Y(": V.Y=O)

ww ... (4)

w.E
(3) and (4) => Y satisfies the wave equation.
Ex.7.5.7 If V.Y = 0, show that V x[V x {V x (V x V)}] = V 4y (or)

asy
curl [curl {curl (curl Y)} ] = V 4y
Sol: We know that
En
V x (V x Y) = V (V .Y) - V 2y [from 7.5.2 (3)]

= - V 2y (": V.Y = 0) gin


Then the given expression
= - U (say)
e eri
= - V x( V x U) = V 2U - V CV .U) ng.
[from 7.5.2 (3)]

= V 2( V 2y) - V (V . U)

= V 4y - V (V . U)
(": V 2y = U)
net
V . U = V .( V 2y)

= {~) ~}{V2V}
= I {i ~ }.(V2(~i + V2i + V k)) 3

= V2(a~ + aV2 + aV3)


ax 0' az
= V2(DivV)= V2(0) = 0 (": Div Y = V . Y = 0)
Hence the problem

Downloaded From : www.EasyEngineering.net


Downloaded From : www.EasyEngineering.net

512 Engineering Mathematics - 1

Exercise 7(d)

(I)

(2)

(3) Show that V-


ry( v. r"r) =-;-:t
2

ry () = ~ry
ww
(4) Show that V-<j> r
d"<j> 2 df
+--
dr- r dr

w .Ea
1 ~'ry
show also that, if V-<j> = 0, then <j> = C] + -- where CI' C 2 are constants
r
I I
(5) If r = xi + )1 + zk,
syE
prove that, curl (k x grad -) + grad
r
(k. grad -) = 0
r
7.6
7.6.1
VECTOR INTEGRATION
Ordinary integration of vectors n gin
(I)
e
If A(u) = A](u)i + A2(u}j + Aiu)k be a vector function of a scalar variable 'u'

eri
(A,(u), Aiu), A 3(u) assumed to be continuous in any given interval), the indefinite
integral of A(u) is given by.

fA(u)du = i fAl(ll)du+} fA" (u)du +k fA3(1l)du ng.


(2) If there exists a vector B(u) such that A(u) =
d
du
n
(B(u)), we can write et
fA(U )du = f:U (B(ll ))dll = B(u) + c
where c is a constant of integration independent of u.
(3) The definite integral of A(u) between the limits 'a' and 'b' in the above, is written as,
h b d b
fA(u)du = f- (B(u))du = B(u)+cl = B(b) - B(a)
a a
du a

7.6.2 Line Integrals:


Let A(x,y,z) be a continuous vector function defined in the entire region of space.
Let c be any curve in the region. Divide c into n intervals by taking points
A = Bo, BI' B2 ... Bn(= B).
Let Pi be any point in the interval B i_, Bi
0" ' •.;.;.

Downloaded From : www.EasyEngineering.net


Downloaded From : www.EasyEngineering.net

Vector Differentiation 513

ww A= 8 0

w
Let ro' rJ ...... rn be the position vectors of points Bo' B)' B2 ...... Bn respectively. Let

.Ea
us consider the sum,

syE
The limit of this sum as n ~ 00 and 1&,:1 ~ 0 is defined as the line integral of A

f A.dr or n
along the curve c and is denoted symbolically by
dr
f A.-dt; gin which is a scalar.
dt

If c is a closed curve, the integral is written as


e fA.dr eri
Cartesian form of line integral: ng.
If A=AJi+A-j+Ai
dr = (dx)i + ({Mi + (dz)k, n et
f A .dr = fA1dx + A2 dy + A3 dz
c

Note: f<l>dr, and fAX dr are also examples of line integrals.

7.6.3 Physical appliations:


(1) Work done by a force
(I) If A represents a force and dr is an element of the path of the particle along a
curve c, then the line integral
{J

fA .dr
J'
(P, Q are 2 points on c)

represents the work done by force A in moving the particle from P to Q.

Downloaded From : www.EasyEngineering.net


Downloaded From : www.EasyEngineering.net

514 Engineering Mathematics - I

(2) flow or circulation:

(2) If A is the electric field strength, the line integral given above i.e., JA.dr, is

called the flow of A along c. If c is a closed curve it is often referred to as circulation


of A around c.
{J

In general, the line integral JA.dr, will depend on the path from P to Q
/'

ww
7.6.4 T"eorem: Prove that the necessary and sufficient condition for the integral fA.dr,
c

w to be independent of the path c joining any two points is that A is a conservative

.Ea
vector field, (or) there exists a scalar field I/> such that A = VI/> (or) curl A = 0,
[i.e., the work done by the force A in moving the particle from one point to another

syE
is independent of the path if A = VI/>]
Proof Let P = (x I ,y I ,Z I)' Q = (x2'Y2,z2) be any two given points on the curve c. Let A = VI/>

(J
n
where I/> is single-valued and has continuous derivatives.

gin
JA.dr
(1) Work done =
I'
e eri
Q G~ 01/> 01/> 01/»
fVI/>.dr= i-+j-+k-
ry
(dxi+dyj+dzk) ng.
p

Qol/>
p

01/>
ox

01/> (J
oz

J-dx+-dy+-dz= fdl/> = I/>(Q)-I/>(p)


n et
/' ox ry oz I'

= ~(x2'Y2,z2) -1/>(xI'Yl'zl)
i.e., the integral depends upon the two points only but not on the path joining them.
(This is true only if I/> is single-valued at all points P and Q).
(2) The integral is independent of the path. Then,
(x,y,z) (x,y,z) dr
I/>(x,y,z) = f A.dr = f A. ds ds
(Xl ,YI ,Zl) (Xl ,)'1 ,zl )

Differentiation with respect to's' gives

dl/> =A. dr ...... (I)


ds ds

Downloaded From : www.EasyEngineering.net


Downloaded From : www.EasyEngineering.net

Vector Differentiation 515

But
d<j> =V<j>dr ...... (2)
ds ds
dr dr
(I) - (2) => (A - V<j». -d = 0, which is true irrespective of-
s ds
:. A= V<j>
Solved Examples
4

Ex. 7.6.5: F(t) = (3t 2-t)j + (2-6t}j - 4tk, find (a) JF{t}dt (b) JF{t}dt
2

Sol:
ww
(a) JF{t}dI=; J{3/
2
-/~t+ j J{2-61}dI-k J41dl

~ (I' w
- I~} +(21-31'); -2t'k+c
.E-~lja )j - cj
(b) fF{t}dt
2 syE=(/3
2
+ (21 -3t 2 2t 2k +
2
= 50; - 32j - 24k

ngi
x/2

Ex.7.6.6 Evaluate: n{3sin e)i + {2cose)j}ie

Sol:
o
Given integral nee
xl2

r x/2

=; J(3sine}de+ j J{2cose}de=-3cosel;
xl2
+2sinejJ
o
ri/2

ing =3;+ 2j

.ne
0 0 0
Ex.7.6.7 The acceleration a ofa particle at any time 't' ~ 0 is given by,
a = e-tj - 6(t + 1}j + (3sint)k
Find the velocity V and displacement r at any time 't' given that V = 0
when t = 0 and r = 0 when t = O.
t
d 2r
Sol: a = e-t ; - 6(t + l}j + (3sint)k = -2
dt

:. V ~ : ~ Jadl ~ - e-t i - 6(1~ +I lj - (3cost)k + C, ..... (1 )

(C 1 is constant of integration)
But V = 0 when t = 0
:.-i-3k+C 1 =0=> C 1 =;+3k
Substituting in (I), we get
Velocity V = (I - e-t)i - (3t2 + 6t}j + (3 - 3cost)k

Downloaded From : www.EasyEngineering.net


Downloaded From : www.EasyEngineering.net

516 Engineering Mathematics - I

Integrating,
r = (I + e-t)i - (t3 + 3t2)j + (3t - 3sint)k + C 2 ..... (2)
(e 2 being constant of integration)
But r = 0 when t = O.
:. (2) => + i + C 2 = 0 :. C 2 = - i
From (2), r = (t - 1 + e-t)i - (t3 + 3t2)j + (3t - 3sint)k
Ex.7.6.8 Show that

ww f d2F
Fx--=Fx-+c
dt 2
dF
dt

w.E
Sol: We know that

~(FxdF)=FX~(dF)+dFxdF
dt dt
asy dt dt dl dl

En
Hence the result. gin
Ex.7.6.9 If A = 2ti + 3t7 - (4t + 1)k, and B = ti + 2j + t 2k, find
2 2 eer
(i) f(A.B}dt
o o
(ii) f(A x B}dt .
ing
Sol: (i) A.B = (2t)t + (3t ) (2) - t2 (4t + 1) = 2t2 + 6t2 - 4t3 - t2 = 7t2 - 4t3
2
.ne
t
i j
2
(ii) A x B = 21 3/
t 2

= (3t4 + 8t +2)i + (--4t2 - t - 2t3)j + (4t - 3t3 )k

196. 62.
= -1--j-4k
5 3

Downloaded From : www.EasyEngineering.net


Downloaded From : www.EasyEngineering.net

Vector Differentiation 517

Ex.7.6.10 If F(2) = ; + 2j - 2k and F(3) = 6; - 2j + 3k,


3 dF
evaluate IF.-du
2 du
Sol: From vector differentiation,

d dF dF
We get, -(F.F)=F.-+-.F=2(dF)
F.- so that
du du du du '

F. dF =~~(F.F)=~~IFI2
'du 2 du 2 du

ww
w .Ea
But F(2) = ; + 2j - 2k ~ IFI2 = I + 4 + 4 = 9, when u = 2
syE
and F(3) = 6; +2j - 3k ~ IFI2 = 36 + 4 + 9 = 49, when u = 3

3 dF I
. IF.-du=-[49-9] =20 ngi
., 2 du 2
nee
Ex. 7.6.11 If F = (.xl - 2y); - 6yzj + 8xz2k,
r ing
evaluate IF.dr from the point (0,0,0) to the
c

point (I, I, I) along the following paths


(1) x = t, Y = t2 , z = t3 . .ne
t
(2) the straight line from (0,0,0) to (I ,0,0), then to (1, I ,0) and then to (1,1, I) and
(3) the straight line joining (0,0,0) to (1, I, I).
Sol. (1) x = t; Y = t 2; z = t 3,.
dx = dt; dy = 2t dt; dz = 3t2 dt·,
when t = 0, the point is (0,0,0), when t = 1, the point is (1,1, I)

IF.dr= I(x 2 -2y}tx-6yzdy+8xz 2 dz


c c

/;1 1
J~2 -2t 2 }it -6.t 2 .t 3(2t}dt + 8t~3 J{3t 2 }it =
2 6 9
J-t dt -12t dt + 24t dt
/;0 o

Downloaded From : www.EasyEngineering.net


Downloaded From : www.EasyEngineering.net

518 Engineering Mathematics - I

Irf 9 6 2 \, 24t lO 12t 7 (3 I


= J~24t -12t -t p t = - - - - - - !
o 10 7 30

12 12 1 252-180-35 37
-----= =
5 7 3 105 105
Aliter: Along C, F = (t2 - 2t2 )j - 6. t 2 t 3j + 8.t.t6 k = - t 2 j - 6tJ + 8t7k
dr = (dx)i + (dy}j + (dz)k = dt i + (2t dt}j + (3t2 dt)k
I

J(- t -12t + 24t ~t


2 6 9
JF.dr = (Taking dot product of F and dr)
c 1=0

ww 37

(2)
105
w.E
Let 0 = (0,0,0), P = (1,0,0), Q = (I, 1,0), R = (I, 1,1 ).
Then along OP, y = 0, z = 0, dy = 0, dz = 0 and x varies from 0 to 1.

2
I
asy 2
I

~ x~
En
:. JF.dr= J(x -2.0}tx-6(OXOXO)+8x(0)2(0)= Jx dx=±
0
.... (I)

gin
Along PQ, x = I, z = 0, dx = 0, dz = 0 and y varies from 0 to 1.
I

:.
I'Q
JF.dr=
y=O e
J(12-2y~-6y(0)dy+8.1.(0)2(0)= Jo=O
eri
.... (2)

Along QR, x = I, Y = 1, dx = 0, dy = 0 and z varies from 0 to I.


I I ng.
QR z=O
2
:. JF.dr= J(12-2.IXO)-6.1.z(0)+8.1.z2.dz= J8z dz=%
0 net.... (3)

Adding (I), (2), (3), fF.dr =~ + 0 +! = 3


3 3

(3) The equation of the straight line joining (0,0,0) to (1,1, I) is f = f


~ = = t (say),
so that x = t,y = t, z = t, dx = dy = dz = dt 't' takes values from 0 to I.
I I

.. JF.dr = n~2 2t)- 611 + 8tJ }it = J(8t 'Vt


2 3 2
- 5t - 21
c o o

Downloaded From : www.EasyEngineering.net


Downloaded From : www.EasyEngineering.net

Vector Differentiation 519

Ex. 7.6.12 Find the total work done by a force F = 2xyi - 4zj + 5xk along the curve x = t2,
Y = 2t + 1, z = t3, from the points t = 1 to t = 2.
Sol: x = t2 , dx = 2t dt; y = 2t + 1, dy = 2dt; z = t\ dz = 3t2 dt

Total work done = fF.dr


c

= f(2xyi-4zj + 5xk}.{(dx}i + (dy}j + (dz}k} = f2xydx-4zdy+ 5xdz


c

2 2

= fl2J 2(21 + I}J2Idl _{4./ 3 )2dt + {5/2 )3/2 dl = f{8/ 4 + 4/ 3 -81 3 + 15/ 4)dl
ww '~I 1

w
{ 2
f
1
}
23/ 4 -4/ 3 11= [1
.Ea 23--/4
5
5
]2 =23- (32-1)-(16-1)= -713- 1 5 =638-
1
5 5 5

syE
Ex.7.6.13 If c is the curve y = 3x2 in the xy - plane and F = (x + 2y)i - xyj,

S'ol: n
evaluate fF.dr, from the point (0,0) to (I ,3).

gin
Since c is a curve in xy - plane, we take r = xi + yj, so that
F.dr= {(x+2y)i-xyj}. {(dx)i+(dy}j} =(x+2y)dx-xydy.
e
1st Method: By taking the curve in parametric coordinates as, er°i
2
x = t, Y = 3t , dx = dt, dy = 6t dt, so that t varies from ng.
to 1 to get the points
(0,0) & (1,3): We have
(1,3) 1 1
n et
fF.dr = fv + 6t 2) dt -/{3t 2) (6tdt) = fv + 6t 2 -181 4)dt
(0,0) '~O 0

t2 6/ 3 18t 5 1 1 18 5 18 11
= -+---1 = -+2--=---=--
2 3 5 0 2 5 2 5 10
2nd Method: >: =3x2, dy = 6x dx and x varies from °to 1.
1
.. fF.dr = f(x + 6x 2}Ix - x.3x 2.6x.dx
c 0

1
~( 2 11
= Jx+6x -18x 4 \px
,
=--
o 10

Downloaded From : www.EasyEngineering.net


Downloaded From : www.EasyEngineering.net

520 Engineering Mathematics - I

Ex. 7.S.14 Find the work done by the force F in moving a particle once around the circle
'C' in xy - plane, ifthe centre of the circle is origin and radius is '2' and
F = (x +y + z)i + (2x + y}j + (2x - y +z)k.
Sol: xy - plane is z = 0,
:. F = (x + y)i + (2x +y}j + (2x - y)k and r = xi +yj ~ dr = (dr)i + (dy}j
F. dr = (x + y)dx + (2x + y)dy.
y
The equation of the circle is x = 2cosS, Y = 2sinS

ww :. dx = - 2sinS dS, dy = 2cosS dS


S varies from 0 to 2n

w.E
Work done = fF.dr

= a 2lt

syE
J(2COSS + 2sin sX- 2sinS)de + (2.2cosS + 2sin SX2cosS)dS
o

-~
2lt
ngi 2lt
J{- 2sin2S -4sin S + Scos S + 2sin 2S}ie = J{scos S - 4sin S ~S
2 2 2 2

2lt
o
2lt
0

nee
rin
= n4(1 + cos2S)- 2(1- cos2S)}iS = J(2 + cos2S)dS = [2S + 3sin 2S~lt
o

g.n
0

= 4n
Ex.7.S.1S Show that the necessary and sufficient condition for a vector field V to be

Sol:
conservative is curl V = 0
a) Necessary condition: If V is conservative, :3 a 'cp' 3 V = V cpo
et
curl V·= curl (V cp) = 0 (see 7.5.2 (1))
b) Sufficient condition: Let V = V I i + V'li + V 3k
i j k
Curl V = V x V = 0 ~ 8/ Ox 8/ By 8/ 8z = 0

~ I(8VBy 8V8z )i = 0
3 _ 2

.... (1)

Downloaded From : www.EasyEngineering.net


Downloaded From : www.EasyEngineering.net

Vector Differentiation 521

The work done by the force field V in moving a particle from (x\,y\,z\) to (x,y,z)

is IV.dr
c

= Iv; (x,y,z)dx + V2 (x,y,z)ay + V3 (x,y,z)dz


c

where V is a path joining (x\,y\,z\) to (x,y,z)


Let us choose a particular path consisting straight line segments from (x\,y\,z\)
to (x,y\,z\) to (x,y,z\) to (x,y,z) and denote the work done along this path by a
scalar function ~(x,y,z);

ww :.
x y z
~(x,y,z) = IV;(X'Yl,ZI)dx+ IV2 (x,y,zJly+ IV3 (x,y,z)dz ..... (2)

w.E XI
From (2), it can be seen that,
Yl

8cI>
8z = Vix,y,z) asy .... (3)

En
gin
= V2 (X,y,ZI)+ V2(X,y,Z~
.z
e eri
= V2(X,y,ZI)- V2 (x,y,z)- V2(X,y,ZI)

ng. .... (4)

net
from (1)

y z
= V\(x,y\,z\) + V;(X,y,ZI~ + V;(x,y,z~
Yl Zl

= V \(x,y\,z\) + V \(x,y,z\) - V \(x,y\,z\) + V \(x,y,z) - V \(x,y\,z\)


= V \(x,y,z) .... (5)

(3),(4),(5) => :i+ : j + : k=V;(x,y,z)i+V2 (x,y,Z)j+V3 (x,y,z)k=V

=> V = V~
Hence the proof.

Downloaded From : www.EasyEngineering.net


Downloaded From : www.EasyEngineering.net

522 Engineering Mathematics - I

Ex. 7.6.16a) Show that F = yi + (2xy +z2)j +2yzk is a conservative force field.
b) Find its scalar potential.
c) Find the work done in moving an object in this field from (1,2,1) to (3, 1,4)

j k
a) V x F= Max a/fJy a/az
Sol:
i 2xy+Z2 2yz
= i(2z - 2z) + j(O - 0) + k(2y -2y) = 0
:. F is a conservative force field.

ww b) Let 'cjl' be the scalar potential of F.

w 1st method:

.Ea
acjl
syE 2 acjl_2 z
ngi
2
:. ax = y .... (1) :=2Xy +z .... (2) az - y .... (3)

cjl = xY + j(y,z), cjl = xY +yz2 + g(z,x), and nee


Integrating (1) w.r.t x, (2) w.r.t. y, & (3) w.r.t. z, respectively, we get,
cjl = yz2 + h(x,y).
These equations will be consistent iff, g, h are taken as
j(y,z) yz2, g(z,x) = 0, h(x,y) xy.
r ing
.ne
= =

Hence cjl = xy + yz2 + constant


2nd Method: Since F is conservative, JF.dr is independent of path joining
c
t
(Xl,yl,zl) to (x,y,z)
using method of problem 7.6.15(b),
x Y Z

cjl = J&12}ix + J(2Xy + z/ }ry + J2yzdz = xy~ ( +(xy 2 + Z12 y) r+yz2 f


XI YI zl Xl .Y] zl

2
= xYI - XlYI + xy +z 12y - ry I - ZI 2y I + yz2 - yz I
= xy2 + yr - x1yl2 - Zl2Yl = xy2 + yz2 + constant.

3rt/ Method: Since F. dr = Vcjl.dr = acjl dx + acjl dy + acjl dz =dcjl ,


ax fJy az

Downloaded From : www.EasyEngineering.net


Downloaded From : www.EasyEngineering.net

Vector Differentiation 523

d~ == cY)dx + (2xy + z2)dy + (2yz)dz == cYdx + 2xydy) + (z 2dy +2yz)dz


== d(xy) +d(yz2) == d(xy2 + yz2)

=> ~ == xy + yz2 +constant


(0,1)8
1'2=(3,1,4)

c) work done == f F.dr == fd~


c IHI,2,1)

(3,1,4)
'=:-:::----i----"'I\( 1,0)
~(P2) - ~(PI) = xy + xz2
ww ==

=(3.1 2 +3.4 2)-(1j2+ 1.1 2)=51-5=46.


I
(1,2,1)

w .Ea
Ex. 7.6.17 Evaluate f(x 3dy + y 2dx) where c is the boundary of the triangle whose vertices

syE
c

are (0,0), (1,0), (0, I).

Sol: Let I == f(x 3dy + y2 dx)


c n gf, inf
I == II +12 +1 3, where II == f, 12 ==
OA AH
e
13 =
HO
eri
(i) Along OA, y == 0, => dy == ° ng.
(ii)
.
•• 11 ==

AlongAB,
Jx 3 •0 + 02 •dx == 0

x+y=1 ,y=l-x=> dy=-dx,xvaries from


n et
I toO.
o
:.1 2 = fx 3 {-dx)+{I-x)2 dx
I

_~o =0_(_~_~)= __1


3
= J(x 2_x 3 +1_2X)dx=_x
o 3 4 I 3 4 12
(iii) Along BO, x =°=> dx =° :. 13 =fo.dy+ i.o =°
1
:. 1 = II + 12 + 13 = - -
12
Ex.7.6.18: If 1= xy2z2 ,evaluate fldr where the curve 'c' is given by

x = (, Y = (2, Z =(3 from t = ° to I .

Downloaded From : www.EasyEngineering.net


Downloaded From : www.EasyEngineering.net

524 Engineering Mathematics - I

Sol: f=xy2 Z2 =/(/2y .(/3 Y=1 11

dr=dx i+dyj+dz k
2 2
dr =(dt)i + (2tdl )j + (3/ dl )k =(i + 2tj + 3/ k )dl
1 1 1 1
Jfdr
c
= Jill
1=0
(i + 2tj + 3/ 2k )dl =i Jill + dl+ j J2/
0 0
12 13
dl+ k J3/ dl
0

./12 I • 2/ 1 3/ 14 I 1. 2. 3
13
=1- J+l-J+k- J=-I+-l+- k
12 0 13 0 14 0 12 13 14
ww
Ex. 7.6.18: If A = 3zi - 2xj + yk , and c is the curve given by

w.E
x = cos I, Y = sin 1 , Z = 2cos I,
JA
evaluate x dr from I =0to I = 1t .
c

i j k asy 2

Sol: A x dr = 3z - 2x y
En
dx dy dz
gin
= (-2x dz- y dY)i+(y dx-3z dZ)j +(3z dy+2x dx)k
x = cos I, Y =sin I, z = 2 cos I e eri
:. (1)
~ dx = (-sin/)dl,dy = (cos I) dl, dz = (-2sin/)dl
~ ng.
Ax dr = i[(-2cos/)(-2sin/) - sinl . cost]dt +j[(sin/)(-sint) - 3(2cost)
(-2sin/)]dl + k[3(2cos/)( cost) + (2cos/)(-sin/)]d/.
=i(3sint cost) dt + j[(12 sinl COS/) -
net
sin2/)] dl + k(6cos2/- 2sint cost) d/.
1C/23 ft/2[ 1
J J
!
Axdr=i J-sin2Idt+ j
.. c o 2 0
6sin2t--(1-cos2/) dl
2
1C12
+ k ~3(1 + cos2/)-sin 2/}it
o

.3 (_COS2/)1C12.[ 1 sin 21 Jft/2 [ 3. COS2/]ft/2


= 1- + J -3cos2/--+-- +k 3/+-sI02/+--
220 24 0 2 20

= {
3
-4 )(-1-1)+ j[-3(-1-1)- : + 0]+k[3; +O+~(-l- ])]

= %;+(6- :}+e; -1)k

Downloaded From : www.EasyEngineering.net


Downloaded From : www.EasyEngineering.net

Vector Differentiation 525

Exercise - 7(e)
4

I) IfU(t)=(2t2 -I)i+3tj+(2-t)k. Find (a) JU(t)dt,(b) Ju(t)dt


2

2/
[Ans:(a) ( ~-I
3
12
2
i+ 3t j + ( 2t-
t2
\06 l
2 k,(b) 3i+18j-2k]
11/2

ww
2) Evaluate: J(6sillu)i -(3cosu)j +uk
o

w.E 1t
[Ans: 6i-3j+-k]
8
2

a
3) If A(s) = si - s2j + (s + l)k, B(s) syE =
2 2

2si + 6sk find (a) JA.Bds, (b) JA x Bds .

ngi o

100
0

4) If A = ti - j + 2tk, B = t2 ;
nee
[Ans: (a) 3' (b) -
tj +2k, C = ; - 2j + 2k, evaluate
48i - 12j + 16k]

rin
-

1 1

(a) J{A.(BxC)}dt,(b) J{Ax(BxC)}dt


o 0 g.n
. 2 6
I
5. 37. 8 k
[Ans' (a) -- (b)
+-} +- ]
3' 3
--I
et
5) The acceleration of a particle a at any time t ~ 0 is given by a = eli + (2cos2t)j +
(2sin2t)k If the velocity V and displacement r are both zero at t = 0, find V and r at
any time t.
[Ans: V = (el - I)i + (sin2t)j + (1 - cos2t)k,

r =(i -t-I)+~(l-COS2t)j +(t -~Sin2t}]


3
6) Evaluate fA. dA du, given that, A(2) = 4i - 2j + 3k and A(3) = 2i + j + 2k
2 du
[Ans: - 10]

Downloaded From : www.EasyEngineering.net


Downloaded From : www.EasyEngineering.net

526 Engineering Mathematics - I

Exercise 7(f)

I. If ~ = xyz, valuate I~dr, where c is the curve x = t3, y = t2, z = t, from t = 0 to I


c

I. I k
[Ans: JI i +4J+ 7 1
2. If F = xi - yzj + z2k, and c is the curve given by x = t, Y = t3, z = t 2, evaluate
(i) IF.dl" and (ii) fF.dr, from t = 0 to t = I.

ww e C

w.E .)
[A ns: ( I
5. 7. 1\ k ..
-71-I5J +1"2 ,(II)
23]
24

asy
3. If A = (2x + 3)i + xyj + (zx - y)k, evaluate fF.dr along c where c is

(a) the curve x


En
= t3,y = 2t2, z = t from t = 0 to I.

gin
(b) The straight lines from (0,0,0) to (1,0,0), then to (1,0, I) and then to (1,2, I)
(c) The straight Iinejoining (0,0,0) and (I ,2, I).

e eri
491
[Ans: (a) 105' (b)
13
(c)
14
2' "3]
4. If A = (3xy - 2r)i + (x - y)j, evaluate ng.
fA.dr along the curve c in xy - plane given
by y = x 3 from the point (0,0) to (2,8).
c

net 1308
[Ans: 3.5]
5. IfF = (2x- y)i + (x- 2y)j, evaluate fA.dl" where c is the closed curve shown in the
c

figure below.
y

Downloaded From : www.EasyEngineering.net


Downloaded From : www.EasyEngineering.net

Vector Differentiation 527

6. If A = (3x + 2y)i + (x + y)j, and c is the boundary of the tringle whose vertices are

(0,0), (1,0), (0, I), evaluate fA.dr.


c

3
[Ans: 2" J
7. If A = (2x + y)i + (3x - 2y)j, compute the circulation of A about the circle C:
xl + 1=4, traversed in the positive direction.
[Ans: 81t]

ww
8. Find the work done in moving a particle in the force field.
F = 2x2i + (2yz - x)j + yk, along (a) the straight line from (0,0,0) to (3, I ,2)

w.E
(b) the space curve x = 3t2, Y = t, z = 3t2 - t from t = 0 to t = I

[Ans: (a)
113
6' (b) 3]
58

a syE
9. (a) Prove that V = (2x siny - 3)i + (x 2 cosy + z2)j + 2(yz + I)k is a conservative
force field. (b) Find the scalar potential ofY. (c) Find the work done in moving an

object in this field from (1,0,-1) to (2, ngi


2"1t ,I).
nee 1t

rin
[Ans: (b) x 2 siny + yz2 - 3x + 2z + constant, (c) 2" + 5]

10. If A = (9xly -
g.n
2xz3 )i + 3x3j - 3x2z2k, (a) prove that fA.dr is independent of the
c

curve 'c' joining two given points. (b) show that there is a differentiable function <jl
such that A = \l<jl and find it.
et
7.7 SURFACE INTEGRALS
7.7.1 Let S be a two-sided surface. Let one side be taken as the positive side. If S is a
closed surface, the outer side is considered as the positive side. Let A be a vector
function. Consider an element of area 'ds' in the surface. Let n be the unit normal
vector to ds in the positive direction. It can be seen that A.n = A cose. (where '8' is
the angle between A alld n and A = IAI) is the normal component of A. Let ds be a
vector whose magnitude is ds and whose direction is that ofn.
:. ds = n ds.

Downloaded From : www.EasyEngineering.net


Downloaded From : www.EasyEngineering.net

528 Engineering Mathematics - I

ww
w.E Then the integral,

ffA.ds = ffA.Il .ds


asy
s
..... (1)

En
is an example of a surface integral which is also called as flux of A over s.
If'f' is a scalar function,

ff4>ds gin ..... (2)

e eri
ng. ..... (3)

ffAx ds = ffA x nds

are some other examples of surface integrals.


net ..... (4)

Note (1) The surface integrals can also be defined in terms oflimits of sums. (see 7.7.2)

(2) The notation If is also used to denote a surface integral over the closed
surface s.

(3) Sometimes the notation f may also be used for surface integrals.
(4) Surface integrals can be conveniently evaluated by expressing them as double
integrals over the projected area of s on one of the coordinate planes
{see 7.7.3)

Downloaded From : www.EasyEngineering.net


Downloaded From : www.EasyEngineering.net

Vector Differentiation 529


/

7.7.2 Definition of surface integral as the limit of a sum:

.,J------...... y

ww
wlet .Ea
The area S is divided into 'L' elements of area Mill' m = 1,2, .... L,
(xm'YIII,zm) be any point in Let A(xm,Ym,zl/.)= AHI

syE
P'II = &\)111.

Let nlll be the positive unit normal to I'1S111 at Pm. Then «(Am.nm) is the normal
component of Am at Pm. Consider the sum,

I
/,
ngi
1/1=1
Am.nIllMm
nee ..... (I)

The limit of the sum (l)as L ~oo such thatthe largestdimens~on of each I'1S", ~ 0
rin
(if the limit exists) is known as the surface integral of the normal component of A

over S an d denote d by , IIA.n.d'} g.n


7.7.3 Evaluation of a surface integral
To evaluate surface integrals, it is convenient to express them as double integrals
e t
taken over the projected area of the surface S on one of the coordinate planes
(xy,yz, or zx planes).
If R is the projection of S on the xy - plane, it can be shown that

ffA.nds = ffA.n d\XdY,


, /I n.k
From 7.7.2, the surface integral is the limit ofdle sum
/,

I
111=1
Am .nmMIII ..... (1)

Downloaded From : www.EasyEngineering.net


Downloaded From : www.EasyEngineering.net

530 Engineering Mathematics - I

;---~----~~~--.Y

ww The projection of ASmon thexy- plane is InmA.S'II/.kl (or) Illm.kIMi'm which is equal

w .Ea
syE
:. The sum (I) becomes

ngi
nee
By the fundamental theorem of integral calculus the limit of this sum as L ~ 00 in

r
such a manner that the largest Llx m and AYm approach zero is

dxdy
ing
IfA.n In.kl
R which is the required result.
.ne
Note: Similarly ifR is the projection ofS on yz and zx planes respectively, it can be seen as

ff A .nds = ffA.n dl'Y~ZI


t
S R nJ

and ffA.nds = ffA.n dz~xl


S R ln.]
7.7.4 Physical interprettion of surface integrals:
Let A denote the velocity of a moving fluid. Let S be a fixed surface in the fluid. Let
ds be an element of surface. Then A.n ds = A.dS represents the amount offluid that
passes normally through dS in unit time at any point. If the direction ofn is outward
or positive, the amount offluid flow is positive. Similarly if dS' is another element for
which n is in the negative direction, the fluid flow is negative at that point.

Downloaded From : www.EasyEngineering.net


Downloaded From : www.EasyEngineering.net

Vector Differentiation 531

to fA.nds and it is known as the total flux of A through the entire surface S.
s
A can be a vector denoting physical quantities such as electric force, magnetic
A

ww n....---...---~C"

w .Ea
syE
force, flux of heat or gravitational force etc. In all these cases, fA.nds
s
denotes

total flux of A through S.


n
Sioved Examples gin
Ex.7.7.5 Evaluate
e eri
fJA.nd\" where A = {x + y2} - 2xj + 2yzk and S is the surface of the
plane 2x + y + 2z = 6 in the first octant. ng.
Sol: A = (x + y)i - 2.>.] + 2yzk
Let ~ = 2x + Y + 2z - 6 z n et
V ~ = 8q>i + aqy + 8q>k = 2i + j + 2k
ax ~ az
V~2i+j+2k
Unit normal n to S = IV~I = ~22 + 12 + 22

2i+ j + 2k
3

Downloaded From : www.EasyEngineering.net


Downloaded From : www.EasyEngineering.net

532 Engineering Mathematics - I

= .![2x+ 2y2 - 2x + 2y(6 - 2x - y)]


3
[Substituting 2z = 6 - 2x - y ( ... 2x + y + 2z = 6)]

= .!(12y-4xy)
3
If R be the projection of S on the xy - plane.

2 dxdy 3
In.kl = -3 => ds = -l/l.kl- = -dxdy
2

ww .. IfA.nds = If(A.n) dXdY


w .Ea
/I /I ln.k I

If .!(12y - 4);y )~dxdy


syE /I 3 2

n =
/I gin
If(6 y - 2Xy}dXlry ..... (I)

To evaluate this double integral over R,


(i) Keep x fixed and integrate w.r.t. y from y e =
eri
0 to (6 - 2x), ii) and then integrate
w.r.t. x from x = 0 to x = 3.
:. Given integral ng.
3 6-2x

J J(6y-2xy)1ydx n et
x=O y=O

3 3 3
TI3i _xy2 r:~x dx =
2 3
= J(3 -xX6- 2xY dx = J(J08-108x+36x -4x }Ix

= [\08x-54x2+12x3-x4]~ =324-486+324-81 =81

Ex.7.7.6 If F = 4xzi - yJ + yzk, evaluate IfF.nds where S is the surface of the cube
s

bounded by x = O~ x = I, Y = 0, Y = 1, z = 0 and z = 1.

Downloaded From : www.EasyEngineering.net


Downloaded From : www.EasyEngineering.net

Vector Differentiation 533

Sol. The surface S can be divided into 6 faces (see figu --, z
o
(i) S\: Face EPFA
(ii) S2: Face OBOC E f---+----(
(iii) S3: Face PFBO
y
(iv) S4: Face OCEA
(v) Ss: Face POCE
x
(vi) S6: Face OBFA

ww IfF.nd5 = IfF.nds + IfF.nds + IfF.nds + IfF.nds + IfF.nds + IfF.nd.,.


S ~ ~ ~ ~ ~ %

w.EJOn S \: n = i x = 1
1 1 1

JJ4zdydz
1 1

J2Z21~
1 1

IfF.nds =
SI 0
asy J(4Zi - y2 j + yzk }idydz =
0 0 0
=
0 0
= J2dY = 2
0

On S2: n = - i, x
1

JJ(-
=

EX-n J
1
0
1 1

HF.n.d.,. =
o 0gin y2 j + yzk i)dydz =
o
J(O}lydz = 0
0

On S3: n = j, y =I
e
J er
1 1

HF.n.ds=J J(4xzi-j+ zk).jdxdz =


1 1

J(-I)1xdz= J-zi
1

ing
1 1

dx= J-Idx=-l

.ne
.'13 0 0 0 0 0 0 0

OnS 4 :n=-j,y=O

:. HF.n.ds =
1

o
1

J J(4xzi).(- j)dxdz J J(O)1ydz


0
=
1

o 0
1

= 0 t
On Ss: n = k, z = 1
1 1 1 1 1 21

:. HF.n.ds=J J(4Xi-/j+yk}kdxdy=
8S 0 0
J Jydxdy= J~ I
0 0 0 0

On S6: n = - k, Z =0
1 1

HF.n.ds
.'16
=
0
J J{- y2 j }(- k )dxdy =
0
0

. JJF.n.ds =2 + 0 - 1 + 0 + ~ + 0 = ~
.. 2 2
s

Downloaded From : www.EasyEngineering.net


Downloaded From : www.EasyEngineering.net

534 Engineering Mathematics - I

Ex.7.7.7 If A = z 2i + xlj - yZk, and S is the surface ofthe cylinder xl + Y = 16 included

in the first octant between z = 0 and z = - 5, evaluate jfA.n.ds


s
Sol: z

f d~- ~~'
,. 5 R Q
r=-----'JL..-- y
'<.
n

ww
,.... 4
- ' z=Q
x

w
Project s on xz plane and let the projection be R. (See figure)

.Ea
HA.n.ds = HA.n dx,d~, ..... (I)
S

The norma I to x 2 +
R

Y = 16 is syE n.)

V(X2y2 )=~(X2 + y2) +~(X2 + y2)j


ax ngi
ay

.
= 2xi+ 2yj
2xi + 2yj 2(xi + yj) nee
xi + yj
Ul1Itnormaln= ~(2xY+(2yy = 2JX2+y2 =-4-
r (.: xl + Y
ing
16)

.ne
Z
2X+X 2 y
A.n = ------'--
4

n.J=
. y
"4 t
:. From (I) J JA.nds
S
= J V2X+X2y dxdz
R 4 y/4

[.: J x
o ~16 _x 2
dx=4,

3
= 4z + 64zf = 820
3 3 0 3

Downloaded From : www.EasyEngineering.net


Downloaded From : www.EasyEngineering.net

Vector Differentiation 535

Ex.7.7.8 Evaluate I I<I>nd~ where S is the surface of problem 7.7.7 above and <I> = x~z
s
dxdz
Sol: We have Ifs <l>nds = If<l>n-
I
II
·1
n.)

xi+ yj y
using n = - 4 - ' n.j = "4' the integral on R.H.S. becomes,

ww
w.E
R asy
-
1
80
64 Z. 64Z.
-l+-}
3 3 f En Z

I 64(z2. Z2 .]1
5
8 25(. .) gin 100(. .)
-.- -1+-)
83 2 2 0
=-.--;;-1+) =-1+.1
3 ~
e3
eri
Ex.7.7.9 If A = yi -+ (x - 2xz)j - xyk, evaluate I I{curIA).nd~·
s ng.
where S is the surface of

Sol:
the sphere x2 +.0 + z2 = 4 above the xy plane

Z n
net

I--~-+--Y

j k
CurlA= a/ax a/fJy a/az
y x-2xz -xy

Downloaded From : www.EasyEngineering.net


Downloaded From : www.EasyEngineering.net

536 Engineering Mathematics - I

= i{~(-xy)-~(x-2xz)} + j{~(y)-~(-xy)} +k{~(x-2xz)-~(y)}


~ & & fu fu ~
= xi + yj - 2zk
The normal to the surface is V (xl + .0 + z2) = 2xi + 2yj +2zk
2xi + 2yj + 2zk
Unit normal n = I
,,(2x)2 + (2y)2(2z)2

xi+ yj+zk

ww
2
x 2y2 _2z2
(Curl A). n = 2

w.EThe projection of S on xy plane is the circle xl +.0 = 4, Z = 0 (see figure)

asy
.. f.f(curl A.n}ds = f [(CUr! A.n) I:'~f

= fpX2 i E
-
ngi
+ 2Z2 ) dxdy
2 z 12

n
/I

eer
ing
2

f f
4x
2
( 2 )
3 x + y2 - 8dxd .ne
I 2 2
x=-2Y=_~4_x2 ,,4-x y
OJ

Changing into polar coordinates by taking x = r cose, y = r sine, dx dy =r dr de, the


t
integral becomes

de= f(8-8}ie=o
8=0

Downloaded From : www.EasyEngineering.net


Downloaded From : www.EasyEngineering.net

Vector Differentiation 537

Ex.7.7.10 Evaluate J JA.nds where A = yzi + zx} + xyk and S is the part of the sphere
s
x 2 + y2 + z2 = 9 which lies in the first octant.
. V x 2 + y2 + Z2
Sol: Untt normal n to S = IV x 2
+ y +z-
' ,

2xi+2y}+2zk xi+ y}+zk


(.: x2 +y2 +z2 = 9)
~4X2 + 4y2 + 4Z2 3

ww A.n = 3xyz; nk=


. -3
z

J
w .E
If R is the projection of S on xy - plane, we have,

f A .nds = f fA.n d1xdY


s /I n.k I
asy
= f f 3xyz dxdy
En
=9 f fxy dxdy
/I z/3 II

gin
The region R is bounded by x-axis, y-axis and the circle x 2 + y2 = 9; z = 0 chang,iog
to polar coordinates, the last integral becomes
",/2 3 rr/23 eer
9 J
8;0 r;O
f(rcos9Xrsin9)rdrd9=9 J[Jr 3 cos9sin9dr}19
0 0 ing
9 fo ~4)31 cos9sin9d9=9.-811[/2fcos9sined9
1[/2(
.ne
t
=
4 0 4 0

(
1[/2
': !cos9Sin9d9=2"
I)
Exercise - 7(g)

I. If F = 18zi - 12x} + 3yk, evaluate J f F.nds where S is that part of the plane
s
2x + 3y + 6z = 12 which is located in the first octant
dxdy
(Hint: Take projection ofS on the xy plane and ds = -1-1
n.k
)
[Ans: 24]

Downloaded From : www.EasyEngineering.net


Downloaded From : www.EasyEngineering.net

538 Engineering Mathematics - I

2. Evaluate f f V.nd~ , where V = yi + 2xj - zk and s is the surface of the plane


s
2x + Y = 6 in the first octant cut off by the plane z = 4
[Ans: 108]

3. If r = xi + yj + zk find the value of the integral f fr.nds over


s
a) the surface S of the unit cube bounded by the coordinate planes and the planes
x = y = z = I and

ww b) the surface of the sphere of radius 'a' with centre at the origin
[Ans: a) 3, b) 41ta3]

w.E
4. Evaluate f fF.d~ over the entire surface ~fthe region above the xy plane bounded
s
by the cone :l-
asy y
= x2 + Y and the plane z = 4, if F = 4xzi + xyzj + 3zk
[Ans: 3201t]

En
5. IfS is the surface of the parabolic cylinder = 8x in the first octant bounded by the

gin
planes y = 4 and z = 6, evaluate f fF.nds, where F = 2yi - zj + x 2k
s

[Hint: f f F .nds = f fF.n e eri


dIY~IZ , where R is the projection ofS on the yz plane]
S R nJ

ng. [Ans: 132]

6. Evaluate f fV.nds over the entire surface S of region bounded by the cylinder
S
net
x2 + :l- = 9, x = 0, y = 0, z = 0 and y = 8 if V = 6zi + (2x + y}j - xk
[Ans: I81t]

7. Evaluate f f A .nds where A = zi + xj - 3yzk and S is the surface of the cylinder


s
x2 + Y = 16 included in the first octant between z = 0 and z = 5
[Ans: 90]
8. If V = yzi + zxj + xyk and S is that part of the sphere x2 + Y +:l- = 1 which lies in
the first octant, find the value of f fV.nds .
s

3
[Ans: "8]

Downloaded From : www.EasyEngineering.net


Downloaded From : www.EasyEngineering.net

Vector Differentiation 539

9. Evaluate JJ{(x 3
- yz2)i _(2x2y)j + 2k}.ds over a cube with edges of length 'r'
s
parallel to the coordinate axes
I
[Ans: 2"]
10. If V = xi + yj + zk, and S is the 'triangle with vertices at (1,0,0), (0, 1,0) and (0,0, I),

find the value of I IV.ds .

ww
s

I
2"]
w.E [Ans:

II. If A = xi - yj + (z2 - I )k, find the value of I IA.nds, where S is the closed surface

asy s
bounded by the planes z = 0, z = I and the cylinder x 2 + y=I
En [Ans: 1t]
7.8 VOLUME INTEGRALS
gin
7.8.1
eer
Consider a closed surface in space enclosing a volume V. Then, integrals of the

form I I IAdv and I I I~dv, [A is a vector function, f is a scalar function] are


v " ing
examples of volume integrals.
7.8.2 Expression of volume integral as the limit of a sum: .ne
Let A be a continuous vector function. Let S be a surface enclosing the region D.
Divide this region D into a finite number of subregions D, ... ,= 1,2, ..... n. t
Let L1v, be the volume of the subregion D, enclusing any point whose position
vector is rio
Consider the sum
II

V = LA~JL1v,
,;1

The limit of this sum as n ~oo such that L1V, ~ 0, is called the volume integral of

A over D and denoted by I I I Adv


J)

If A = A I (x,y,z)i + Aix,y,z}j + A 3(x,y,z)k,

Downloaded From : www.EasyEngineering.net


Downloaded From : www.EasyEngineering.net

540 Engineering Mathematics - I

so that dv = dx dy dz

f f fAdv = iff fA,(x,y,z}lxdydz + j f f f A2(X,y,z}!xaydz +


D D D

k f f f A3(X,y,z}lxdydz
f)

Solved Examples

(2x 2 - 3z)i - 2xyj - 4xk, evaluate f f fV.Fdv where V


ww
Ex.7.8.3 If F =
v
IS the closed

VF =
w.E
region bounded by the planes x = 0, y = 0, z = 0 and 2x +2y + z = 4.

~(2X2 -3z)-~(2xy)-~(4x)=4x-2x=2x
Sol: . ax
asy
ry az

.. f f fV.Fdv= f f En
2 2-x4-2x-2y 2 2-x 4-2x-2y

f 2xdzdydx= f f [ f 2xdz ]dydx


v x=Oy=o z=O
gin
x=o y=o z=O
2 2-A 4-2x-2y 2 2-x

A=O y=O Z=O x=O y=O


e
2 2-x

x=O y=O
2
eri
f f 2xz l dydx = f f 2x(4-2x-2y}!ytlx= f[ f(8x-4x -4xy)t(y]dx

2 2-x 2
ng.
f[8x(2 - x)- 4x 2(2 - x)- 2x(2 - X)2}1x
= f8xy-4x2y-2xy2fdx
x=O y=O x=O net
64 8
=16--+8=-
3 3

Ex.7.8.4 Evaluate ff j(V.A}tv over the region bounded by x + Y = 4, z = 0 and z = 3,


2

v
where A = 4xi - 2yj + z2k.

a(4x ) --(2y
V.A=- a 2 a 2 )=4-4y+2z
)+-(z
Sol: ax ry az

x=2 y=J4-x 2 3
.. fff(v.A}!v= fff(4-4y+2z}lv=
v v x=-2 y=-v4-x~
f
f [J(4-4y+2z)dzf/ytlx
12 z=O

Downloaded From : www.EasyEngineering.net


Downloaded From : www.EasyEngineering.net

Vector Differentiation 541

2 )'~f4-x2
dydx==
x~-2
J[ JC 2l - 12 y)dy]dx
;--- ?
y=-Y4-r-

x r.-? x 2 n
==84[--v4-x 2 +2sin- ' C-)] ==84[O+2C-)-O]==84n
220 2

ww JJ
Ex.7.8.5 Evaluate
J'
J<I0v taken over the rectangular parallelopiped 0 s:; x < a, 0 s:; y < b,

w.E
os:; z < c and ~ == 2{x + Y + z)

Sol: fJf¥v ~ ff f2{x+ Y+ z}Jv ~ ,U[}{x + asy }YdY y + z)

En
gin
a
== J2exy+ cy2 +e 2YI
b

eer
a
dx == JC2bex+eb2 +e 2b)dx==bex2 +(b 2e+be 2)xl
a

x=O 0

== a 2 be + (J(b 2 e + be 2) == abe {a +b + e)
x=o

ing 0

Ex. 7.8.6 If ~ == 4y + 2xz, evaluate .ne


JJJ~dv over the region in the first octant bounded by
x2 +1 = 9, z = 0, z = 2.
J'
t
Sol: JJJ~dv== JvJJ{4y+2xz}lv
I'

J.=3y=J9-x 2 z=2 ~2 3
?

J J [J(2xz+4y}lz]dydx==
2
J
JC4yz+xz )f dydx
x=O y=o ==0 0 0 0

3 ~x2 3 ~2
= J[ J(8y + 4xX/y]dx = JC4y2 +4xy)1 dx
o 0 0 0

3
= J[4(9-x2)+4x~9-x2]dx==108
o

Downloaded From : www.EasyEngineering.net


Downloaded From : www.EasyEngineering.net

542 Engineering Mathematics - I

Ex.7.8.7 Evaluate f f fFdv where F = xzi - 2xj + 2y.k and V is the region bounded by
v
the surfaces x = 0, y = 0, y = 6, z =.x2, and z = 4
Sol:

z Q s
\:

ww y=
.Q...~
z=x
2
z=4 ,I
w.E \ '\
~y =6

asLy_________ J//
y

x
En
gin
The region V is covered by (see the figure) (a) keeping x andy fixed and integrating
-i <

e eri
from z = .x2 to z = 4 (base top of column PQ) (b) then by keeping x fixed and
integrating from y = 0 to y = 6 (R to S in the slab) and (c) finally integrating from x
°
= to x = 2 (where z =.x2 meets z = 4).
ng.
:. The required integral is

2 6 4
net
f f[ f(xzi-2xj+2y 2k)dz}1ydx
x=Oy=O z=x2

264 264 264


=i f f[ fxzdz]dydx+j f f[ f- 2xdz ]dydx+k f f[ f2y dz}1ydx
2

x=O y=O z=x2 x=O y=O z=x2 x=O y=O z=x 2

26 24 26 4 26 4
= i f f x; J dydx+ j f f- 2xz J dydx+k f f2 y 2 z J dydx
x=O y=O z=x2 x=O y=O z=x2 x=O y=O z=x2

Downloaded From : www.EasyEngineering.net


Downloaded From : www.EasyEngineering.net

Vector Differentiation 543

26 5 26 26

=i J[ J(8X-~)dy}lx+j J[ J(2x 3
-8x)dy]dx+k J[ J(8i- 2x2 y)dy]dx
x=() y=O x=o y=O ~=o y~o

X5 2 8 y3 2X2 3
= i
2
J8xy-~
2
6
I dx+ j J2x'y-8xyl
)'~()
6

)'~O
dx+k J-'--_Y-
3 3
6
I
y=O
£Ix
x=o x=()

2 2 2

J( 48x - 3x
ww
5
=i )dx + j J(l2x 3 - 48x)dx + k J(576 - 144x2 )dx
o 0 0

w.E
1
= i(24x 2 -:;- x 6 )
2
I
2
+ j(3X4 - 24x2) I + k(576x - 48x 3 ) I
2

4~i
~

+ 768k
asy
x=o ~=O x=o

En
= 64i -
Ex.7.8.8 Find the volume of the region common to the intersecting cylinders x 2 + r =b2

and x2 +z2 = b2 .
gin
Sol:
z 2 e
x +z ::: b
2 2
eri
/ I
/
/'" ng.
net
",

I I-
II
V - 2 2
_''x +y ::: b
2

V
~o y

Required volume is equal to 8 times the volume of the region shown in the figure
(as the axes cut the volume into 8 equal parts one in each octant)

Downloaded From : www.EasyEngineering.net


Downloaded From : www.EasyEngineering.net

544 Engineering Mathematics - I

Exercise - 7(h)

ww 1. Evaluate Iff( 2x + y) dv where V is the closed region bounded by the cylinder

w.E z = 4 - x 2 and the planes x = 0, y = 2 and z = 0 . I ADS: - I


80
3
2.
planes asy
If A = (2X2 - 3z)i - 2xyj - 4xk , and V is the closed region bounded by the

En Iff(
x=O,y=O,z=Oand 2x + 2y + z = 4,findthevalueof curlA )dv

gin [ADS: 1u -k) )


Ifff dv eer f = 45x 2 y and v is the closed region bounded by the
ing
3. Evaluate where

planes
4x+2y+z = 8,x = O,y = 0 apd z = 0 [ADS: 128)
.ne
4. If A = 2xzi - xj + y2 k and v is the region bounded by the surfaces
x = O,y = O,y= 6, t
z= x 2
and z = 4, find the value of Iff Adv ADS: 128i-24j+384k

5. Evaluate ffI(Div A)dv taken over the rectangular parallelepiped,

o~ x ~ 1, 0~y ~ 2,
o~ z ~ 3 , where A = (X2 - yz) i + (i - zx) j + ( Z2 - xY) k [ADS:3 6 1

6. Evaluate Iff(Div F)dv for the volume of a cube with edges of length unity
parallel to the coordinate axes where F = (x 3 - yz2 ) i - ( 2X2Y ) j + 2k
[ADS: 1/3)

Downloaded From : www.EasyEngineering.net


Downloaded From : www.EasyEngineering.net

Vector Differentiation 545

7.9 CURVILINEAR COORDINATES


7.9.0 The students are already familiar with the coordinate systems such as i) cartesian
ii) Polar and iii) parametric.
A study of yet another coordinate system called "curvilinear coordinate system"
will be now taken up.
7.9.1 Transformation of coordinates
Suppose the rectangular coordinates (x,y,z) of any point be expressed as functions
of variables vI' v2' v3 as

ww ...... ( I)

w.E
Let the above system of equations (i) be solved for vi' v2' v3 and another system of
equations

asy
Ifthe functions in (I) and (2) are assumed to be single valued and to have continuous
..... (2)

En
derivatives, the correspondence between the variablesx,y, z and vI' v 2' v3 is unique.

gin
Thus, to a given point P with cartesian coordinates (x,y,z), we can associate from
(2) a unIque set of coordinates (vl'v 2,v3 ) which are called curvilinear coordinates.

e eri
The systems of equations (I) and (2) give the "transformation of coordinates".
7.9.2 Orthogonal Curvilinear coordinates
z V3 curve ng.
net
........ ;
>-----------~-------------y
"

x
The surfaces vI = ci' v2 = c 2 ' v3 = c 3. (cl'c2, c 3 beIng constants) are called
'coordinate surfaces'. (see figure).
Each pair of the above surfaces intersect in curves called 'coordinate curves or
lines'. i.e., vI = c I and v 2 = c2 intersect in v3 curve, v 2 = c2' v3 = c 3' vI = c I in v 2
curve.

Downloaded From : www.EasyEngineering.net


Downloaded From : www.EasyEngineering.net

546 Engineering Mathematics - I

If the coordinate surfaces taken in pairs intersect at right angles the coordinate
system is called 'an orthogonal curvilinear coordinate system'.
Note: The coordinate surfaces described above are similiarto the coordinate planes of the
rectangular system and the coordinate curves are similar to the coordinate axes.
Infact, the rectangular coordinate system is also an example of curvilinear system
with VI = x, v2 = y, v3 = z; c I = c2 = c 3 = 0
7.9.3 Unit vectors of a curvilinear system
Consider the position vector r = xi + yj + zk, of a point P. using (I), it can be written
as r = r(v l ,v2,v3)

ww
(i) A tangent vector to the VI curve at P(for which v2' v3 are constants) is ~. If e l is
av,
w.E
.
the Ul1lt tangent vector we have e I = I I'
ar/av
ar

a syE av,

ngi
lilly if nee
ar , ar are tangent vectors to the v2 and v3 curves at P respectively and e2,
av aV3
2

e3 are unit vectors in these directions, rin


ar
we can write , -av = A2 e2 , -
ar
av3 = A3e3 g.n
where
2

et
The unit vectors e I' e2 , e3 are in the directions of increasing v\' v2' v3 respectively.
The quantities A\, A2 , A3 are called 'scale factors'.
(ii) 'Vv, ' is a vector at P normal to the surface VI = c 1. If EI is a unit vector in this
direction,

VV 2 Vv
lilly the unit vectors, E2 = IVv 1' E3 = IVv: I at P are respectively normal the surfaces
2

v2 = c 2' and v3 = cy

Downloaded From : www.EasyEngineering.net


Downloaded From : www.EasyEngineering.net

Vector Differentiation 547

V3 curve

ww
w.E
Thus, in general, at a given point P there exist two sets of unit vectors,
(i) el' e2, e 3 tangential to the coordinate curves and

a syE
(ii) E], E2, E3 are normal to the coordinate surfaces (see figure)
7.9.4 Representations of a vector

ngi
(a) A vector A can be represented in terms of the unit base vectors e], e2 , e3 or
EI' E2, E3 in the following forms:
(i) A = ale] + a 2e2 +a3e3 nee
(or) (ii) A = A]E] + A2E2 + A3E3' rin
g.n
Here a\, a2 , a3 or AI' A 2, A3 are the components of A in the two systems
. 8r 8r
av, av2 av3
8r
respectively (b) The vectors - , - , - and Vv" Vv J , Vv3 , are called
- et
'Unitary base vectors (need not be unit vectors in general). We can also represent
A in one of the following forms; as

Here
CI' c2' c3 are called the 'contravariant components of A and CI' C 2, C 3 are
called the 'Covariant components of A

Downloaded From : www.EasyEngineering.net


Downloaded From : www.EasyEngineering.net

548 Engineering Mathematics - I

7.9.5 Arc length, area (surface) and volume elements


(i) Let r = r(vl'v2,v3)

:. dr= ~dvJ + ar dV 2 + ar dV3 =AJ(dvJe J+A 2 (dv 2 )e2 +A 3(dv 3)e3 ..... (3)
8v J aV2 aV3
Since ds 2 = dr. dr, the differential ds of arc length is given by,

ds 2 = AI2dvl2 +A}dv} +A/dv/ for an orthogonal coordinate system


(.,' e l .e2 = e2·e3 = e3.e l = 0)

ww (ii) Let dvl' dv 2, be vectors specifying arc-elements along VI and v2 curves


respectively at point P.

w.E
i.e., dV I = dr along VI curve and dV 2 = dr along v2 curve.

a
Since v2 ' v3 are constants along VI curve and vI' v3 are constants along v2

syE
curve, we have, from (3),
..... (4)

ngi
The area of the parallelogram formed with the vectors of( 4) as adjacent sides is

nee
the area element or surface element 'ds 3 ' on the surface v3 = c3 (see fig) and it
is given by

ds 3 = Idv i x dv21 = IAIA2dvldv2e31 = AIA2dv l dv2


rin V2 curve

Oily ds I = A2A3dv2dv3

= A3Aldv3dvi g.n
dS 2

are surface elements on surfaces


VI = c i and v2 = c2 respectively.
et
(iii) The volume element 'dv' for an orthogonal curvilinear coordinate system is
equal to the volume of the parallelopiped whose coterminous edges are dv I' dv 2 ,
dv 3

i.e., Aldvle l, ~dv2e2' A3dv3e3


(see figure)

:. dv = I(Aldv1e(). (A2dv 2e2) x (A3dv3e3~


= AIA2A3 dv(dv2dv3 le(.e 2 x e31
= A(A2A3 dv 1dv2dv3 (.,' le 1.e2 x e31'; 1)

Downloaded From : www.EasyEngineering.net


Downloaded From : www.EasyEngineering.net

Vector Differentiation 549

7.9.6 EXPRESSIONS FOR GRADIENT, DIVERGENCE AND CURL IN AN


ORTHOGONAL CURVILINEAR COORDINATE SYSTEM:
7.9.6(1) Gradient of a scalar function ~:

Prove that V'~ (i.e. grad~) = [ Ia~c7v


~
I~ I~
el +~ c7v e2 +~ c7v e3
1
I I 2 2 3 3

Proof: Let V'~ = gle l + g2e2 +g3e3' where gl' g2' g3 are to be determined

ar
dr= -dvI +-dv2 +-dv3
ar ar
ww =
c7v1 c7v 2 c7v3
(A1el)dv 1 + (A 2e2)dv2 + (A 3e3)dv3

w d~ =
.Ea
V'~.dr,

d~ = A1g1dv 1 + A2g 2dv2 + A3~3dv3 ..... (A)

Again, d~ = ~ dVI + ~
av l
savyE av 2
dv 2 + ~
3
d,V3 ..... (8)

n gin
e eri
ng.
n e
Note: The above expression indicates that V'
el
= [ ~a;
~ e2 ~
+ ~ c7v + ~ c7v
e3 ~ 1 t
I I 2 2 3 3

7.9.6(2) Ifvl' v2' v3 are orthogon~1 coordinates, prove that


(a) lV'vII = A,I,(I= 1,2,3)
(b) e l = E1(I= 1,2,3)
We know that, (from 7.9.6 (I))

V~=~ a~ +!2~ +~ a~
Al c7v1 A2 c7v2 A3 c7v3

Downloaded From : www.EasyEngineering.net


Downloaded From : www.EasyEngineering.net

550 Engineering Mathematics - I

Hence

lilly I V I= -:;:
n
v 2
e
= ~ IVV3 I= -:;:
2

2
-] e
1\,2, = ~-]
3

3
1\,3 , which proves (a)

(by definition, see 7.9.3(ii))

ww
w y,
.Ea
/= 1,2,3. [from (a)]

sayE
7.9.S. (3) Prove that, in orthogonal curvilinear coordinates,
1
V.(A,eJ=
A]A2A3 Ov]
ngi
(A,AzA3) and hence show that,

nee
rin
g.n
VV 2 xVV3 = -1- (e2 xe3 ) = -
A2 A3
e, -
AzA3 e t
..... (C)

e] = A2A3 (VV2 X Vv 3)
lilly e2 = A3A, (VV3 x Vv,) and e3 = A,A 2(Vv, X r
VV 2
V.(A]e,)= VAA 2 A3 (Vv 2 x Vv 3)
= V(A,A 2A3}(Vv2 x Vv 3)+ A,A zA3 {V.(Vvz x Vv 3 )}

= V(A,A2A3).~
A2 A3
+ A'A2A3{V.~}
A2 A3
..... (D)

(Using (C))

Now,

Downloaded From : www.EasyEngineering.net


Downloaded From : www.EasyEngineering.net

Vector Differentiation 551

(Writing the expression for V )

= _I ~(_I_)+O+O
AI Ovl A2 A3

=0

ww =
1 a (AIA2 A3)
w.E
..... (E)
AIA2A3 Ovl
(Operating V)

lilly, we get, asy a


V.A 2 e2 ==
1
(A2A3AI) ..... (F)

En AIA2 A3 Ov2

and
gin ..... (G)

Hence, if A == AIel + A2e2 + A3e3, e eri


Div A = V.(Ale l + A2e2 + A3e3)
V.(Ale l )+ V.(A 2e2 )+ V.(AJ e3)
ng.
=

net
7.9.6 (4) Prove that in orthogonal curvilinear coordinates,

Vx(AleJ==~~(AIAJ-~~(AIAI) and hence,


A3 AI Ov3 AIA2 Ov2

Downloaded From : www.EasyEngineering.net


Downloaded From : www.EasyEngineering.net

552 Engineering Mathematics - I

e
.: VVI = -.L, we have,.
AI
V x (Ale l )= V x (A,A,Vv,)
=V(A,AI)xVv, +(A,A,)(VxVv,)

(from 7.5.2 (i) curl grad = vI = 0)

[!l~(AIAI)+ e ~(AIAJ+!2.~(AIAJlx!l
ww =
AI oV I
z
A Ov 1..,3 oV zAI 2 3

w .Ea
(substituting for V)

syE ..... (i)


lilly, it can be shown that
V x (A 2 e2)= ~~(A2AJ--e'-~(A2AJ
ngi
1..,11..,2 Ov l AzA3 OV3
nee ..... (ii)

and
rin
Vx (A3e3 ) = - el - -0- (A3A3 ) - -ez- -0- (A3A3 )
1..,21..,3 Ov z 1..,31..,1 Ov l
g.n ..... (iii)

:. curl A = V x A = V x (Aiel + A2 e2 + A3e3)


= Vx (Ale,)+ V(A 2e2)+ V(A 3 e3)
e t
= _e_1
{~(A3A3)-~(A2AJ1+~{~(A'AJ-~(A3A3)1
A2 A3 oV2 Ov3 f 1..,31..,1 Ov3 Ovl f
(Adding R.H.S. of (i), (ii) and (iii))

Ale, A2e2 A3e3


--olov, olOv2 olOv3
1..,11..,21..,3 "I
A,/\', A2A2 A3 A3

Downloaded From : www.EasyEngineering.net


Downloaded From : www.EasyEngineering.net

Vector Differentiation 553

Ex. 7.9.6. (5) Derive an expression for V2~ in orthogonal curvilinear coordinates.

Sol: (from 7.9.6(1»

A=_I~
I AI Ov I '

ww
.. V2~ = V.V~ = V.A

w .Ea
syE
ngi
Note: In the expressions for V ,
nee
grad~, Div A, curl A, and V2~ derived in 7.9.6(1) to

7.9.6(5), if we put vI = X, v 2 = y, v3 = z; AI =A2 = A3 = I, and e l = i, e2 = j, e3 = k;

rin
we obtain the corresponding expressions in (rectangular) Cartesian coordinate
system. [The reader is advised to verify]
7.9.7 Spherical Polar coordinates (Spherical Coordinates) g.n
7.9.7(1}
"spherical polar coordinate system".
z
e
If we take vI = r, v2 = e, v3 =~, the curvilinear coordinate system becomes a
t

Downloaded From : www.EasyEngineering.net


Downloaded From : www.EasyEngineering.net

554 Engineering Mathematics - I

The equations of transformation are,

x = r sin ecos~,}
y = rsinesin~
..... (i)
z =rcose

where r ~ 0, O:s ~ < 2n, and O:s e:s n .

7.9.7(2) Coordinate surfaces and coordinate curves.


(i) The coordinate surfaces (level surfaces) in a spherical coordinate system are

ww r = c] which are spheres with centre at.origin [or origin itself if c] = 0]


e
w.E
= c2 which are cones with verex at origin
(lines if c2 = 0 or nand xy-plane if c2 = n12)
~
asy
= c3 which are planes through the z-axis
(ii)
En
The coordinates curves are:
(a) Intersection ofe = c2 and ~ = c3 is the r--curve and it is a line
(b) Intersection of r = c] and ~ gin
= c3 is the e--curve. It is semi circle if c] :j:. 0

7.9.7(3)
(c) Intersection of r ",,-c] and e
Scale factors:
=
e
c2 is the
eri
~urve. It is a circle (or point).

r = xi + yj +zk ng.
= (rsinecos~)i + (rsinesin~)j + (rcose)k

~ = Or = (sinecos~)i + (sinesin~)j + (cose)k


net
Ovl Or

A =
] 8r
18rl=~(sinecos~)2+(sinesin~?+(cose?
=1

8r = 8r =(rcosecos~)i+(rcosesin~)j-(rsine)k
Ov2 as

=r

Downloaded From : www.EasyEngineering.net


Downloaded From : www.EasyEngineering.net

Vector Differentiation 555

- or = -or = (-rsm. 0 Sill",


. .t.).1+ (rSIl1
. 0 cos",.t.) }.
Ov 3 0<1>

A =
3
lorl=~(-"sinOsin<l»2
0<1>
+ (rsinOcos<l»2

= r sinO
: .. The scale factors of spherical coordinate system are
A( = I, A2 = r, A3 = r sinO.

ww
7.9.7(4) Base vectors: The base vectors e (' e 2, e 3 are taken for the sake of convenience

w as er , eo' etjl respectively. They are,

e
r .Ea
= e
(
= _I ~=_I or = (sin Ocos<l»i + (sin Osin<l»j + (cosO)k
AI Ov l AI or
eo = e2 =_1
syE
or = ~[(rcosOcos<l»i + (rcosOsin<l»j -(rsinO)k]
AI GO r
ngi
eel> =e3 =_1 or
= (cosOcos<l»i+(cosOsin<j»j-(sinO)k
nee
=-~-(-rsinOsin<l»i+(rsinOcos<l»j
and
A3 0<1> rsmO
rin
= (- sin <I»i + (cos <I> )j
g.n
e
From the above expressions, it can be observed that

and
e,..e s = es.eq. = er.eq. = 0

er xes =eq.; efJ xeq. =e,.; eq. xe r =eo;


t
[students are advised to verify the above equations]
which show that "the spherical polar coordinate system is a right-handed orthogonal
coordinate system".
7.9.7(5) Arc length, Area element, and volume element in spherical
coordinates:
A( = I, A2 = r, A3 = r sinO
~ 2d 2
.) dS 2 = 11.1 ~2d 2 ~ 2d 2 (from 7.9.5(1))
I VI + 11.2 V2 + 11.3 V3

= 12(dr)2 +r2(dO)2 + (rsinO)2(d<l>f


gives the arc length 'ds'.

Downloaded From : www.EasyEngineering.net


Downloaded From : www.EasyEngineering.net

556 Engineering Mathematics - I

and

ds r , ds s ' dStjl are the area elements on the respective surfaces


r = cl' e = c2' and cj> = c3.
iii) From 7.9.5(3), dv = 1..)1..21..3 dv)dv2dv3

ww = I.r.{rsin e)dr.d9.dcj>

w.E
i.e., dv = r2 sin edr.d9.dcj> is the volume element.

7.9.8 Cylindrical polar coordinates (cylindrical coordinates)


7.9.8(1)
a
In this case, we take,
syE
ngi
Then the curvilinear coordinate system becomes a 'cylindrical coordinate system'.

nee
rin
g.n
et
The equations of trans,formation are

x=pcose, y=psine, z=z.

where

7.9.8(2) Scale factors:


r = xi + yj +zk
= (pcose)i + (psin e)j + zk

Downloaded From : www.EasyEngineering.net


Downloaded From : www.EasyEngineering.net

Vector Differentiation 557

-8r = -8r = (cos 8) I.+ (.


sm 8)·}
av l 8p

8r I= -J cos 8 2 + sm
AI = 8p •
82 = I
I
-8r= -8r= (-psm8
. ) i+ (pcos8 ) j
8v2 as

ww=1:1=~(-psin8)2
A2 + (pcos8)2 =p

w.E 8r = 8r = k;
8v) 8z
A1 = I8r I= 1
8z

asy
:. scale factors are I, p, 1.

E
7.9.8(3) Coordinate surfaces and coordinate curves:
ngi
(i) The coordinate surfaces (level surfaces) in a cylindrical coordinate system are.

nee
p = cl' which are cylinders coaxal with z-axis (or z-axis itself if c 1 = 0)
8 = c2' which are planes through z-axis
rin
and z = c3' which are planes ~r to z-axis
g.n
(ii) Coordinate curves are:
(a) Intersection of 8 = c2 and z = c3 is the p-curve which is a straight line. e t
(b) Intersection of p = c 1 and z = c3 is the 8-curve which is a circle (or Point)
(c) Intersection of p = c 1 and 8 = c2 is the z-curve which is a straight line.
7.9.8(4) Base vectors: (ep,eo,e z )

ep = _I 8r = (cos 8 )i + (sin 8)j


AI 8p

eo =_1 8r =~[(-psine)i+(pcose)j]
1<.2 as p

= (-sin8)i+(cose))

Downloaded From : www.EasyEngineering.net


Downloaded From : www.EasyEngineering.net

558 Engineering Mathematics - I

1 8r
e =--=k·
~ A3 8z '

which shows that cylindrical coordinate system is a righthanded orthogonal coordinate


system.

ww
7.9.8(5) Arc length,Area element and volume element in cylindrical coordinate system:

(i)

w.E = 12 (dpy + p2(de)2 + 12 (dzY

= asy
(dpy + p2(d8)2 + (dZ)2

En
gives arc length ods'.

gin
eer
ing
ds p' ds s' ds z represent the area elements on the surfaces p .ne
= c l ' 8 = c2 ' and z = c 3
respectively.
(iii) Volume element 'dv': t
= l.p.dp.dedz = pdp.de.dz.

7.9.9: EXPRESSIONS FOR GRADIENT, DIVERGENCE, CURL, LAPLACIAN


AND JACOBIAN IN SPHERICAL AND CYLINDRICAL COORDINATES:
7.9.9(1) Gradient of a scalar function 'f

18/ 18/
Grad f= V j=---e l + - - e2 +---e3
18/
(from 7.9.6(1))
AI av l A2 av 2 A3 av
3

Downloaded From : www.EasyEngineering.net


Downloaded From : www.EasyEngineering.net

Vector Differentiation 559

a) spherical coordinates:
Al = 1, A2 = r, A3 = rsin 0

el =C r =(sin8cos~)i+(sin8sin~)j+(cos8)k

e2 = Co = (cos8cos~)i + (cos8sin ~)j ~(sin 8)k

e, =e<l> =(~sin~)i+(cos~)j

v f= of e +! Wc +_I_ofe
or r as r 0 rsine as <I>

ww b) cylindrical coordinates:
Al = 1, A2 = p, A) = 1

w .Ea
el =ep =(cose)i+(sine)j

e2 =eo =(~sine)i+(cose)j

syE
ngi
7.9.9(2) Divergence of a vector function
nee
If A = AIel + A2e2 + A3 e3, the expression for Div A is given by r ing
Div A= 1 [~(AIA2A3)+~(A2A3AI)+~(A3AIA2)l(from 7.9.6(3» .ne
AIA2 A3 8v1
(a) In spherical coordinates
8v 8v2 3

t
:. Div A = 2 1.
r sll1e or
[~~2 sin eA )+ ~(rsineA2)+
I
as
~(rA3)l
o~ J
(b) In cylindrical coordinates
Al =1, A2 =p,A 3 =1, VI =p,v 2 =e, V3 =Z

:. Div A = - 1[0-(pAI)+-(A2)+-(pAJ
p op
0 0 J
as oz

Downloaded From : www.EasyEngineering.net


Downloaded From : www.EasyEngineering.net

560 Engineering Mathematics - I

7.9.9(3) Laplacian (V 2 ): Let f be a scalar function. Then the expression for V2 j m


curvilinear coordinates is given by

[from 7.9.6(5)]
a) In spherical coordinates:
Ai =1, 1.,2 =r, 1.,3 =rsiIl9, =r, v2 =9, v)<l>
ww
Vi

1 [ -a ( r2sm9-
a ) +-
a (.sm9- j
a ( -1- aj
OJ ) +- - )]
w V2j =

.Ea
r2 sin 9 ar
.
ar 09 09 a<l> sin 9 O<J>

a2j aj 1 a2j a2f


= -- +- - +- - -
ar2 syE
2
2 + - - - + ---::---:----
r ar r2 ae 2
cot 9 aj
r2 ae r2
1
sin 9 a<j>2

b) In cylindrical coordinates: ngi


(after ditferntiation and simplification)

Al = I, 1.,2 = p, 1.,3 = 1, VI = p, V2
nee
= 9, V3 =Z

V2j ![~(p Of)+~(! Of)+~(p Of)] rin


..
=
pap op OOpOO OZ GZ
g.n
a2f 1 af
= - - + - - + - - -2+ - -
ap2 p ap 2
1 a2j
p2 09
a2f
az
e t
(after differentiation and simplification)

7.9.9(4) Curl of a vector function A.


If A = Aiel + A2 e2 + A3 e3 '

(from 7.9.6(4»

a) In spherical coordinates:
Ai =1, 1.,2 =r, A) =r~';n9, VI =r, v 2 =9, v) =<j>

Downloaded From : www.EasyEngineering.net


Downloaded From : www.EasyEngineering.net

Vector Differentiation 561

er rea {rsin8}eq,
1
ajar ajae aj~
.. curl A = r2 sin 8
AI rA2 (r sin 8 )A3

b) In cylindrical coordinates:
AI =1, A2 = p, A3 = 1, VI =P, v2 = 8, V3 = Z

ww
w.E
7.9.9(5) Jacobian:

asy
(i) Orthogonal curvilinear coordinates:
Let x = x{V"V 2 ,V3); Y = Y{V"V 2 ,V3); Z = z{V"V 2 ,V3) represent an orthogonal

En
curvilinear coordinate system

J( gin
axjavl ayjav l azjavl
Jacobian = x,y,z ) =
v" v2 ' V3 e
axjav2 ayjav2 azjav2
axjav3 ayjav3 azjav3
eri
since r = xi +yj + zk
ng.
net

[Scalar Triple Product]

Downloaded From : www.EasyEngineering.net


Downloaded From : www.EasyEngineering.net

562 Engineering Mathematics - I

(ii) Spherical coordinates:


x=rsinOcos<\>
y=rsinOsin<\>
z = rcosO, are the equations of transformation of spherical coordinate system.

. =.,(x,y,z)
Jacobian --
r,O,<\>

ww ax/ar ay/ar az/iJr


ax/a8 ay/ao az/80
w.E ax/a<\> ay/a<\> az/a<\>

asy sin Ocos<\> sinOsin<\> cosO

EnrcosOcos<\> rcosOsin <\> - rsin 0

gin
-rsinOsin<\> rsinOcos<\> 0

= r2 sinO r2 sinO]
(iii) Cylindrical coordinates: eer
(or) [J = AIA2A3 = I.r.r sinO =

x = pcosO,y = psin 0, z = z, are the equations of transformation.


ing
ax/ap ay/ap az/ap
.ne
Jacobian = J[x,y,z) = ax/88 ay/88 az/88
p,O,z. ax/az ay/az az/az t
eosO sinO 0
-psinO peosO 0 = p
o 0

[(or) J = AIA2A3 = Lp.l = p]

The summary of important results of this chapter is given in a table in the next
page for the convenience of the student.

Downloaded From : www.EasyEngineering.net


Downloaded From : www.EasyEngineering.net

Cartesian Spherical Cylindrical


Coordinates Coordinates Coordinates
(x, y, z) (r, 0, ¢) (p ,0, z)

1)
ww
Equations of
transformation
of coordinates
x=x
y=y
z=z
x =r sinO coS¢
y =r sinO sin¢
z = r cosO
x =pcosO
y =psinO
z=z

2)
w.E
Scale factors AI = 1, A2= I,A. 3= 1

3) Base vectors i, j, k

asy e,= (sinO coS¢)i + (sinO sin¢)j + (cosO)k ep = (cosO)i + (sinO)j


ee= (cosO coS¢)i + (cosO sin¢)j-(sinO)k ee= HinO)i + (cosO)j
e~= (-sin¢)i + (coS¢)j ez=k

4) Jacobian (J)
8(x, y, z)
a(x,y, z)
E ngi
8(x, y, z) _ 2 • Ll
a(r. 0, ¢) - r smu

5) (Arc Length)2 (ds)2 =(dx)2 + (dy)2 + (dz)2 (dS)2 nee


=(dr)2 + r2(d~)2 + r1 sin 2O(d¢)2
6) Area elements ds, =dy dz ds , = r2 sinO dO d¢ rindsp =pdO d-

g
~

on the coordinate ds 2 =dz dx ds e= r si nO d¢ dr ds Q=d'"-, dp


surfaces ds 3 =dx dy dSIj>= r dr dO =
ds z pdpdO
.ne
7) Volume
element (dv,\
dv =dx dy dz dv = r1 sinO dr dO d¢ dv = pdpdOdz
t
Downloaded From : www.EasyEngineering.net
Downloaded From : www.EasyEngineering.net

Cartesian Spherical Cylindrical

ww Coordinates
(x, y, z)
Coordinates
(r, 0, ¢)
Coordinates
(p ,0, z)

8) Grauf
w .Ea
a~ a~
syE
go ng
~ O[ ~
()A I
9) Div A -+-+- Div A = (r2sinO AI)
Ox ay clZ ....2
rsm (Jr

~~ (r A3)]
i + (r sinO A2) +

nee
I 2:J. rin
10) Laplacian

(v2j)
!!i.!J:i.iEJ.
or + iJy2 + iJ z 2
~ 2!Ji.

+
cotO!Ji.
tJr2 -r ar +r2- ()02 + -
r2

!Ei
- ao
g.n
f2:[ + I .!Ji. + I Sf
Up2 Ii up 7Y a(J2
+iEJ.
at
m
::J
co
::J
(l)

et -
(l)
2
r2sin O U¢2 ~.
::J
co
s::
Q)
=r
j k er reo (r sinO)e¢ eI' peo e: (l)
I 3Q)
11) CuriA
a/{)x OliJy alaz r2sinO
alar
AI
alfJo
rA2
()/af/J
(r sin{})A3
-p cl/(Jp
AI
(J/r)O
pA2
i.JI (Jz
A3
-
oen·
AI A2 A3

Downloaded From : www.EasyEngineering.net


Downloaded From : www.EasyEngineering.net

Vector Differentiation 565

Solved examples
Ex. 7.9.10 Obtain the equations of transformation from cylindrical to cartesian coordinates.
Sol: Equations of transformation from cartesian to cylindrical coordinates are
x=pcos8 ..... (1), y=psin8 ..... (2) z=z ..... (3)
from (I) and (2), we get, p2 = x 2 + y2

~p=~X2 + y2 ,p> 0

ww y
from (I) and (2), we also get - = tan 8
x

w .Ea ~8=tan-l(~)

sp=~X2+y2,
:. The required equations are

yE 8=tan-'(~} z=z

Note: ngi
If a point lies on z-axis, x = 0, y = 0 => 8 is indeterminate. These points on z-axis

nee
are known as singular points of the transformation.
Ex. 7.9.11 Represent the vector A = 2xi - yj +z2k in cylindrical polar coordinates.
ri
Sol: The base vectors in cylindrical coordinates are
ep =(cos8)i+(sin8)j ng. ...... (\)
eo = (-sin8)i+(cos8)j

ez = k
n et
..... (2)
..... (3)
Solving (\) and (2),. we get
i=(cos8)ep -(sin8h

j = (sin8)ep + (cosS)eo
Then A = 2[(cosS)ep -(sin8)eo~-[(sinS)ep +(cos8~o}y+z2e~.
A = (2xcosS - ysinS)ep -[2xsin8+ ycosSh + z2e~

= (2pcos 2S - psin 28 ~p - [2psin8cos8 + psin8cos8to + Z2 ez


(substitutingx= pcos8, y= psin8)

= p(2cos2S-sin28~p -(3psinScos8)eo +z 2e:

Downloaded From : www.EasyEngineering.net


Downloaded From : www.EasyEngineering.net

566 Engineering Mathematics - I

Aliter: Since cylindrical coordinates form an orthogonal coordinate system, we can write A
as A = a,ep + a2ee + a3ez .•..• (I) whereat'~, a 3 are to be determined.
From (1) we find,
A.ep =a" A.eo =a2 ; A.ez =a3 ..... (2)
For cylindrical coordinates, we have,
x=pcos8, y=psin8, Z=Z
given A becomes,
A= (2pcos8)i-(psin8)j+z 2 k ..... (3)

ww ep = (cos8)i+(sin8)j }
Also, eo = (-sin8)i+ (cos8)j ..... (4)

w.E
ez =k
using (3) and (4) in (2), we get

a
a, = 2pcos 2 8 - psin 2 8;
syE
a 2 =-2psin8cos8-psin8cos8=-3psin8cos8; a 3 =Z2

A = a,ep + a2 ee + a3 ez
ngi
= (2 cos 28 - sin 2 8) pep - (3p sin8 cos8) eo + z2 ez

Ex. 7.9.12 nee


Represent the vector A = xyi - zj + xzk in the spherical coordinate system.
Sol: Since the base vectors er, eo'
A a,er + a2 eO + a3eq,
ecj>
rin
are mutually orthogonal, we can write
..... (J)
g.n
=
where aI' a 2, a 3 are to be found.
From (I) we get, A.er =a,. A.ee =a2 • A.eq, =a3
For spherical coordinates, we have,
x = r sin8 cos<j>, y = r sin8 sin<j>, z = r cos8
et
••••• (2)

..... (3)
A = {r 2 sin 2 8sin <j> cos <j»i - {rcos8)j + {r 2 sin 8cos8cos<j»k ..... (4)
Also we have,
er = {sin 8cos<j»i + {sin 8sin <j»j + {cos 8 )k
ee ={cos8cos<j»i + (cos 8cos 4> )j - {sin 8)k
eq, = (-sin4»i+ {cosiJj ..... (5)
Using (4) and (5) in (2), we get,
a, = 1'2 sin 3 8sin 4>cos 2 4> - rsin 8cos8sin<j> + 1'2 sin 8cos 2 8cos4> .
a2 = r2 sin 2 8cos8sin <j>cos 2 <j> - rcos 2 8sin cp - r2 sin 2 8cos8coscp
a3 = _1'2 sin 2 8sin 2 cpcos<j> - r cos 8 cos cp

Downloaded From : www.EasyEngineering.net


Downloaded From : www.EasyEngineering.net

Vector Differentiation 567

Hence
A = aler + a 2ee + 01 eq,

= {r 2
sin} 8sin $cos 1 $ - rsin 8cos8sin $ + r2 sin Ocos 2 8cosc/>k
+ {r 2 sin 2 8cos8sin $cos 2 $ - rcos 2 8sin $ - r2 sin 2 8cos8cos$ ~o
1 2 2
+ (-r sin 8sin $cos$-rcos8cos$h
Ex. 7.9.13 Prove the following:

(i) e = Seo+ {sin 8 )~eq,


r

ww eo (ii) = -Ser + {cos8)~eq,

w
Sol:
(iii) eq, = (- sin 8 )~er - (cos8 )~eo

.Ea
where' • ' denotes differentiation w.r.t. time 't'.
(i) We know that

er ={sin 8(- sin$)$+


syE"} {
er = (sin 8cos$)i + (sin 8sin $)j + (cosS)k

"}
" cos8cos$8
ngi i + cos8.sin$.8+
" sinOcosc/>.$ j - (sin8)8k
"

=
nee
e{(cos 8 cos $)i + {cos8sin$)j - (sin 8 )k} + <i>{{- sin8sinc/»i + {sin8cos$)j }
"
= 8e "
e + sin 8.$eq,
rin
(ii) ee ={cos8cos$}i + (cos8sin C/>}j - {sin 8}k
g
coseco.~++ .n
ee =( - sin8.8cos$ -cos8sin$~ } + ( - sin8.8s;". + e t
)j -( cOS8.e)k

= - e{(sin 8cos$)i + (sin 8sin$}j + (cos 8 )k} + cos8.+{(- sinc/»i + {cos$)j}


" "
= - 8e r + cos8$eq,
(iii) eo!> =(-sin$)i+{cos$}j

eo!> =-cos$~i-sin$~j ..... (i)

(- sin 8 }~er - cos8~ee

= ~[- sin 8{sin 8cos$i + sin 8sin $ j + cos8k}


- cos 8 {cos 8cos c/> i + cos8sin c/> j - sin 8 k}]

Downloaded From : www.EasyEngineering.net


Downloaded From : www.EasyEngineering.net

568 Engineering Mathematics - I

o 0

= - coscj).cj)i - sin cj).cj) j


= e~ from (i)

Ex. 7.9.14 If ep , eo are base vectors in cylindrical coordinates, show that

(i)(ep)=ee o and (ii)(eo)=-eep,


where'.' denotes differentiation w.r.t. 't'

ww
Sol: ep = (cosS)i+ (sinS)j

w.E
asy
= e{(-sinS)i + (cosS)j}

En
gin
eo = -sin Si + cosS j, eo = - S(cosS)i -e(sin S)j = -Sep

Sol: eer
Ex. 7.9.15 Express the velocity V and acceleration a ofa particle in cylindrical coordinates.
In cartesian coordiantes, the position .vector r = xi + yj + zk .
. dr dx. dy. dz
VelOCIty V = -=-I+-j+-k
dt dt dl dt ing
d 2r
-=-1+-, j+-k
d 2 x. d 2y d 2z .ne
Acceleration a= 2
dt2 2
dt dr dt
In cylindrical coordinates, x = pcosS, y = psin S , z = z and
t
ep=(cosS)i+(sinS)j, eo =(-sinS)i+(cosS)j, ez =k
~'. i = cosSep - sin Seo ; j :::: (sin S)ep + (cos e)eo (Solving above equations for i andj)
:. r = (pcosS XcosSe p - sin See)+ psin e(sin Sep + cosSee)+ zez = pep + ze z

dr dp de dz
. V = -=-e +P-P +-e [
.: dez =~(k)=O]
.. dt dt p dt dt Z dt dt

o
= pep (0) 0
+p Sea +zez (from Ex. 7.9.14)

Downloaded From : www.EasyEngineering.net


Downloaded From : www.EasyEngineering.net

Vector Differentiation 569 .

dV d. . . .... .. .. ....
a = dt = dt [pep + pOeo + zeJ = pep + pep + pOeo + pOeo + pOeo + ze z

(since ez = 0)
= pBen + pep + pBeo + pBeo + pB(-Bep ) + ze z ' (using results of (ex 7.9.14»

[p- pe 2 ]e p +[pO + 2pB]eo + zez

ww
Ex. 7.9.16 If t = xyz,
coordinates
find 'gradf' in (a) Cylindrical coordinates (b) Spherical

Solution:w .Ea
syE
In cylindrical coordinates (p,O,z), we have, x = pcosO,y = psinO,z = Z

and
ngi ..... ( I)

f = xyz = p2 Z sin 0 cos 0 = p


2
Z sin 20 nee
2
rin
af = pz sin 20· af = p2 Z cos 20. at = p2 sin 20
ap 'ao 'az 2 g.n
e 2
:. (I) gives, gradf = Vf = (pz sin 20)e p + (pz cos 20)eo + (~ sin 20)ez
t
(b) In spherical coordinates (r,O,tjJ), we have,

x = rsinOcostjJ,y = rsinOsintjJ,z = rcosO, and

Vf= af e +! af e +_1_ af e ..... (2)


ar r r ao 0 r sin 0 atjJ ;
Here f = xyz = 3r 2sin 2 OcosOcostjJsintjJ

:. af ='csin2tjJ{-sin 3 0+cos02sinOcosO}
ao 2

Downloaded From : www.EasyEngineering.net


Downloaded From : www.EasyEngineering.net

579 Engineering Mathematics - I

aj =rJsin20cosO.cos2~
o~
:. from (2)
)

~l=(3r2sin20cosOsin+cos~~,. + '; sin2~{-sin30+2sinOcos20}ee

+ (r2 sinOcosOcos2~~~.
ww
E.7.9.17 Itf= pzcosO (in cylindrical coordinates) find Vf.

w.E
Sol: f= pz cosO.

-aj = zcoso of = -pzsin 0 aj = pcosO


op
aj
asy
, 00

I aj of
' oz

Hence Vf =
En
op ep +p ae eo + oz ez

gin
= (zcosO)ep -(zsinO)eo + (pcosO}ez
Ex.7.9.18 Iff= ,.'2 sin2Gsin+ in spherical coordinates, find Vf.
eer
Sol: aj = 2rsin20sin4>
or ing
-8f = 2r2 cos 20sin 4>
ae .ne
aj =r2 sin20cos~
o~
t
8j Jo/ 1 Of
.. ~l = or e,. +-; ae ce + rsinO o~ e.p
= (2rsin20sinq,)e,. +(2rcos20sin~)eij + (2rcosOcos4»e.p

Ex. 7.9.19 Show that the vectorfieldA = z{(sin O)ep + (cosoh }-(pcosO}ez, in cylindrical
polar coord inates is solenoidal.
Sol: IfA= Aiel' +A2 co +AJe z then, AI =zsinO; A2 =zcosO; A3 =-pcosO

Downloaded From : www.EasyEngineering.net


Downloaded From : www.EasyEngineering.net

Vector Differentiation 571

= ~[~(pz sinO)+~(zcOsO)+~(- p2 coso)l


p ap as aZ J
= ~[zsinO-zsinO+O] = 0
p
:. A is solenoidal

1 I I )
Ex.7.9.20 IfA= ( r-cosOer +-eo +-.-Oe+ ,in spherical coordinates, find Div A.
r rS1l1

I I

ww
Sol: Here A =r 2 cosO, A2 =-, A3 = - . -
I r rS1l10

w .Ea
syE
= I
3
[4r
-sin20+cosO+O ng1i
nee
1.
,.- S1l1 0 2

r ing
= 4rcosO+-2-
cotO
r .ne
Ex.7.9.21 Ifj= p2Z2cos20, show that V2j=2p2cos20
Sol: In cylindrical coordinates,
t
2 af 1 af
V j = - + - - + - -2- + -
1 a2 f a2 f
..... (I)
ap2 2
p ap p2 ao az
j = p2 Z 2 cos20

2
Qf = 2 pz 2 cos2 0 . - j = 2 z 2 cos 20 .
a - at ')
-
ap , ap2 , as = -2p- Z 2 si1120·'
_'J

Downloaded From : www.EasyEngineering.net


Downloaded From : www.EasyEngineering.net

572 Engineering Mathematics - I

:. From (1) we get,


yo2 f = 2Z2 cos 26 + 2Z2 cos 29 - 4Z2 cos 29 + 2p2 cos 29 = 2p2 cos 29 .
Ex.7.9.22 Using spherical coordinates, show that yo2r" = n{n + l)r,,-2 when n is a
constant and r = 0 if n<2
Sol: In spherical coordinates,
2 2
yo2f=a2f+~Of +_1 a f + cot 9 Of + 1 d f
..... (I)
ar2 r ar r2 00 2 r2 a9 r2 sin 20 d~2
Here, f= rn
2

ww
..
Of = nrfl-I. a f = n{n- 1)r"- 2
8r ' Br2

w .Ea
.. (I)
syE
~ yo2f =V2r" = n{n-l)r"- 2 +~.nr"-I
r
= n{n -1 )r"-2 + 211r"- = n{n + I )r"- 2 .
2

Ex.7.t.23 Iff= r 2 sin29+cos 2 «p, find yo2f.ngi


Sol: Of = 2rsin29. a
or '
2
~ = 2sin 29 nee
ar~

r ing
.ne
Of
-=-sin2+
0+
a2f
-=-2cos2~
2
' 0+
a 2 f 20f 1 a 2f cot90f a 2f
t
yo2f - --+--+---+----+-::---::--
2
.. - ar2 r ar r2 00 ,.2 a9 r2 sin 9 a~2
2

+ [CO/9x2r2cos29+
r2 r
2 ~
sm~9
? X{-2COS2«P)]

= .2sin 29 +~(2rsin29)+ ~(- 41'2 sin 29)+ co~9 x -2cos2«p


r r r

= 2sin 29+ 4sin 29 -4sin 29 + 2cot9.cos20 -4cosec 29.cos2«p


r

= 2sin 29 + 2 cot 9.cos 29 -4cosec 2 9.cos2«P


r

Downloaded From : www.EasyEngineering.net


Downloaded From : www.EasyEngineering.net

Vector Differentiation 573

Ex.7.9.24 If A is a vector field, find curl A where,

(\) A= {r:~sino~,+(~cosO}e+ r l2eq,


(2) A = {zcosO)ep -{ZSiIlO)eo

Sol: If A = Ale, + A2eO + A3eq., We have, in spherical coordinates,

reo e, {rsinO)eq,
1
CurlA= ~.alar a/ae a/a~

ww r-smO
Al rA2 (rsinO}A3

w.E
Here, AI =r sinO, A2
2
=~cosO,
r
A =-
3 r2

asy
e, {rsin O)eq,
:. Curl A = 2
\

r sin 0 r2 sin 0
ajar
En a/ae
cosO
a/a$
(I/r}sine
gin
== ~.
r smO ae
\ [- a {( -smO
r
a (cosO )}e,+ {-a (r 2smO
1. ) - -
a$ a$
. a (sinO)}
)-- - - rea
ar r e eri
ng.
net
= 7
r- smO r
1. [~coso.e, +-;-sinO.re
r e _r2 coso.rsinoeq,]

= 2.
1 [\-cosO.e, +--ee
sin 0 -r 3 smO.cosOeq,
• ]
r smO r r
cotO I
= -3-e, +]ee -rcosOeq,
r r

ep pea ez
\
we have, Curl A = - a/ Bp alae a/az
p AI A2 A3

Downloaded From : www.EasyEngineering.net


Downloaded From : www.EasyEngineering.net

574 Engineering Mathematics - I

Here, A I = Z cosO, A2 = -z sine, A3 = o.

e peo ez
I P
:. Curl A = - a/ap a/ae a/az
p zcose -pzsinO 0

a a .}e + {-(zcose)--(o)
= -I [{ -(o)--(-pzsme) a a}
poO az Paz ap pee

ww
w.E = ~[psineep +pcosOeo +(-zsinO+zsine)ezl
p
asy
= (sinO)ep + (cose)eo

En +(~p2sin2e)e= is
gin
Ex.7.9.25 Show that the vectorfieldA= (pzsin2e)ep + (pz cos 2e)ee

irrotational.
e eri
=~p2sin2e
Sol: Here, AI =pzsin2e, A) =pzcos2e, A)
- 2
ng.
net
ep peo ez
a/ap alae a/az
p pzsin2e p2zcos2e ~p2sin2e
2

Downloaded From : www.EasyEngineering.net


Downloaded From : www.EasyEngineering.net

Vector Differentiation 575

:. A is irrotational.

Ex.7.9.26 Show that div (curl A) = 0, where A is a vector function, in orthogonal curvilinear

Sol:
ww
coord inates.
Let A = Aiel + A2e 2 + Aj e3

w
Then

.Ea Ale] A 3 e3

syE %v2
A2A2
Ojov)
A)A,

ngi
nee
rin
g.n
. . '"
Dlv j = ~ A A A
I
I
2 3
[~(
Ov V··A A ) I
I
I 2 3
e t

=0

Downloaded From : www.EasyEngineering.net


Downloaded From : www.EasyEngineering.net

576 Engineering Mathematics - •

Exercise - 7(i)
l. Show that the spherical coordinate system is an orthogonal system.
2. Show that the cylindrical coordinate system is an orthogonal system.

[Hint for I and 2: Show that e,.e 2 = e2 .e3 = e3 .e, = 0 etc 1


3. Express the vector, F = 2xi - 3y.j +z,Yk, in cylindrical polar coordinates.
2 2
[Ans: F= (2pcos S-3p 2 sin S}p -~sjn2S+3p2sin2Sh +(pzcosskJ
4. Represent the vector, A = xyi + yzj +zxk, ill spherical polar coordinates.
ww [Ans: A = r2 sinS{sin 2 Scos 2 ~sin~+sinScosSsin2 ~+COS2 Scos~k
w.E + (sin Scos8cos
2
~sin~ + cos 2 8sin 2 ~ - sin ScosScos~~o

a syE
+ (cosSsin~cos~-sinSsin2 ~cos~}~ 1

5. Express the vectors a) A = yi + xj and b) F = zi - 2xj + yk, in cyl indrical coordinates.


[Ans: a) A = peo b) F ngi
= (zcosS - p sin2S)e p - (zsinS + 2pcos2S)eo + (psinS)ezJ
6. Express the vectors a) A
coord inates.
= - yi + xj, b) F
nee
= xi + 2y.i + yzk, in spherical polar

rin
[Ans: a) A = r sinS eq, b) rsinS [{sinS(I + sin2~) + rcos2~ sin~}er

g.n
+ {cosS(l + sin2~)- rsinScosS sin~}e() + sin~cos~ eq,l
7. Find the gradients of the fol/owing function

i)f= pz sinS ii)f=


sinS
-2-
r
et
[Ans: i) Vf = (zsin S)ep + (z cos S)eo + (psinO)ez

ii) Vf=~[(-2sinS)er + (cosO)eoJ


r

iii) Vf =(2rsin
2
~}r - (rsin 2S)eo -(~cosecscos2 ~sin~ )e~1
8. Find the divergence ofth fol/owing vector functions.
i) A = (cosS + sinS) ep + (cosS - sinS)e o + ez

ii) F = ~2 sinS}r + (~COSO )eo + rl2 e~ , iii) V = (pcosO)er + (psinO)eq,

Downloaded From : www.EasyEngineering.net


Downloaded From : www.EasyEngineering.net

Vector Differentiation 577

.. 4 . 0 1 cos 20
IAns: (i) 0 (II) rsm +2-.-- (iii) 3cosO )
r Sill 0
9. Show that the following vector fields are solenoidal.

(i) A= (zcosO)c" -( z - sin O)co(ii) F = (~cot O)c r -~c() + (r cosO)c¢


r r
10. Find the curl of the following vector fields:

(i) A= (zsin O)C" + (zcosO)ctJ -(pcosO)c z

ww +(~COSB +(~
(ii) F =(rsinO)c r )cr )e¢

w.E
...
(III)V
1
= -tan
,.
0
-e¢
2

asy
IAns:(i) (sinO-cosO)c p +(sin¢+cos¢)Cf) (ii) (,.\ cot 0 jer -(cosB)c¢ (iii) 1~2 c¢)
En
11. If ' f'
that 'Vf' is irrotational. gin
is a scalar function in orthogonal curvil inear coordinates v" v2 , V3 prove

7.10 GREEN'S THEOREM IN THE PLANE e eri


7.10.1 Green's Theorem:
ng.
Let
(i)
(ii)
net
R be a closed region of the xy plane bounded by a simple closed curve C
P(x,y,) and Q(x,y) be continuolls functions of x and y having continolls

derivatives in R. Then r Pdx + Qdy =


.l
JI
aQ
c - ap )d'Cdy
11 ax ay
where C is in the positive direction.
Proof: Y
E

a ___ .lI __ ~ _ _7
I A
I
I
I
o~----~c------------~--x

Downloaded From : www.EasyEngineering.net


Downloaded From : www.EasyEngineering.net

578 Engineering Mathematics - I

Let the equations of the curves DAB and DEB (see the figure) be respectively
y = fl (x) and y = f2 (x). Let R be the region bounded by the curve C. Then,

oP " [ 12(X) oP }
f f=-dxdy = f f -dy ix
II CY x=c y=fI(x) CY

ww
w i.e., .Ea
fp(x,y)ix = - f f~xdy
c II CY
syE
lilly if the equations of the curves ADE be and ABE be taken respectively as
..... (1)

x = gl(Y) and x = g2(Y)' we have, ngi


nee
rin
h a h
g.n
= fiQ(g2,y)-Q(g\,y)}iy= fQ(g\,y)iy+ fQ(g2,y)iy
a h a e t
c

Thus fQ(x,y)iy = I~Qdxdy


c
f II ~
..... (2)

Adding (1) and (2),

J pdx +Qdy = f' oQ _ oP xdy


'! }lox oy rt

Downloaded From : www.EasyEngineering.net


Downloaded From : www.EasyEngineering.net

Vector Differentiation 579

7.10.2 Vector notation of Green's theorem


Green's Theorem in the plane can be put in vector notation in the following way.
Let F = P(x, y)i + Q(x, y)j
and r = xi + yj, so that
dr = (dx)i + (dy)j
F. dr = P dx + Q dy

j k

ww Again,VxF=%x
P
o/ay
Q
%z
°
w.E = (
OQ OP)
ox oy k, so that

(v x F).k = oQ _ oP
asy
ox oy
En
fF .dr JJ(V
gin
Taking dR = dxdy, Green's theorem in the plane can be stated in the vector form as,

=
II
eer
x F)JaIR

7.10.3 Physical interpretation of Green's theorem ing ..... (A)

I. Let F denote the force field acting on a particle.

fF .dr represents the work done in movi.g the particle around a closed .ne
Then

curve C.
t
:. From (A) it follows that the work done is determined by curl F = V x F.
2. In particular, if V x F = 0 i.e., ifF is conservative (or F = V t)

Then fF.dr = O. i.e., the work done is independent of the path.

3. Conversely, if the integral is independent of the path, i.e., if

fF .dr =0, then vx F=O


In the plane, V x F = 0 is equivalent to saying that : = ~~ where F = Pi +Qj.

Downloaded From : www.EasyEngineering.net


Downloaded From : www.EasyEngineering.net

580 Engineering Mathematics - I

7.10.4 Application of Green's theorem to the evaluation of area of a simple


closed curve.

The area bounded by a simple closed curve C = lfxu:y - ydx


c
Proof By Green's theorem, we have,

.fPdx + Qdy fI!~{auax - apay rtXl~Y


I"
==

ww if we put P =
.I.e., -ap =-1,
- y, and Q = x,

aQ_ == I, au _ap =2,


w.E ay
:. We get,
ax ax ay

fXdY - ydx = asy


ff2dxdy = 2A , where A is the required area.
I" II

En
i.e., A = ~fxdy -
2 (' gin
ydx .

eer Solved Examples

ing
.ne
~ ~

Ex. 7.10.5 Find the area of the ellipse ~+L = I.


a 2 h2
Sol: Parametric equations of the ellipse are,
x = a cosO, y = b sinO
dx = - a sine de, dy = b cose dO.
t
:. By Green's theorem,

Area of the ellipse = lfXdY - ydx


L

I 2n
=- f(acosehcos 0 + bsinea sine}le
2 0

I 2n
== - fahdO = rrah
2 0

Downloaded From : www.EasyEngineering.net


Downloaded From : www.EasyEngineering.net

Vector Differentiation 581

Ex. 7.10.6 Evaluate f(y - sinx}dx + (cosx}1y, a) directly and b) lIsing Green's theorem,

where c is the boundary of the triangle in xy-plane whose vertices are (0,0),

(%,0) and (%,1) traversed in the positive direction.

Sol: Let 1= f{y-sinx}d\" + {cosx)dy

ww a)alongOA: y=o, dy=O


1!
X =-
2

w.E 1['2
f(y - sin x}1x + {cos x }dy = f- sin xdx = cosxi
n<2

OA
asy 0 0
=-1

En
alongAB: d\"=O
g° in
.. f(y -sinx}lx + (cosx~/y = fOdy
Ali
=
e eri
2 ng.
along BO: Equation of 013 is y = 2x , dy = -dx ,
7t
1t
net
f(y-sinx}dx+cosxdy=
/JO
1[(2:
11 ~
-situ )+;cosx}x

x
2
= -+cosx+-stnxi0 2. =(0+ 1 +0)- (1t 1t
-+0+- =1----. 2) 2
1t 1t 11/2 4 1t 4 1t

:. I = Sum of the integrals along OA, AB and 130

7t 2 7t 2
=-1 +0+1- - - - =- - - -
4 7t 4 7t

Downloaded From : www.EasyEngineering.net


Downloaded From : www.EasyEngineering.net

582 Engineering Mathematics - I

(b) By Green's theorem,

J Pdx + Qdy = Ii( 8Q - ~~ldX dy, Here P = y-sinx, Q = cosx


'jc I
I
dx dy

8Q = _ sin x ap = 1
ax 'ay

wwf [ = !,f( ~sinx~ l)dr try <i[[ (~sinx~ l)dy Jd< = ~i ~ysinx~ y'l d<
f
w.E =
7r

o
2( 2x. 2x 2
--smx-- x = --
1C 1C
7r

1C X~O
f xsmx d'(--2 f xd'(
2.

1C ()
7r 2

a syE
ngi
Ex.6.10.7 Evaluate f(3x+4y)dx+(2x-3y)dy where C is the circle in xy plane with

nee
centre at origin and radius 2 units.
rin
Sol: By Green's theorem, g.n
fPdx + Qdy =
II
8Q 8Pl dxdy,
Ii(---
dx dy
et
Here P=3x+4y, Q=2x-3y ; 8~=4 aQ =2
Oy 'ax
The given integral = ff(2-4)dxdy

= -2 ffdxdy = -2A,
II

where A is the area of the circle C.

= -2 x 1C x 22 = -81C

Downloaded From : www.EasyEngineering.net


Downloaded From : www.EasyEngineering.net

Vector Differentiation 583

7.10.8 Verity Green's theorem in the plane for the integral 1(3x 2 - 8y2 )dx + 4( 4y - 6xy)dy

where C is the boundary of the region given by

(I) (2) x = 0, y = 0, x + Y= I
(I) The given region is shown in the figure below.

ww
w.E ._~ ______ X

a syE
The points of intersection of y = x 2 and y =
ngfxi are (0, 0) and (I, I).
We have to integrate [ along
(I) y = x 2 from 0 to A.
fx
nee
(2) along y = from A to 0, and add the two values.
rin
Alongy=x2, dy=2xdx
x~1

20x 4 + 8x 3 )dx = x 3
g.n I .
:.1= f (3x 2
AU

Alongy =
- 8X4)dx + (4x 2 - 6x')2xdx = f(3x 2

fx ; x = y, dt = 2ydy
X~O
- -
0
et
4x 5 + 2X4 I =-1

yeO II 0 5
:.1= J(3 y 4_8 y 2jdy+(4y-6x 3 )dy= J(6yS-22y3+4Y)(2Y~/y=y6_~y4+2y21=-
AO )' I
2 I 2

5 3
:. 1=-1+-=-
2 2
By Green's Theorem,

1Pdt + Qdy =
c
Ii OQ. OP)dXd
)l ox ~v Y
Here, P = 3x2 - 8y ; Q = 4y - 6xy

oQ _ oP =-6v-(-16v)=IOy
ox oy . .
Downloaded From : www.EasyEngineering.net
Downloaded From : www.EasyEngineering.net

584 Engineering Mathematics - I

:.I=f flOydxdy= f
I[E 1 I
flOydydx= fSy
2 J~l dx =
x2
I 4 3
fCSx-Sx )dx=-
2
R x=O y=x 2 x=O x=o

Green's theorem is verified.


2. The given region is shown in the figure below.

ww (0,1)8

w.E x=O
o~--~~----------x
y=0 A(1,0)

AlongOA,
a y = syE
0 ::) dy = 0

:. Given integral =
I
2 3
f 3x dx = x 1= I ngi
I

AlongAB, y =
x=o

I - x ::) dy =
0

- dx nee
:. Given integral
rin
=
o
f~X2 - 8(1- x 2 )}dx +
0
{4(1- x) - 6x(1- x)C-dx)} = fC-Ilx 2 + 26x -12) dx 'g.n
x=1

II 3 2 0 II 8
=--x +13x -12xl =--13+12=-
x=1

et
3 I 3 3
AlongBO, x = 0 ::) dx = 0
y=O 0
Given integral = f 4ydy = 2y21 =-2
y=1 I

8 S
:. The given integral = 1 + "3 - 2 ="3
By green's theorem,

1Pdx + Qdy = f JlnaQ.


C
ap)dXdY
ax ayR

Downloaded From : www.EasyEngineering.net


Downloaded From : www.EasyEngineering.net

Vector Differentiation 585

Here, P = 3x2 - 8y2, Q = 4y - 6xy


aQ ap
ax· ay = -6y - (-16y) = lOy

= 5(1-x)3i =-~(O-I)=~

ww Hence the theorem is verified.


3x-1 0 3 3

w
Ex. 7.10.9 Evaluate.Ea(2,1)

J(10x
4
- 2x/ dx - 3xl.v dy along the path x4 - 6xl - 4.0 = 0

Sol:
(0,0)

syE
P = IOx4 - 2xl, Q = 3x2.0
ar
-=-6xy-
ay
~
=-
8Q
ax
ngi
nee
The integral is independent of the path. Hence we can use any path. For example,

rin
if we use the path from points (0, 0) to (2, 0) and then from (2, 0) to (2, 1); we can
evaluate the integral.
(i) From (0, 0) to (2, 0) ; y= 0, dy = 0 g.n
.
:. The tntegral =
2
J
o
4
lOx dx - 2x
512
0 = 64
e t
(ii) From (2, 0) to (2, I); x= 2, dx = 0
2 2 2 2
4 5
:. The integral = JIOx dx = 2x J= 64
o 0 0
J 12y 2 dy = - 4/1 =-4

:. The value of the integral = 64 - 4 = 60

Aliter: Since ar = aQ, we know that {I Ox4 - 2x1)dx - 3xl.0 dy} is an exact
ay ax
differential of (2x 5 - xlI).
(2.1) (2,1) 2 3
2 5 2
The given integral = J d(2x 5 - x /)= 2x - x y3 J= 2.25 - 2.1 = 60
(0,0) (0,0)
Downloaded From : www.EasyEngineering.net
Downloaded From : www.EasyEngineering.net

586 Engineering Mathematics - I

X
Ex.7.10.10 VerifY Green's theorem for f(e- siny)dx + (e-X cosy) dy where C is the
c

boundary of the rectangle whose vertices are (0, 0) (n, 0) (n, ~ ) and (0, ~ )
2 2
traversed in the +ve direction.
Sol:
y

ww
w.E
I

O(~O~,O~)----~'~----~A~(n~,O~)-x

a
By Green's theorm,
sInyE
fPdx+ Qdy =
c )l ax
8Q - 8P)dXdY
ay ngi
Here, P = e-x siny, Q = e-x cosy
nee
-
aQ
8x
ap
- -
ay
= - e-x cosy - e-x cosy = -2e-x cosy
rin
fPdx+Qdy= If -2e x cosydxdy g.n
c

=-2
R

x x/2
f [ f e-xcosdyJdx=-2 f eX siny fdx
x x/2 et
x=o y=O x=() 0

x
=-2 fe-xdx=2e xl: =2(e- X
-1) .....(i)
c

Again,fPdx+Qdy=f f+ f+ f+ f
c c OA AB BD DO

Along OA ; y = 0, dy = 0

.. fPdx+Qdy=O
OA

Alo~gAB: x = n, dx= 0

Downloaded From : www.EasyEngineering.net


Downloaded From : www.EasyEngineering.net

Vector Differentiation 587

1tI2

f Pdx+ Qdy = Je 11 cosydy = e- 1I sin YI~/2 = e 11

AB 0

AlongBD;

o t 0
p dx+Qdy= fe-xdx=~ =-1 + e
J -- 1 11
11

Along DO ;
BD

x = 0, dx = °
11

ww JP dx + Qdy = feos ydy =


o
sinYI~/2 = - 1

w
DO 11/2

.Ea c
..... (ii)

J p dx + Qdy =
syH(E
:. From (i) and (ii) it is proved that

8Q - 8P) dxdy
c

Hence the theorem is verified.


R 8x
ngi 8y

Ex. 7.10.11 Apply Green's theorem to obtain the area bounded by the curve
°. nee
x2/3 + y2/ 3 = a 2/3 , a>
The parametric equations of the curve are x = a cos3e, y = sin 3e. A rough sketch of rin
Sol:
the curve is given below:
g.n
e t
A'
x~'--~~~~~~-----x
(6 =rc) A(6 =0)

,
y

x = acos3e, y = asin 3e
dx = -3 acos 2e sine de, dy = 3a sin 2ecose de
By Green's theorem, the area bounded by a simple closed curve C is given by

!J(xdy- ydx)
2c

Downloaded From : www.EasyEngineering.net


Downloaded From : www.EasyEngineering.net

588 Engineering Mathematics - I

:. The area bounded by the given curve C


21<
= ! J{acos 30.3asin 2 0cosO+asin 30.3acos 2 0sinO}dO
2 {)=o

3 3 3
ar
3 2 21t
= - JCos Osm OdO=-
2·2 af·}2 br
(lr n:a
sm-20dO=-lJ-cos40)dO=--
2 2x 2

ww 2 0 8 0

Verify Green's theorem for J(2xy-x 2)dx+(x+y2)dy, where C is the


16 () 8

w.E
Ex.7.10.12
c
closed curve in xy-plane bounded by the curves y = x 2 and = x. y
Sol: By Green's theorem,
asy y

JPdx+Qdy= Jfc
aQ
En
- ap)dxdy
ax ay
A(1,1)

gin
C R

2 aQ ap
Here, P = 2xy - x , Q = x +,1 - - -ry
Y, ax
=

e
1 - 2x

eri o-=:;;~-----x

J aQ of
J[ f (1-2x)dyJdx= JY-2xyl~:;; dxn
I y=~ I

R
jr---)dxdy=
ax ry x=() y=x2 ()
g.n
I
= jr-v'x -2x-v'x _x 2 +2X3 )dx = '!:x 2 -2.'l:x 2 -~+2.~
o 3 5 3 40
3 5 3 4 I
e t
1 4 1 1 1
=-----+-=- ..... (i)
2 5 3 2 30
Again fPdx+Qdy= fCPdx+Qdy)+ J(Pdx+Qdy)
r ~ ~

Along C I' Y = x2, dy = 2xdx, "x" varies from 0 to 1


1 I
:. fpdx+Qdy= f C2x 3 -x2)dX+CX+X4)2xdx= fC2x 3 +X2 +2X5)dx
(') x=o 0

Downloaded From : www.EasyEngineering.net


Downloaded From : www.EasyEngineering.net

Vector Differentiation 589

AlongC 2, x=y, dx=2ydy, 'y' varies from I to 0


o 0
:. JPdx+Qdy= J(2/_y4)2ydy+(y2+ y 2)dy= J(4 y 4_2 y 5+2 y 2)dy
(', y=1 I

..... (ii)

wwFrom (i) and (ii), the theorem is verified.

w.E
asyExercise -7(j)

1. En
Evaluate 1(x 2 + y2)dx+3xy 2dy, (a) directly (b) by Green's theorem, where cis

the circle x 2 + gin


y = 4, traversed in the +ve direction.
(Ans: 121t)
e eri
2.
ng.
Evaluate 1(x 2 + 2xy)dx + (x 2 y + 3)dy around the boundary C of the region given

by y = 8x and x = 2, (a) directly and (b) by Green's theorem.


(Ans: 128) net
3. VerifY Green's theorem for the integral 1(3x 2 + 2y)dx - (x + 3cos y)dy where Cis

the boundary of the parallelogram with vertices at (0,0), (2, 0), (3, I) and (1, I)
(Ans: -6)

(n,2)
2
4. Evaluate J(6xy - /)dx+ (3x - 2xy)dy along the cycloid x = 8 -sin8,
(0,0)

y = l-cos8. VerifY the result by using Green's theorem.


(Ans : 61t 2 - 41t)

Downloaded From : www.EasyEngineering.net


Downloaded From : www.EasyEngineering.net

590 Engineering Mathematics - I

5. Using Green's theorem find the area bounded by one arch of the cycloid
x=a(B-sinB), y=a(l-cosB),a>O, and x-axis.

[Ans: 3J[a 2 ]

6. Evaluate f{2x 2 - y2 )dx + (x 2 + i) by Green' theorem where C is the boundary of

ww c

the surface in the xy plane enclosed by x axis and the semi-circle y = ..Jl- x 2
w.E [Ans: 4/3]

7.
asy
Evaluate f(cos x sin y - xy)dx + sin x cos y,using Green's theorem where c is the

En
c

[Ans: 0] gin
2_e
8. eri&2
VerifY Green's Theorem in the plane for f{x xy 3 )dx+ -2xy)where C is the

ng. c

[Ans: 8]
n
square with vertices at ( 0,0) , ( 2,0) and (0 ,2)

et
(2,1)
9. Evaluate f (12 x3- 2x/ ) dx - 3x2y2 dy along the path x3- y3 + Y - 4xy = 0
(0,0)

[ Hint: Proceed as in aliter of 7.1 0.9]

4 2 31(2,1)
[Ans: 3x - x y
(0,0)
= 44 ]

Downloaded From : www.EasyEngineering.net


Downloaded From : www.EasyEngineering.net

Vector Differentiation 591

7.11 Gauss Divergence Theorem


7.11.1 Gauss' Divergence Theorem
Let (I) V be the volume bounded by a closed surface S
(2) A be a vector function of position with continuous derivatives.

Then, fffV .A .dv ffA .nds 1A .ds


= =

where n is positive unit normal (outside drawn normal) to S.

ww
Proof:

w z
.Ea
syE
ngi
nee
r ..... ,
t------y

ing
x
R

.ne
S is a closed surface
t
Let any line II lel to the coordinate axes cut S in at most two points.
Let z = gl(x, y) and z = gix, y) be the equations of the upper portion SI and lower
portion S2 respectively.
Let R be the projection of S on the xy-plane.
If A = Al i + A::J + A3 k

Downloaded From : www.EasyEngineering.net


Downloaded From : www.EasyEngineering.net

592 Engineering Mathematics - I

Then,

oA OA
fff OZ 3 dv = fff OZ 3 dzdydx
v v

Iq(x,y) oA ff z~g,
ff[ S 0/ dz] dydt= AJX,y,Z)1 dydx
II z~g2( x,y) II z~g2

= f f[A 3 (x,y,g,)- A 3 (x,y,g2)]dydx

ww R

For the upper portion SI' dy dx = (cosyl)dS I = k. n l dS I

w.E
since the normal n l to SI makes an acute angle YI with k.
For the lower portion S2' dydx
asy = (-cOSY2) dS 2 = --k.n 2 dS 2,
since the normal n 2 to S2 makes an obtuse angle Y2 with k.

En
:. f f A3(X,y,g, )£lydx =:f f A 3k .n, dS, and
R "
gin
e eri
So that, f f[A 3 (x,y,g,) -A3 (x, y, g2) ]dydx
II 8, ng.
= f f A 3k .n,dS, + f fA3k.n2dS2
S2

s
f
= fA3k.ndS net
..... (I)

Similarly, by projecting S on the other coordinate planes, we get,

f f fO~, dv= f fAl nds ..... (2)


v •

oA2
and f f f dv = f f A 2j.n ds ..... (3)
v cry s

(I) + (2) + (3) =>

fff(o~, + O~2 + o~3)dV= ff(A ,i+A 2j+A3k).ndS


v s

i.e. f f fV.A dv = f fA.n dS which proves the theorem.


S

Downloaded From : www.EasyEngineering.net


Downloaded From : www.EasyEngineering.net

Vector Differentiation 593

7.11.2 (a) Express Gauss' Theorem in words and (b) obtain its Cartesian form.
(a) Gauss' Theorem states that "The surface integral of the normal component
of vector A taken over a closed surface is equal to the integral of the
divergence of A taken over the volume enclosed by the surface.
(b) Let A=A 1i+A 2 j+A 3k,

. aA aA? aA 3
Then dlv A = V.A = ~-I + -"'- + ---
ax ay az
Let the unit normal n to S make angles a,P,r with the +ve coordinate axes so

ww
that, cosa=n. i, cosp =n. j, cosr=n.kand cosa, cosp, cosr are the
direction cosines of n

w .Ea
n = ( cos a )i +( cos P)+( cos r ) [ n. i = "I = cos a etc.]

syE
:. A.II =Alcosa+A 2 cosP+A 3 cosr

JJr{ aA + aA + aA )dXdYdZ = ngi


Hence the divergence theorem can be written in Cartesian form as,

J J(A I cosa + A2 cos p + A3 cosr)dr;


v Jl ax ay az s
nee
JIA,dydz + A 2dzdxr+ A dzdy
=
s
ing3

Ex. 7.11.3: Evaluate using


Solved Examples

the divergence theorem = J J F .n .ne d~ where


s
F = 2xyi + yz2 j + xzk and S is the surface of the parallelepiped
t
bounded by x = 0, y = 0, z = 0, x = 2, Y = 1 and z =3 .

Sol: By the divergence theorem; HF.n ds = JHV.Fdv


s

Here, V.F =~(2Xy)+~(yz2)+~(XZ)


ax ay ay
=(2Y+Z2 +x)

I IF.n ds = HI(2Y+Z2 +x)dv


s

Downloaded From : www.EasyEngineering.net


Downloaded From : www.EasyEngineering.net

594 Engineering Mathematics - I

2 I 3 2 I 3

I I Ie 2 y + Z2 + x)dxdydz = I I [ I(2y + Z2 + x)dz] dxdy


X;O y;O z;O X;O y;O =;0

2 I 3 3 2 I

= II2Yz+~+xz dxdy== Ifc 6 y+9+3x)dxdy


00 3 0 00

2 I 2 I 3 22 2
= I[ I (6y+9+3x)try]d\" = I 3i +9y+3xyl dx== I(12+3x)d\"=12x+ ;

ww X;O y;O

=24+6= 30
0 0 0 0

Sol:
w
Ex. 7.11.4 Verify the divergence theorem for F = (4xy)i - (j2)j + (xz )k, over the cub(

.Ea
bounded by x = 0, x = I, Y = 0, Y = 1, z = 0 and z = 1.
By the divergence theorem

II F.n dS == I I IV.F dv
\ V
syE
Here F
a a 2 a
= (4xy)i - (j2)j + (xz)k
ngi
V.F==-(4xy)--(y )+-(xz) =4y-2y+x=x+2y
ax ay az
nee
rin
I I I I I I

I I IV.Fdv == I I Ie x + 2y)dzdydx == I I [ Ie x + 2y)dz ]a:vdx


v x;Oy;Oz;O x;Oy;O =;0

I I

fi xz + 2 y z1 dydx
g.n
I I
== I

I
x;Oy;O

I I 2
e 3
t
== If I(x+2y)dy]dx=
X;O y;O x;O
Ixy+y
2
1

0
dx=
x;o
IeX+I)d~==~ +xl~==2

To evaluate IIF.ndS z

The surface S contains 6 faces (see figure)


S) - Face EPFA, S2 - Face OBDC
S3 - Face PFBD, S4 - Face OCEA
Ss - Face PDCE, S6 - Face OBFA J---+--+---y

Downloaded From : www.EasyEngineering.net


Downloaded From : www.EasyEngineering.net

Vector Differentiation 595

The surface integral IIF.ndS is equal to the sum of the surface integrals on the
above 6 faces.
on S" n = i, x = I, F.n = 4y; dS = dy dz
J J J J J J

:. JJF.ndS= J J 4ydydz= J[ J4ydz]dy= J 4yzl~dy= J 4ydY=2/1~ =2


y;O :;0 F-O :~O y~O 0

on S2' n = -i, x = 0, F.n = °


IIF.n~S=O
ww
:.

on S3' n = i, y = I, F.n = -I, dS = dx dz.


w.E I I J

I[ I-Idz]dx= I-z dx= I-dx = --xl~ =-1


I

:. I IF.ndS=
a°sy
on S4' n = -i, y = 0, F.n =
:. I IF.ndS=o En
84 gin
on S5' 11 = k, z = I, F.n = x ; dS = dx ely
J J J J
e I J eri
1 2 1

:. I I F.lldS = I I xdxdy = I [ Ix dy 1 dx = I xy dx = I xdx = ~ =


X~O y=o 2 0 ng. ~

net
85 X;O y;O x;(} 0 x=O

on S6' n = k, z = 0, F.n = °
:. I IF.ndS = °
Hence :. II F.ndS = 2 - 1 + ~ =%
8

The divergence theorem is verified

Ex.S.11.5 : If r = xi + Yi + zk, evaluate :. IIr.n dS where S is any closed surface.


s
Sol: By the divergence theorem
:. I Ir.n dS = I I Iv." dv ,where V is the volume enclosing S.
s v

Downloaded From : www.EasyEngineering.net


Downloaded From : www.EasyEngineering.net

596 Engineering Mathematics - I

000
V.r = -(x) + -(y) + -(z) = 3
ox 0' oz
:. I Ir.n ds = I I I3 dv = 3v

Ex.7.11.6 : If '\jI' is a scalar function, prove that I I IV\jIdv = I I'P n ds

Sol: By divergence theorem


ww I I IV.Adv = I IA.n ds ..... (I)

w.E Take A = \jIc, C = cli + c-j + c3k is any constant vector.

asy
:.(l)=> JIIV.(lPc)dv= fI('Pc).nds ..... (2)

En
v s

But,

gin
eero )+-(c~)+-(c
0 0
= (V \jf).C [ ":V.c=-(c
ox l

ing
oy- OZ3 )=0 ,
= c. V \jf
.ne
cl' c2 ' c3' being constant]
and (\jIc).n = c.(\jJl1)
so that (2) becomes, t
[ ": a.b = b.a]

IJI(c.V'P) dv = JI{c.('Pn)}ds

=> c. JI IV'Pdv = c. JI'-IJn ds

=> JI IVlPdv = JI'Pn ds (.: c is arbitrary)


v

Ex.7.11.7 : Ifs is a closed surface enclosing a volume V and ifF = (Ix)i + (my)j + (llz)k,

I, m, n being constants, evaluate JIF.nds

Downloaded From : www.EasyEngineering.net


Downloaded From : www.EasyEngineering.net

Vector Differentiation 597

Sol: By Gauss' theorem

HF.nds= Hfv .Fdv

a a a
V.F = -(Ix) + -(my) + -(nz) = I + m + n
ax ry az
:. HF.nds = Hfu + m + 11) dv

=(/+m+n)V
Ex.7.11.8: IfE=curlA,evaluate HE.nds wheresisanyclosedsurface
ww
Sol: By Gauss' theorem,

w.E HE.n ds = Hf(div E) dv where V is the volume enclosed by s


.\' V

But Div E = div (curl A) = 0


:. HE.nds=O asy [ See 7.5.2 (2) ]

En
gin
Ex. 7 .11.9 : If s any closed surface and n is unit +ve normal to s, show that Hn ds = 0

Sol: Consider a constant vector


a = ali + ~ + a3k e eri
( i.e. al' a2, a3 are constants)
Then, a·[Hndv] = H(a.n)ds
ng.
= Hf(V.a)dv net
(by divergence theorem)

=0
a.
[ ':V.a=-) +_2 +_3 =0]
aa aa
ax ay az
Thus a. a.[ Hn dv] = 0 is true where a is any arbitrary vector.

:. Hnds=O

Ex.7.11.10 : Evaluate H(F.n)ds, where s is the region bownded byy2 = lx, x = 2, Z = 0,


s
1
Z = 3, and F = 2xi + 3yj + "3 z3k, using Gauss' theorelll

Downloaded From : www.EasyEngineering.net


Downloaded From : www.EasyEngineering.net

598 Engineering Mathematics - I

Sol: By the divergence theorem,


ff(F .n) ds = fff(V.F)dv .... (1)
s v

V.F =~(2X)+~(3y)~~(Z3)
ax ay ay 3
= 5+z 2

From (I), ff(l1~.Il)ds = fff(5+z 2 )dv,


s

ww
f fw.
2 2.[i-; 3 3

5z + ~
=
E
x=O Y= 2 Ex 0
dydx

asy ~--------r-----X

En
2 gin
) 2

= f 96J2xdx = 96J2 ~
x=o eer0
= 192J2

Ex. 7.11.11 Evaluate ing


ff( 2xi - 31 j + Z2 k ).n ds over the surface bounded by

x 2 + y2 = 1, z = 0,
s
Z = 2, using Gauss' theorem. .ne
Sol: By the divergence theorem,
ff(F .n) ds = fff(V.F)dv ,
t
s
Here,F= 2xi-3y2j+Z2k

DivF =~(2x)-~(31)+~(z2)
ax ay ay
= 2-6y+2z
+1 M 2
ff(F.n)ds= f f f(2-6y+2z)dzdydx
x=ly=~O

Downloaded From : www.EasyEngineering.net


Downloaded From : www.EasyEngineering.net

Vector Differentiation 599

16[~~I-X2 +~sin-1 XJ1


ww
=
2 2 .1
=8n

w.E
Ex.7.11.12: Evaluate using the divergence theorem H(F.Il)dS where S is the surface

of the sphere x 2 + I + z2 h2 in the first octant and F = yi + zj + xk


Sol: By divergence theorem, asy =

f
:. H F.1l d~' = HV'·F dv En ..... (i)

F = yi + zj + xk
v

gin
V'.F= 0
e eri
:. H(V'.F) dv = 0
z ng. ..... (ii)

net
~--+B--Y

Let us evaluate the surface integrals over the faces OAB, OBC and OCA.
b Jj;2~)
f fF.n ds = - f fx dx(ry (,,' n = -k)
OA B x=O y=O

Downloaded From : www.EasyEngineering.net


Downloaded From : www.EasyEngineering.net

600 Engineering Mathematics - I

Similarly If F.ndv = If F.nds + If F.nds + If F.nds + If F.nds


DAB OAB ABC DCA ABC

3
= -Jrb + If F.nds .... (iii)
AIlC

From (i), (ii), and (iii), we get


3
0= -Jrb + If F.nds
ABC

ww If F.nds = Jrb 3

w.E
ABC

Ex. 7.11.13: VerifY divergence theorem for F=4xi-2y2j+z2k taken over the

2
x + / = 4, asy
region bounded by

Z = 0 and z = 3.
Sol: By the divergence theorem, we have En
Iff DivFdv = gin
IfF.~ ds .... ( I )
s
er
=~(4x)-~(2/)+~(Z2) e
..
.,.
(1) DivF
Ox By 8z
=4-4y+2z
ing
L.R.S. 0[(1) ~ ,t~~l,JL (4-4y+2z)dz ]dydt .ne
t
~ ,Ll" [4z-4yz+z'JC dydx~ )11 (21-12y)dy Jdx
2

=42 f ~4_X2dx=84Jr
x~-2

[Do the integration w.r.t.x yourself, taking x = 2sin ()]


(2) Evaluate of surface integral IfF.~ ds
s

Downloaded From : www.EasyEngineering.net


Downloaded From : www.EasyEngineering.net

Vector Differentiation 601

Z=3

ww x

(a)
w.E
The given surface of the cylinder can be divided into 3parts, namely
8 1 : the circular surface z = 0

(b) asy
S2: the surface z == 3 (circular) and

Eni
I

(c) S3: the cylindrical portion of 8: Xl + = 4, z == 0, z =3


gin
we now find JJF.~ ds over SI ,82 , S3 .If we add them, we get R.H.S of (I).

(a) on SI:Z=.O; e
~=-k; F.~=-(4xi-2ij).k=0;:. JJF.~ds'=O.
eri
s,

- - - ( ) dxdy ng.
(b) on S2:z==3; n=k ; F.n= 4xi-2y2j+9k .k=9; d\'=-I':' -I =dxdy

:. JJF.1i (is' = JJ9dxdy = 9A , where A is the area of the circle


n.k
net
s, s,

x 2 + y2 = 4, = 9J'l' (22) = 36J'l'


(c) on S3: Let fjJ =x2+i-4=0;

- V fjJ 2 ( xi + yi) xi + yi ( . 2 2
n =- - = = SInce x + y = 4)
!VfjJ! 2~X2 + i 2

4 x2 -2y 3 22 3
F.n= =x-y;
2
To evaluate JJF.1i ds, take x = 2cosB,y = 2sinB,
S3

Downloaded From : www.EasyEngineering.net


Downloaded From : www.EasyEngineering.net

602 Engineering Mathematics - I

and ds = 2dO dz ; limits of z are 0 to 3 and those of 0 are 0 to 27r .


2lf 3

Hence fJF.~ ds = f f (8cos 0-8sin 0)2d 0 dz


2 3

S3 (}=02=0

2" 3 2"
= 16 f [( cos 0-sin O)z ]
2 3
dO = 48 f (cos 0-sin O)dB =487r
2 3

(}=o 2=0 (}=O

[ Do the integration w.r.t. e yourself]


ww
:. R.H.Sof(I)= 0+367r+487r=847r;:. L.H.S= R.H.S
Hence the theorem is verified.

w.E Exercise - 7K

1.
asy
Verify Gauss's divergence theorem for A = (x 2 - yz)i + (i - zx) j + (Z2 - xy)k

En
taken over the rectangular parallalopiped 0 ~ x ~ 2, 0 ~ y ~ 3, 0 ~ z ~ 1.[Ans:36]

2.
--
gin
Use the divergence theorem to find ff F.ndS, where
s

f = (3x + 2Z2)i - (Z2 - 2 y) j


eer + (/ - 2z)k and S is the surface of the sphere with

3.
centre at (2, -I, 3) and radius 2 units. [Ans:32 TC]
Verify the divergence theorem for the ing
vector function,
A = (4xz)i - (i)j + (yz)k , taken over the
x = 0, x = 1, y = 0, y = 1, z = 0 and z = 1.[Ans:3/2]
unit cube
.ne
bounded by

4. If r = xi + yj + zk , and S is the surface of the rectangular parallelepiped bounded


by planes x = O,y = O,z = O,x = a,y = band z = c, find the value of ffr.ndS
t
s
using Gauss's theorem. Verity your answer by direct evaluation of the integral.
[Ans:3abc]

5. Use the divergence theorem to evaluate ffA.nds for A = (2x)i-(2y)j + (3z)k

where s is the sphere given by (x _1)2 + i + Z2 = 1 [Ans:4 7r]

6. If V=(lx)i+(my)j+(nz)k and I, m, n being constants show that

ff V.ds = 327r (l + m + n). where S IS the surface the sphere


s 3
(X-3)2 +(y-2i +(z-I)2 = 4.
Downloaded From : www.EasyEngineering.net
Downloaded From : www.EasyEngineering.net

Vector Differentiation 603

7.12 Stoke's Theorem


7.12.1 Stoke's Theorem
Let (1) S be an open, two-sided surface bounded by a simple closed curve C.
(2) A be a vector function having continuous derivatives

Then, fA.dr = H(Y' x A).nds = H(Y' x A).d\'


where C travels in the +ve direction and n is the unit +ve (outward drawn) normal to

ww S.

w.E
Proof:

asy z

En
gin
eer
ing
.ne
Let S be the surface. Let the projections of S on the coordinate planes be regions
bounded by simple closed curves. t
Let 'R' the projection ofS on xy plane be bounded by C l . (see the figure above).
Let the equation of S be z = <l>1(x, y) where <1>1 is a single valued, continuous and
differentiable function.

j
Then Y'x(Ali)=alax alay
AI 0

Downloaded From : www.EasyEngineering.net


Downloaded From : www.EasyEngineering.net

604 Engineering Mathematics - I

{V x (A]i}}.n ds =(8A] _ 8A] ).nds = {8A] (n.}) _ 8A3 (n.k)}ds (.)


8z 8y 8z 8y ..... 1

The position vector r of any point on S can be taken as


r = xi + y) + zk

= xi + y} + t/>/x, y)k
8r _. 0+]
and - - } + - k
ay 8y

ww
But
8r
ay being the vector tangent to S, it is .i, to n.
w 8r
.. ay' n = 0 .Ea
~ n.} =
3</>]
8y (n.k)
syE
-

. 8AI 84>1 8AI


:. (I) => { V x A\,)} . n ds
ngi
= (---n.k--n.k)ds
8z ay 8y

nee ..... (ii)

on S, A\(x y, z) = A\(x, y, 4>\(Y» = G(x, y) (say) r ing


8AI 8AI 8z aa
-+-.-=-
ay 8z 8y 8y .ne
:. (ii) becomes, t
aa aa
{ V x (A\i) ] .n ds = - ay (n.k) ds = - 8y dxdy r·: n.k. ds = dxdy]

H(VxA\i)}.nds=
,I
Jf- aaay dxdy=fCdx-,
R ('
by Green's theorem in the plane. Now

at each point (x, y) ofC, the value ofG is the same as the value of AI at each point
(x, y, z) of C, and since dx is same for both curves, we have

Downloaded From : www.EasyEngineering.net


Downloaded From : www.EasyEngineering.net

Vector Differentiation 605

Hence, H{V x(Ali}}·n ds = fAldx ..... (iii)


s c

lilly by projecting S on yz and zx planes, it can be shown that,


H{V x (A2i)}·n ds = fA 2dY ..... (iv)
s c

H{V x (A3 k )}.n ds = f A 3dz ..... (v)


s c

ww
Adding (iii), (iv) and (v), HVx A.n ds = fc A. dr
w.E s

(since A. dr = AI dx + A 2dy + A 3dz)

a
Hence the theorem is proved.
syE
ngi
Ex.7.12.1 :(a) Express Stoke's theorem in words and (b) obtain its cartesian form.
Sol :(a)
nee
The I ine integral of the tangential component of a vector A taken around a simple
closed curve C is equal to the surface integral of the normal component of curl
A taken over a surface Shaving C as its boundary.
rin
(b) As in 7.11.2(b)
A=A l i+Aj+A3k g.n
n = (cosa)i + (cos~)j + (cosy)k

) k
et
Then, Vx A = ajax ajay ajaz
Al A2 A3

=(aA 3 _ aA2)i + (aA I _ aA3)} + (aA 2 _ aA I )k


ay az az ax ax ay

aA 3 aA 2 aA aA 3 aA 2 aA I
(V x A)Jl =( - - -)cosa + ( -I - -)cos~ +( - - -)cos y
ay az az ax ax ay

Downloaded From : www.EasyEngineering.net


Downloaded From : www.EasyEngineering.net

606 Engine~ring Mathematics - I

Hence the cartesian form of the Stoke's theorem can be stated as,

=fA ,dx+A 2dy+A 3dz


c
Solved Examples
Ex.7.12.3 : VerifY Stoke's Theorem for A = (x - 2y)i + yilj + y2zk, where S is the upper

ww
Sol:
half of the sphere xl + y2 + z2 = 1 and C is its boundary.
The boundary of the projection ofS in the xy-plane is a circle with centre at origin

w.E and unit radius. Its parametric equations are x = cosS, y = sinS, Z = 0, 0:::: S < 2n
dx = (-sinS)d9, dy = (cosS)dS.

asy
f Adr =f(x-2y)dx+ yz 2dy+ y2 zdz
c c
En
21t
= f[cosS - 2sinS] (- sinS )dS gin (.: z =0)
e~()

e eri
-_ 2~[-sin2S
21t

o
JI
2
+ I -cos2S]dS-
- cos2S
- - + S -sin2SI
4
- - -_2 n
2 0 ng.
i j k
net
Y' x A = ajax ajry ajaz = 2k
(x-2y) yz2 y2 z

H(Y' x A).nds = H2(n.k )ds = 2 Hdxdy


s R

(n.k ds = dxdy and R is the projection of S on the xy plane)

Hence Stoke's theorem is verified.

Downloaded From : www.EasyEngineering.net


Downloaded From : www.EasyEngineering.net

Vector Differentiation 607

Ex.7.12.4: Prove that a necessary and sufficient condition that fF.dr for every closed
("

curve C is that '\I x F =0 .

Proof: (a) The condition is necessary: Let '\IF = 0;

Then by Stoke's theorem, fF.dr = If('\I x F).n ds = 0


(" S

ww (b) The condition is sufficient:

w
Suppose that fF.dr
(' .Ea =0 around every closed path c.

Assume that '\I x F "* 0 syE


at some point P. then, assuming that '\IF is continuous,

n
there exists a region with P as its interior point where '\IF = 0 .Let S be surface contained
gin
in this region and let the normal n to S at each point has the same direction as '\IF .

Then '\IF = an, (a being a +ve constant);


e eri
Let C be the boundary of S.

Then by Stokes theorem, f F .dr = If ('\IF).n ds =a Ifn.n ds > 0 ng.


c s s

Which is a contradiction to the hypothesis that f F.dr =0 ;


n
:. '\IF =0
et
Note: It follows that '\IF = 0 is also a necessary and sufficient condition for the line
1'2
integral f F.dr to be independent of path joining the points ~ and ~ .(see 7.6.4)
I;

Ex.7.12.S If r = xi + yj + zk , show that f r .dr = 0


c

By Stoke's theorem, Jr.dr = If(Curl r).n ds ..... (1)


c ('

Downloaded From : www.EasyEngineering.net


Downloaded From : www.EasyEngineering.net

608 Engineering Mathematics - I

i j k
But curl r= a/ax a/ay a/dz =0
x y z

:. (1) => Jr.dr = 0


('

Ex.7.12.6: If'f and 'g' are scalar functions, show that


Jf( grad g). dr =- Jg(grad j).dr

ww ('

Sol: By stoke's theorem,


('

w ('
.Ea
J{grad (fg)·dr = Jficurl{grad(fg)}ln ds = 0, since curl grad (fg) = 0
S

.. J{grad (fg)}·dr =0
(' syE ..... (1)

But grad(fg) =
ngi
f(grad g) + g(grad j) ..... (2) /

Ex.7.12.7:
Hence the result
nee
[from (1) and (2)]
If A is any vector function, prove by stoke's theorem that div curl A = O.

rin
Sol: Let V be any volume enclosed by a closed surface S. Then by Gauss' divergence
theorem. We get,
JJJV.(curl A)dv = JJ(curl A).nds g.n
e ..... (1)

t
Divide the surface S into two portions SI and S2 by a closed curve C.
Then JJ(curl A ).n ds
= JJ(curl A).ndst + JJ(curi A).n dS 2

= JA.dr - JA.dr
c c

Downloaded From : www.EasyEngineering.net


Downloaded From : www.EasyEngineering.net

Vector Differentiation 609

= 0, by Soke's theorem, since the +ve directions along the boundaries ofS, andS 2
are opposite.
:. (I) ~ fffV.(clirl A)dv=O

Since this is true for all volume elements V, we have,


V . curl A = 0 ~ div(curl A) = 0
Ex.7.12.8: Use Stoke's theorem and prove that curl gradJ= 0, where 'f' is a scalar
function.
Sol: IfS is a surface enclosed by a simple closed curve C, we have, by Stoke's theorem,

ww H{curl(grad J)}.nd.. = f(grad J).dr .... (I)

w
('

Now, grad! dr
.Ea
=(8f i + aJ j + 8f k ).( dx i + dy j + dz k)
ax 0' az
syE
= aJ dx+ 8f dy+ 8f dz=dJ
ax ay az
A
n gin
:. f(grad J).dr = fdf
(' A e eri
= f(A) - f(A) = 0, where A is any point on C
:. (1) ~ ff{curl(grad f)}·nds =0 ng.
Since this equation is true for all surface elements S,
n et
we have, curl (grad j) = 0
Ex.7.12.9: Veri1)r stoke's theorem for A = yj - 2xyj taken round the rectangle bounded
by x = ±b, y = 0, y = a
y
F~ __~__~y=a
______~B

x=b
x=-b

--____ ~----~--~----~~--L---~~X
D A y=O

Downloaded From : www.EasyEngineering.net


Downloaded From : www.EasyEngineering.net

610 Engineering Mathematics - I

i j k
Sol: curl A = ajax ajay ajaz = - 4yk
y2 -2xy 0

For the given surface S, n = k


:. (curl A). n = - 4y
Hence ff(curIA).n ds

a a

ww
h h U

= ff- 4 ydxdy= f[ f -4ydx]dy= f -4xyl dx= f- 8bydy


~h
s y=o J.=~h 0 0

w .Ea = -4by 2/ =-4a 2b


o
a
..... (I)

J
fA.dr
C
= f+ f+ f+
DA AB BF syE
FD

fA.dr = y2 dx - 2xydy
ngi
AlongDA, y = 0, dy = 0, => JA.dr=O
DA nee ( .: A.dr = 0)

AlongAB, x= b, dx = 0
rin
:.
a
2
f A.dr= f-2bydy=-by2/: =-a b
g.n
AB

AlongBF,
-b
y=O

y=a,dy=O e t
:. JA.dr= Ja 2
dx = -2a2b
BF b

Along FD, x = -b, dx = 0


o 0
2
:. fA.dr= f 2bydy=-bi/ =-a b
FD a a

:. fA.dr= 0 - a 2 b - 2a2 b - a 2 b = - 4a2b ..... (2)


C

From (1) and (2), fA.dr = ffcurl(A).nds


c s
Hence the theorem is verified

Downloaded From : www.EasyEngineering.net


Downloaded From : www.EasyEngineering.net

Vector Differentiation 611

Ex.7.12.10: Use stoke's theorem to evaluate the integral fA.dr where A = 2y.i +3xY-
('

(2x +z)k, and C is the boundary of the triangle whose vertices are (0,0,0), (2,0,0),
(2,2,0).

) k
Sol: Curl A = ajax ajay ajaz
2
2/ 3x -2x-z
= 2} + (6x - 4y)k

ww
Since the z-coordinate of each vertex of the triangle is zero, the triangle lies in the
xy-plane.

w
:. n = k.

.Ea
:. (curl A). n = 6x - 4y
consider the triangle in xy-plane.
syE
Equation of the straight line OB isy = x
By Stoke's theorem,

f
n gin
c
f A .dr = f(CurlA)nds
s e o
(0,0)

eri
A(2,O)

ng.
n et
2

I 2

= f6xy - 2/ dx = f(6x - 2X2 )dx = 4 ~


x=O 0
2
3

1= 332 .
Ex. 7.12.11 Use Stoke's theorem to evaluate ff(curlA)nds, where A = 2yi + (x- 2zx)j
+ xyk, and S is the surface of the sphere .xl + Y. + z2 = b 2 above the xy-plane.
Sol: The boundary C of the surface S is the circle.xl + y. + z2 = b2, Z = o.
The parametric equations of C are x = bcose, y = bsine, z = 0, 0 ~ e < 27t
By Stoke's theorem, we have,

fsf(CurIA).nds =cf A.dr

Downloaded From : www.EasyEngineering.net


Downloaded From : www.EasyEngineering.net

612 Engineering Mathematics - I

= f2ydx+(x-2zx}dy+xydz = f2ydx+xdy (0: z=O,dz=OonC)


(' ('

2"
f(2bsin8)(-bsin8)d8+bcos8.bcos8.d8 [-: x = bcos8 => dx = - bsin8 d8
o
and y = bsin8 => dy = bcos8 d8]

= h 2 ](cos 2 8 -2sin 2 8~8 = b 2 2][1 + cos 28 - (l-cos28)18


o 0 2 r
ww t h221t b2 [
= - r(-1+3cos28)=+- -8+
3sin28]- b2
=--.2n=-nb 2
J"

w
Ex.7.12.12
2

.Ea
222
o

Apply Stoke's theorem to evaluate f A.dr .where A = (x- y)i + (2y + z}j +

syE
(y-z)k and C is the boundary ofthe triangle whose vertices are (~, 0, 0) (O,~, 0)
(O,O,~)
ngi z
and
nee
Sol: LetA= (~,O,O) B= (O,~,O) C= (O,O,~). rin
The equation of the plane ABC is (by intercept form),
g.n y

x y z
1/6 + 1/3 + 1/2 = I =>6x+3y+2z= 1
The direction ratios of the normal to (1) are 6, 3, 2
e t
..... (I)

.. . 6 3 2
:. D IrectlOn cosmes are -,-,-
777

Ifn is the unit normal to the plane, n = §..i +l j +3. k


7 7 7
A = (x - y)i + (2y + z}j + (y - z)k
i j k
VxA= a/ax aj8y a/az =k
x-y 2y+z y-z

.. (Vx A}.n =3.


7

Downloaded From : www.EasyEngineering.net


Downloaded From : www.EasyEngineering.net

Vector Differentiation 613

:. By Stoke's theorem,

fA.dr = Ifs (VF).n ds = ~7sIfds = ~7


('
(Area of triangle ABC) ..... (2)

To find the area of triangle ABC:

AB=
(H+(H ~ ~
ww AC= (H+(H ~~
!,
w Direction ratios of AB are -1,

.Ea
Direction ratios of AC are
6 3
-1 2
15' 15'
0 .

. . ratios
Direction
syflO'E , flO
_-1
. 0 f AC a r e- .~-
0- ..

cos CAB = (
n
Ts )(~ ) )sogin +0 +0 =

sin CAB = .:i.-- e eri


EO
.JlO
ng.
:. Area of triangle ABC = 1. AB.AC sin CAB = 1.. 15
2 2 6
._7_ = 2.
6 J50 72 net
JA.dr=~x-2..=_1 [from(2)]
(' 7 72 36
Ex.7.12.13Evaluate If(curl A).n ds taken over the portion s of the surface
s
X2+/+Z2_2/x+ft=0 above the xy plane z 0, if
A = ~::CX2 + /-z2)i and verity Stoke's theorem.
Solution: Let'S' denote the portion of the surface, x 2 + y2 + Z2 - 2ft + ft = 0 above
the xy-plane z= O.
The surface S meets the xy-plane in the circle 'C', whose equations are
2
x + / - 2ft = 0, Z = O.

Downloaded From : www.EasyEngineering.net


Downloaded From : www.EasyEngineering.net

614 Engineering Mathematics - I

:. The parametric equations of 'C' can be taken as

x == j + j cosS, y== jsinS, z=o (0~S<2n;)

Let S) denote the plane region bounded by C.lfS) is the surface consisting ofS and
SI' SI is a closed surface.
:. From example 7.11.8 on divergence theorem, we have

ww =0

w.E
i.e., f f(Curl A).2 ds + f f(Curl A).n ds = 0
s
[.: S) consists of Sand S)]

asy
I.e., f f(Curl A).n ds - f f(curIA).kds == 0
s
En '"

s gin
f f(Curl A).nds = f f(curIA).kds ..... (I)

eer
Now, curl A= a/at a/ay
j k
a/az ing
.ne
:. (curl A). k
=

= -
i(- 2y - 2z) + j(-2z - 2x) + k(- 2x - 2y)
2(x + y)
t
:. From (I), ff{curIA).nds=-2 f f(x+ y}is
. " .'I'
Polar equation ofS) is r = ~cosS.

:. changing to polar coordinates,


It 2/eos9

ff(curlA).nds=-2 f f(rcosS+ rsinS)rdedr


s, 9=0 r~O

Downloaded From : www.EasyEngineering.net


Downloaded From : www.EasyEngineering.net

Vector Differentiation 615

8(,1t 16f31t 1t
3 4
= - 2 x -'- f(eosO + sin e )eos 8de = --3- feos OdO [.: feos 3 Osin Od8 = 0]
3 0 0 o

16f3 1t,12 4 32f3 3 I n .3


= - - x 2 feos Od8=---.-x-x-=-2nj ..... (2)
3 0 3 4 2 2

Again, fA.dr
('

f(x 2 + i - Z2 }h: + (i + Z2 - x2~y + (Z2 + x2 -- y2 )1z


wwf(x i }Ix x
=
C

w.E
=
C
2
+ + (y2 - 2)1y
[.: on C z = 0, dz = 0]

ftU 2"

asy + f cos 8)2 + f2 sin 2 e](- fsine)de


o
E +
21t
ngi
f [f2 sin 2e- (f + f cos8)21f cose)de
fffiki +
o
n(I ee
cosS)' + sin' S}(- sinS) + {'in'S""
rin
+ cosS' )}coril]nIl

f3[2I_ 2sin ede +2I_ 2sin 2ede.+ 2I_ 2cosed8 + 2I_ cos g.n
e
= 3 eaU

21t
0

21t 2
0

1
0
t
- f(I+Cos2e)de+ fSin ecos8de (on simplification)
o 0

= f'(-2n)=-2nf3 (all other integrals vanish) ..... (3)

from (2) and (3), we have

f f(CurIA).nd\· = fA.dr
s ('
which verifies Stoke's theorem.

Downloaded From : www.EasyEngineering.net


Downloaded From : www.EasyEngineering.net

616 Engineering Mathematics - I

Exercise-7(1)

1. f F=(xe )i+(3/)j-(z)dz,and
X 2
C is the X +y2=9,z=2, evaluate fF.dr
('

using Stoke's theorem.


[Ans: OJ

2. Apply Stoke's theorem to obtain the value of the integral fV.dr ,where
('

ww
v = (3/)i + (2X2)j -(x+ 2z)k and C is the boundary of the triangle whose
veltices are (0,0,0),(1,0,0) and (I, I ,0)

3.
w.E
Verify Stoke's theorem for F = (2x - y)i - (yz 2)j - (/ z)k if S is the upper halt
[Ans: I]

asy
surface of the sphere x 2 + y2 + Z1 = I and C is the boundary.

En [Ans: Jr]
4. If F = (y - z
gin
+ 2)i + (yz + 4)j - (xz)k and S represents the surface of the cube
x = 0, y = 0, z = 0, x = 2, y = 2, z = 2 above the xy plane , verify that
f e eri
ff(Curl F).ds = F.dr ,C being the boundary of S traversed in the +ve direction.
s c

ng.
[Ans: Each integral =-4]

5. Find the value of the integral f(yz )dx + (zx )dy + (xy )dz ,using Stoke's theorem
c
where C is the Curve x 2 + y2 = 4, z = y2
net
[Ans:O]

Verify Stoke's theorem for the function V = (3x )i + (2xy)j , integrated along the
2
6.
square x = 0, y = 0, x = I,y =) in thexy-plane.. [Ans: 0]

7. Evaluate by Stoke's theorem the integral where


c
A= (2sinz)i-(3cosx)j+(siny)k; where C is the boundary of the rectangle
o~ x ~ Jr, 0 ~ Y ~ 1, z = 0
[Ans:6]

Downloaded From : www.EasyEngineering.net


Downloaded From : www.EasyEngineering.net

Vector Differentiation 617

Exercise - 7(m)
1. If !(x,y,z)=x 1y lllzn-1 , find the directional derivative of f at (1,1,1) in the
direction of (i+2j+2k)
1
[Ans: -(I + 2m+ 2n)]
3
2. Find the acute angle between the surfaces

x 2 + i + Z2 = 6 and 3xyz + i Z- xy + 3 = °
at (1, -I, 2)

ww IAns: cos'( ~}

3.
w.E
If r = xi + yj + zk , and p, q are constant vectors, show that
Div{(rx p)xq} = -2(p,q)
asy
4.
En
If F=(x 2y)i-(iz)j+(z2x )k,findcurIFat(1,-2,3)

S.
gin
If !=xyz(x+y+z),provethat curl gradf=O

6. A fluid motion is given by V=(z3)i-(i)j+(3xz 2 )k. Show that it


irrotational. Find its velocity potential rjJ sllch that V eer
= V rjJ
IS

ing 3
[Ans: rjJ = xz 3 _l_ + c ]

.ne 3

7. Evaluate
(2,1)

J(4x -12x 2i)dx-(8x y)dy along the path x 3-3xi =2/


(0,0)
3 3
t
[Ans: 16]
2
8. If V=(xy)i-(yz)j+(zx )k and S is the surface of the cube bounded by
x = 0, x = 2, y = 0, y = 2, z = 0, z = 2 , evaluate JV.n ds
s
[Ans: 32/3]

9. If ! = 4x + yz , evaluate HJ!dv over the region in the first octant bounded by


v
x-, + y 2 = 1, z = 0, z = 3 [Ans: 11/2]

Downloaded From : www.EasyEngineering.net


Downloaded From : www.EasyEngineering.net

618 Engineering Mathematics - I

10. Express F = xi - i j + zk in (a) cylindrical polar coordinates (b) spherical polar

coordinates.

+(r sin OcosOcos 2 ¢ sin 2 OcosBsin 3 ¢ - rsinBcosB)eo


ww _,.2

+ (-rsinBsin¢cos¢-r 2 sin 2 Bsin 2 ¢cos¢)e~]


w.E
Iff = pz sin 2B ,find grad f in cylindrical coordinates
II.
asy
En
[(zsin 20)ep + (2zcos 2B)eo + (psin 2B)ez

12. If A
r gin
= (rcosB}er -(!sinB)eo +re~, find the curlA in spherical coordinates

e eri
[Ans: (cotB)e r +2eo -(2sinB)e~]

13. Verify Green's theorem in the plane for f(x 3 ng.


y2)dx + (x 2 - 2xy)dy ,where C is

net
-
('

the boundary of the squart: bounded by O:S x:S 1, O:s y:s 1 [Ans:1]

14. Using Gauss divergence theorem, prove that If F.n ds = TCr


2
P, where
s

f = (y2 Z ); + (xz)j + (z2)k ,and S is the surface bounded by

x 2 + y2 = r2 , z = 0, and z = I

15. Show that the Stoke's theorem, when restricted to the xy-p/ane, is Green's theorem

in the plane.

(Hint: In Stoke's theorem, take A = Pi + Qj; n = k; and ds = dxdy)

Downloaded From : www.EasyEngineering.net


Downloaded From : www.EasyEngineering.net

Vector Differentiation 619

Exercise-7(n)

I. Choose the correct answer in the following questions

The tangent vector at the point t=1 on the curve x = t + 1, y = 4t - 3, z = t 3 is


2
I.

(a) 2i-4j+3k (b) 2i+4j+3k (c) 2i-4j-3k (d) 2i+4j-3k [b]

2. The magnitude of acceleration at 0 = 0 on the curve


x = 2cos30,y = 2sin 30,z = 30 is

ww (a) 6 (b) 9 (c) 18

1= xyz, the value of Igrad II at the point (I, 2, -I) is


(d) 3 [c]

3. if

(a) 0
w.E (b) I (c) 2 (d) 3 Cd]
- 4. asy
-, The maximum rate of change of f = x/ + yz + zx 2 at the point (I, I, I) is

Jli En Cd]
gin
(a) (b) 0 (c) 3 (d) none

5. The angle between the normals to the sphere x 2 + y2 + Z2 = 9 at the points (1,2,
2) and (2, 1,2)is
e eri
1C
(b) -
2 ng.
(d) -
1C

4
[a]

6. If a is a constant vector and r = xi + yj + zk , then '\l(a.r) is net


(a) 0 (b) a (c) r (d) r [b]

7. If (x + 3y)i + (2 - 3y)j + (x + az)k is a solinoidal vector, the value of a is

(a) 0 (b) 1 (c) 2 (d) 3 [c]

8. If r = xi + yj + zk, and a = ~ r, div ii =


3

(a) 0 (b) I (c) -1 (d) 2 [b]

Downloaded From : www.EasyEngineering.net


Downloaded From : www.EasyEngineering.net

620 Engineering Mathematics - I

Exercise - 7(0)
I. Iff(x,yx) = X lymz"_I, find the directional derivative of fat (1,1,1) in the direction of
I
(i+2j+2k) [Ans: "3 (I + 2m + 2n)

2. Find the acute angle between the surfaces x 2 + y +:? = 6 and

3xyz + yz - xy + 3 = 0 at (1,-1,2) [Ans: cos


_,(Fs]
3]

ww
3. If r = xi + yj + zk, and p, q, are constant vectors show that

w.E
Div {r x p} x q} =- 2(p.q)
4. If F = (ry)i - (y2z)j + (:?x)k, find curl curl F at (1,-2,3)

a syE
5. Iff= xyz(x + y+ z), prove that curl gradf= 0
[Ans: 6i - 2j + 4k]

ngi
6. A fluid motion is given by Y = (z3)i - (y2)j + (3xz 2)k. Show that it is irrotational.
+such that Y =
Find its velocity potential V~

nee y3

rin
[Ans: ~ = xz 3 - -
3
+c ]

7. Evaluate
(2,1)
3
f{4x -12x2 y2 )Ix - {8x y}Jy
3
along the path x 3 - 3xy g.n
2;
et
=
(0,0)

[Ans: 16]
8. IfY = (xy)i - (yz}j + (zr)k and S is the surface of the cube bounded by x = 0, x =

2 , y=O " y=2 z=Oandz=2 , evaluate f V .nds


s
[Ans: 32/3]

9. If f = 4x + yz, evaluate f f ffdv over the region in the first octant bounded by
v

x 2 + y2 = I, z = 0, z = 3

[Ans: 1112]

Downloaded From : www.EasyEngineering.net


Downloaded From : www.EasyEngineering.net

Vector Differentiation 621

10. Express F = xi - y.j + zk in


a) cylindrical polar coordinates
b) spherical polar coordinates.
[Ans: a) p(cos28 - pSin38)e p - psin8cos8(1 + psin8).ea + ze z
b) (rsin 28cos2<jl- ~sin30sin3<jl + rcos20)e r + (rsinOcosOcos2<jl- ~sin20cos8sin3<jl­
rsin8cos8)ea + (- rsin8sincj>coscj> - ~sin28sin2cj>coscj»e.]
11. If/= pzsin28, find grad/in cylindrical coordinates

ww [Ans: (zsin28)e p + (2xcos20)e a + (psin28)e z]

w
12. If A = (rcosO}er -(-;,sin 0

.Ea
)eo + re4>, find curl A in spherical coordinates
[Ans: (cot8)e r + 2eo - (sin8)e.]

syE
13. Verify Green's theorem in the plane for J(x 3 -l )tx + (x 2 - 2xy}ty, where C is the

boundary of the square bounded by 0 ~ x


c
ngi ~ I, 0 ~ Y ~ I
nee [Ans: 1]

14. Using Gauss' divergence theorem, prove that J rin1tr2[2 ,


JF.nds = where
s
g.n
F = (y2 Z} + (xz)j + (Z2 ~ , and S is the surface bounded by
z=l e
x2 + y2 = r2 , z =0, and
t
15. Show that the Stoke's theorem, when restricted to the xy plane, is Green's theorem
in the plane
[Hint: In Stoke's theorem, take A = Pi + Qj; n = k; and ds = dx dy]

Downloaded From : www.EasyEngineering.net


Downloaded From : www.EasyEngineering.net

ww
w.E
a syE
"This page is Intentionally Left Blank"
ngi
nee
rin
g.n
et

Downloaded From : www.EasyEngineering.net


Downloaded From : www.EasyEngineering.net

8 ww
w .Ea
Laplace Transforms
syE
8.1 ngi
Laplace Transforms

Introd uction nee


r ing
The theory of Laplace transform is an essential part ofthe mathematical background
required by engineers, physists and mathematicians. It gives an easy and effective

.ne
means for solving certain types of differential and integral equations. It is the foundation
of the modern form of operational calculus, which was originated in an attempt to
justify certain operational methods used by an electrical engineer Oliver Heavinide, in
the latter part of the 19th century for solving equations in electromagnetic theory.
t
The Laplace transform reduces the problem of solving a differential equation to
an algebraic problem. It is particularly useful for solving problems where the mechanical
or electrical driving force has discontinuties, acts for a short time only or is periodic
but not merely a sine or cosine function.

8.1.1 A class of linear transformations of particular importance is that of the integral


transforms. The Laplace transformation (L.T) is a special case of an integral
transformation and is defined h~'
cO

F(s) = L [f{t)1 = Je-stf(t) dt


o

Downloaded From : www.EasyEngineering.net


Downloaded From : www.EasyEngineering.net

624 Engineering Mathematics - I

Def. Piecewise continuity ofa function:


A function off(t) is sectionally continuous (Piecewise continuous) in an interval
o~ t ~ b iff(t) is continuous over every finite interval 0 ~ t ~ b except possibly at a
finite number of points, where there are jump discontinuties and at which the
function approaches different limits from left and right.

f(t)

ww a

w.E
Sectionally continuous function
Def. asy
Functions of exponential Order:-

En
A function f(t) is said to be of exponential order cr, as t ~ 00 if 3 M > 0

gin
3/f(t)/ < Meat, \;f

t ~ o.
t~0 .... (1)
or equivalently, le--crtf(t)1 ~ M, \;f

eer
t2
Remark: eat and sinbt are of exponential order, while e
ing
is not of exponential order since

.ne
8.1.2
which can be made larger than any given constant by increasing 't' indefinitely.
Conditions for L. T to exist
By using above remarks we can prove the conditions that are sufficient for the
t
existence of Laplace transforms.
Theorem: Iff(t) is piecewise continuous in every finite integral [0, N] for N > 0,
and of exponential order cr,that is If(t)1 ~ Meat, \;f t ~ 0 and M> o. (from I)
then the Laplace transform ofF(t) exists for all s> 0'

00 00

Proof: LetI = /F(s)1 = Je-stf(t)dt ~ ~ f(t)/ e-stdt ~ JM eate-stdt


o 0
00

0
=-=-s
M
cr
Here, s > 0' since I is non-negative everywhere. This shows that above conditions
are sufficient for the existence of L [f(t)].

Downloaded From : www.EasyEngineering.net


Downloaded From : www.EasyEngineering.net

Laplace Transforms 625

Properties of Laplace Transforms


8.1.3 Linearity property: Let f,(t) and fit) be two functions on [0, co] such that the
Laplace transforms L[f,(t)] and Lltit)] exist. If k, and k2 are two constants,
then
L[k/,(t) + klit)] = k,L[f,(t)J + k2 L[fit)J ..... (i)
This property is valid since

00 00 00

=
f[ktf; (t) + k 2 f2(t)]e-'1 dt fk\.!; (t)e-'I elt + fk2.f; (t)e -,I lit = k\L[j; (I)] + k 2 L[f2(t)]

ww
0 0 0

This result can be extended to the linear combinations of more than two functions.
~

8.1.4
w
Iff(t) is piecewise continuous on [0, N] for each N > 0, and ttt) is of exponential

.Ea
order 0", that is by 8.10.1 (1), If(t)1 ::s Meat, V t ~ 0 and M > 0 and ifL [f(t)] = F(s)
then we have
(1) L [eatf(t)]

,
(2) L[f (tn. e-aSF(s)
syE
= F(s-a) V s > 0" + a va> 0 ..... (i)

ngi
=

f(t -a) t > a}


where f,(t) = { 0 t < a (a>O)
nee
and (3) L (ttat» = ±F(~) (for any a> 0)
rin
Proof(l)
_a_ g.n
e t
--~------~----~~s

From definition, we have


00

L[f(t)] = fe-Mf(t) dt = F(s)


o
00 00

so that L[eatf(t)] = fe-ot[eal f(t)dt = fe-(s-a)1 f(t)dt = F(s - a)


o 0

Downloaded From : www.EasyEngineering.net


Downloaded From : www.EasyEngineering.net

626 Engineering Mathematics - I

Th is property shows sh ifting on the s-axis and is called the first shifting property.
This means that replacing s by s - a in the Laplace transform corresponds to
multiplying the original function by eat
00 a 00

(2) L(fl(t) = fe-Ilf; (t)dt = fe-'If; (t)dt + fe-sl.t;(t)dt


o 0 a
a 00

= fe-.II.Odt+ fe-lIf(/-a) dt from definition offl(t)


o a

ww
00 00

= fe-·llf(t-a)dt== fe-I(p+a)f(p)dp (where t = p+a)


o 0

w.E 00

= e -a.1 fe -P' f(p )dp == e -a.1 F(s)

asy o
This property indicates shifting on the t-axis and is called the second shifting property

(3)
00

En d
L[f(at)] = fe-'I f(at)dt == fe-·I(p/a) f(p)-'£
00

[taking t = pia]
o
g
~ }e-IP/af(p)dP==~F(~) in
o a

a0
=
a a eer
This property is known as change of scale property.
ing
8.1.5 Laplace Transforms of standard functions
I .ne
(a)

Pro()f:
L (e-at ) = --

r
we have L(e-a) = Je
s+a

t -.1/
.e -aidt-
_ rJe -(Ha)1
t--- e
(s> a)

d _ - 1 [ -(Ha)I]OO
0
_ 1
---
t
...... ( I)

o 0 s+a s+a
. 1
Ifa=OtheL(I)= - (s> 0) ..... (2)
s
Also changing the sign of , a' we get
I
L(eat ) = -- ..... (3)
s-a
a s
(b) L(sinat) = 2 2 and L(cosat) = J 2 (s> 0)
S +a s- +a
1 s+ia
Pro()f: L(cosat+isinat) = L(e lat ) = --.- ==-)-- from (a)
S -lU s- + a 2

Downloaded From : www.EasyEngineering.net


Downloaded From : www.EasyEngineering.net

Laplace Transforms 627

Equating real and imaginary parts we get


a s
L(sinat) = 2 2 ,L(cosat) = 2 2 ..... (4)
S +a s +a
a s
(c) L(sinhat) =
S
2
-a
2 and L(coshat) =
S
2
-(I
2 (s> laD

Proof:
.
L(smhat)=L
(e _e-
OI
1
ol
01 11 -01
=-L(e )--L(e )
222

1[1 I]
ww - -
2 s-a
-----
s+a
from (a)

w.E
a
Thus (sinhat) = 2 2
S -a

a
similarly we can show that L(coshat) =

I(n + 1) syE s
2
s
-(I
2 •••. (5)

(d) L(tn) = ----;+]


s
[(n + I) > 0 and s> 0]
ngi
Proof:
nee
OOfe-P.pnd =T(n+l) rin
o S
n+1 'P
S
11+1

g.n
Since T(n+I)= fe-xxndx
00

o
and T(n+ I) = n! ifn is a +ve integer
et
n!
:. L(tn) = ----;;-+J ..... (6)
s
From (6) we have
1 1
L(I)= -, L(t) = -2 ,
s s
Solved Examples

8.1.6 Find the Laplace transform of


1. 2t3 + cos4t + e-2t 4. 3t2 + e- t + sin 32t
2. e2t + 4t3 - sin2tcos3t 5. cos32t
3. cos(wt + ~)

Downloaded From : www.EasyEngineering.net


Downloaded From : www.EasyEngineering.net

628 Engineering Mathematics - I

6 s 1 12 s 1
Sol: I. L(2t3 + cos4t + e-2t ) = 2x-+ + - - =-+ +--
S4 S2 + 16 S + 2 S4 S2 + 16 S + 2
2. L(e2t ) + 4L(t3 ) - L(sin2tcos3t)

I 6 (5
S - 2 + 4x7 - S2 + 25 - n 2 + 1
1)
1
[.: sin2tcos3t = "2 (sin5t - sint)]
3. L cos(wt + 13) = L[coswt.cosj3 - sinwtsinj3]

ww =cosfJL(coswt)-sinfJL(sinwt)= cos fJ
+w
L[3t2+e-t+sin 32t] = 3L(t2 ) + L(e-t ) + L (sin 32t)
S
2
S
2
.
-sm
fJ
S
2
w
+w
2

w.E
4.
2 1
=3.-+--+
3 3
- -?--

asy S3 s+1 2(S2+4) 2s-+36


2 1 3 1 3 1

En =-+--+--------
S3 S + 1 2 S2 + 4 2 S2 + 36

(.: L(sin32t) =
1
4" L[3sin2t - gin
sin6t]

5.
3
L(cos 2t) = L
[cos6t + 3cos2t]
4 e eri
1 (s 3S) ng.
= 4" S2 + 36 + S2 + 4

Exercise 8(A)
net
Find the Laplace transform of
8 3 1
1. 4t2 + sin3t + e-2t Ans. - + - - + - -
S3 S2 +9 s+2
1 4
2. (sin2t - cos2tf Ans. -
S S2 + 16

3. Ans. !(!+_S_)
2 S S2 -64

4.

5.

Downloaded From : www.EasyEngineering.net


Downloaded From : www.EasyEngineering.net

Laplace Transforms 629

6. cos 2 (2bt) 1(1


Ans. - - + ? ?
2 s s- + 16b-
s)
162
7. Ans. -----::,---------:---
(S2 -81)(.~·2 -9)

a b
8. sinhat- sinbt An s. -?::-------::-)
s- -(r

S(S2 - 28)
9.
ww Ans. -----:----::---
2
(S2 - 36)(5 - 4)

10.
w.E
coshat - cosbt Ans.
S
2
s
-(I
2
s

8.1.7 asy
Examples on properties 8.1.4
Find the Laplace transform of
En
1. e-btcosat
4. e-3t sin3t
2. t 3 e-4t
gin
5. e-2t[2cos3t-3sin3t]
Sol.
S
e eri
1. L( cosat) =)
s- +0
2

ng.
net
(from 8.1.4( I) first shifting theorem)

6
2. L(t3 ) = 4
s
By shifting theorem 8.1.4(1)
6
L(e-4t. t 3) = - - -
(s + 4)4

. 3
3. L(sm3t) = -2--
S +9

by 8.1.4(1),
3
2t
L( e sin3t) = (5 _ 2)2 + 9.

Downloaded From : www.EasyEngineering.net


Downloaded From : www.EasyEngineering.net

630 Engil~eering Mathematics - I

4. e-3t .sin3t

L(sin3t) =- L [~(3Sin
4
1 - Sin3/)] = ~ L(sin I) - ~ L(sin 3/)
4 4
3 1 1 3

By 8.1.4(1)

-3t . - ~ I
L(e sm3t) - 4· 3)2
(s+ +I
5.
ww e-2 t(2cos3t - 3sin3t)

L(2cos3t) = 2.-2-
s

w.E s +9
9
L(3sin3t) = ~9
8 +
asy
By shifting theorem 8.1.4( I)
2L(e-2tcos3t) - 3L(e-2t sin3t)
En
= (s
2(s + 2) 9
+ 2)2 + 9 - (8 + 2)2 + 9 gin
8.1.8 If L[f(t)] = ~9 .
3s
+
find L[f(3t)] e eri
ng.
8

38
SoL 'L[f(t)] = S2 +9

I
s
3") 38
net
By 8.1.4 (3), L[f(3t)] = 3(8)2 8
2
+27
- +9
3

Exercise 8(8)
Find the Laplace Transform of
2 8-3
1. Ans. 4cosa +sin-----,:-----
(s - 3)2 + 16 (8 - 3)2 + 16

2.

3. cosh2t.cos2t Ans ~[ 8-2 + __8_+_2_]


. 2 (8 - 2)2 + 4 (8 + 2)2 + 4

Downloaded From : www.EasyEngineering.net


Downloaded From : www.EasyEngineering.net

Laplace Transforms 631

a(s2 + 2( 2)
4. coshat sinat Ans.
S4 + 2a 4
S(S2 - 7)
5. cos3t.cosh4t Ans.
S4 + 625 -14s 2
2s-5
6. e- 21(2cos3t-3sin3t) Ans.
S2 +4s+ 13
S3
7. coshat cosat Ans.
S4 + 4a 4

ww
8. elsin4t.cos2t Ans.
3
(s _1)2 +36

w.E Ans.
5s 2 -3s+2

asy n+ 1
S3

En Ans.
(8 + a)"+1 + (.\'2 + 1)(s2 + a)

8.1.9
gin
Property (4) : Effect of multiplication by t
If L[f(t)l = F(s) then L[tf(t)] = -FI(s)
00
eer
Proof: We know that F(s) = Je-"f(t)dt.
o
ing
differentiating with respect to s
00 a 00
.ne
F(s)= J-[e-"f(t)]dt= Je-"'f(t)dt
o as
00
0
t
:. -FI(s) = Je-" [if(t)]dl = L[if(t)]
o
8.1.10 Examples

I. - -3- L(Ie -2')_


L( e-2')_ d 1 _
--------_o_ 1
s+2' ds s + 2 (s + 2)2

2,
d d ( 2
L(lcosat)=(-I)-[L(cosal)]=-- 2 = 2
2S2
2 2
s)
ds ds s + a (s + a )

3. L(t sin I) = __d


d..
(_I_) =
S2 +1
---,-2_s---,-
(S2 + 1)2

Downloaded From : www.EasyEngineering.net


Downloaded From : www.EasyEngineering.net

632 Engineering Mathematics - I

a
4. L(atsinat-cosat)=aL(tsinat)-L(COsat)=a[-dd ( 2-
S S +a
2)]- s 2+a 2
S

2
2as] s 2a s s
= a [ (S2 +a2f~ - (S2 +a 2) = (S2 +a 2 )2 - (S2 +l?)
5. L(1+tet)3 = L[I+3et+3t2e2t+t3e3t]
= L(I) + 3L(tet)+ 3L(t2e 2t) + L(t3e3t)
2 3
1 d 1 d 1 d ( 1 ) '1 3 6 6
=~-3 ds (s-I) +3 ds 2 (s-2)ds 3 s-2 =2+ (s-1)2 + (S-2)3 + (s-3)4
ww
6.
w.E
asy
d En
L(te4t sin2t) = -L[e sm 2/1 = - -
41' d [ 2 ] by shifting property

J gin
7. 2
ds ds (s-4) +4

=-
d [ 2
ds S2 -85+20
eer 45 -16
= (S2 -8s+20)2

8.
d
2
d [
2
s -1 ] ing
L(t2etcos4t) = ds2 L(e cos4/) = d~2 (s _1)2 + 16 by shifting theorem
l

=_~[ 2 .ne
t
3
S2-2S-15]= (2s -6s -90s+94)
ds (S2 - 2s + 17)2 (S2 - 2s + 17)3

Exercise 8(C)

Find the Laplace transform of the following:

1. Ie-"'sjn2. Ans. H~ (s 3)' - (X~~::15;),]


6s-18
3t
I 2. te- sin3t Ans. (S2 _ 6s + 18)2

2s+4
3. Ans, (S2 + 4s + 5)2

Downloaded From : www.EasyEngineering.net


Downloaded From : www.EasyEngineering.net

Laplace Transforms 633

9+ 12s -3s 2
4. 2tsin3t-3tcos3t Ans. 2 2
(s +9)

35-3
5. tetsin2tcost Ans. 2 ?
(s - 2s + 10)-

8.1.11 Property (5) If L[f(t)] = f(s), then L U.f(t)] = J/(5) ds


,

ww
Proof:
00

f(s)= fe-'I/(t)dt

w .Ea
Integrating bothsides w.r.t 's' between the limits s,
integration on right hand side (R.H.S),
00 and changing the order of

J/(S)ds
"tL
= syE
'1Je-.\II(t)dS]dt = J-[e~\1 l(t)] 00 dt = Je-\I I~t) dr
0

ngi 0 0

nee
8.1.12 Examples
rin
Find the Laplace Transforms of the following

~(1-cosal) ~(e-I sinal) g.n


1.
I
2.
I
e t
1 s
Sol.1. L(I-cosat) = L(l) - L(cosat) = - - 2 2
S S +a

(l I s
)K
L -(I-cosat) = - - 2 ] [
1 2
2 ds= logs--log(s2+a) ]00
1
=-Iog(2? 1
s + a-
2
t , s s +a 2 0 2 s

a
2. L(e-tsinat) = 2 2
(s+l) +a

I ISlOat)
L -(e-
[t
. ]
= oof a2 [-1
(S+I)]OO
2ds= tan - 1t
- =--tan s+1
--=cot _I(S+I)
-- -1
s (s + 1) + a a ,2 a a

Downloaded From : www.EasyEngineering.net


Downloaded From : www.EasyEngineering.net

634 Engineering Mathematics - I

1 1
3. L(1-e2 t) = L(I) - L(e2t) = ---
s s-2

I
L[-(1-e
t
21]
) = ----
s s-2
X
I I)ds= [logs-log(s-2)Js100 =Iog--
(s - 2)
s
s

2
4. L(e-3tsin2t) = ---::---
(S+3)2 +4

L(!e- 3/
Sin2t)= } 22 ds=tan- I (S+3)00

ww _1(S+3) _1(S+3)
t s (s + 3) + 4 2 s

w.E 1t
=-=tan
2
- - =cot
2
--
2

5.
asy
L(eal ) = --,
1
s-a
I
L(e ht )= - -
s-b
X E
1 al -e bl]
L [ -(e
t
n) =
s
-I- - -1)
s-a s-b
- ds= [log(s-a)-Iog(s-bh,.

gin
100

=
[
s-a
log( - )]
s-b
00

s
s-a
s-b
(
s-b )
=-IOg(-)=IOg-
s-a e eri
Exercise 8(0) ng.
Find the Laplace transfonn ofthe following: net
1.
1
-(1-cost)
t
1 (hI]
Ans. 2"log - s -

I _I •
2. -e SlOt Ans. coc 1 (s+l)
t
1
3. - (-at
e - e-hI) Ans.log C+b)
--
t s+a

4. ! (e-3tsin2t) S-3)
Ans. coC I ( -2-
t

5. !
t
(sin2t) Ans.
1 (.'+4]
"4 log ~

Downloaded From : www.EasyEngineering.net


Downloaded From : www.EasyEngineering.net

Laplace Transforms 635

6.
I
- (coat - cosbt) I
Ans. -log (S2+b 2)
2 2
t 2 S +(1

7.
I
- (I - cos3t) I
Ans. -log (S2 +9)
--2-
t 2 S

8. ! (sin3t) Ans. 4"I [3cot


_)
S- cot
_)
"3s]
t
8.2.1
ww Theorem: Iff(t) is continous ¥ t ~ 0 and of exponential order, say 0" and has a
derivative ~(t) which is piecewise continuous on every finite interval [0, N] for
each N > 0, then the Lapalce transform of the derivative ~(t) exists for s > 0" and
w.E
L{~(t)} = sLf(t) - f(O)

Proof: L(f(t)] =
aje-'Isy
00

f(t)dt
o
00
En
L(~(t)] =
o
je-'Ifl(t)dt
gin
Integration by parts gives
00
e eri
L(~(t)] = [e-.'I f(1)1 + s je-'I f(t)dt
o ng.
Since f(t) is of an exponential order, the integrand in first integral on the R.H.S. is
zero at the upper limit when s> 0' and iff(O) at the lower limit. Thus, we have
00
net
L( ~(t)] = 0 - f(O) + S je -;t f(t)dt =' sLf(t) - f(O)
o
If Lt f(t) = f(O+) exists, but is not equal to f(O), which mayor may not exist then
1-)0

L(~(t)] = sLf(t) - f(O+).


Note: Iff(t) and its derivatives ~(t), ~I(t) .............. fI- 1 (t) are continous ¥ t ~ 0 and are
functions of exponential order for some M > 0 and 0', and the derivative fI(t) is
Piecewise continous on every finite interval in the range t ~ 0, then the Laplace
transform offl(t) exists when s> 0' and is given by
L[fI(t)] = s"[Lf(t) - s" - If(O) - s" - 2 ~(O) ............... _tn - 1(0)]
If n = 2, L [~I(t)] = s2[L(f(t»] - sf(O) -~(O) and so on.

Downloaded From : www.EasyEngineering.net


Downloaded From : www.EasyEngineering.net

636 Engineering Mathematics - I

8.2.2 Example

Prove that L(cosat) = S2 + a2

Sol. We can show this with the help of above theorem


Let f(t) = cosat => f(O) = 1
then [I(t) = -asint => fl(O) = 0
[l1(t) = -a2cosat

ww Hence, we have
L[fll(t)] = s2Lf(t) - sf(O) - fl(O)

w.E :. L[-a 2cosat]


2
= s2L(cosat) - s - 0

s2L(cosat) + a L(cosat) = s
= s2L(cosat) - s

:. L( cosat) = 2 asy
s
2

8.2.3
+a S

En
Theorem: Iff is Piecewise continous on [0, N] for each N > 0 and is of exponential
I I
gin
order cr, then L ff(u)du = -L[/(/)] v s > CT,

I
o s
eer
Proof. Let «p(t) = ff(u)du. Then we have «p1(t) = f(t) and f(O) = 0;
o ing
L[~(t)l =s.L[~(t)l-~(O) =s[L(~(t»l =SL[!f(U)du 1 .ne
III t
:. L Iff(u)du = -L[¢I(/)] = -[L(f(/»]
[
o s s
8.2.4 Examples
Find the Laplace transforms of
I

I. fsin 2pdp
o
I

2. fp cosh p dp
o
3. r
I

o p

mp
dp
I
rp .
sm P
4. je . - - d'P
o P
2
Sol. 1. L(sin2t) = Z--4 = f(s) , say
s +
112
fsin 2pdp =- f(s) = --=-2--
o S s(s +4)

Downloaded From : www.EasyEngineering.net


Downloaded From : www.EasyEngineering.net

Laplace Transforms 637

2.

I 2 )
I (s + (1-)
o J
then L pcosh pdp = - I(s) =- )
8
)
(8- -a-)s

3.
.
L(smt)= ~I
I
s +
; L
(I .
-SlOt
t
)
= J-2-
00

s +1
0\
I ds

ww
L 'JSin
w.E
p dOp=-Ijo() I -I
--
o P s
s =-cot
s
asy
s

4.
I
L(sint)= -2-1 En I
... L( etsint) = --.,,----
s +
gin (s _1)2 + I

I
L [ -ee'
t
sint) ] = ooJ
, (s
ds2
-I) +1 e eri
ng.
'f sin p 1 1 1
:. L e P --dp = - I(s) = -cot- (s -1)
net
o P s s

Exercise 8(E)

Find the Laplace Transform of


I I
I
I. Jcos2pdp
0
3
2. Jpe- sin 4 pdp
0
3. Jp sin 3pdp
I

s 8(s + 3) I _I
Ans. I. 2. 3. -cot s
S(S2 + 4) S(S2 + 6s + 25)2 s

Downloaded From : www.EasyEngineering.net


Downloaded From : www.EasyEngineering.net

638 Engineering Mathematics - I

Laplace Transforms of Periodic Functions


8.2.5 A function f(t) in said to be periodic with period p>O iff(t+p) = f(t) for all t>O
Since periodic appear in many practical problems and in most cases are more
complicated than single sine or cosine functions, we find it useful to establish the
following result:

I I'
L([f(t)] = I-e- Ps fe-Slf(t)dt
o

ww
Proof: Let f(t) be a periodic function of period 'p' so that f(t) = f(t+p) = f(t+2p) = ...... .

w.E
00 I' 21'
L[f(t)] = fe -si f(t)dt = fe -s/ f(t)dt + fe -si f(t)dt + ............... .
o o I'

writing t = u+p
asy
21'

En
I'
now fe-ollf(t)dt= fe-o,(u+p)f(u+ p)du
I' 0
gin
I'
= e- Ps fe-OVlf(t + p)dt e
(change u to t)
eri
o
ng.
= e -1"
I'

o
f -vlfCt )dt
Je 0 [.: f(t+p)=f(t)]
net
31' I'
similarly, fe-oIl f(t)dt = e- 21'S fe-ollf(t)dt and so on
21' 0

I'
L[f(t)] = (1+e-ps+e-2ps + .................... 00) fe-ollf(t)
o
The expression within parantheses is a G.P. with c.r. = e-PS

:. L[f(t)] = 1_:-1'$ Je-Slf(t)dt


o

Downloaded From : www.EasyEngineering.net


Downloaded From : www.EasyEngineering.net

Laplace Transforms 639

Solved Problems

8.2.6 Find the Laplace Transform of


t
l. f(t) =k- for 0 < t < P and f(t) = f(t+p)
P
2. f(t) = 1 for 0 < t < a and f(t) = -1, a < t < 2a and f(t) is periodic with period 2a
Sol. Since f(t) is periodic with period p.
P P k
L[f(l)] = Je-"I.~dt
ww 1. Je-s1f(l)dt=
l-e- sp
o
1
l-e-"P
0
p

w.E
asy
En
2. f(t) is peirodic with peirod 2a

1 2a si gin 1 [a 2a. 1
L[f(t)] = Je- f(t)dt =
1- e-2as 0 eer
1 _ e- 2as
Je-sl.l.dt + Je-·'I (-I)dt
0 0

_ 1 [{_e-SI}a
SI
{_e- }2aj_l 1 (l -a,)2 ing
-1_e-2a, - s - 0 - - s - 0 - s l_e-2as ' -e
.ne
- 1s [11+- ee -2as
--.
-2as
]_ 1[/f - e-'T --tanh
--.
S ~
1 [-as ]
s-~ 2
1- t
e 2 +e 2

8.2.7 Find the Laplace transform of the rectangular wave shown in the figure:

f(t)

1s
o b 2b 3b

Rectangular wave

Downloaded From : www.EasyEngineering.net


Downloaded From : www.EasyEngineering.net

640 Engineering Mathematics - I

Sol: The period of the given function is '2b'.

Hence L[f(t)]=
I 2h.
-2h, Je-Mj(/)dt=
I [ h .S1 2b
-2h.\ J1.e- j(t)dt+ J(-I)e-S1dt
1
l-e 0 l-e 0 b

hs -bs
- -
e 2 -e 2

ww
w.E I bs
=-tanh-

8.2.8
s 2
asy
What is the Laplace transform of the staircase function (fig(a)) given by

En
f( t) = n+ I, np < t « n+ 1)p for n = 0, 1, 2 ............ .
Sol.
gin
The easient way of getting the Laplace transform of the staircase function is to
consider it as the difference between the two functions shown in Fig(b) the

eer
transform ofthe linear function (t + p) can easily be found by using first shifting,
P
second shifting and change of scale properties. ing
Thus, we have
.ne
f(t) f(t)
t
o P 2p 3p

(a) Staircse function (b) solution of the stair case function (c) Saw-tooth function.

Downloaded From : www.EasyEngineering.net


Downloaded From : www.EasyEngineering.net

Laplace Transforms 641

t
The function fl(t) = - is called a saw-tooth function (fig(c» and its transform is
p
obtained as shown below.

1 P 1 1 [ -,I ]P
L[ft(t)] = _, fe-'I~d'= _.- _e_(_st_1)
1- e Ip p 1- e "P p S2
o 0

1- (1 + ps)e - P'
ps2(1-e-"P)

ww This can be simplified to

L[ I' (I)] - ~[1 + ps - p 1


w.E
JI - P S2

Thus from (1) and (2), we get


s(1-e- PS ) ..... (2)

a
L[f(t)]=~(~+P)_~(I+PS_
p s- s P S2
p- 1
s(1-e P") syE
1
ngi
nee
Exercise 8(F)
rin
1. If Lf(t) = <I>(s), Prove that
g.n
2.
L[~II(t)] = s3<1>(s) - s2 <I>(s) - s<l>l(s) - <l>1I(s)

If L[~I(t)] = Tan-I (+), f(O) =,2 and ~(O) = -1 find L[f(t)]


et

3. A periodic function f(t) of period 2a is defind by


f(t) = t, for 0 .'S t .'S a,
= 2a - t, for a < t < 2a

show that L[f(t)] = 1 tanh 7 (as)


2

Downloaded From : www.EasyEngineering.net


Downloaded From : www.EasyEngineering.net

642 Engineering Mathematics - I

4. Iff(t) = t2, ° < t < 2 and f(t+2) = f(t)


find L[f(t)].

5. Iff(s) = L(f(t»), Prove that

L[Sdl SJ'(U)duj == F~) and hence generalise the result.


o 0 s
8.2.9 Laplace transform of the unit step function
ww The unit step function orthe Heaviside function is defined as
°
w.E
H(t-a) =
H(t-a) =
ift<a,
lift < a

a
where a is a non-negative constant. In particular, when a = 0, we have
O,i/ 1 < ° sy
H(t)= [l,ifl>O
En
gin
The unit step function is the basic building block of certain functions whose
knowledge greately increases the usefulness of the Laplace transformation.

eer
It is easy to show that the Laplace transform of H(t-a) is given by

L(H(t-a»==-
e- U \
s i ng.
00 a 00

L[H(t - a) == fe-vI H(t - a)dt = fe-VI O.dl + fe-·\/l.d'


o 0 a
net
00

-e -sl e-as
s o s

. 1
In Particular, when a=O, we get L(H(t» = -
s
8.2.10 Heaviside shift theorem: If Lf(t) = f(s), then L[f(t-a)H(t-a) = e-as f(s).
Proof: By defnition, we have
00 a 00

L[f(t - a)H(t - a») == fe- si f(t - a)H(t - a)dt == fe- si f(t - a)O.dt + fe- si f(t - a)l.dt
o 0 0
00 00

== fe -s(p+a) f(p )dp == e -sa fe -sp (p )dp


(consider p = t-a)
o 0

Downloaded From : www.EasyEngineering.net


Downloaded From : www.EasyEngineering.net

Laplace Transforms 643

8.2.1 Examples
Find the L.T. of the function

f(t) =1 } if 0 < t < 1t


1. =0 if1t<t<21t
=sint ift>21t

SoL Let us represent f(t) interms of unit step functions as flt)=H(t)-H(t-1t)+H(t-21t)


sint

ww 1 -It' -2 It'
L[f(t)] = L[H(t)] - L[H(t-1t)] + L[H(t-21t)sint] = __ _e _ + _e_

2. w.E
Find L.T. oft2H(t-3)
s s s

asy
Let us express <I>(t) = t 2 as a function of(t-3) by using Talor's series, which states
that

_ fI En (x-a)2 II
f(x) - f(a) + (x-a) (a) + 2!
gin
f (a) + ................. + ............. .

¢(t)=94-(t - 3).6+
(t 3)2
2!
.2 e eri
Now, by the above result ng.
L[f(t).H(t-a)] = L[9+6(t-3) + (t-3fH(t-3)]
= L[f(t-3).H(t-3)] = e-3S Lf(t) where f(t) =
net
9+6t+t2

L[t2H(t-a)] = e-3s [ -9 + -6? + 32 + ......... .]


s s- s

3. Express F(t) = (t-2)2 when t>2 and f(t) = 0 when 0<t<2 in terms of Heavis ide unit
step function and find its Laplace transform
Sol. It is easy to see that
F(t) = (t-2)2 = (t-2f.H(t-2) (since for t<2, H(t-2)=0 and for ~2, H(t-2)=1)
L[f(t).H(t-2)] = L[(t-2)2H(t-2)] = e-2s .Lf(t) where f(t) = t2

-2,\ 2!
=e .3
s

Downloaded From : www.EasyEngineering.net


Downloaded From : www.EasyEngineering.net

644 Engineering Mathematics - I

Exercise 8(G)

I. Find the Laplace tranform off(t) using the unit step function, where

I ifO<t<1
2 ifl<I<3
f(t) =
4 if3<t<4
-2 if 4 <I

ww I e-I 2e-31 6e-~1


Ans. - + - + - - - - -
J
w.E [s s s s

2.
asy
Determine the Laplace transform of the following function:
6-2e-7t'i -4e-2""
(i) f(t) =

En
6-2H(t-1t) - 4H(t-21t) Ans.------
s

(ii) f(t) = e2I H(t-log5) gin 5 2-,


Ans. - -

e eri
5-s

e
-(1\'

(iii) f(t) = (t-a)H(t-a) Ans. -2


s
ng.
(iv) f(t) = t2 H(t-2) Ans.
e- "(2 4s 4s 2 )
2
+
--'-----.:3--...:...
S
+
net
1
(v) f(t) = H(t-e t ) Ans. -
s

(vi) f(t) = tH(t-2)

25 38 42 24]
(vii) f(t) = (l +2t-3t2 +4t3)H(t-2) Ans. e-2s - + - + - + -
[ s S2 S3 S4

1
(viii) f(t) = sint.H(t-1t) Ans. e-ltS --
S2 +1

Downloaded From : www.EasyEngineering.net


Downloaded From : www.EasyEngineering.net

Laplace Transforms 645

8.3.1 Inverse Laplace Transforms


If L[f(t)] = F(s) then f(t) is called the inverse Laplace transform of F(s) and is
denoted by L-I[F(s)] = f(t)
if fl and f2 are continuous functios on [0, 00] such that
L[fl(t)] L[f2(t)] or FI(s) = F2(s) (v- s>O) then fl(t) = fit) (v- t::::O)
=

Some of the important properties of the inverse laplace transforms are analogous to
the corresponding properties of Laplace transforms.
1. Linearity Property: If kl' k2 are any constants and F I(s) and Fis) are the Laplace

ww transforms of fl(t) and fit) respectives, then


L-I[kIFI(s) + k2Fis)] = klfl(t) + k/i t).
2.
w.E
First shifiing property: If L- I [F(s)] = f(t), then

asy jor t > a


for t < a

3.
En
Change of scale property: if L-' [F(s)] = f(t), then

gin
4.
e
Inverse Laplace transforms of derivatives: If L-'[F(s)] = f(t) theneri
ng.
5. Inverse Laplace transfonn of integrals: If L-'[F(s)] = f(t) then
net

6. Multiplication by's': If L-'[F(s)] = f(t) and f(O) = 0, then


L-[sF(s)] = fI(t)
7. Division by's': If L-I [F(s)] = f(t) then

Downloaded From : www.EasyEngineering.net


Downloaded From : www.EasyEngineering.net

646 Engineering Mathematics - I

. result can be extended to L- I ( - -


This F(S»)
s"
I I I

= ff..·················ff(u)du".
00 0

(the proof is left to the student)


8.3.2 Table of inverse transforms

L[f(t)] L- I [F(s)]

ww L(1) = -
I
L- I (~) = 1

w.E
L(e-at)
s
1
L- I (_I ) e-at

a
= -- =
s+a s+a

L(tn )
n! syE 1-1 ( I) + I)!- t"

ngi
= -
S"+I S'1+1 - (n
if 'a' is a positive integer

L(sinat) =
a
rl ( neI e ~
2 2) = sinat
S2 +a 2 s +a
rin a

L(cosat) =
S2
s
+a 2
L-I ( 2 S
s +a g2) = cosat
.ne
L(sinhat) =
S2
a
_a 2 L_ I ( 21
s -a
2)=~sinhat
a t
s
L(coshat) = L- I ( s 2 ) coshat
S2 _a 2 S2 _a

8.3.3 Using the above results inverse Laplace transforms of some simple functions can b~
obtained as below.
Find the inverse Laplace transform of:
2s+3 3s+5 4._1_
I. 2 -- 3. 2
• S2 +9 9s -25 4s+5
I 3 2s+ I s+3
5. 4s-5 6.- 7. (s + 1) 8·~4
s+ 4 s +

Downloaded From : www.EasyEngineering.net


Downloaded From : www.EasyEngineering.net

Laplace Transforms 647

Sol. From the above table, we get

I.

2.

= 2cos3t + sin3t

ww 3~+5
3. I
L- (
9s- - 25
)=L-I( 3s
9s 2 - 25
)+L-I( 2 5
9s - 25
)=~COSh(~)+~sinh(~)
3 3 3 3

w.E
4.
asy
En
5. gin
eer
L- (_3_)
I = 3rl_1_ = 3e- 41 ing
6.
s+4 s+4
.ne
7. L-I 2s+1
1
(s- + 1)
-2L-I(_s
- 1
s- + 1
) + L-I(_l
2
s +1
)_" .
- .... cost+smt t
8. L-I( S+3) -I(
-1--
s- + 4
=L s ) +L
-1--
s- + 4
-I( 3 ) =cos2t+-sm21
-1--'
s- + 4
3 .
2

Exercise 8(H)

Find the invere Laplace transform of:

2 1 1 4s+ 15
1. -+-+--
S S3 S +4 5. 16s 2 _ 25

Downloaded From : www.EasyEngineering.net


Downloaded From : www.EasyEngineering.net

648 Engineering Mathematics - I

[2
Ans. 1.2+ - + e-4t 2. 3cos4t + sin4t 3. 2cosh3t+sinh3t
2
,4
4.1 + t 2 + -
24

Examples 8.3.4
Find the inverse Laplace transform of:
6s-4 I s-2
1. 4. + -=-2- - -

ww
--"-2- - - 2
s -4s+ 20 2. (s _ 3)3 3. (s _ 2)4 (s-l) s -4s+5

Sol.
w.E
asy
En
gin
e eri
ng.
4 L- I
. (
1 ) + L-I
(S-2)4
(s - 2)
(s2-4s+5)
= L-I( I
(s-2i
) + L- I ( (s - 2)
s2-4s+4+1
)
net
=L -I( 1 ') ) + L
(s-2t
-l( S-2)
(S-2)2 + 12
= ')
et.t + e~t.cost

Exercise 8(1)

Find the inverse Laplace transform of:

s+4 (ii) s +6 4s+ 12


(i)---- (iii) -=----
S2 -4s+ 13 S2 -Ss +25 S2 +Ss + 16

Downloaded From : www.EasyEngineering.net


Downloaded From : www.EasyEngineering.net

Laplace Transforms 649

B.3.5 Method of partial fractions:


Whenever possible we exress the given function F(s) into the sum of linear or
quadratic partial fractions as
A Bs+C
F(s) = r + ? 2 r and then use the standard results to find the inverse L.T.
(s + a) (s- + a )

Examples (I) Find 4] for all s>O,


r l [ S3 +
s +s

ww
Sol. By partial fraction expansion, we have

w.E s+ 1 A Bs+C
---=-+---
S(S2 + I) S S2 +1

asy
s+1 = A(s2+1) + (Bs+c)s = As2 + A + Bs2 + cs
comparing coefficients of different powers of s from both sides
O=A+B
En
)= c
:. I = A, B =-1 gin
.
s+1 1 -s+1
=-+-- eer
.. S(S2 + I) S S2 + 1
By the linearity property of L-I, we have ing
- - = L-l(I)
L-l(S+l) - - L-1(-s- ) + L-1(-I- ) .ne
=I -cost+smt
.

2.
S3 + s ' S S2 + 1

2S2 -6s+5
S2 + I
t
If L[f(t)] = S3 + 6s2 + lIs -6 find [f(t)]

Sol. By partial fractions we have

J(s) = 2S2 -6s+5 =_A_+_B_+_C_


S3 -6s 2 +Ils-6 (s-l) (s-2) (s-3)

:. 2S2 -6s + 5 = A(s -2)(s -3)+ B(s -l)(s -3)+C(s -l)(s -2)
put s = I we get
I =2A A= 1/2
put s = 2
B=-1

Downloaded From : www.EasyEngineering.net


Downloaded From : www.EasyEngineering.net

650 Engineering Mathematics - I

Similarly s = 3 gives
C = 5/2

.
.. L-I[f'( S.)]_
--I L- I - -
I - L- I ( -I- ) +-
5 L- I ( -A- )_ 1 1 -e 21 +-e
--e 5 31
. 2 (s-I) 8-2 2 8-3 2 2

3s+ I
3. If Lf(t) = ( )( 2 I ' find f(t)
s-I s + )
Sol. By partial fractions we have

ww j(s)=~+ Bs+C
s -1 S2 + I

w:. Solving for A, B C

.Ea
we have A = 2, B = -2, C = -I

S -1 S2 + I syE
L- I [j(s)] = 2C l _1__ 2CI _s_ + L- I _)_1_
s- + I

= 2e t - 2cost + sint
ngi
4. If L[f(t)] =
s+2
(s + 3)(s + 1)3 find f(t) nee
=~_I__ ~_I_+~ rin
Sol. j(s) = s+2 I +1 1
(s+3)(s+I)3 8s+3 8s+1 4(8+1)2 2(8+1)3
g.n
(after expressing the function in partial fractions)

:.rl[f(8)]=~rl-I--~rl-I-+ 1
8 8+38 8+14(8+1)-
? + I 1
2(8+1)3
e t
=~e-31 - ~e-I -t ~/e-I +t 2e-1 = ~[(2t2 + 21 -1)e-' + e-31 ]
8 8 4 8
(S2 + 2s - 42)
5. If L[f(t)] = (S2 + 28 + 5)(8 2 + 2s + 2) find f{t)

as+b c8+d
Sol: Let f(s) = 82 + 28 + 5 + S2 + 28 + 2

:.S2 + 28 -4 =(a8 +b)(8 2 + 28 + 2)+ (cs + d)(S2 + 2s + 5)


Equating the coefficients of like powers of s we get a = 0, b = 3, c = 0, d = -2.

Downloaded From : www.EasyEngineering.net


Downloaded From : www.EasyEngineering.net

Laplace Transforms 651

3 2 3 2
:·f(s)= s2+2s+5 s2+2s+2 (s+I)2+22 (s+I)2+12

:.L -I( f(s)]=3L -I( 12 ? ) -2L


(s+l) +2-
-I[ 12 ?
(s+l) +1-
1

6. ww If L[f(t)] =
2s
4s2 + 16 ' find f(t)

Sol.
w
-I[ .E
By change of scale property we have

L 2
2s
4s +16 2 asy
] =-cos
1 21 since L- I [ 2 S
s +16
] = cos4t

En
e- 5•
7. If L[f(t)] =

gin
(s _ 2)4 find f(t)

Sol. We know rl[ 1 L-I[_I] e2/.~ e~t3e21e


1 = e 21 = =
(s - 2)4
rin
S4 3! 6

Thus by second shifting property

1 {Ig
g .ne
L- I
[
e -5s
(S-2)4
= -(I - 5)3e2(t - 5)
t
ift>5
if 1 < 5

Exercise 8(J)

Determine the inverse Laplace transform of each of the following functions.

10g(~)
3 s 2s+ 1
(i) (ii) (iii) (iv)
s+2 s2+2s+6 s(s + 1) s+2

s-5 e -2 .. S2
(v) (vi) (vii)
S2 +6s+ 13 s(s + 1) (S2 + 9)(S2 + 16)

Downloaded From : www.EasyEngineering.net


Downloaded From : www.EasyEngineering.net

652 Engineering Mathematics - I

2S2 + s -I 0
(viii) -(ix) log (I + Sl2 ) (x) 4
(s - 4)(S2 + 2s + 2) (8 + 64)

s
(xi) (xii)
s(s + I)(s + 2)(s + 3) (s4+s2+1)

2
8 + lOs + 13 s
(xiii) (xiv) 2
2 (8 + 1)(s2 +4)
8 -I 0(8 - 5s + 6)

ww
(xv)
8+29
(xvi)
5s+3

w.E
(S+4)(S2 +9) (s -1)(s2 + 2s + 5)

48+4 S2 + 1
(xvii)
(8 _1)2(8 + 2)
asy (xviii)
s3 +3s 2 +2s

Ans. En . ) -1 (-2/
( IV e -e -3/)

gin t

(v) e-3tcos2t - 4e-3tsin2t e


(vi) 4(1 - e-2t)(t - 2)
eri(~Sin4t
(vii) -%Sin3t)

ng.
(viii) e4t - e-t(cos2t+2sint)
2
(ix) -(I-cost)
t
(x)
I
"8
ne
sin2t sinh2t

t
. I
6 2
I _/ 1 -2/ 1 -3/
(XI) -.l--e +-e
2
--e
6
. J32(. TJ5 t.sln'ht]2"
(XII) Sin

(xiii) 12et - 37e2t + 25e3t (xiv) "3I (cost - cos2t) (xv) e--4t - cos3t + "35 sin3t

(xviii) .! - 2- et + ~ e-2l
2 2

Downloaded From : www.EasyEngineering.net


Downloaded From : www.EasyEngineering.net

Laplace Transforms 653

8.4 Convolution Theorem


If L[f(t)] = F(s) and L[g(t)] = G(s) the inverse transfonn of F(s). G(s) denoted by
H(t), is called the convolution of f and g is defind as given below.
I

H(t) = (f*g)(t) = f(t-u)g(u)dll,vtzO ..... ( I)


o
The convolution operation * has the following properties.
(f*g)(t) = (g*f)*t), v t ~ 0 (commutative)
[f*(g*h)](t) [(f*g)*h](t) v- t ~ 0 (associative)

ww
=

[1'*(g+h)](t) = (f*g)(t) + (f*h)(t) v- t ~ 0 (dstributive)


8.4.1
w
Theorem: Let f(t) and get) be the inverse transforms of F(s) and G(s) respectively,

.Ea
and satify the hypothesis of the existence theorem (8.10.2) then the the inverse
transform h(t) of the product F(s) G(s) is the convolution of f(t) and get).

syE I

h(t) = (f*g)(t) = fl(t-u)g(ll)du


o
I ngi
i.e. L-'[F(s)G(s)] =
o
fl(t - u)g(u)du
nee ..... (i)

Proof: It is sufficient if we show that

1f
rin
F(s) G(s) ~ L[ (l- u)g(u)du 1 g.n
e t
Now F(s)G(s) ~ [fe" f( v)dv] [V· g(u)du ]

=
00
rrllo
Ile-(U+V)S J(v)g(u)dvdu = le-S(V+U) J(V)dV]g(U)dU'
where the integration is extended over the first quadrant (u ~ 0, v .:=:: 0) in the
uv-plane. We now introduce a new variable 't' in the inner integral of the last expression
by taking v = t-u so that u + v = t and dv = dt (u being fixed during this integration).
Thus

F(s)G(s) = rrllule->t J(t - U)dt]g(U)dU = Ile->t J(t -u)g(u)dtdu


0 U

Downloaded From : www.EasyEngineering.net


Downloaded From : www.EasyEngineering.net

654 Engineering Mathematics - I

al a

F(s)G(s)= fTIe-SI!(t-u)g(u)du]dt= fe- jlr{t-u)g(u)du}tt


s, I
00 0 0

8.4.2 Examples

ww ~ 2 )2l
(I) Evaluate r' [ s (s 1+ I using the convolution theorem

w.E
L-'C, )~Ia r'((S~I)' )~/e-'
Sol.
syE
we have

By the convolution theorem


and

ngi
=L-
I
[ 2(
nee U
1 y]= J(ue- -{t-u}du= J(ut-u )e- )du
s s+1 0 0
2 U

= [(ut -u2)e-u - (t-2u)e-u + (-2)(-e-U) = t-e-t + 2e- 1 + t-2


rin
(2) ~valuate L- I
[ 2 S 2 2] using the convolution theorem.
(s +2 ) g.n
Sol. Let F(s) = ~4
s s
' G(s) = - 2 - and Let ~s) = F(s) G(s)
e t
s + s +4

_I s
f(t) = L-I [F(s)] = L (-2-) = cos2t
s +4

1
g(t) = L-l[G(s)] = r-I (-2-_) = ! sin2t
s +4 2

f(u).;: cos2u CIld g(t-u) = 2"1 sin2(t -u)

Downloaded From : www.EasyEngineering.net


Downloaded From : www.EasyEngineering.net

Laplace Transforms 655

I I
C l [tp(S)] = JCOS 2u -sin 2(1 - u) du
o 2
I

= ~ JSin 2t + sin 2(t - 2u )dll


40

=~[sin2t.U+~COS2(t-2U]1 ~tsin2u
4 2 04

(3)
ww
Evaluate L-
I
~ 1using convolution theorem.
w.E
[
s s+4

1
Sol. Let F(s) =
vs+4
~
asy G(s) = - and choose
s
¢i...s) = F(s).G(s)
En
.. f(t) = L- [F(s)] =
I
r
I
~
= e-4t
gin
CI_1_ =e-41
s~
t~ = r;;
r- 1
e-
41

(s + 4)

1
2

e eri
2 '\I1tl

g(t) = L-1[G(s)] = L- 1 (-)


s
=1
ng.
f(u) =
e --4u
rand g(t-u) = 1
'\I1tU
net
By convolution theorem
1
Je-
I -4u 2!i
x2
L-1[$(s)] = Jer- du = r dx
o '\I 1tU '\I 1t 0

where 4u = .xl 2du =xdx

(where erf(t) J
I

= 2r e- x
2
dx
'\I1t 0

(erft stands for error function)

Downloaded From : www.EasyEngineering.net


Downloaded From : www.EasyEngineering.net

656 Engineering Mathematics - I

Exercise 8(K)

Using the convolution theorem find the inverse Laplace transforms of

(v) log«s+2» (vi) 1 ?


S +1 (s + 1)(s- + 1)

ww 1 1

w.E (ii) "3 (2sin2t-sint) (iii) "3 tsint

(iv) Ie
-/I
-e
u
-211

asy
du I f4 -cost-smt
(vi) -Le . ]
o
En 2

8.4.3 Applications to differential equations


gin
Suppose we wish to find the particular solution of the differential equation
lay" + /3y' + yy = J(t)
which satisfiestheinitial conditionsy(O) Yo and yeO) = Yo e
= eri ..... (1)

ng.
Taking the Lapa1ce transform on bothsides of (l) and using its linearity, it follows
that
aL(y") + /3L(y') + yL(y) =
but L(y") = s2L(y) -sy(O) _yl(O)
L[J(t)] net
L(y') = sL(y) - yeO)
a[s2L(y) - sy(O) - yl(O)] + /3[sL(y) - yeO)] + yL(y) = L[f(t)]
and hence
(as 2 + /3s + y)L(y) - L[f(t)] + (as + /3)Yn + ayJ
=> L(y) = L[/(t)] + (as + /3)yo + aYb
as 2 + /3s +y

The function L[f(t)] is a specific function of s since f(t) is known; and since a, /3, y.
}'0 and y J are constants, L(y) is completely known as a function of s. The inverse
Laplace transform of L(y) will be the solution of the given diff. eqn.
(Here it is assumed that the functions f( t), y, y' and y" must have Laplace transforms).

Downloaded From : www.EasyEngineering.net


Downloaded From : www.EasyEngineering.net

Laplace Transforms 657

Solved Examples

8.4.4
2
' d th
Fm e soiutlOn d2
' 0f - y + -dy - 2Y = smt
' w h'IC h satls e Imtla I cond'ItlOI1S
'files t h'" , y = 0,
dt dt
y' = 0 when t = 0
Sol. Applying the Laplace transforms to both sides of the given equation and by linearity,
we have
L(yW) + L(y') - 2L(y) = L(sin t)

ww 1
As we know that L(sint) = -2-1 ;s > 0
s +

w .Ea
[s2L(y) -sy(O) - y' (0)] + [sL(y) - yeO)] -2 L(y)
1
= -2-1
s +

syE
Given conditions yeO) = 0, y' (0) = 0

=> s2L(y) + sL(y) -2 L(y) =


ngi -2-
1
s +1

L(y)= ?
1
(s- + S - 2)(S2 + 1)
=-----:---
nee
(s -IXs + 2)(S2 + 1)

we have the partial fraction expansion rin


1 A B C . +D
- - - - - - - : - - = - - + - - + -":-- g.n
where
(s -1)(s + 2)(S2 + I)

A
6' 15 10
s -I s + 2

=.! B =-=! ,C =-=! and D = - 3


10
S2 + I
e t
Therefore we get
• 1 1
yet) = L- 1 - - - - - - 1
(s -1)(s + 2)(S2 + 1)

as the solution to the given initial value problem,

Downloaded From : www.EasyEngineering.net


Downloaded From : www.EasyEngineering.net

658 Engineering Mathematics - I

8.4.5 Example
Solve the initial value problem

y"-3y'+2y=e 31 .y=l,y'=0 wheret=O

Sol. Applying Laplace transform to both sides of the differential equation, we get
1
[s2Ly(t) - sy(O) - y' (0)] -3 [sLy(t) - yeO)] +2 L(t) = L(e3!) = -
s-3
Applying the given conditions, we get

ww (s2 - 3s + 2) Ly(t) = s-3 + -


1

w.E s-J
s-3
I

asy
:. L[y(t)] = (S2 _ 3s + 2) + (s - 3)(S2 - 3s + 2)

En 1
:. L(y(t)] = (s _ 3)(S2 _. 3s + 2) + (8 2-
s-3
3s + 2)

gin 1 8-3
--------+------
(8-1)(s-2)(s-3) (s-I)(8-2)
eer .... (1)

ing
To find yet), we expand each term on the right hand side of (I) in partial functions,
Thus

------=----+---
ABC .ne
(s-I)(s-2)(s-3) s-1 s-2 s-3

1 = A(s-2) (s-3) + B(s-l) (s-3) + C(s-I)(s-2)


t
I
substituting s = I gives A = 2" ;similarly substituting s = 2 gets B = -1 and substituting

I
s = 3 gives c ="2 hence

- - -1- - - = 1
--- +----
(s-I)(s-2)(s-3) 2(8-1) s-2 2(8-3)
Similarly we can also write
s-3 D E
----=-+--
(s-I)(s-2) s-1 s-2

Downloaded From : www.EasyEngineering.net


Downloaded From : www.EasyEngineering.net

Laplace Transforms 659

s-3 = D (s-2) R(s-I)


Taking s =: 2, we get E =:-1
similarly s = I we get D = 2

I I I I I
Hence L[f(t)] = --+ +---
2(s-l) s-2 2{s-3) s-I s-2

221
- - - - + -,-----
- 2( s - I ) s - 2 2{s - 3)

ww
w.E
asy
Example 8.4.6
En
Sol.
gin
Solve yll + 3yl + 2y = 2t3 + 2t + 2 with yeO) = 2, yl(O)

eer
Applying Laplace transform to both sides of the given equation we get
L(yll) + 3L(yl) + 2L(y) = 2L(t2) + 2L(t) + 2

? I 4 2 2
[s-Ly(t) - sy(O) - yeO)] + 3 [sL[y(t)] - yeO)] + 2Ly(t) = 3 + -2 + - ing .... (1)
_ s
.ne s s

using the given conditions (1) gives

(
2 3 2
s + s+ )Ly t
()
=
2(2+s+s2)
S
3
2
+ s+
6
=
3
2(S4 +3s +S2 +s+2)
S
3
t

By partial fractions,

yet) = 3L-I(~) -2r L\ ) 2L-ILI


1
+ 3) - L-Ie ~ 2)
yet) =3 - 2t + t 2 - e-2t

Downloaded From : www.EasyEngineering.net


Downloaded From : www.EasyEngineering.net

660 Engineering Mathematics - I

Exercise 8(1)

Using Laplace transform method solve the following differential equations with
the given conditions.
I. yll + 2yl - 3y = 0 at, y = 0, yl = 4 when x = 0
2. yll + 4y = 4t at y = I, yl = 0 when t = 0
3. yll - 2yl + Y = et at y = 2, yl = -I when t = 0

ww 4. yll + k2y = coskt at y = 0, yl = "2


I
when t = 0

w.E 5. yll- y = 1 at y = -I, yl = I when t = 0


yll + 2yl + 3y = 3e-t sint; yeO) = yl(O) = 0
6.
7. asy
xii + 4xl = -8t given x(O) = 0, xl(O) = 0
8.
En
yll + 9y = 18t given yeO) = 0, yl(O) = 0

9. gin"2
yll + Y = sint given yeO) = -I, yl(O) =
1

eer
Ans.l. y = eX - e-3x
4
1
8
I
ing I
2. y= - t- - sint + cost 3. y = 2et - tet + -2 t2et

1. k 1 .
4. Y = 2k Sill t + "2 tSll1t 5. y = -1 + et - cosht .ne
6. y = 3e-tsint

7. x =
-I
8 +"2
1 2
t- t +
I
"8 e-4t 8. y = 2t -
2
"3 sin3t
t

Downloaded From : www.EasyEngineering.net


Downloaded From : www.EasyEngineering.net

ww
w.E
a syE
ngi
nee
rin
g.n
et

**Note : Other Websites/Blogs Owners we requested you, Please do not Copy


(or) Republish this Material.
This copy is NOT FOR SALE.
**Disclimers : EasyEngineering does not own this book/materials, neither created nor
scanned. we provide the links which is already available on the internet. For any quarries,
Disclaimer are requested to kindly contact us. We assured you we will do our best. We DO
NOT SUPPORT PIRACY, this copy was provided for students who are financially troubled but deserving
to learn. From : www.EasyEngineering.net
Downloaded Thank You and God Bless!

You might also like